Download AANPChapters1-14 - Teddie Joe Snodgrass

Survey
yes no Was this document useful for you?
   Thank you for your participation!

* Your assessment is very important for improving the workof artificial intelligence, which forms the content of this project

Document related concepts

Nurse–client relationship wikipedia , lookup

Evidence-based nursing wikipedia , lookup

Patient advocacy wikipedia , lookup

Electronic prescribing wikipedia , lookup

Transcript
What material does the program include?
Information covered in this course:
1.
2.
3.
4.
5.
6.
7.
8.
9.
10.
11.
12.
13.
14.
15.
16.
17.
18.
19.
20.
Core concentration areas pertaining to specialty area:
Taking the Certification Examination, FNP Professional Role
Health Promotion/ Disease Prevention
Head, Eyes, Ears, Nose and Throat Disorders
Respiratory Disorders
Cardiovascular System Disorders
Endocrine Disorders
Psychiatric/ Mental Health Conditions
Renal and Urologic Conditions
Infectious Diseases
Hematological Disorders
Sexually Transmitted Diseases
Dermatological Disorders
Musculoskeletal Disorders
Neurologic System Disorders
Gastrointestinal System Disorders
Male and Female Reproductive Disorders
Content slide handouts can be accessed and printed with registration.
Contact hours are awarded with completion and processing of verification documentation.
No contact hour credit will be awarded after 8/31/2013.
A maximum of 19.43 contact hours may be earned by learners who successfully complete this continuing nursing education
activity.
ANA’s Center for Continuing Education and Professional Development is accredited as a provider of continuing nursing
education by the American Nurses Credentialing Center’s Commission on Accreditation.
ANCC Provider Number 0023
ANA’s Center for Continuing Education and Professional Development is approved by the California Board of Registered
Nursing, Provider Number CEP6178 for 23.32 contact hours (50 minute contact hour).
ANA’s Center for Continuing Education & Professional Development includes ANCC’s Credentialing Knowledge Center.
How much does it cost? $400 allows unlimited use for ninety days from the date of purchase.
What technical capabilities do I need? All you need to get going is an e-mail address and access to the Internet (with Flash
Player.) However, in order to access all content and materials available with the course, a sound card with speakers (or
headphones) and printer are also required. Click here for minimum technical requirements.
When can I take this course? The Web-based course is available for enrollment at your convenience. You can review the
material in the course as many times as you like during this ninety-day period.
How do I access the course? Simply click on the enrollment link at the top right of this page. You will be asked to register
and submit your credit card information. Once you are done you will receive a confirmation e-mail which will give you login
information and a direct link to the Web course.
Note: You will use the log-in and password that you create during the purchase process to log in to the course. After
this, you can to access all materials at anytime through the Web.
1
Family Practice Board Exam Topics:
•Atopic Dermatitis
•Eczema
•Human Immunodeficiency Virus
•Paget's Disease
•Systemic Lupus Erythematosus
•Hyperlipidemia
•Congestive Heart Failure
•Aortic Stenosis
•Myocardial Infarction
•Lichen Planus
•Treatment of Psoriasis
•Impetigo
•Acute Appendicitis
•Management of Hypothermia
•Type 2 Diabetes Mellitus
•Hyperthyroidism
•Hypocalcemia
•Cushing's Disease
•Peptic Ulcer Disease
•Esophageal Dysphagia
•Lactose Intolerance
•Cirrhosis of The Liver
•Epigastric Hernias
•Dementia
•Parkinson's Disease
•Osteoporosis
•Sideroblastic Anemia
•Sickle Cell Disease
•Thalassemia
•Von Willebrand's Disease
•Multiple Myeloma
•Polycythemia Vera
•Chronic Leukemia
•Hodgkin's Lymphoma
•Bacterial Meningitis
•Conjunctivitis
•Hepatitis B (HBV)
•Methicillin-Resistant S Aureus
•Nephrotic Syndrome
•Addison's Disease
•Hypernatremia
•Metabolic Acidosis
•Nephrolithiasis
•Temporal Arteritis
•Friedreich's Ataxia
•Horner's Syndrome
•Crohn's Disease
•Hyperemesis Gravidarum
•Macrosomia
•Sarcoidosis
2
Quiz 1 annc
1. Which of the following foods would NOT be a dietary trigger influencing the onset or severity of migraine
symptoms?
A. Stilton cheeses
B. Pickled herring
C. Broad beans
D. Kale
-----Explanation:
Correct answer: Kale
There are many foods that are potential dietary triggers influencing the onset or severity of migraine symptoms. Kale and
other fresh green vegetables are not one of these types of foods. Other triggers include: bananas, citrus foods, pizza,
chicken liver and freshly baked yeast products.
2.
A nurse practitioner (NP) has worked at a large hospital as an RN and now, as a new NP. She has developed an NPmanaged clinic for hospital employees and is employed by the hospital. The NP is described as a(n):
A. Risk taker
B. Nurse specialist
C. Intrapreneur
D. Entrepreneur
The Correct answer is: Intrapreneur
An intrapreneur is someone who is able to carve out a specialty role within an existing organization, healthcare setting, or
business setting. An entrepreneur is someone who assumes the financial and personal risks of owning their own business,
which they also operate.
3.
When evaluating the carotids, with correct procedure for auscultating is:
A. Use the bell of the stethoscope.
B. Position the client at a 30-degree angle, and press firmly with bell of the stethoscope.
C. Use the diaphragm of the stethoscope.
D. Place the stethoscope 1 inch off the area above the sternocleidomastoid muscle.
Explanation: The correct answer is to use the bell of the stethoscope. The correct procedure is to listen for carotid bruits with
the bell of the stethoscope, which brings out low-frequency sounds and filters out high-frequency sound. The bell should be
placed very lightly on the neck with just enough pressure to seal the edge.
11. Which of the following is a typical sign in a person who develops type 2 diabetes?
A. polyuria
B. polydipsia
C. polyphagia
D. all of the above
---------------------------------------------------------------Explanation: Correct answer: all of the above
All of the first three choices are signs/symptoms experienced by a person with type 2 diabetes. Other associated
symptoms include: parasthesia, dysesthesias, blurred vision, vaginal candidiasis, or fungal skin infections.
12. Which of the following is the most common cause of stasis ulcers?
A. diabetes mellitus
B. fungal infections
C. venous insufficiency
D. arterial insufficiency
Correct answer: venous insufficiency
Stasis ulcers are most commonly caused by venous insufficiency. They are far less commonly caused by arterial
insufficiency, diabetes mellitus and fungal infections.
13. A 66-year-old African-American male with Type II Diabetes is in the office for a check up and medication refills. You
conduct a diabetic foot screen for evaluation. Which question is not part of the Carville Diabetic Foot Screen?
A. Does the foot have an abnormal shape?
3
B. Are the nails thick, too long, or overgrown?
C. Has there been a change in the foot since the last evaluation?
D. Does the foot hurt when walking?
The correct answer is: "Does the foot hurt when walking?"
The Carville Diabetic Foot Screen does not include the question, “Does the foot hurt when walking?” The Diabetic Foot
Screen helps practitioners to determine if a patient with diabetes mellitus has a current and severe foot issue or has risk factors
for acute foot issues. The five questions of the Carville Diabetic Foot Screen include:
Does the foot have an abnormal shape?
Are the nails thick, too long, or overgrown?
Has there been a change in the foot since the last evaluation?
Is there weakness in the ankle or foot?
Is there a foot ulcer now or a history of foot ulcer?
14. You are examining a 12 month old infant at a well-baby checkup. You would expect his heart rate to be in which of the
following ranges?
A. 55 – 95 beats per min.
B. 80 – 160 beats per min.
C. 65 – 105 beats per min.
D. 70 – 190 beats per min.
Correct answer: 80 – 160 beats per min.
The normal range for heart rate for a 12 month old infant is 80 – 160 beats per min. This is a resting measurement.
15. Marketing refers to determining the needs and desires of the prospective consumer and designing NP services to meet
those needs. The key elements of marketing include the four Ps which are:
A. product, price, place, and prescription
B. profession, place, price and production
C. product, price, place and promotion
D. profession, price, place and prescription
Correct answer: product, price, place and promotion
The key elements of marketing include the four Ps. These are: product (the unique role of the NP); price (cost
advantage in the NP-delivered care); place (competitive advantage of a practice site or after-hours services); and
promotion (processes devoted to negotiating a position in practice).
16. You have a patient whom you suspect has rubella. You would expect to see which of the following types of rash on this
patient?
A. pinpoint to 1 cm oval or round pink macules and/or papules
B. bright red to purple macules and papules
C. bright red confluent lacy rash
D. sandpaper-like red papules
Correct answer: pinpoint to 1 cm oval or round pink macules and/or papules
Rubella is marked by pinpoint to 1 cm oval or round pink macules and/or papules. The rash begins on the face and neck and
spreads rapidly to the trunk and extremities.
17. When screening for tuberculosis (TB) which of the following is least likely to be used?
A. sputum culture
B. PPD
C. Mantoux test
D. none of the above
Correct answer: sputum culture
4
When screening for TB, the purified protein derivative (PPD) test or the Mantoux test are used. Sputum cultures or
chest X-rays are not used for screening.
18. In terms of the scope of practice for an NP, which of the following statements is incorrect?
A. The scope of practice defines a specific legal scope determined by state statutes, boards of nursing, educational
preparation and common practice within a community.
B. General scope of practice is specified in many published professional documents.
C. Prescriptive authority is recognized as within the scope of practice for nurse practitioners in all 50 states.
D. Scope of practice is always defined by state statutes enacted by the state legislature.
Correct answer: Scope of practice is always defined by state statutes enacted by the state legislature.
Some states define scope of practice by state statutes enacted by the state legislature. In other states, the legislature
gives the board of nursing the authority to define the scope of NP practice. Either way is enforceable.
19. The most common cause of Cushing’s disease in children is which of the following?
A. hypersecretion of ACTH
B. adrenal adenomas
C. long-term corticotropic or glucocorticoid use
D. nonpituitary neoplasms
Correct answer: long-term corticotropic or glucocorticoid use
All of the choices are causes of Cushing’s disease. However, the most common cause in children is long-term
corticotropic or glucocorticoid use.
20. Which of the following does not play a significant role in the development of varicose veins?
A. diabetes mellitus
B. wearing constricting garments
C. weakness of the walls of the vein
D. pregnancy
Correct answer: diabetes mellitus
Diabetes does not play a significant role in the development of varicose veins. An inherited venous defect of either a valvular
incompetence or a weakness in the walls of the vessel likely plays a significant role. In addition, situations that cause high
venous pressure, such as wearing constricting garments and pregnancy, contribute to their development.
21. Lead poisoning remains a serious public health problem. Which of the following is the least likely to present a lead
hazard?
A. lead-glazed pottery
B. lead-soldered vessels used for cooking
C. fumes from burnt casings of batteries
D. paint in a home built in 1995
Correct answer: paint in a home built in 1995
This is the least likely source to present a lead hazard. Lead-based paint has not been available in the USA for
more than 30 years. Therefore, a home built in 1995 would not contain lead-based paint
22. An acute viral illness caused by the Herpes viride family, that presents as an erythematosus, maculopapular rash
appearing on the trunk is which of the following?
23. rubella
24. rosacea
25. rubeola
26. roseola
Correct answer: roseola
5
Roseola is an acute viral illness caused by an infection of the Herpes viride family, with most cases caused by human
herpes virus. Infection occurs throughout the year and as often in males as in females.
23. For a healthcare professional to obtain blood at the request of law enforcement officials without a patient’s consent which
of the following must be present?
A. The suspect must be under arrest.
B. A delay in drawing blood would lead to destruction of evidence.
C. The test is performed in a reasonable manner.
D. all of the above
Correct answer: all of the above
All of the first three choices are factors that must be present. Two other conditions must also be present: the
likelihood that the blood drawn will produce evidence for criminal prosecution; and the test is reasonable and not
medically contraindicated.
A.
B.
C.
D.
24. White papules found on the gum line resembling an erupting tooth are which of the following?
Epstein’s pearls
epispadias
hypospadias
Moro lesions
Correct answer: Epstein’s pearls
Epstein's pearls are small white or yellow cystic vesicles (1 to 3 mm in size) often seen in the mouth of newborn
infants. They do not require treatment because they resolve spontaneously over the first few weeks of life.
25. Which of the following parts of Medicare pays 80% of durable medical equipment such as wheelchairs and walkers?
A. Part D
B. Part A
C. Part B
D. Medicare does not pay for durable medical equipment.
Correct answer: Part B
Medicare Part B pays for medically necessary services or supplies (outpatient). This includes durable medical
equipment such as wheelchairs and walkers.
26. Which of the following statements about tendonitis and its treatment is false?
A. With rotator cuff involvement, the likelihood of concurrent bursitis is low.
B. When the hand or wrist is affected, splinting and NSAIDs are reasonable first-line therapies.
C. Achilles tendonitis may necessitate treatment with a posterior splint.
D. There is a 10% risk of tendon rupture with recurrent Achilles tendonitis.
Correct answer: With rotator cuff involvement, the likelihood of concurrent bursitis is low.
This statement is false. With rotator cuff involvement, the likelihood of concurrent bursitis is high. Treatment
includes limiting overhead movement and intrabursal corticosteroid injection.
27. A female patient has a history of vaginal itching and heavy white discharge. She reports no recent sexual activity and
upon examination the nurse practitioner (NP) discovers a red edematous vulva and white patches on the vaginal
walls. The NP expects what factor to be in the patient's history?
A. early menopause
B. recent antibiotic use
C. vegetarian diet
D. recent diarrhea
The Correct answer is: Recent antibiotic use
6
These findings are consistent with candidiasis and the majority of women, almost half, develop a vaginal infection due to
candidiasis, after completing antibiotics. Further, candidiasis is not a sexually transmitted disease.
Question 28
You are counseling a mother who has her 7-month-old infant in the office with the complaint of "spitting up his formula." The
nurse practitioner (NP) knows this to be a common thing for infants but the mother conveys she has "put him on goat's milk."
The NP is concerned because she knows that goat's milk places the infant at risk of developing:
scurvy
rickets
megaloblastic anemia
botulism
The Correct answer is:
Megaloblastic anemia
This condition can develop secondary to folic acid deficiency. Goat's milk does not contain the appropriate level of folic acid
needed for the infant. Scurvy is caused by a lack of ascorbic acid and rickets is a result of a diet lacking in Vitamin D.
Botulism is food poisoning due to an endotoxin produced by the bacillus Clostridium botulinum.
Question 29
You are treating a patient for a knee sprain. The nurse practitioner understands that initial management includes:
anti-anxiety medications
rest, ice, compression, and elevation
activity as tolerated
X-rays
The Correct answer is:
Rest, ice, compression and elevation
Rest, ice, compression, and elevation, which is often abbreviated as RICE, is a standard initial management for a knee sprain,
along with anti-inflammatory medications. X- rays are not necessary for a sprain. Further, activity is usually withheld for 48
hours.
Question 30
Your elderly patient has tan-colored macules on the dorsum of the forearms. You understand that these are which of the
following?
seborrheic keratoses
senile purpura
lentigines
actinic keratoses
Correct answer:
lentigines
Lentigines are also known as “liver spots.” They are tan-to-brown colored macules on the dorsum of the hands and forearms
from sun damage. They are more common in light skin and are benign.
Question 31
The FNP has a 70-year-old female patient who has come in because she began to feel weak and dizzy. After examination the
FNP suspects atrial fibrillation. The FNP’s plan of diagnosis and treatment for this woman would include which of the
following?
doing a 12-lead electrocardiogram (EKG)
prescribing an anticoagulant
monitoring response to anticoagulation with the INR
all of the above
Correct answer:
all of the above
The diagnosis and treatment for this woman would include all of the first three choices. The anticoagulation guidelines are
atrial fibrillation: INR 2.0 – 3.0; synthetic valves: INR 2.5 to 3.5.
7
Question 32
Which of the following is NOT an absolute contraindication to postmenopausal estrogen therapy?
seizure disorder
unexplained vaginal bleeding
acute liver disease
neruo-ophthalmologic vascular disease
Correct answer:
seizure disorder
All of the choices are absolute contraindications to postmenopausal estrogen therapy except seizure disorders. In the case of
seizure disorders, estrogen therapy can be used with caution, considering if the benefit outweighs the risk.
Question 33
The FNP has a 67-year-old male patient with type 2 diabetes. In managing this patient’s care which of the following lab tests
should be done every 3 – 6 months?
thyroid assessment
urine microalbumin/creatinine
serum creatinine
A1c
Correct answer:
A1c
The A1c test should be done every 3 – 6 months. The goal for this test is a result of ≤ 6.9% (ADA) or ≤ 6.5% (AACE).
Question 34
The first-line treatment for osteoarthritis in European nations that is a nutritional supplement here in the United States is which
of the following?
timed antacid use
scralfate
glucosamine
misoprostol
Correct answer:
glucosamine
In many European nations glucosamine is a first-line treatment of OA. In the United States it is an over-the-counter nutritional
supplement. The results of research studies have differed on the effectiveness of its use, with many reporting no improvement
in arthritis symptoms.
Question 35
You are seeing a teenage girl who comes to the clinic and tells you she was raped late last night by her date. The immediate
action taken by the nurse practitioner is:
Send patient to emergency room for examination
Call her parents so they can be with her.
Perfom a pelvic examination to determine her injuries.
Send her immediately for counseling to help her deal with this situation.
The Correct answer is:
Send patient to emergency room for examination
This patient should be examined by emergency room personnel who are specifically trained to collect appropriate evidence
needed to testify in court regarding rape. The exam cannot be done in the office unless the nurse practitioner has protocol and
training in this area. The patient decides whom to call, and should be offered rape crisis resources.
Question 36
You are doing a complete physical exam on a 25-year-old female patient with sleep apnea. Which of the following are you
most likely to find in this patient?
8
bruit
paroxysms of sweating
increased blood pressure
atrial fibrillation
Correct answer:
increased blood pressure
Increased blood pressure is the only choice that might be found in a patient with sleep apnea. Bruit might be found in renal
artery stenosis. Paroxysms of sweating may be found in a person with pheochromocytoma. Atrial fibrillation might be found
in a person with hyperthyroidism.
Question 37
Which of the following is NOT classified as a generalized seizure?
absence seizures
myoclonic seizures
status epilepticus
tonic seizures
Correct answer:
status epilepticus
Status epilepticus is considered an unclassified epileptic seizure. Other generalized seizures (convulsive or nonconvulsive)
include petit mal, clonic, tonic-clonic, atonic seizures and infantile spasms.
Question 38
Of the following hormones, which one stimulates the ovaries to ovulate?
TSH
GH
ACTH
LH
Correct answer:LH
LH (luteinizing hormone) stimulates the ovaries to ovulate. It is also responsible for the production of progesterone by the
corpus lutea
Question 39
In males, puberty is considered precocious if it occurs before:
12 years of age
9 years of age
14 years of age
10 years of age
Correct answer:
9 years of age
In boys, puberty starts at 9 years of age. It is considered precocious puberty if it begins before 9 years of age. In girls puberty
starts at 8 years of age and is considered precocious if it begins before 8 years of age.
Question 40
You are treating a 38-year-old mildly obese female with peptic ulcer disease symptoms. The most cost effective, sensitive and
specific test for Helicobacter pylori infection is:
fecal DNA testing
organism-specific stool antigen testing
serological testing for antigen related to the infection
stool Gram stain, looking for the offending organism
The Correct answer is:
Organism-specific stool antigen testing
9
The most cost-effective method of diagnosing H. pylori infection is the organism-specific stool antigen testing. Serological
testing is also available but has limitations. H. pylori is transmitted via the oral-fecal and oral-oral route. It is a gram-negative,
spiral shaped organism that is seen in duodenal ulcer and gastric ulcer disease.
Question 41
The most common cause of acute diarrhea in children is which of the following?
Escheria coli
rotavirus
salmonella
shigella
Correct answer:
rotavirus
The most common cause of acute diarrhea in children is rotavirus. it causes 25% of cases. Other pathogens include Norwalklike viruses, enteric adenoviruses, astroviruses, and caliciviruses. Bacterial pathogens include salmonella, shigella, yersinia,
campylobacter, and more.
Question 42
The process by which a voluntary, non-governmental agency or organization appraises and grants status to institutions and
programs or services which meet predetermined structure, process and outcome criteria is which of the following?
accreditation
certification
licensure
scope of practice
Correct answer:
accreditation
Accreditation is the process by which a voluntary, non-governmental agency or organization appraises and grants status to
institutions and programs or services which meet predetermined structure, process and outcome criteria. The purpose is to
assure that the organization has met specific standards.
Question 43
The FNP has a patient who is hypertensive. The FNP is making some suggestions of lifestyle modifications as recommended
by the JNC-7 report. Which of the following recommendations is likely to produce the best average systolic blood pressure
reduction rate?
moderation of alcohol consumption
aerobic physical activity
dietary sodium reduction
weight reduction
Correct answer:
weight reduction
Weight reduction will give the patient a 5 – 20 mm Hg reduction for every 10 kg of weight lost. Moderation of alcohol
consumption will give a 2 – 4 mm Hg reduction; aerobic physical activity will give a 4 – 9 mm Hg reduction; and dietary
sodium reduction will give a 2 – 8 mm Hg reduction.
Question 44
The FNP has a pregnant patient who is Rh negative and whose baby is Rh positive. The treatment for the infant antepartum
includes which of the following?
transfusion with Rh negative blood
phototherapy
transfusion of packed red blood cells
administration of gamma globulin
Correct answer:
transfusion with Rh negative blood
Once a diagnosis has been established, antenatal treatment includes transfusion of the fetus with Rh negative blood. All of the
other choices are parts of postpartum interventions.
10
Question 45
Communication theory emphasizes all of the following EXCEPT:
interaction of individuals that includes both verbal and nonverbal communication
emotional support, shared information, and instruction
that messages are not time-bound
that communication conveys values and beliefs between members and the external environment
Correct answer:
that messages are not time-bound
On the contrary, communication theory emphasizes that messages are time-bound. They must be appreciated within the
context of the sender.
Question 46
Your patient has been diagnosed with Bell’s palsy. The cranial nerve involved in this disease is which of the following?
CN V
CN VI
CN VII
CN VIII
Correct answer:
CN VII
CN VII is the facial nerve. Dysfunction of this nerve gives the characteristic findings of Bell’s palsy (facial asymmetry, droop
of mouth, absent nasolabial fold, impaired eyelid movement).
Question 47
All of the following are factors that would protect a person from the possibility of gastroenteritis EXCEPT:
normal bacterial flora of the intestine
acidity of the stomach
normal motility of the GI tract
taking antacids
Correct answer:
taking antacids
Loss of normal gastric acidity, such as in those taking antacids or H2-receptor antagonists, is a factor that increases
susceptibility to gastroenteritis. Gastric surgery, increased age, and poor physical condition are also factors contributing to
susceptibility.
Question 48
A nurse practitioner (NP) has been infected with the HIV virus during her emergency department working days in the early
1980s. She is now employee in a private clinic and performs wellness exams. Which of the following is an ethical statement?
The NP is under no obligation to inform anyone.
The NP is under obligation to inform the patient if she performs invasive procedures.
The NP is obligated to inform her patients of her HIV status.
The NP is obligated to inform her employer of her HIV status.
The Correct answer is:
The NP is under no obligation to inform anyone
The nurse practitioner's health information is protected, therefore, she is not obligated to inform her patients, employer, or the
state board of nursing as long as she is able to perform her job without imposing a risk to anyone
Question 49
Which of the following statements about Managed Care Organizations (MCOs) is false?
MCO is an umbrella term that may include HMOs and other forms of health plans.
MCOs reimburse primary care providers on a fee-for-service basis, a capitated basis or a combination of both.
11
MCOs do not credential providers; that is done by the federal government.
MCOs sell a priced package of health services to their clients.
Correct answer:
MCOs do not credential providers; that is done by the federal government.
This is the false statement. MCOs do credential providers. They collect educational, license, malpractice, employment and
certification data on each provider and make a judgment that a provider is or is not adequately prepared to care for the MCO’s
patients.
Question 50
Which statement is TRUE regarding the professional role in reading evidence-based medicine articles?
Nurse practitioners should have sufficient research skills to be able to critically evaluate and participate in outcome studies
that will relate to their clinical practice.
The trend is to base decisions on intuition and tradition, not research.
Clinical trials have yet to gain acceptance as valid foundations on which care can be provided.
The use of evidence-based research in clinical practice is not common.
The Correct answer is:
Nurse practitioners should have sufficient research skills to be able to critically evaluate and participate in outcome studies
that will relate to their clinical practice
To reduce the number of conflicting or varying recommendations for the diagnosis and treatment of common problems, the
trend is going to evidence-based research from randomized controlled research trials and meta-analyses of these trials.
Outcome studies are replacing tradition, intuition and preference for how problems should be treated. The use of research in
clinical practice is now commonplace.
Question 51
A nurse practitioner wants to make a change in the state's law concerning the dispensing of prescription medications by nurse
practitioners. Who would this case need to be taken to?
state board of nursing
state board of pharmacy
nursing specialty organization
state legislature
The Correct answer is:
State legislature
Healthcare policy within the states is codified or enacted into law by the respective state legislature. The boards should have
significant input in the process, such as providing the research data and expert "testimony" that the legislature needs to make
informed decisions. However, boards, such as the state nursing boards, do not have the governing power to pass state laws.
Question 52
You are conducting a Mini-Mental exam (MME) on a patient. You ask the patient to spell “world” backward. Which of the
following MME activities is being used?
recall
orientation
attention and calculation
copying
Correct answer:
attention and calculation
Attention and calculation activities of the MME include asking the patient to spell a work backwards. It also includes serial 7s
(asking a patient to subtract 7 from 1000 and on down).
Question 53
An adult female comes to your clinic complaining of a rash on her hands. You ask her if there have been any changes in
detergent, lotion, etc. that come in contact with her hands. She tells you that she changed her brand of dish detergent recently
and that she washes all her dishes by hand. Which of the following would you diagnose for this patient?
contact dermatitis
12
atopic dermatitis
scabies
psoriasis
Correct answer:
contact dermatitis
Contact dermatitis is a cutaneous reaction to an external substance such as an irritant or allergen. It appears as an asymmetric
distribution of red, raised, and/or inflamed rash, or rash only on exposed areas. 80% of cases are due to universal irritants such
as soap and detergent.
Question 54
In terms of practice environment, what would a CAM treatment be?
a treatment used in mass casualty situations
a treatment used when patients ask for a specific treatment
a complementary and alternative medicine treatment
none of the above
Correct answer:
a complementary and alternative medicine treatment
These days there is a greater recognition of the use by patients of complementary and alternative modalities and medicines.
FNPs as providers need to learn about common CAM treatments.
Question 55
Which of the following is least likely to exacerbate edema in your patient?
laxative abuse
corticosteroids
calcium agonists
diuretics
Correct answer:
diuretics
Diuretics are indicated for marked peripheral edema, pulmonary edema, CHG, and inadequate dietary salt restriction. Abuse of
diuretics, however, can lead to edema.
Question 56
Attachment, in terms of infant psychosocial development, is the enduring and specific affective bond that develops over the
first year of life. In an insecure-disorganized attachment the underlying process/emotion is which of the following?
anger
anxiety
confusion/dysfunction
ambivalence
Correct answer:
confusion/dysfunction
In an insecure-disorganized attachment the underlying process is confusion and dysfunction. In a secure attachment, the
underlying emotion is love. In an insecure-avoidant attachment, the underlying emotion is anger. In an insecure-anxious
attachment the underlying emotion is anxiety/ambivalence.
Question 57
A 35-year-old Caucasian female is in the office with rosacea. She is asking about treatment options. What is the recommended
treatment for her?
topical 5-fluorouracil
low-dose tetracycline
oral ketoconazole
Dilantin
13
The Correct answer is:
Low-dose tetracycline
Treatment with systemic low-dose tetracycline is a very effective measure for rosacea. Further, topical treatment with
metronidazole or a low-dose steroid cream may also be helpful. In some cases, ketoconazole, the antifungal cream and not an
oral form, is used to treat rosacea. Topical 5-fluorouracil is used to treat actinic keratosis.
Question 58
Which of the following specifically addresses the need to promote consistent access to quality advanced practice nursing care
within states and across state lines?
The Joint Commission
The Advanced Practice Registered Nurse Compact
ANA
HHS
Correct answer:
The Advanced Practice Registered Nurse Compact
Beginning in 1999, the National Council of State boards of Nursing (NCSBN) began implementation of an interstate compact
for nursing practice to reduce state-to-state discrepancies in nursing requirements to practice. The Advanced Practice
Registered Nurse Compact addresses the need to promote consistent access to quality advanced practice nursing care within
states and across state lines.
Question 59
A diabetic patient with proteinuria has been placed on an ACE inhibitor. How soon can the anti-proteinuric effect of the ACE
inhibitor be realized in this patient?
6 to 8 weeks
3 months
6 months
3 to 5 years
The Correct answer is:
6 to 8 weeks
The effect can be realized as early as 6 to 8 weeks after starting an ACE inhibitor or ARB. A second agent can be added if the
ACE or ARB reaches maximum dosage and the goal of proteinuria, which is less than 500 mg/dL, has not been achieved.
Question 60
Which of the following would NOT be a common reason for developing a scope of practice in state law for an NP?
to allow NPs to perform at their level of education and training
to assure safe and appropriate care with an outcome as good as expected
to avoid any charges of practicing medicine without a license
to place accountability for benefits or harm to a patient squarely on the NP
Correct answer:
to assure safe and appropriate care with an outcome as good as expected
This is a reason for developing a standard of care rather than scope of practice. Other reasons for developing a scope of
practice in state law for an NP include: to get reimbursement for physician services, when provided by an NP; to avoid
imputation of liability for medical malpractice to someone other than the NP, usually a physician, and to establish that the NP
is a professional entity, not just a “nonphysician.”
Question 61
A patient in the clinic is found to have 2 palpable, tender, left pre-auricular nodes that are about 0.5 cm in diameter. You
expect to find what in this patient?
ulceration on the tongue
ear infection
sore throat
conjunctivitis
The correct answer is conjunctiviitis. The eyes are drained by the pre-auricular lymph nodes and they can be palpated near the
ear. These swell in response to eye infections, allergies, or foreign bodies in the eye.
14
Question 62
A patient you diagnosed with hypothyroidism was started on levothyroxine. At what interval should the nurse practitioner
reassess her TSH?
1 to 2 weeks
2 to 4 weeks
4 to 6 weeks
6 to 8 weeks
The Correct answer is:
6 to 8 weeks
In the treatment of hypothyroidism, T4 replacement is needed in the form of levothyroxine (Synthroid or Levoxyl). The initial
dosage for an adult is 75 to 125 mcg. For an elderly person, the dose is 75% less than the adult dosage. Because of the long
half-life of levothyroxine, the effects of a dosage adjustment or initiation would not cause a change in TSH for approximately
five to six drug half-lives, or about 6 to 8 weeks.
Question 63
There are issues that the nurse practitioner must address in negotiation with managed care organizations, health systems, or
professional practice groups. Which of the following is NOT one of these issues?
clinical privileges
participation in bonus payment plans
malpractice insurance
patients' bill of rights
The Correct answer is:
Patients' bill of rights
The patients' bill of rights has nothing to do with negotiation with managed care organizations, health systems or professional
groups. The remaining answer choices are all included as important issues as well as percentage of payment to collaborating
physician, contracts between the nurse practitioner, employer and payer, and risk sharing.
Question 64
The ability of a patient and family to understand and act on health information is known as which of the following?
family systems theory
complementary medicines
collaborative education
health literacy
Correct answer:
health literacy
Health literacy is recognized as one of the largest contributors to health outcome. It is the ability of a patient and family to
understand and act on health information.
Question 65
Which of the following statements about heart murmurs is least accurate?
Midsystolic murmurs usually originate in semilunar valves.
Pansystolic murmurs are usually associated with regurgitation.
Murmurs originating in the left side of the heart change more with respiration.
Middiastolic/presystolic murmurs are associated with atrioventricular valves.
Correct answer:
Murmurs originating in the left side of the heart change more with respiration.
This is not accurate. Murmurs originating in the right side of the heart change more with respiration. Examples of this type of
murmur are pulmonic or tricuspid murmurs.
Question 66
You are examining a 4-year-old child who attends daycare when his mother and father are working. He has a rash with a
“slapped cheek” appearance. Your most likely diagnosis for this child will be which of the following?
15
roseola
chicken pox
Fifth disease
scabies
Correct answer:
Fifth disease
Fifth disease is a mild, self-limiting viral disease characterized by erythematous, macular rash first appearing on the cheeks
and ears, leading to a “slapped cheek” appearance. One of the factors contributing to this disease is institutional style day care.
Question 67
Although nurse practitioners and physician’s assistants may function similarly, there are differences between NPs and PAs.
Which of the following statements about NPs in relation to PAs is incorrect?
A PA has a job description, while an NP has a scope of practice.
PAs practice medicine under the license of a physician, never independently.
NPs are always primary care providers.
NPs practice under their own licenses.
Correct answer:
NPs are always primary care providers.
Some state’s laws specifically authorize nurse practitioners to be primary care providers (PCPs). Other states do not prohibit a
nurse practitioner from being designated as a PCP. State law governs whether the NP can be a primary care provider.
Physician’s assistants are never primary care providers.
Question 68
A pulse oximetry of less than what percent indicates the need for supplemental oxygen therapy?
98%
95%
94%
90%
Correct answer:
90%
Pulse oximetry determines oxygen saturation. Less than 90% indicates the need for supplemental oxygen therapy.
Question 69
What do you expect to find in the cerebral spinal fluid (CSF) when evaluating a patient who has aseptic or viral meningitis?
low opening pressure
glucose at about 30% serum levels
low protein level
predominance of lymphocytes
The Correct answer is:
Predominance of lymphocytes
To eliminate or support a diagnosis of meningitis, a lumbar puncture to evaluate cerebral spinal fluid is often done.
Pleocytosis, defined as a white blood cell (WBC) count of more than 5 cells/mm of CSF, is an expected finding in meningitis
caused by bacterial, viral, tubercular, fungal, or protozoan infection. An elevated CSF opening pressure is also a nearly
universal finding, not a low one. Normal glucose levels and normal or high protein levels are also noted findings with
meningitis.
Question 70
Which test is the most important diagnostic lab value to diagnose iron deficiency anemia?
serum folate level
serum ferritin level
red blood cell (RBC) count
direct Coombs
16
The Correct answer is:
Serum ferritin level
Serum ferritin levels correlate with the total body iron stores because it is the major iron storage protein. Its value is reduced in
iron deficiency anemia. Serum folate measures the folic acid level. Direct Coombs measures in vivo RBC coating by
immunoglobulins.
Question 71
You have a patient who has hypertension which has been controlled by medication. You believe that this patient has the right
and responsibility for selecting self-care behaviors that will be consistent with keeping her blood pressure under control. You
believe in which of the following theories?
compliance perspective
fulfillment perspective
transtheoretical perspective
empowerment perspective
Correct answer:
empowerment perspective
The empowerment perspective assumes that the informed client is the “expert” in his or her own disease and, as such, has the
right and responsibility for selecting self-care behaviors consistent with good or better health within the context of daily
lifestyle. Efforts to understand the client’s perspective, acknowledge feelings, and educate for optimal decision making are
cornerstones of this model.
Question 72
You have a patient who is 26 weeks pregnant. She presents with what you diagnose as Fifth Disease. You understand that all
of the following are special considerations for this patient EXCEPT:
Infections during pregnancy may cause fetal hydrops and death.
The risks associated with Fifth Disease are lower in this patient since she was exposed after 20 weeks of pregnancy.
The risk of fetal hydrops and death is relatively high in pregnant women.
B19 infected fetuses may be treated with intrauterine blood transfusions.
Correct answer:
The risk of fetal hydrops and death is relatively high in pregnant women.
The risk of fetal hydrops and death is relatively low. The highest risk is if the mother is exposed during the first 20 weeks of
pregnancy
Question 73
Your patient brings her 6-year-old female child to you. The child is complaining of abdominal pain and swelling of her hands
and feet. She tells you that the child had a “strep throat” a couple of weeks ago. She has been running a fever and acts very
tired. Which of the following diseases/disorders is most consistent with these symptoms?
systemic lupus
acute glomerulonephritis
congestive heart failure
bacterial endocarditis
Correct answer:
acute glomerulonephritis
Acute glomerulonephritis is the result of an immune response, usually a streptococcal infection. It is common in children with
the peak ages being 2 – 6 years
Question 74
The FNP has an African-American female patient who is 75 years old. She complains of a gradual loss of her peripheral
vision. Examination indicates increased IOP. Which of the following is the most likely cause of her symptoms?
subconjunctival hemorrhage
primary closed-angle glaucoma
primary open-angle glaucoma
macular degeneration
17
Correct answer:
primary open-angle glaucoma
Primary open-angle glaucoma has a gradual onset of increased IOP due to blockage of drainage of the aqueous humor. The
optic nerve undergoes ischemic damage resulting in permanent visual loss. It is the most common type of glaucoma and is
most commonly seen in elderly patients, especially those of African background or diabetics.
Question 75
A patient receives another person's medication in error. This was done by an LPN in the office and did not result in a lifethreatening situation for the patient. What is an appropriate way for the nurse practitioner to document this?
Patient received wrong medication. Incident report was filed. LPN was disciplined.
X mg of Y drug was administered to the patient. No adverse effects were noted. Physician notified.
LPN inadvertently administered Y drug to the wrong patient. Supervisor notified. Family threatening litigation.
Patient was given X mg of Y drug in error.
The Correct answer is:
X mg of Y drug was administered to the patient. No adverse effects were noted. Physician notified
Documenting, "X mg of Y drug was administered to the patient. No adverse effects were noted. Physician notified," is the
most accurate note because it is factual. Also, with this noted documentation, the writer is avoiding blame or assuming
liability. The other notes, identified in the remaining answer choices, would be "red flags" mentioning error, litigation, and
incident report.
Question 76
You are testing a 5-year-old child for visual acuity. You use the Snellen chart. The child should be standing at what distance
from the chart?
5 feet
10 feet
15 feet
20 feet
Correct answer:
20 feet
You should obtain visual acuity (V/A) and binocular vision by age 4 to 5 years. The child using the Snellen chart should be at
20 feet when reading it. A failed test is V/A 20/40 or greater in either eye, or if there is a two-line discrepancy between the
eyes.
Question 77
The FNP has recommended isometric exercises for a patient who is recovering from a joint injury. All of the following
statements about isometric exercises are true EXCEPT:
They are controlled and sustained contraction and relaxation of a muscle group.
They are helpful for those with osteoporosis.
They are less stressful on joints than regular exercise.
They are usually done first before active exercise post injury.
Correct answer:
They are helpful for those with osteoporosis.
Isometric exercises are non-weight-bearing exercises. Only weight-bearing exercises work for osteoporosis (for instance,
walking or biking).
Question 78
In terms of physiologic changes during pregnancy, which of the following is the least accurate statement?
There is increased peristalsis from progesterone effects.
Plasma volume increases 50% by the end of the third trimester.
Decreased systolic and diastolic pressure occurs in the first trimester.
Cardiac output increases by 1/3 by the last two trimesters.
Correct answer:
There is increased peristalsis from progesterone effects.
18
This is not accurate. There is decreased peristalsis from progesterone effects. This results in things like heartburn and
constipation
Question 79
A 16-year-old mildly obese adolescent presents requesting a letter stating she should not participate in gym class because of
her asthma. The most appropriate response is to:
Excuse her from outdoor activities only as to avoid pollen exposure.
Write the note to excuse her from indoor activities only to avoid dust mite exposure.
Write the note because gym class participation could trigger asthma symptoms.
Remind her that with appropriate asthma care, she can participate in gym class and that exercise is part of a healthy lifestyle.
The Correct answer is:
Remind her that with appropriate asthma care, she can participate in gym class and that exercise is part of a healthy lifestyle
Asthma is a common chronic disorder that is complex but treatable. Exercise is a necessary part of a healthy lifestyle,
especially for a developing teen that is overweight. With the use of a short-acting beta2 agonist inhaler, this condition can be
controlled and managed during activity.
Question 80
In terms of newborn vision, which of the following statements is correct?
Newborns are farsighted.
Newborns can focus best at a distance of about 24 inches.
Preferred colors are yellow and blue.
Newborns have excessive tears when crying.
Correct answer:
Newborns can focus best at a distance of about 24 inches.
This is the correct statement. Newborns are nearsighted. Their preferred colors are black, white and red; and they have no tears
when crying since lacrimal ducts mature by age 2 – 3 months.
Question 81
A young male is in the clinic and he mentions suicidal thoughts. All of the following are part of the suicidal risk assessment
EXCEPT:
Approach with a declaration such as: "It sounds as if you are having a really hard time."
Always ask "Do you have a plan?"
There is no need for intervention if the patient is not tearful and distraught.
Pay attention to affect and mood.
The Correct answer is:
There is no need for intervention if the patient is not tearful and distraught
The patient does not have to be distraught and tearful to be suicidal. Often, they present with a soft low voice and have
downcast eyes with poor eye contact
Question 82
For the patient with mild ulcerative colitis, the nurse practitioner knows the primary therapy is:
amoxicillin
metronidazole
sulfasalazine
cephalexin
The Correct answer is:
Sulfasalazine
Sulfasalazine is the treatment recommended for patients with mild ulcerative colitis. Additionally, it results in symptomatic
improvement in 50-75% of patients diagnosed with mild ulcerative colitis. If no response is seen after 2-4 weeks, the addition
of prednisone is recommended. The remaining answer choices are not first line initial therapy.
Question 83
19
Which of the following is NOT a violation of the Stark Acts?
referring a patient covered by Medicare to a clinical laboratory where the physician or an immediate family member of the
physician has a financial relationship
referring a patient to certain designated health services when the physician has a financial relationship with the facility
offering the services
referring a patient to another physician in the same group practice
none of the above
Correct answer:
referring a patient to another physician in the same group practice
Under the Stark Acts, a physician cannot refer a patient covered by Medicare to a clinical laboratory where the physician or an
immediate family member of the physician has a financial relationship. Nor may a physician refer a patient to certain
designated health services when the physician has a financial relationship with the facility offering the services. He can,
however, refer a patient to another physician in the same group practice.
Question 84
Which of the following changes are you least likely to see in a menopausal woman?
larger nipples that protrude more
atrophy of the vulva
sleep disorder
increased incidence of cystocele and rectocele
Correct answer:
larger nipples that protrude more
You are not likely to see that a menopausal woman’s nipples enlarge and protrude more. Changes related to the breast include
nipples that become flatter and smaller. Also there is size and shape change in the breasts and atrophy of glandular tissue.
Question 85
A 23-year-old thin Caucasian female presents in the clinic with a new diagnosis of Crohn's disease. She has recently started
medication prescribed by her gastroenterologist and asks you to tell her what else to do. You know that proper
nonpharmacological management of this condition includes all of the following EXCEPT:
Avoid dairy products that contain lactose if they exacerbate symptoms.
A low protein is recommended.
Stress management is important, including relaxation techniques.
A low-roughage diet is recommended during acute exacerbations.
Correct answer:
A low protein diet is recommended.
A diet high in protein and vitamins, particularly B12, is recommended for the patient with Crohn's disease.
Question 86
The FNP has a patient with a gradual onset of numbness starting in the hands and feet. The CBC on this patient reveals a
macrocytic anemia and a peripheral smear shows multisegmented neutrophils. This patient most likely has which of the
following conditions/diseases?
neutropenia
vitamin B12 deficiency
thrombocytopenia
non-Hodgkin’s lymphoma
Correct answer:
vitamin B12 deficiency
Vitamin B12 deficiency is indicated since the patient has a gradual onset of paresthesias (numbness, tingling or pricking)
starting in the hands and feet with a CBC that reveals a macrocytic anemia and a peripheral smear that shows multisegmented
20
neutrophils. Pernicious anemia is the most common cause of vitamin B12 deficiency.
Question 87
The state boards of nursing do all but which of the following?
do audits of NP practice
carry out the statutes of the state
write and administer rules and regulations for nursing practice, based on statute
ensure that NP qualifications are up to date
Correct answer:
do audits of NP practice
State boards of nursing do not do audits of NP practices. They also do not test NPs.
Question 88
Which of the following would NOT be a common cause of epilepsy in older adults?
cerebrovascular disease
premature birth
neoplasms
head trauma
Correct answer:
premature birth
Premature birth is a factor that would increase susceptibility to seizures, but is not a common cause in the elderly population.
In addition to the other three choices, metabolic derangements are also a common cause.
Question 89
Which of the following measures would be directed particularly against the reservoirs of infection?
insect spraying
water purification
immunization
improved nutrition
Correct answer:
insect spraying
Measures directed against the reservoirs of infection include: insect spraying, isolation and quarantine. Water purification is a
measure to interrupt the transmission of organisms. Immunization and improved nutrition are measures that reduce host
susceptibility.
Question 90
When a patient is receiving antithyroid medication, the nurse practitioner has knowledge that:
The drugs are somewhat expensive and have serious cardiac and hematologic side effects.
Patients remain on drug therapy for 1-2 years and then the medication is gradually withdrawn.
Lifelong daily treatment is necessary to keep TSH levels within the normal range.
Antithyroid medications do not cross the placebo membrane structure.
The Correct answer is:
Patients remain on drug therapy for 1-2 years and then the medication is gradually withdrawn
Antithyroid medications are relatively inexpensive. The client remains on the medications for 1-2 years with the hope of a
permanent remission of symptoms when they are withdrawn.
Question 91
A patient is having a seizure with rigid extension of her arms and legs and sudden jerking movements with loss of
consciousness. She is having which of the following types of seizure?
myoclonic
tonic-clonic
simple partial
complex partial
21
Correct answer:
tonic-clonic
A tonic-clonic (grand mal) seizure is characterized by rigid extension of her arms and legs and sudden jerking movements with
loss of consciousness. The onset of this type of seizure can happen at any age. In adults, a new onset may be found in patients
with a brain tumor, post head injury or alcohol withdrawal
Question 92
If a clinician is doing a research study that measures the impact of a fall prevention program on the number of falls among
older adults it would be which of the following types of research?
experimental research
descriptive research
historical research
correlational research
Correct answer:
experimental research
Experimental research is research aimed at determining the effect of one or more variables. The fall prevention research falls
into this category. For a true experimental design to exist, the sample must be randomly selected; there must be manipulation
of the independent variable by the researcher as is usually seen in intervention studies; and there must be a control group.
Question 93
Researchers try to substantiate causality so that the patient outcomes can be more consistently predicted. For the nurse
practitioner, the ability to predict the outcomes for every patient would mean:
no further studies would need to be conducted about treatments
the patient's individual qualities would not need to be considered
treatments would not need to be individualized for each patient
the appropriate treatment would be prescribed for patients
The Correct answer is:
The appropriate treatment would be prescribed for patients
The ability to always predict patient outcomes would mean that everyone would receive the exact same treatment. Treatments
would still have to be individualized, and individual qualities would need to be considered
Question 94
You are treating a patient with acute sinusitis. You understand that the “gold standard” first line medication for this condition
is which of the following?
cephalosporins
trimethoprim sulfamethoxazole
Ceftin
amoxicillin
Correct answer:
amoxicillin
Acute sinusitis is an infection of the sinuses by Gram-positive and Gram-negative bacteria. For treating acute sinusitis,
amoxicillin is the “gold standard” for any age group.
Question 95
Which of the following statements about dysphagia is least accurate?
It can be attributed to normal aging.
It may be a structural or neuromuscular problem.
It may present as difficulty swallowing solids or liquids.
It may present as chest pain.
Correct answer:
It can be attributed to normal aging.
22
This is the least accurate statement. Dysphagia is not attributed to normal aging. It is a difficulty in swallowing and having
food pass from the mouth down the esophagus to the stomach.
Question 96
A 11-year-old girl is brought into the office with clinical symptoms and a diagnostic history for scabies. The nurse practitioner
knows that she was probably infected:
1-3 days ago
1 week ago
2 weeks ago
3-4 weeks ago
The Correct answer is:
3-4 weeks ago
The incubation period for scabies is about 3-4 weeks after primary infection. Further, the patient with scabies will develop
symptoms in 1-3 days. The classic complaint from patients with scabies is nocturnal pruritus.
Question 97
Persons who are allergic to latex products are often allergic to all of the following EXCEPT:
apples
kiwi
bananas
avocado
The Correct answer is:
Apples
It has been noted that those persons with latex allergy also often have a cross-allergy to kiwi, bananas and avocado. Apples
pose no cross-allergy threat, making this the right answer in this case.
Question 98
A 65-year-old female has come to the clinic complaining of a scaly red rash on one of her nipples that will not go away. She
tells the FNP that it has become crusty and sometimes bleeds. Examination shows a discharge from the nipple and a small
lump. Which of the following diseases/conditions is likely to be diagnosed?
breast cancer
fibrocystic breast
Lymphogranuloma Venereum
Paget’s disease of the breast
Correct answer:
Paget’s disease of the breast
Paget’s disease of the breast is often reported by older females. The disease presents as a scaly rash resembling eczema on the
nipple or the nipple and areola that does not heal. It can eventually have crusting, ulceration and/or bleeding and is sometimes
itchy.
Question 99
Which of the following is least likely to precipitate an attack of pancreatitis?
a high protein diet
high fat diet
heavy meal
alcohol ingestion
Correct answer:
a high protein diet
A high fat diet, heavy meal, or alcohol ingestion may precipitate an attack of pancreatitis. A high protein diet will not. Alcohol
abstinence should be stressed to any patient with pancreatitis.
Question 100
23
A patient is diagnosed today with pregnancy and her last pregnancy was 3 years ago. At that time she had a protective rubella
titer. What should be done about evaluating a rubella titer today?
It should be evaluated to make sure it is still protective.
She should be vaccinated now to insure protection.
She does not need one because it was protective 3 years ago.
There is no need to get one since she has been pregnant before.
The Correct answer is:
She does not need one because it was protective 3 years ago
Standard practice across the U.S. is to insure that a protective rubella titer exists in women who are pregnant. Re-evaluation is
not necessary if the pregnant patient had a protective rubella titer.
24
Test 1
Question 1
You are treating a patient with asthma in the clinic. They tell you they received a new medication in an emergency
room when they were out of town visiting family. Which of the following should be avoided in a patient with asthma?
naproxen
topical hydrocortisone
amlodipine
timolol ophthalmic drops
The Correct answer is:
Timolol ophthalmic drops
Timolol ophthalmic solution drop is a beta blocker which is known to precipitate asthma exacerbation in patients. Even though
it is administered in the eye, there is considerable absorption through the mucous membranes, resulting in systemic effects.
The remaining answer choices have no specific contraindications for patients with asthma.
Question 2
In terms of informed consent, the patient is informed of all of the following EXCEPT:
the insurance coverage for the procedure or treatment
the purpose of the procedure or treatment
the success rate or failure for the suggested treatment or procedure
the prognosis and success rate
Correct answer:
the insurance coverage for the procedure or treatment
Insurance coverage, although important in any case, is not a part of informed consent. In addition to the other choices,
informed consent addresses the risk vs. benefits of the procedure and alternatives to the procedure or treatment.
Question 3
Nurse practitioners are at risk for which of the following professional mishaps?
business
clinical
both business and clinical
none of the above
Correct answer:
both business and clinical
Nurse practitioners are at risk for two categories of professional mishap: clinical and business mishap. There can, however, be
an overlap between clinical and business problems.
Question 4
Which of the following drugs for treating anxiety has the most rapid onset of action?
diazepam
clonazepam
alprazolam
oxazepam
Correct answer:
diazepam
Diazepam (Valium) has a rapid onset of action and a relatively sustained effect. The other choices all have a slow onset of
action and a relatively or highly sustained effect.
25
Question 5
The nurse practitioner (NP) understands that the research process is similar to the processes that the NP use to provide patient
care in that both are decision-making processes. The steps of these processes in the correct order are:
Notification, subject selection, consent verification and study implementation
Study scope, literature review, study protocol and study design
Research construction, research compilation, research conclusion, research implementation
Assessing, planning, implementing and evaluating
The Correct answer is:
Assessing, planning, implementing and evaluating
When conducting research, the nurse uses the process steps of assessing, planning, implementing and evaluating. During the
assessing stage of the research, the nurse identifies the problem and performs a literature review. The nurse practitioner also
identifies any variables that may need to be later measured in the study. With planning, the nurse performs research to create a
hypothesis, determine measurements for the variables of the study and then decide how the sample for the study is selected.
The nurse practitioner then collects the data. During the evaluation or evaluating stage, the nurse practitioner analyzes or
evaluates the data, makes a conclusion about the findings and determines how to communicate the findings.
Question 6
Which of the following is NOT one of the ethical concepts for nurses?
beneficence
human dignity
confidentiality
risk
Correct answer:
risk
Risk is not one of the six ethical concepts for nurses. The six ethical concepts are: beneficence, non-malfeasance,
confidentiality, accountability, human dignity, and compassion.
Question 7
A 42-year-old non-smoker is diagnosed with mild community-acquired pneumonia. He is otherwise healthy and does not need
hospitalization at this time. Which antibiotic can be used for empirical treatment according to the 2007 Infectious Disease
Society of America?
azithromycin
baclofen
oxybutynin
Zoloft
The Correct answer is:
Azithromycin
These guidelines promote doxycycline or macrolide use, such as azithromycin, clarithromycin and erythromycin, for the initial
treatment of uncomplicated pneumonia in patients who are otherwise healthy and have not had recent antibiotic therapy. The
remaining answer choices, oxybutynin and baclofen, are not antibiotics but are drugs used for treating symptoms associated
with other conditions such as multiple sclerosis. For example, in patients with multiple sclerosis, baclofen is often prescribed
for muscle spasms. Then, oxybutynin helps control the urinary incontinence. Further, Zoloft is an antidepressant medication.
Question 8
Which of the following parts of the eye are used for color perception?
cones
rods
fundus
macula
26
Correct answer:
cones
The cones are for color perception. The fundus is the interior surface of the eye opposite the lens. The rods are for detecting
light and for depth perception. The macula is for central vision
Question 9
The most cost-effective method of diagnosing H. pylori infection is which of the following?
stool antigen testing
UGI endoscopy
serological testing
biopsy
Correct answer:
stool antigen testing
Stool antigen testing is the most cost-effective way to diagnose H. pylori infection. It is particularly effective when it is
coupled with a clinical presentation consistent with peptic ulcer disease
Question 10
When counseling a patient with benign prostatic hypertrophy, the nurse practitioner knows to advise:
The patient to limit intake of fluids in the evening, and avoid large quantities in a short time frame.
The patient that there are no surgical options available.
The patient to void at least twice a day.
The patient to avoid certain bladder irritants such as cranberry juice, clear sodas, and blood pressure pills.
The Correct answer is:
The patient to limit intake of fluids in the evening, and avoid large quantities in a short time frame
It is important to tell the patient that if severe symptoms persist, surgical options such as transurethral resection of the prostate
(TURP) are available. Also, the nurse practitioner should encourage the patient to void frequently. Further, the nurse
practitioner should advise the patient to avoid certain bladder irritants like antihistamines, anticholinergics, coffee, alcohol,
decongestants, and tricyclic antidepressants.
Question 11
A 68-year-old female patient has been under your care for years. She is a heavy smoker and has frequent episodes of
pneumonia. The nurse practitioner realizes that age-associated changes that increase the risk of pneumonia in the elderly
include an increase in:
diameter of the trachea and bronchi
lung parenchyma
cough forcefulness
compliance of the chest wall
Correct answer:
diameter of the trachea and bronchi
This is an age-associated physiologic change that occurs in the elderly. Other changes include less elastic lung parenchyma,
less forceful cough, and decreased compliance of the chest wall.
Question 12
You are seeing a 2-year-old male child in the clinic with community acquired pneumonia (CAP). When managing this patient,
you know that what percentage of children have an episode of pneumonia by age 5?
less than 10%
10%
27
20%
30%
The correct answer is:
20%
Pneumonia is considered to be a disease that primarily affects older adults and the chronically ill. However, about 20% of
children develop pneumonia by age five
Question 13
According to guidelines, which of the following might be considered least important in determining the level of visit to bill for
evaluation and management of a patient (not counseling)?
examination
time
history taking
medical decision making
Correct answer:
time
Time is a minor consideration in determining the level of visit to bill, according to guidelines, if a clinician is billing an office
visit for evaluation and management. If a visit is primarily counseling, however, time matters and should be documented.
Question 14
When communicating with Hispanic Americans, the nurse practitioner understands that:
Avoiding eye contact with a person of authority indicates disrespect.
Dramatic body language is avoided.
Direct confrontation is expected.
Members tend to be verbally expressive yet value confidentiality.
The Correct answer is:
Members tend to be verbally expressive yet value confidentiality
Avoiding eye contact with a person in authority indicates respect and attentiveness with the Hispanic American patient
population. Dramatic body language, such as gestures and facial expression, are often used to express emotion or pain. Direct
confrontation is disrespectful and not expected with the members.
Question 15
When treating an adult male with alcoholic cirrhosis diagnosed by liver biopsy, the nurse practitioner has to counsel on the
progression of the disease and its management. It is important that the nurse practitioner inform him that the disease is
irreversible at this point and that the following will generally halt the progression of the disease:
long-term, low dose corticosteroids
abstinence from alcohol
vitamin B supplements
maintenance of a nutritious diet
The Correct answer is:
Abstinence from alcohol
Refraining from alcohol is the most important measure for the patient with alcoholic cirrhosis to undertake. The client's diet
should be full of adequate protein and calories and salt should be restricted if fluid retention occurs. Vitamin supplementation
is necessary. Long-term corticosteroids should be avoided.
Question 16
A 22-year-old female suspects she is pregnant. In order to establish pregnancy, a test of the urine or blood is routinely
performed. The nurse practitioner understands that this can be done with predictable results at what time?
within 3 days after conception
28
within 7 days after conception
1-2 weeks after conception
35 days after last menses
Correct answer:
1-2 weeks after conception
Whether performed on urine or blood, the presence of hCG is evaluated to identify pregnancy and can be found in high
quantities in the first morning urine or at any time in a serum sample. Both tests are highly sensitive, but if the pregnancy is
early, the serum test is more specific.
Question 17
Which of the following are elements of a broad-based risk management program?
System of contract review that helps to avoid assuming liabilities that should be borne by others.
Hazardous materials compliance program as part of a comprehensive safety and security system.
Early-warning/incident reporting program to identify elements of risk.
All of the above.
The Correct answer is:
All of the above
These and other elements combine to produce a program of systematic risk identification, analysis, treatment, and evaluation,
with the overall goal of loss prevention.
Question 18
When evaluating a patient for the need for a bone density screening test, it is important for the nurse practitioner to understand
that:
All patients over the age of 40 need to be screened.
A risk factor for osteoporosis is being of African American descent.
This condition is common in individuals with estrogen deficiency.
The patient must be NPO for the exam.
The correct answer is:
This condition is common in individuals with estrogen deficiency
Osteoporosis is more common in people on chronic steroids, those with an androgen or estrogen deficiency, and individuals
with hypogonadism. It is more common in Caucasian and Asian women and not all patients over the age of 40 need to be
screened. The patient does not need to be NPO for the exam. However, some testing equipment analyzes the lumbar spine.
Further, no calcium pills three days prior to the test are recommended.
Question 19
In which of the following cases would the FNP refer a patient to a specialist for low back pain? A patient with:
acute neck pain
bladder dysfunction
reduced range of movement
stiffness
Correct answer:
bladder dysfunction
If the patient has a bladder dysfunction, or a limb or bowel dysfunction, prompt referral to a specialist is needed. Surgery is
usually considered only if severe symptoms persist beyond 3 months.
Question 20
29
Konrad Lorenz is best known for the theory that:
there are biologically-programmed periods (sensitive periods) predisposed for particular learning.
maternal separation and social isolation resulted in dramatic impairment of social-emotional development.
promoted rooming-in and father participation in the birth of a child.
neutral stimulus associated with a meaningful one over time leads to a “conditioned” response that can be elicited by neutral
stimulus alone as though it were the meaningful one.
Correct answer:
there are biologically-programmed periods (sensitive periods) predisposed for particular learning.
Konrad Lorenz is best known for the theory that there are biologically-programmed periods (sensitive periods) predisposed for
particular learning. It is one of the many ethologic theories developed in the 20th century.
Question 21
A 52-year-old male patient presents in the office and you notice a yellowish plaque on his upper eyelid that is painless. What
should the nurse practitioner look for?
liver function studies
lipid levels
sedimentation rate
vision in the affected eye
The Correct answer is:
Lipid levels
The yellowish painless plaque on the patient's upper eyelid is descriptive of xanthelasma. Additionally with xanthelasma, the
plaque is slightly raised and well-circumscribed on the upper eyelid. Further, one or both eyelids may be affected.
Xanthelasma is often associated with lipid disorders but may occur independent of any systemic or local disease. However,
xanthelasma does not affect vision
Question 22
A 45-year-old woman complains to the FNP that her 80-year-old mother is too demanding and that it has created a great stress
in her life. According to Havighurst’s developmental tasks, this complaint reflects which of the following?
a need for independence
a need to adjust to an aging parent
establishing an explicit affiliation with one’s own age group
a need to achieve emotional independence
Correct answer:
a need to adjust to an aging parent
One of Havighurst’s developmental tasks for 30 – 60 year old persons is adjusting to an aging parent. Other tasks in this age
group include: achieving adult social and civic responsibility, developing adult leisure time activities and relating oneself to
one’s spouse as a person
Question 23
The FNP has a patient who desires to quit smoking. She asks the FNP if there is anything that can help her in trying to quit.
The FNP understands that all of the following are true in regard to smoking cessation EXCEPT:
Bupropion, an atypical antidepressant, is used for smoking cessation.
Bupropion can be used with nicotine products such as gum or nasal sprays.
Bupropion is safe to use for persons with hypertension or seizures.
Nicotine patches should not be used with other nicotine products.
Correct answer:
Bupropion is safe to use for persons with hypertension or seizures.
This is incorrect. Bupropion (Zyban) is contraindicated in persons with hypertension or a history of seizures
Question 22
30
A 45-year-old woman complains to the FNP that her 80-year-old mother is too demanding and that it has created a
great stress in her life. According to Havighurst’s developmental tasks, this complaint reflects which of the following?
a need for independence
a need to adjust to an aging parent
establishing an explicit affiliation with one’s own age group
a need to achieve emotional independence
One of Havighurst’s developmental tasks for 30 – 60 year old persons is adjusting to an aging parent. Other tasks in this age
group include: achieving adult social and civic responsibility, developing adult leisure time activities and relating oneself to
one’s spouse as a person.
Question 23
The FNP has a patient who desires to quit smoking. She asks the FNP if there is anything that can help her in trying to
quit. The FNP understands that all of the following are true in regard to smoking cessation EXCEPT:
Bupropion, an atypical antidepressant, is used for smoking cessation.
Bupropion can be used with nicotine products such as gum or nasal sprays.
Bupropion is safe to use for persons with hypertension or seizures.
Nicotine patches should not be used with other nicotine products.
This is incorrect. Bupropion (Zyban) is contraindicated in persons with hypertension or a history of seizures
Question 24
The FNP has a patient with a H. Pylori-Negative ulcer. Which of the following therapies would not be prescribed for
this patient?
H2 blockers
antibiotics
proton pump inhibitor if necessary
lifestyle changes
Antibiotics would not be prescribed because there is no infection in H. Pylori-Negative ulcers. After encouraging lifestyle
changes, the FNP would prescribe an H2 blocker as first line therapy. If this does not give relief or gives a poor result, the FNP
may switch to a proton pump inhibitor.
Question 25
The theory that maternal separation and social isolation results in dramatic impairment of social-emotional development was
developed by which of the following persons?
Konrad Lorenz
John Bowlby
Harry Harlow
Marshall Klaus and John Kennel
Correct answer:
Harry Harlow
Harry Harlow was part of the Wisconsin primate laboratory’s classic rhesus monkey experiments on maternal deprivation. He
found that maternal separation and social isolation results in dramatic impairment of social-emotional development. Physical
contact and comfort are necessary for normal social and emotional
Question 26
In terms of a migraine headache without aura which of the following is NOT true?
It is usually bilateral, although occasionally unilateral.
The female to male ratio with this type of headache is 3:1.
During the headache nausea or vomiting may occur.
Photophobia occurs with this type of headache.
Correct answer:
It is usually bilateral, although occasionally unilateral.
This is not true. The opposite is true. Migraine headaches without aura are usually unilateral, although occasionally they are
31
bilateral.
Question 27
A mother is asking questions regarding giving her 4-month-old infant fruit juice. The nurse practitioner understands that the
following statements are accurate concerning fruit juice EXCEPT:
Fruit juice is an excellent replacement for infant formula if an infant is allergic to cow's milk.
Juice should not be given before 6 months of age and intake should be limited to 1 to 2 ounces.
Pureed fruit should be given instead of juice if possible.
Fruit juice should not replace breast milk or infant formula in the diet because it is high in carbohydrate and low in the other
nutrients.
The Correct answer is:
Fruit juice is an excellent replacement for infant formula if an infant is allergic to cow's milk
Fruit juice is not an excellent alternative to infant formula. If an infant is allergic to cow’s milk, the nurse practitioner can
suggest to the mother other acceptable alternatives, such as feeding the infant soy-milk formula. Further, fruit juice should
only be given to infants who are 6 months old and older. Therefore, the nurse practitioner should inform the mother that the 4month-old is not at an appropriate age to drink fruit juice.
Question 28
What type of insurance coverage does the clinic need that is purchased by an organization to address employee job-related
injuries?
business interruption insurance
directors and officers insurance
workers' compensation insurance
professional liability insurance
The Correct answer is:
Workers' compensation insurance
Professional liability insurance is acquired to protect the organization from lawsuits by clients arising from negligent acts of
employees. Business interruption coverage is usually purchased in tandem with fire insurance and it will reimburse an
organization for losses sustained from a catastrophic event. Workers' compensation is the line of coverage that protects
employees after on-the-job injuries.
Question 29
Part of health promotion for the adult patient is to recommend an exercise regimen. Which of the following is a good
recommendation for the adult patient?
The intensity or component of the program should be increased to build stamina if the patient is unable to talk while
exercising.
Contraindications for exercise include asthma, obesity, and hypertension.
The goal of exercise for the adult is to sustain target heart rate for 50 minutes for maximum cardiopulmonary conditioning.
The focus should be on fundamental fitness, not sport-specific skills.
The Correct answer is:
The focus should be on fundamental fitness, not sport-specific skills
The intensity should be decreased if the patient is unable to talk while exercising and increased as the patient develops
tolerance. A person can exercise with asthma, obesity, and hypertension. It is highly recommended. The goal of exercise is to
get the heart rate up at the target range for 30 minutes during a session.
Question 30
Several research studies based on the transactional model of development emerged in the 1980s. One of these models was the
Kauai longitudinal study from birth to adulthood to explore perinatal risk and environmental factors on subsequent
developmental outcomes and resiliency. Which of the following researchers conducted this study?
Alan Sroufe
Michael Rutter
Lawrence Kohlberg
Emmy Werner
32
Correct answer:
Emmy Werner
The transaction model was first described in 1975 as a “continuum of reproductive risk and caretaking casualties” by Arnold
Dameroff and Michael Chandler. Emmy Werner produced the Kauai longitudinal study from birth to adulthood to explore
perinatal risk and environmental factors on subsequent developmental outcomes and resiliency based on the transactional
model.
Question 31
Which of the following statements about structural functional theory of families is the least accurate?
Rank order within families is not a component of structure.
Families are social systems.
In optimally functioning families, members take on predictable roles.
Disease or ill health can interfere with the family’s ability to carry out its internal functions.
Correct answer:
Rank order within families is not a component of structure.
All of the statements are accurate except this one. Internal family subsystems function as a microcosm of society, reflecting
the larger sphere of human needs. Rank order within families is a component of structure, such as the ordering of children by
birth in the family.
Question 32
Which of the family theories of assessment and intervention outlines eight consecutive stages in the family life cycle?
general systems theory
structural functional theory
communication theory
developmental theory
Correct answer:
developmental theory
Developmental theory explains human growth and development according to theorists such as Erikson, Piaget and
Havinghurst. The model outlines the eight consecutive states in the family life cycle that offers a predictive overview of the
activities that occur in families over time and serves as a basis for anticipatory guidance when assessing and teaching families.
They are: beginning family, childbearing family, family with preschool children, families with schoolchildren, family with
teenagers, launching center family, family with middle-aged parents and family with old age and retirement.
Question 33
A mother comes into the office with her 2-year-old child who attends daycare. The child is pulling at her ears, running a fever,
and has bright red bulging tympanic membranes bilaterally. The nurse practitioner knows that with acute otitis media (AOM)
the risk factors include:
Chinese race, previous otitis media, many siblings
Summer season, full-time day care, premature at birth
higher socioeconomic level, full-time day care, allergies
second-hand smoke, attending day care, American Indians and Eskimos
The Correct answer is:
Second-hand smoke, attending day care, American Indians and Eskimos
The key factor that contributes to acute otitis media (AOM) is a dysfunctional eustachian tube that allow bacteria from the
nasopharynx into the middle ear. American Indians and Eskimos have more repetitive and severe otitis media than members of
other races. Further, this condition occurs more often during the fall, winter, and spring months than summer. Children who
attend day care centers have a greater risk of acquiring acute otitis media as do members of lower socioeconomic levels. Other
risks for the development of acute otitis media are second-hand smoke, having a large number of siblings, male gender, and
developmental abnormalities.
Question 34
Margaret Mahler developed the neo-Freudian theory of psychological birth of the infant. Which of the following phases of
Mahler’s “psychological birth” takes place at about 24 – 36 months?
consolidation
33
rapprochement
differentiation and practicing
symbiosis
Correct answer:
consolidation
According to Mahler consolidation is part of the separation-individuation phase that takes place from 6 to 36 months of age.
Consolidation takes place from 24 – 36 months of age. It is defined by the increased ability to cope with separations through
symbolic play.
Which of the following tests is a diagnostic test?
hemoglobin test
Mantoux test
MRI scan
fasting blood glucose test
Correct answer:
MRI scan
Diagnostic tests are more specific and/or sensitive than screening tests. They give objective proof that a disease process or
abnormal condition is present. Other diagnostic tests include: tissue biopsies, cultures and CT scans.
Question 36
You are treating a patient with a persistent cough. Which of the following is NOT an accurate statement regarding when to
consult or refer this patient?
Refer if patient is at high risk for malignancy.
Refer if patient refuses to stop smoking.
Refer to a pulmonologist if bronchoscopy is indicated.
Refer if cough is accompanied by blood for at least 2-3 days after there has been treatment initiated for a respiratory infection.
The Correct answer is:
Refer if patient refuses to stop smoking
The patient's refusal to stop tobacco use is not a reason for referral or consultation, but a reason to counsel.
Question 37
The U.S. Department of Agriculture (USDA) food pyramid and other research suggests all of the following EXCEPT:
diet should have substantial fiber
use sweets, fats, and oils sparingly
vegetables: 6-8 servings daily
fruits: 2-4 servings daily
The Correct answer is:
Vegetables: 6-8 servings daily
The current USDA recommendation for vegetables is 2-3 servings daily. The remaining answer choices are all current
recommendations.
Question 38
In terms of the course of disease development, the subclinical stage is which of the following?
stage of disability
stage of susceptibility
stage of presymptomatic disease
stage of clinical disease
Correct answer:
stage of presymptomatic disease
The subclinical stage of disease is also known as the stage of presymptomatic disease. It is the early stage of disease when
physiologic changes have begun but no signs or symptoms are observed.
34
Question 39
Which of the following medications is contraindicated as a treatment for acute gouty arthritis?
aspirin
naproxen
indomethacin
naproxen sodium
Correct answer:
aspirin
Aspirin is contraindicated as a treatment for gout because it can precipitate gout. A loading dose of an NSAID, such as
naproxen, 750 mg or indomethacin, 50 mg, followed by lower doses, can be helpful.
Question 40
Documentation is part of the health care process. The nurse practitioner understands that the following are all types of
procedures that may trigger a signed consent requirement except for:
HIV tests
a minor surgical procedure
an abortion
a DEXA scan
The Correct answer is:
A DEXA scan
The DEXA scan does not require additional consent. However, the remaining answer choices are all invasive and sensitive
tests and procedures requiring documented consent.
Question 41
In which of the following situations might there be an exception to guaranteed confidentiality of a patient?
in cases of suspected child abuse
when next of kin requests a patient’s records
when discussing the patient with another medical professional not involved in the care of the patient
none of the above
Correct answer:
in cases of suspected child abuse
There are exceptions to guaranteed confidentiality when society determines that the need for information outweighs the
principle of confidentiality. These situations include: when records are released to insurance companies; to attorneys involved
in litigation; in answering court orders, subpoenas or summonses; in meeting state requirements for mandatory reporting of
diseases or conditions; in cases of suspected child abuse; or if a patient reveals an intent to harm someone.
Question 42
Certain medications have been known to cause drug-induced macrocytosis without existing anemia. All of the following are in
this category EXCEPT:
Altace (ACE inhibitor)
carbamazepine (Tegretol)
valproic acid (Depakote)
zidovudine (AZT)
The Correct answer is:
Altace (ACE inhibitor)
Abnormally large or macrocytic cells due to an altered RNA:DNA ratio results in macrocytosis without anemia. Valproic acid
(Depakote), zidovudine (AZT) and carbamazepine (Tegretol) can cause macrocytosis as well as alcohol and phenytoin
(Dilantin).
Risk factors for oral cancer include all of the following EXCEPT:
recent trauma to the mouth
smoking
35
alcohol abuse
human papillomavirus type 16
Correct answer:
recent trauma to the mouth
This is not a risk factor for oral cancer. The risk factors include advancing age, tobacco and alcohol abuse and most recently,
they include chronic infection with human papillomavirus type 16.
Question 44
The nurse practitioner knows that certain persons may require additional immunizations. The following are all in this category
EXCEPT:
food handlers
animal workers
factory workers
travelers
The Correct answer is:
Factory workers
Many persons with special circumstances require additional immunizations that the general public does not. Factory workers
are not in this category. However, the other remaining answer choices all have special needs due to increased risks.
Question 45
You are volunteering at a homeless clinic to gain clinical experience. The nurse practitioner knows that which statement is true
regarding this?
Volunteerism negates susceptibility to lawsuits.
Malpractice insurance will be needed.
Malpractice coverage will be provided by the state where the clinic is located.
Malpractice insurance is not necessary when working volunteer status.
The Correct answer is:
Malpractice insurance will be needed
The nurse practitioner should always have malpractice insurance. The "good Samaritan" law protects professional volunteers
in some states, but in the event of being sued, professional liability should be carried.
Question 46
A 25-year-old female patient comes to the clinic with a “rash” on the inner wrist area. She tells the FNP that it is extremely
itchy, especially at night. The FNP tells her that she has scabies. The FNP understands that all of the following are true in
regard to this disease EXCEPT:
Kwell (lindane) is the preferred treatment.
Permethrin 5% can be used as a treatment by applying to the entire body and head and washing off after 8 to 14 hours.
The entire household must be treated.
Linens and clothes must be washed in very hot water.
Correct answer:
Kwell (lindane) is the preferred treatment.
All of the choices are true except this one. Kwell may be used as an alternative treatment but it is out of favor as a treatment
due to neurotoxicity.
Question 47
A 57-year-old Asian female is in the office inquiring about the symptoms and treatment options for menopause. She describes
"hot flashes" that are occurring frequently throughout the day. The nurse practitioner knows to tell her that these are related to:
fluctuating estrogen levels
low estrogen levels
fluctuating progesterone levels
low progesterone levels
The Correct answer is:
Fluctuating estrogen levels
Low levels of estrogen alone do not produce these hot flashes. A rapid change in estrogen levels produce the vasomotor
symptoms that are referred to as hot flashes.
36
Question 48
You are seeing a patient in the clinic with gastric ulcer. The nurse practitioner has knowledge that which of the following is
least likely to be found in this patient?
The patient is younger than 50 years of age.
The patient has previously used H2RA or antacids.
The patient has a history of long-term naproxen use.
The patient smokes cigarettes.
The Correct answer is:
The patient is younger than 50 years of age
Contributing factors and risks of gastric ulcers are nonsteroidal anti-inflammatory drug (NSAID) and corticosteroid use,
cigarette smoking, and peak incidence in the fifth and sixth decades of life. The presenting signs and symptoms are pain with
meals or immediately afterwards, nausea, vomiting, and weight loss.
Question 49
Which of the following is a TRUE statement about the Healthy People 2010?
This model applies only to people of the United States of America.
This model is developed and formulated by a special committee formed by an alliance of all the state health departments of the
United States of America.
This model's objectives are not only applicable nationally, but also internationally.
This model has an objective to help people internationally achieve only physical health.
The Correct answer is:
This document applies only to people of the United States of America
The Healthy People model provides science-based, 10-year national objective for improving the health of citizens in the
United States. The Healthy People model assesses progress to: (1) promotes collaborations across sectors; (2) help individuals
make better and knowledgeable health decisions; and (3) evaluate the effects of health prevention strategies. Healthy People
2010 is not developed by a committee formed by state health departments, but by the U.S. Department of Health and Human
Services. Also, this model does not apply to people internationally, only individuals living within the United States.
Question 50
Which of the following SSRIs has the shortest half-life?
Prozac
Fontex
Lexapro
Paxil
Correct answer:
Paxil
Paxil has the shortest half-life of the SSRIs (selective serotonin reuptake inhibitors) in the choices. Prozac has the longest.
Question 51
The theory of aging that holds that after repeated use and damage, body structures and functions wear out because of stress is
which of the following?
wear-and-tear theory
free-radical theory
immune theory
activity theory
Correct answer:
wear-and-tear theory
There are a number of theories of aging. They can be sociologic theories, biologic theories or psychologic theories. The wearand-tear theory is a biologic theory that holds that after repeated use and damage, body structures and functions wear out
because of stress.
Question 52
You are seeing a 3-year-old female child in the office. Her mother is concerned about her not being able to elevate her left arm
without crying out. A clavicular fracture is suspected. Which of the following is NOT true regarding a fracture of the clavicle?
This is one of the most common fractures that occur in childhood.
37
The patient may avoid moving the arm on the injured side or may angle the head toward the injured side to relax the trapezius
muscle.
There is usually no visible or palpable deformity upon examination and it is rarely seen on an x-ray.
The mechanisms of injury is usually a fall on the outstretched hand or direct blow to the shoulder area.
The Correct answer is:
There is usually no visible or palpable deformity upon examination and it is rarely seen on an x-ray
There is most always a visible and a palpable deformity seen and the x-ray findings will show a visible separation of the
clavicular bone. Further, the remaining answer choices are all true of this type of fracture.
Question 53
Which of the following is NOT one of the basic requirements/allowances of the HIPAA privacy rule?
Providers releasing patient information for marketing purposes must explain to the patient how the information will be used, to
whom it will be disclosed, and the time frame.
Providers and their staff are restricted to conveying the “minimum necessary information” about patients.
Providers may disclose health information to oversight agencies.
In general, providers may disclose psychotherapy notes without patient consent.
Correct answer:
In general, providers may disclose psychotherapy notes without patient consent.
There are special rules for psychotherapy notes. In general, patient authorization is required in order to disclose psychotherapy
notes to carry out treatment, payment or healthcare operations.
Question 54
Which of the following is least likely to be considered a risk factor for heart disease?
BMI <25 kg/m2
hypertension
dyslipidemia
microalbuminuria
Correct answer:
BMI <25 kg/m2
All of the choices are risk factors for heart disease except this one. A BMI of <25 kg/m2 is normal. A BMI of 30 kg/m2 or
more is considered obesity and this is a risk factor for heart disease.
Question 55
The Family Nurse Practitioner has the duty to disclose certain information to the patient as part of the informed consent
process. Exceptions to this duty would include all of the following EXCEPT:
It is a definite emergency situation.
The healthcare provider believes that the information would be harmful to the patient and invokes the therapeutic privilege.
The patient has waived the right to receive informed consent.
The patient is 80-years-old or older and unable to comprehend the complex medical facts.
The Correct answer is:
The patient is 80-years-old or older and unable to comprehend the complex medical facts.
All patients are entitled to medical information regardless of age. The nurse practitioner may treat in any of the other
circumstances listed in the remaining answer choices.
Question 56
The earliest age that an average child would be able to copy a triangle, know his colors, and count on his fingers is:
age 2 years
age 3 years
age 4 years
age 5 years
The Correct answer is:
Age 5 years
Common developmental tasks for the 5-year-old child would include the ability to draw a person with a body, head, and arms
and recognize most letters and print some. Further, the child knows how to skip, copy a triangle, count on fingers and knows
38
colors.
Question 57
The mean weight gain of a female in puberty (10 – 14 years) is which of the following?
52 lbs
38 lbs
30 lbs
55 lbs
Correct answer:
38 lbs
The weight gain for a female during puberty (10 to 14 years) is between 15 and 55 lbs. The mean weight gain is 38 lbs. Males
gain 15 – 65 lbs during puberty (11 – 16 years) and the mean weight gain is 52 lbs.
Question 58
When managing the patient on Lithium therapy, which of the following would be least important for the nurse practitioner to
evaluate?
urinalysis, electrolytes
alanine aminotransferase, aspartate aminotransferase, lactate dehydrogenase
thyroid-stimulating hormone, triiodothyronine, thyroxine
blood urea nitrogen, creatinine
The Correct answer is:
Alanine aminotransferase, aspartate aminotransferase, lactate dehydrogenase
It is not important to evaluate liver function before initiating treatment with lithium. However, lithium has adverse side effects
on renal, cardiac, and thyroid function. Baseline electrolytes are also important to obtain.
Question 59
A patient comes into the clinic for a physical examination. In the course of the examination the FNP finds that the patient has a
blood pressure reading of 165/101. This would be considered which stage of hypertension?
normal
stage 1
stage 2
prehypertension
Correct answer:
stage 2
Stage 2 hypertension is when the systolic pressure is > 160 mmHg and the diastolic pressure is > 100 mmHg. Most patients
with stage 2 hypertension will require a two drug combination.
Question 60
In terms of the human papilloma virus (HPV) which of the following statements is incorrect?
HPV has 70 subtypes and only a few are oncogenic.
HIV-positive women are at higher risk for aggressive HPV disease.
HIV-positive women should have Pap smears at least once a year.
The Pap smear is the cervical cancer screening test.
Correct answer:
HIV-positive women should have Pap smears at least once a year.
This statement is incorrect. HIV-positive women should have Pap smears every three to six months to monitor. There is a
higher incidence of cervical cancer in these women.
Question 61
A patient-provider relationship can be established in which of the following situations?
over the telephone
at a party
by providing sample medication
all of the above
Correct answer:
all of the above
A patient-provider relationship can be established in all of the situations given. It can also be established by supervising
39
another’s treatment and by giving advice or opinions to family or friends, as well as in the usual professional way.
Question 62
A 65-year-old female comes into the clinic with a history of low back pain with recent onset. Her gait is antalgic and she
reports new loss of bladder function since the onset of back pain this morning. What should be done?
Keep non-weight bearing until x-rays are completed.
Refer to a neurologist.
Refer to the emergency department.
Order physical therapy.
The Correct answer is:
Refer to the emergency department
The symptoms indicate cauda equina syndrome which is a medical emergency. With cauda equina syndrome, urinary retention
with overflow incontinence is typical. Additional symptoms of cauda equina syndrome are saddle anesthesia, sciatica down
both legs, and leg weakness.
Question 63
When counseling a new mother regarding fever in her infant or child, the nurse practitioner knows that:
Most fevers over 40°C or 104°F are of bacterial origin.
Fevers could cause convulsions in children between 6 months and 6 years of age.
Fevers of 40° or 104° can cause brain damage in children 6 months of age and younger.
Children between the ages of 6 months and 6 years of age are susceptible to heart damage from a fever.
The Correct answer is:
Fevers could cause convulsions in children between 6 months and 6 years of age
Febrile seizures are benign and do not lead to brain damage. These type of febrile illnesses are usually the result of a virus
rather than a bacterial infection and are associated with a high fever.
Question 64
In terms of normal heart sounds the S2 (diastole) is characterized by which of the following?
closure of the aortic and pulmonic valves
closure of the mitral and tricuspid valves
occurrence during early diastole (ventricular gallop)
occurrence during late diastole (atrial gallop)
Correct answer:
closure of the aortic and pulmonic valves
S2 (diastole) is characterized by closure of the aortic and pulmonic valves. S1 is characterized by the closure of the mitral and
tricuspid valves. S3 heart sound occurs during early diastole. S4 occurs during late diastole.
Question 65
It is January and you are seeing influenza in the community. The nurse practitioner is aware of what fact regarding the "flu" or
influenza?
It is characterized by a slow, insidious onset of chills, fever, and muscle aches.
In older adults, the flu may persist for several weeks with symptoms of lack of energy and fatigue.
Influenza can be caused by receiving a flu vaccine when one's resistance is low.
This is primarily contagious in the early autumn and spring and children have the greatest risk.
The Correct answer is:
In older adults, the flu may persist for several weeks with symptoms of lack of energy and fatigue
Influenza is a highly contagious respiratory infection that occurs epidemically during the winter months. The elderly
individual diagnosed with the flu has different presenting symptoms and are affected longer. Further, flu symptoms are
headache, fatigue, high temperatures, muscle aches, cough, rhinorrhea, and laryngitis. Flu vaccines do not cause the flu and are
not made with live virus.
Question 66
When seeing a 33-year-old man in the clinic with acute cocaine intoxication, you inquire about:
chest pain
internal dream states
40
fragmentation
muscle cramps
The Correct answer is:
Chest pain
Cocaine is a potent sympathomimetic and can increase heart rate and myocardial contractility. This causes coronary vessel
constriction that could lead to myocardial ischemia. Inquiring about chest pain is prudent in caring for a patient with cocaine
abuse.
Question 67
A nurse practitioner is HIV positive and she is employed in a privately owned clinic. She performs wellness exams on
ambulatory adults. What is required of this professional?
She is under obligation to inform the patient of her HIV status if she performs invasive procedures.
She is under no obligation to inform anyone of her HIV status.
She is obligated to inform her employer of her HIV status.
She is obligated to inform her patients of her HIV status.
The Correct answer is:
She is under no obligation to inform anyone of her HIV status
The nurse practitioner's health information is protected health information, just as her patient's health information is protected.
She is not under obligation to inform anyone of her HIV status, including her employer, the patient, or the State Board of
Nursing. As long as the performance of her job does not impose unnecessary health risks to anyone, she is protected by this
law.
Question 68
Which of the following best defines a hospitalist?
a physician or nurse practitioner who specializes in the care of hospitalized patients
a member of a hospital accreditation board
any physician or nurse practitioner who has hospital privileges
none of the above
Correct answer:
a physician or nurse practitioner who specializes in the care of hospitalized patients
To increase efficiency, physicians and nurse practitioners can specialize in the care of hospitalized patients. These clinicians
are called hospitalists
Question 69
The FNP has an elderly patient who complains of headaches on one temple. She tells the FNP that her scalp is tender and she
is having vision problems in the eye that is on the same side as the headache. Your examination shows an indurated, reddened,
cord-like temporal artery that is tender to touch and this is located on the same side as the headache. Lab tests show that the
sedimentation rate for this patient is elevated. Which of the following diseases/conditions would the FNP likely diagnose?
temporal arteritis
acute bacterial meningitis
migraine headache
trigeminal neuralgia
Correct answer:
temporal arteritis
When a patient has a new onset of headaches on one temple with all of the signs and symptoms mentioned in the question,
temporal arteritis is likely to be diagnosed. This type of headache is more common in older adults.
Question 70
Drug interactions are common with oral iron therapy. Which of the following drugs is NOT matched with its correct effect
when taken with oral iron?
Levodopa: decreased iron effect
Tetracyclines: decreased tetracycline and iron effect
Antacids: decreased iron absorption
Caffeine: decreased iron absorption
Correct answer:
Levodopa: decreased iron effect
41
When levodopa is taken with oral iron therapy there is the effect of both decreased levodopa and decreased iron. Use of these
medications should be separated by as much time as possible and levodopa dosage should be increased if needed.
Question 71
In terms of inflammatory bowel disease, which of the following statements is incorrect?
A person with ulcerative colitis has approximately a 10% chance of having a flare in 2 years after achieving disease remission.
A person with Chrohn’s disease has approximately a 40% chance of having a flare in 2 years after achieving disease
remission.
With ulcerative colitis, colorectal cancer risk is greatly increased afer about a decade of disease.
With Chrohn’s disease there is an increased risk for small bowel malignancy.
Correct answer:
A person with ulcerative colitis has approximately a 10% chance of having a flare in 2 years after achieving disease remission.
This statement is incorrect. A person with ulcerative colitis has approximately a 50% chance of having a flare in 2 years after
achieving disease remission. The incidence of a flare within 2 years in someone with Chrohn’s disease is less (40%).
Question 72
A 40-year-old female comes to the clinic with a rash on her hand. She tells the FNP that the rash is itchy and has begun to
“leak.” The FNP understands that this rash is most likely which of the following?
psoriasis
acute cellulitis
eripselas
eczema
Correct answer:
eczema
Eczema is a chronic inherited skin disorder marked by extremely pruritic rashes that are located on the hands, flexural folds
and the neck. The classic rash starts as multiple small vesicles that rupture, leaving painful bright red weepy lesions.
Question 73
The following statement is correct about the tetanus toxoid immunization:
DTaP and diphtheria and tetanus toxoid (DT) are safe to give to adults.
Tetanus toxoid fluid provides the same antitoxin antibody titers as tetanus toxoid.
Tetanus and diphtheria toxoid (Td) induces more persistent antitoxin antibody titers.
The recommended dose for the adult is 1 ml IM.
The correct answer is:
Tetanus and diphtheria toxoid (Td) induces more persistent antitoxin antibody titers
The tetanus and diphtheria toxoid induces a more persistent antitoxin antibody titers than tetanus toxoid fluid. Td and Tdap are
recommended for older children, adolescents, and adults. The diphtheria, tetanus, and pertussis (DTaP) vaccine and diphtheria
and tetanus toxoid (DT) vaccine are for use in children under 7 years of age and should NOT be given to adults. Note that
uppercase letters indicate higher amounts of the particular component.
Question 74
Which of the following is one of the factors in Bowen’s Family System Theory?
adequacy of growth
changes in social roles, such as children caring for parents
connecting one’s past family experiences with current behaviors
confrontation with death
Correct answer:
connecting one’s past family experiences with current behaviors
Part of Bowen’s Family System Theory is connecting one’s past family experiences with current behaviors. Also part of the
theory are: identifying multiple factors interacting across time that influence family functioning; and identifying the
interactions among biological, genetic, psychological and sociological factors that influence human behaviors.
Question 75
Which of the following is TRUE regarding current factors that support the greater emphasis on health promotion and disease
prevention?
Only 20% of illness and disease is related to lifestyle and unhealthy decisions.
42
Healthy People 2010 was developed as a outline of the prevention agenda by the American Nurses Association.
Tobacco use, substance use, and obesity are considered some of the leading health indicators.
Preventive services include treating active diseases that could spread out into the community.
The Correct answer is:
Tobacco use, substance use, and obesity are considered some of the leading health indicators
It is reported that 50% of illness and disease is related to lifestyle and unhealthy decisions. Healthy People 2010 is the
prevention agenda, and not an outline, for the nation that was established by the federal government through the U.S.
Department of Health. Preventive services are infant care, immunizations, and sexually transmitted disease services, not the
treatment of active disease states.
Question 76
You are treating an elderly lady with chronic constipation. What is the first action for patient management?
Add dietary fiber and increase fluids.
Use an oil retention enema.
Stop all constipating medications when possible.
Add sorbitol solution daily.
The Correct answer is:
Stop all constipating medications when possible
The first action the nurse practitioner should take to manage the patient's constipation is to stop all medications that are known
to cause constipation. Then, the nurse practitioner should re-assess the patient's symptoms at a later date. If symptoms should
persist, 5 to 25 grams of dietary fiber should be added along with increasing fluid intake. Physical activity should be increased
as tolerated.
Question 77
One of the neo-Freudian theories of human growth and development is Erik Erikson’s theory of life span psychosocial
development. In this theory, school age children would be in which of the following groups?
autonomy vs. shame and doubt
initiative vs. guilt
industry vs. inferiority
intimacy vs. isolation
Correct answer:
industry vs. inferiority
School age children would be in the “industry vs. inferiority” stage of psychosocial development. Toddlers are in the
“autonomy vs. shame and doubt” stage. Preschoolers are in the “initiative vs. guilt” stage. Young adults are in the” intimacy
vs. isolation” stage.
Question 78
In terms of educating the older population about health maintenance and preventive care which of the following statements is
false?
Teach them to rise to an upright position slowly because of the possibility of postural hypotension.
Encourage use of over-the-counter medications for financial savings.
Encourage use of support systems.
Encourage counseling to prevent illness.
Correct answer:
Encourage use of over-the-counter medications for financial savings.
This is false. The FNP should discourage the use of over-the-counter medications as they may interact with prescribed drugs.
The FNP should review all medications yearly and encourage the patient to throw away outdated drugs and prescriptions.
Question 79
The Centers for Disease Control and Prevention (CDC) recommends screening all women for sexually transmitted diseases
between the ages of:
12 and 16
16 and 20
20 and 24
24 and 30
Correct answer:
20 and 24
43
The CDC recommends screening all women for sexually transmitted diseases between the ages of 20 and 24. This would be
considered a secondary prevention measure to detect disease early to minimize bodily damage.
Question 80
In advising a male patient with hypertension about lifestyle changes (according to JNC7) to help control the hypertension, the
FNP would tell the patient to limit his alcohol intake to:
< 4 drinks per day
< 4 drinks per week
< 1 drink per day
< 2 drinks per day
Correct answer:
< 2 drinks per day
For moderation of alcohol consumption to control hypertension in men, the recommendation according to JNC7 is < 2 drinks
per day. For women and lighter weight persons alcoholic drinks should be limited to < 1 drink per day.
Question 81
You are doing a well-child examination and the child is 3-years-old. The nurse practitioner has an understanding that head
circumference should be measured up until what age?
12 months
15 months
24 months
36 months
The Correct answer is:
24 months
Head circumference is routinely measured at each periodic well-child visit until he has reached 2 years of age. During the first
24 months of life, the head grows most quickly, and its growth should be measured for adequacy. This should be recorded on a
growth chart so that changes can be evaluated and appropriate growth can be determined. The CDC charts show head growth
up to 36 months, but the standard recommendations for routine measurements generally specify the first 24 months.
Question 82
When taking an approach to patient education and counseling, it is important that the nurse practitioner does not:
suggest large changes in behavior for optimal health promotion
evaluate the contribution of the patient's belief system to the problem
give accurate, specific information
inform the patient about the purpose and benefit of an intervention
The Correct answer is:
Suggest large changes in behavior for optimal health promotion
Small changes in behavior should be suggested as to not overwhelm the patient. The remaining answer choices are orderly
approaches and effective education and counseling techniques.
Question 83
If an FNP breaches the standard of care with a patient but the patient suffers no injury because of it, which of the following is
true?
The FNP has committed malpractice.
There is no malpractice because the patient will not be able to prove all of the elements of malpractice in this case.
The patient will be able to prove 2 elements of malpractice; this will be enough to win a suit and damages for the breach of the
standard of care.
none of the above
Correct answer:
There is no malpractice because the patient will not be able to prove all of the elements of malpractice in this case.
There are four elements to malpractice. They are: duty, breach of duty, causation and injury. The breach of the standard of care
would fall under breach of duty, but there was no injury, so there can be no causation and therefore, there is no malpractice.
Question 84
The nurse practitioner who works with geriatric patients needs to know that the most commonly reported form of elder
maltreatment is:
sexual exploitation
44
neglect
physical abuse
financial exploitation
The Correct answer is:
Neglect
Elder mistreatment is likely to be more prevalent in a cultural community milieu where tolerance of aggressive behavior and
negative beliefs about aging and elderly adults exist. Neglect is the most commonly encountered type of elder maltreatment.
Additionally, in times of economic difficulty, the rate of financial exploitation usually increases.
Question 85
Which of the following is NOT a factor in putting a woman at a higher risk of breast cancer?
genetics
first degree relative with breast cancer
longer exposure to estrogen
thin stature
Correct answer:
thin stature
This is not a factor that puts a woman at a higher risk of breast cancer. Obesity is a factor, however, since adipose tissue can
synthesize small amounts of estrogen.
Question 86
Which of the following is not a stage of the epidemiologic principle and natural history of disease?
stage of susceptibility (prepathological)
stage of presymptomatic disease (subclinical)
stage of recovery (improvement)
stage of disability (or death)
The correct answer is stage of recovery (improvement). This is not part of the course of disease development. The third stage
is the stage of clinical disease.
Question 87
Enuresis is involuntary urination after a child has reached the age when bladder control is usually attained. Management of
primary nocturnal enuresis includes all of the following EXCEPT:
limiting fluid intake before bedtime
double voiding before bedtime
avoiding punishment
withholding verbal praise so as not to call attention to the problem
Correct answer:
withholding verbal praise so as not to call attention to the problem
Verbal praise is an excellent means of motivational therapy for a child with primary nocturnal enuresis. Other means of
motivational therapy include a reward system and keeping a dryness calendar.
Question 88
A 62-year-old patient with Type II diabetes has been diagnosed with hypertension. Which anti-hypertensive drug is the
recommended choice in this case?
ACE inhibitor
calcium channel blocker
diuretic
beta-blocker
The Correct answer is:
ACE inhibitor
These antihypertensive agents, ACE inhibitors, enhance renal function in patients with diabetes.
Question 89
In terms of billing for Medicare patients which of the following statements is incorrect?
There are no consequences for selecting an inappropriate code for a patient visit if done unintentionally.
Each CPT code has corresponding levels of required history taking, physical examination and medical decision making.
NPs are responsible for ensuring that the billing for their services matches the level of care given.
The consequence of selecting an inappropriate code may include loss or restriction of the NP’s license b the board of nursing.
45
Correct answer:
There are no consequences for selecting an inappropriate code for a patient visit if done unintentionally.
NP’s are expected to know how to bill correctly. Ignorance is a poor defense. There may be consequences for selecting an
inappropriate code if auditors find that more errors were made in overcoding than undercoding visits.
Question 90
A 76-year-old African-American female patient presents with a complaint of dyspnea, fatigue, cough, and dependent edema
that has been worsening over the past few days. In planning treatment, the nurse practitioner considers:
prescription for furosemide (Lasix) 40 mg PO qd
addition of a calcium channel blocker to the patient's medications
streptokinase therapy
referral for hospitalization for evaluation of heart function
The correct answer:
Referral for hospitalization for evaluation of heart function
The patient's symptoms, dyspnea, fatigue, cough, and dependent edema, indicated early pulmonary edema. Patients with
pulmonary edema require hospitalization for treatment and stabilization. Further, patients with this condition should not be
treated as an outpatient. Calcium channel blockers are of little benefit in heart failure.
Question 91
A 60-year-old male comes to the clinic complaining of a sudden onset of severe frontal headache with eye pain. He tells the
FNP that his vision is blurred and his eyes are tearing. He also sees halos around lights. Which of the following conditions
would the FNP most likely suspect?
primary closed-angle glaucoma
primary open-angle glaucoma
subconjunctival hemorrhage
macular degeneration
er:
primary closed-angle glaucoma
Primary closed-angle glaucoma is a sudden blockage of aqueous humor causing markedly increased IOP causing ischemia and
permanent loss of vision. The symptoms reported by the patient point to primary closed-angle glaucoma more than any of the
other choices.
Question 92
Which of the following is correct regarding pediatric screening recommendations?
All full-term infants need a phenylketonuria (PKU) test at age 6 months.
Anemia screening with hemoglobin or hematocrit is recommended once between 1 and 9 months and once during adolescence
for menstruating females.
Hearing screening is not necessary after initial newborn screening unless indicated.
Blood pressure should be checked with each well-child visit, starting at age 1.
Anemia screening with hemoglobin or hematocrit is recommended once between 1 and 9 months and once during adolescence
for menstruating females
Anemia screening allows for the detection of iron deficiency anemia and congenital anemias. The PKU is done prior to
discharge from the hospital or within 1-2 weeks. Hearing should be screened at ages 3, 4, 5, 10, 12, 15, and 18 years of age by
means of audiometric evaluation. Blood pressure screening is recommended annually beginning at 3 years of age.
Question 93
A male infant is in the office with clubfoot. The nurse practitioner has knowledge that:
It can be corrected with casting and exercises.
It involves the foot only.
It always requires an urgent neurosurgical referral.
It involves the foot and lower extremity.
It involves the foot and lower extremity
Clubfoot is known as talipes equinovarus. This condition occurs when the foot is plantar flexed and the forefoot and sole are
thrust medially. Clubfoot involves the foot and lower extremity and is an urgent orthopedic referral, not a neurosurgical
referral. The initial management involves casting and splinting, and not exercises, with surgical treatment within 3-6 months.
The majority of the cases of patients with clubfoot will improve with taping, splinting, and/or casting when early intervention
46
occurs.
Question 94
Which of the following characteristics would be unusual in a patient with uncomplicated gallbladder disease?
guarding
positive Murphy's sign
nausea
fever
The Correct answer is:
Fever
Fever is an unusual symptom in a patient with uncomplicated gallbladder disease. This may be seen in an episode of acute
cholecystitis. A positive Murphy's sign is usually elicited when the gallbladder is inflamed.
Question 95
All of the following are accurate regarding documentation EXCEPT:
You should document your criticism of a fellow healthcare provider's clinical decision in the patient's medical record. This
will protect you if your treatment needs to be defended later.
You should use standard abbreviations in the medical record so that subsequent readers will have no doubt as to your meaning
and intent.
You should document a patient's noncompliant medical behavior in the medical record.
You should document telephone conversations with the patient and/or family in the medical record and be particularly specific
about recording medication changes.
The Correct answer is:
You should document your criticism of a fellow healthcare provider's clinical decision in the patient's medical record. This
will protect you if your treatment needs to be defended later.
Ridiculing another provider in the patient's medical record is never appropriate. It will allow the plaintiffs' lawyers an avenue
to find wrongdoing. The nurse practitioner should deal with the provider directly, preferably in person.
Question 96
A liability policy which pays claims only during the period that the policy is active is termed:
claims made policy
bobtail coverage
tail coverage
liability protection
The Correct answer is:
Claims made policy
Liability insurance which covers the holder only during the time of the active policy is known as a "claims made" policy. This
kind of policy is usually less expensive than other policies that will protect the policyholder against unknown claims at the end
of the policy period.
Question 97
The process of monitoring patients who have certain high-cost or chronic conditions to help third party payers and/or hospitals
to manage their health care costs is called:
case management
risk management
utilization review
none of the above
Correct answer:
case management
Case management is the process of monitoring patients who have certain high-cost or chronic conditions to help third party
payers and/or hospitals to manage their health care costs. Case management is very useful for third party payers and hospitals.
Question 98
A 64-year-old Caucasian male has the following lipid level results:
Total cholesterol 240 mg/dL
LDL 140 mg/dL
HDL 35 mg/dL
Triglycerides 201 mg/dL
What class of medications will normalize his lipid elevations and decrease his risk of a cardiac event?
fibric acids
47
bile acid sequestrants
niacin
statins
The Correct answer is:
Statins
The only medication class that lowers elevated lipid levels and has proven efficacy in lowering the risk of cardiac events, even
for primary prevention, is statin medications. Further, statin therapy has been shown to reduce overall mortality due to
cardiovascular deaths.
Question 99
A patient comes to the clinic with a burn on his leg that is about three times the palmar surface of his hand. The body surface
area (BSA) of the burn can be estimated at:
3%
6%
9%
12%
Correct answer:
3%
A person’s palmar surface represents a BSA of 1% throughout his life span. Therefore, a burn that is 3 times the palmar
surface represents a BSA of 3%.
Question 100
Which of the following is NOT a factor in determining the scope of practice for nurse practitioners?
common practice within a community
federal statutes, rules and regulations
state statutes, rules and regulations
boards of nursing
Correct answer:
federal statutes, rules and regulations
Federal statutes, rules and regulations do not determine the scope of practice for nurse practitioners. The specific legal scope
of practice for a nurse practitioner is determined b state statutes, rules and regulations, boards of nursing, common practice
within a community and educational preparation. For instance an adult nurse practitioner would not be legally authorized to
care for children.
48
Test 2 ancc
Question 1
Which of the following osteoporosis treatments stops bone loss but does not rebuild bone?
Fosamax
Evista
Teriparatide
Miacalcin
Correct answer:
Miacalcin
Miacalcin (calcitonin salmon) stops bone loss but does not rebuild bone. It is a good choice for relief of bone pain from
vertebral fractures.
Question 2
Coping styles (defense mechanisms) are automatic psychological processes that protect the individual against anxiety and
from the awareness of internal and external dangers and stressors. Which of the following coping styles involves attributing
one’s own thoughts or impulses to another person?
displacement
projection
undoing
introjection
Correct answer:
projection
Projection is attributing one’s own thoughts or impulses to another person. An example of this would be when a secretary has
sexual feelings toward her boss and tells her friends that her boss is “coming on to her.”
Question 3
Which of the following is NOT a screening guideline for hyperlipidemia?
complete lipid profile every 5 years starting at the age of 20
over age 40, screen every 2 – 3 years
total cholesterol should be < 169 mg/dL
preexisting hyperlipidemia patients – screened annually or more often
Correct answer:
total cholesterol should be < 169 mg/dL
This is not correct. Total cholesterol should be < 199 mg/dL. Borderline is between 200 and 239 mg/dL; high is > 240 mg/dL.
Question 4
Of the fundamental domains of human growth and development, which one includes fine and gross motor abilities?
psychological and social domain
cognitive domain
physical domain
none of the above
Correct answer:
physical domain
Fine and gross motor abilities fall into the physical domain. Also parts of the physical domain are: genetic factors, physical
stature and appearance, nutritional status, and physical health and well- being.
Question 5
At about what age can the FNP expect to see an infant keep his back straight when pulled to sitting?
1 – 2 months
3 – 4 months
5 months
9 – 11 months
Correct answer:
49
5 months
At 5 months an infant will typically keep his back straight when he is pulled to a sitting position. At this age infants also bear
weight on their legs when standing, play with their feet and sit with support.
Question 6
The condition pictured above is which of the following
vitiligo
xanthelasma
seborrheic keratoses
melasma
Correct answer:
xanthelasma
Xanthelasma are raised and yellow soft plaques that are located under the brow on the upper and/or lower lids of the eyes,
usually on the nasal side. They may be a sign of hyperlipidemia if the person is younger than 40 years of age.
Question 7
The FNP has a new patient who is taking a monoamine oxidase inhibitor (MAOI) for depression. The FNP knows that this
patient should avoid all but which of the following foods?
aged cheese
red wine
chocolate
leafy green vegetables
Correct answer:
leafy green vegetables
Patients who take MAOIs must avoid foods with high tyramine. These include aged cheese, red wine, chocolate and fermented
food such as beer.
Question 8
Which of the following statements about pulmonary tuberculosis (TB) is false?
About 75% of individuals exposed to the causative organism of TB become infected.
TB is a chronic bacterial infection.
TB is transmitted through aerosolized droplets.
The stage of TB known as primary TB is usually symptom-free.
Correct answer:
About 75% of individuals exposed to the causative organism of TB become infected.
This statement is false. About 30% of individuals exposed to the causative organism of TB become infected. In an
immunocompetent host, when the organism is acquired, an immune reaction ensues to help contain the infection within
granulomas.
Question 9
A 12-year-old girl is brought to the clinic by her mother with inflammation of the oral mucosa involving the tongue, mucosa,
and gingiva. A diagnosis of gingivostomatitis is made. The nurse practitioner knows that all of the following are management
interventions for this condition EXCEPT:
Oral anesthetics will help with symptoms but should be used with caution.
If the case is severe, corticosteroids should be considered.
Coating agents such as diphenhydramine hydrochloride elixir with Kaopectate or Maalox 1:1 may bring relief.
Avoid cool fluids and popsicles because they worsen the severity of this condition.
The Correct answer is:
Avoid cool fluids and popsicles because they worsen the severity of this condition
The patient should consume cool fluids and popsicles and not avoid eating them. Cool fluids and popsicles provide soothing
and help with the healing process. Antipyretics and analgesics may be necessary, as well as oral antiviral agents and
antibiotics.
50
Question 10
Tobacco use is a major health hazard. Which of the following statements in regard to identifying and treating tobacco
dependence is incorrect?
Brief tobacco dependence treatment has been shown to be effective.
The combination of counseling and medication is more effective than when each is used alone.
Individual, group and telephone counseling increase in effectiveness with treatment intensity.
Generally the risk associated with medications to treat tobacco dependence is more than that associated with continued
tobacco use.
Correct answer:
Generally the risk associated with medications to treat tobacco dependence is more than that associated with continued
tobacco use.
This statement is incorrect. Numerous effective medications are available for tobacco dependence and clinicians should
encourage their use by all patients who are attempting to quit smoking. The use of these medications reliably increases longterm smoking abstinence rates. Generally, the risk associated with the use of these medications is less than that associated with
continued tobacco use.
Question 11
The FNP is going over some dietary facts for a person with a risk of heart disease. The American Heart Association suggests
all of the following dietary restrictions EXCEPT:
cholesterol: < 100 mg per day
total fat: 25-35%
saturated fat: < 7%
protein: 15%
Correct answer:
cholesterol: < 100 mg per day
The American Heart Association recommends that a person have less than 200 mg of cholesterol per day. In addition to the
guidelines in the other three choices, the AHA suggests monounsaturated fat: up to 20%; polyunsaturated fat: up to 10%; and
carbohydrates (from complex carbohydrate foods): 50-60%.
Question 12
In terms of infant growth and development from birth to 2 years, major tasks in the cognitive development domain include all
of the following EXCEPT:
presence of blink reflex in newborns
preconceptual thinking in older infants
a greater auditory acuity for high rather than low frequency sounds as infants
binocular vision
Correct answer:
preconceptual thinking in older infants
Preconceptual thinking does not take place as a cognitive development until the ages of 2 to 4. All of the other choices are
cognitive developments that take place from birth to 2 years.
Question 13
Erickson’s stages of psychosocial development holds that how well individuals accomplish developmental tasks will
determine their success in accomplishing other tasks as they get older. Failure in which of the following tasks leads to despair?
identity
iintimacy
generativity
ego identity
Correct answer:
ego identity
Failure in ego identity leads to despair according to Erickson’s stages of psychosocial development. Failure in identity leads to
role confusion. Failure in intimacy leads to additional role confusion. Failure in generativity leads to stagnation.
51
Question 14
A 72-year-old male patient with longstanding hypertension takes an ACE inhibitor and a thiazide diuretic daily. Today his
blood pressure is 128/88 and his pulse is 98. He has developed dyspnea on exertion and peripheral edema over the past several
days. The nurse practitioner knows that these symptoms demonstrate:
fluid or sodium excess
development of congestive heart failure
noncompliance with medication
the need for better blood pressure management
The Correct answer is:
Development of congestive heart failure (CHF)
The symptoms of dyspnea on exertion and peripheral edema are symptoms of mild CHF. Longstanding hypertension is a
major risk factor for the development of CHF. Noncompliance with medications or sodium/fluid excess may produce
peripheral edema, but it is highly unlikely that a 72-year-old would have these symptoms in the absence of CHF.
Question 15
A father brings his 14-year-old son into the clinic with complaints of knee pain. The boy plays basketball and states that it gets
worse with jumping. It is important for the nurse practitioner to consider Osgood's Schlatter's disease and to know that:
The pain is the result of microtears of the patellar tendon and overuse of the quadricep muscles of the knees.
This is a condition caused by stress and trauma of the femoral tendon.
This condition is more common in girls but boys have it too.
Athletic individuals are not at risk for this condition.
The Correct answer is:
The pain is the result of microtears of the patellar tendon and overuse of the quadricep muscles of the knees
Osgood's Schlatter's disease is more common in boys from ages 10 to 17 years. Repetitive stress and trauma from excessive
traction to the patellar tendon causes overstress or overuse of the quadricep muscles. Further, this condition is more commonly
seen in physically active adolescents.
Question 16
What is TRUE about adolescent male's puberty and growth and development?
Spermarche is defined as the first ejaculation and the average age is 15.
The majority of somatic changes occur from age 10 to age 14.
Delayed puberty is represented by no testicular growth by age 12.
Puberty typically starts around age 12.
The Correct answer is:
The majority of somatic changes occur from age 10 to age 14
Growth spurts in boys occur one year later than girls, with the majority of the somatic changes seen from age 10 to age 14.
Further, spermarche is defined as the first ejaculation in males, with the average age at 13.4 years and not 15 years of age.
Delayed puberty is represented by no testicular growth by age 14. Precocious puberty typically starts before age 9. Then,
puberty starts at age 9 and not age 12.
Question 17
A patient with hypertension is in the office and you have prescribed Cozaar. Cozaar, if taken with which of the following
drugs, can put a patient at risk for developing a renal calculi?
Tagament
Nizoral
Diflucan
Oxipurinol
Correct answer:
Oxipurinol
Cozaar is an Angiotensin II Receptor Antagonist prescribed for the treatment of hypertension. This drug blocks the material in
the body that constricts or narrows the blood vessels, then relaxes the blood vessels, allowing the blood to flow more freely
and results in a lowered blood pressure. The common side effects of Cozaar are dizziness, diarrhea, muscle cramps, heartburn,
stuffy nose, pain in the leg, knee or the back. Some serious side effects include chest pain, difficulty breathing and swelling in
the face, throat or tongue. Cozaar, if taken with certain other drugs, can cause a potential drug interaction. For example, when
52
Cozaar is taken with Tagament, Cozaar increases the serum level of Tagament. Then, if Cozaar is taken with Diflucan,
Diflucan reduces the efficacy of Cozaar. When Cozaar is taken with Oxipurinol, the patient has an increased risk of
developing renal calculi. Further, if the patient drinks grapefruit juice with Cozaar, grapefruit juice causes a delay in the
absorption and decreases the serum levels of the Cozaar.
Question 18
According to the Code of Ethics for Nurses, the philosophical basis for informed consent in health care is which of the
following?
respect
participation
self-determination
dignity
Correct answer:
self-determination
Self-determination, also known as autonomy, is the philosophical basis for informed consent in health care. Patients have the
moral and legal right to determine what will be done to them.
Question 19
The verification of the more abstract nursing theories (such as Martha Rogers) is often hampered by:
prior studies that did not support the theory
lack of adequate laboratory settings for conducting experiments
adapting the medical model to nursing care
lack of adequate measures for the theoretic concepts
The Correct answer is:
Lack of adequate measures for the theoretic concepts
Prior studies always provide information about the theory, even when conducted in other countries. The lack of adequate
measures for theoretic concepts is a major roadblock in many areas of research. More abstract concept tends not to be studied
in laboratory settings.
Question 20
When assessing a female factory worker for carpal tunnel syndrome (CTS) the nurse practitioner has knowledge of:
The Phalen’s Sign involves tapping the anterior wrist briskly to elicit a positive finding of paresthesia or reproduce symptoms.
The Tinel's Sign involves full flexion of the wrist for about 60 seconds.
Adult patients who use their hands frequently for a job are generally affected.
Numbness and tingling of the lateral two fingers (fourth and fifth digits) is expected.
The Correct answer is:
Adult patients who use their hands frequently for a job are generally affected
Jobs like typing, factory work, and writing put patients at risk for carpal tunnel syndrome. It is more common in females. The
Tinel's Sign involves tapping the anterior wrist and the Phalen's Sign involves full flexion of the wrist for 60 seconds.
Numbness and tingling is described in the thumb, index finger and middle finger, not the later two, the fourth and fifth fingers.
Question 21
The fee paid by a managed care organization (MCO) to a healthcare provider, per patient, per month, for care of an MCO
member is which of the following?
“incident to” services
capitation
fee for service
none of the above
Correct answer:
capitation
Capitation is the fee paid by a managed care organization (MCO) to a healthcare provider, per patient, per month, for care of
an MCO member. Capitated fees for primary care vary, based on a patient’s age and sex. An NP wishing to provide care for a
Medicare patient enrolled in an MCO applies to the MCO for admission to the organization’s provider panel.
Question 22
You are discussing sleep with an aging adult. It is important the nurse practitioner understands:
Increased delta or stage IV sleep occurs with age.
53
Decreased sleep latency occurs with age.
An aging adult has decreased rapid-eye-movement (REM) sleep.
There are decreased nocturnal awakenings with the older adult.
The Correct answer is:
An aging adult has decreased rapid-eye-movement (REM) sleep
REM sleep is associated with skeletal muscle atonia and dreaming. It occurs in three of four regular spaced 10 to 15 minute
cycles and starts 120 minutes after the onset of sleep. With age, REM decreases. Delta sleep, or stage IV, is a deep sleep and
also decreases with age. Nocturnal awakening and sleep latency increases with age.
Question 23
Which of the following persons would NOT be considered incompetent to give consent?
a patient who is unconscious
a patient who is mentally retarded
an emancipated minor
a patient who is under the influence of sedative drugs
Correct answer:
an emancipated minor
Minor children cannot consent to treatment. However, emancipated minors (those that are married, for example) are competent
to give consent.
Question 24
The nurse practitioner must be skilled in understanding when referral is necessary. Which of the following patients would need
to be referred to a gastroenterologist?
a patient with persistent GERD that doesn't respond to H2 blockers
a patient with bright red rectal bleeding
a patient with significant weight loss and history of frequent blood loss
a patient with mild nausea and dysuria
The Correct answer is:
A patient with significant weight loss and history of frequent blood loss
These symptoms indicate referral since they are worrisome for cancer. Mild to moderate GERD that doesn't respond to H2
blockers is common, so a proton pump inhibitor could be implemented. Bright red rectal bleeding usually is related to
hemorrhoids, which can be treated in primary care. Nausea and dysuria needs further evaluation and can be treated in primary
care.
Question 25
In a pregnant woman if there is vaginal bleeding and cramping and the cervix remains closed it is which of the following types
of spontaneous abortions?
inevitable abortion
threatened abortion
complete abortion
incomplete abortion
Correct answer:
threatened abortion
In a threatened abortion there is vaginal bleeding and cramping but the cervix remains closed. The pregnancy may be able to
be salvaged.
Question 26
In terms of the male genitals, what is it called when the testicle is elevated toward the body in response to stroking the
ipsilateral inner thigh?
cremasteric reflex
transillumination
spermatogenesis
stepping reflex
Correct answer:
cremasteric reflex
The cremasteric reflex happens when the testicle is elevated toward the body in response to stroking the ipsilateral inner thigh
(or the thigh on the same side as the testicle).
54
Question 27
All of the following persons were involved in the ethologic theory of human growth and development EXCEPT:
Konrad Lorenz
Carl Rogers
Harry Harlow
Mary Ainsworth
Correct answer:
Carl Rogers
Carl Rogers was a promoter of the humanistic theory of human growth and development rather than the ethologic theory. The
other choices are those who were involved in ethologic theories along with John Bowlby, Marshall Klaus and John Kennel.
Question 28
Under the Final Rule of HIPAA all covered entities must do all of the following EXCEPT:
notify patients orally and face-to-face of their rights under the rules
appoint a privacy officer
conduct training for staff about policies
issue policies regarding handling of and protection of patient information
Correct answer:
notify patients orally and face-to face of their rights under the rules
Under the Final Rule of HIPAA all covered entities must notify patient in writing of their rights under the rules. Patients must
also authorize in writing any release of their individually identifiable information for marketing purposes.
Question 29
In a child with bacterial meningitis, the typical CSF response includes a median WBC count of:
500 cells/mm3 with 80 – 85% neutrophils
1200 cells/mm3 with 90 – 95% neutrophils
1500 cells/mm3 with 85 – 95% neutrophils
1800 cells/mm3 with 80 – 85% neutrophils
Correct answer:
1200 cells/mm3 with 90 – 95% neutrophils
Lumbar puncture should be part of the evaluation of a febrile younger child who has an altered neurological examination.
Typical CSF response in bacterial meningitis includes a median WBC of 1200 cells/mm3 with 90 – 95% neutrophils
Question 30
Insulin should be used in treating type 2 diabetes in all but which of the following situations?
in all patients
at the time of diagnosis to help achieve initial glycemic control
when 2 or more oral agents at optimized dose are inadequate
when acutely ill
Correct answer:
in all patients
In type 1 diabetes insulin would be used in all patients. It would not be used in all Type 2 diabetes patients; it would only be
used in certain circumstances such as those outlined in the other three choices.
Question 31
A 52-year-old female patient who wears a right lower leg brace weighs 100 lb, is 65 inches tall, and her vital signs are normal.
She has developed post-polio symptoms. The nurse practitioner understands she needs to:
Avoid exposure to cold or chilling, which may cause a loss of strength in the affected muscle.
Reduce the amount of time using the brace for joint support to prevent further loss of strength.
Exercise all muscle groups vigorously to prevent disuse syndrome.
Gain weight to prevent further disability.
The Correct answer is:
Avoid exposure to cold or chilling, which may cause a loss of strength in the affected muscle
In addition to avoiding gaining weight and exercising to the point of muscle pain, regular health maintenance visits are
necessary. Cold temperatures can cause a loss of muscle strength in the affected muscle groups and should be avoided. The
brace should be used along with periodic evaluation of muscle strength and function.
Question 32
55
Loss of corneal reflex is in part seen as a dysfunction of which cranial nerve (CN)?
III
IV
V
VI
The Correct answer is:
V
The trigeminal nerve assists with three types of sensation (temperature, pain, and tactile). Further, the trigeminal nerve has
three subdivisions known as branches.
Question 33
Part of the well-child examination is a lead risk assessment. A positive response to any risk questions indicates risk of lead
exposure. A positive response to which of the following questions would be an indicator of a risk for lead poisoning?
Does your child live in or regularly visit a house that was built before 2005?
Does your child live in a home built before 1978 that is undergoing renovations?
Does your child have a playmate that has had lead poisoning?
All of the above
Correct answer:
"Does your child live in a home built before 1978 that is undergoing renovations?"
A positive answer to this questions would be a risk indicator for lead exposure. If the home was built before 1978 and is
undergoing renovations, then the child may be at risk for lead exposure.
Question 34
You are assessing a newborn at a 2-week well-child evaluation. Which of the following is not seen at this age?
light-sensitive eyes
lack of a defensive blink
best vision at a range of 8 to 12 inches
presence of a red reflex
The Correct answer is:
Lack of a defensive blink
A newborn will move all extremities, have preference for high pitched sounds, react to sound by blinking and turning.
Additionally, the remaining answer choices should be present when assessing a 2-week-old well child.
Question 35
A 88-year-old Caucasian female is brought to the nurse practitioner by her family. The client has a history of dementia.
However, the family states the 88-year-old's mental status appears worse than usual. What course of action would the nurse
practitioner consider?
Order an immediate magnetic resonance imaging (MRI) scan.
Perform a comprehensive medication review.
Order a urine analysis (UA).
Help the family look for a nursing home.
The Correct answer is:
Order a urine analysis (UA)
In the elderly, one of the most common reasons for acute mental health status change, especially decline or worsening, is acute
infection. This is usually a urinary tract infection. Since elders do not usually run temperatures, and advanced aging does not
allow for recognition of acute symptoms, the diagnosis is difficult.
Question 36
According the CDC classification of overweight and obesity, a person with a BMI (kg/m2) of 27.0 would be classified as:
healthy weight
overweight
obese
extremely obese
Correct answer:
overweight
56
Persons with a BMI (kg/ m2) of 25 – 29.9 are classified as overweight. A person of healthy weight would have a BMI of 18.5
– 24.9.
Question 37
When treating skin disorders, the nurse practitioner knows that which of the following is the LEAST potent topical
corticosteroid?
hydrocortisone 2.5%
fluocinonide 0.05%
betamethasone dipropionate 0.1%
desoximetasone 0.25%
The Correct answer is:
Hydrocortisone 2.5%
The medications listed as answer choices desoximetasone 0.25%, betamethasone dipropionate 0.1% and fluocinonide 0.05%
are all medium or high potency corticosteroids. Hydrocortisone 2.5% is a low potency agent.
Question 38
When discussing Human Immunodeficiency virus (HIV) testing with a patient, the nurse practitioner knows:
Sexually active homosexual men are the only ones at risk for HIV.
Receiving blood products from 1985 to 1995 is a risk factor for HIV.
The screening ELISA test (enzyme-linked immunosorbent assay) detects antibodies and is 99% sensitive and specific.
There is no need to recheck the ELISA if negative on the first test.
The Correct answer is:
The screening ELISA test (enzyme-linked immunosorbent assay) detects antibodies and is 99% sensitive and specific
Persons with a history of multiple sex partners, homelessness, sexually transmitted diseases (STD's), and IV drug use are also
at risk for HIV. The receipt of blood products from 1975 to 1985 is a risk factor as well. The ELISA is recommended to be
rechecked at a six month window period.
Question 39
The leading cause of death in children 12 months to 9 years is:
heart disease
unintentional injuries
congenital anomalies
cancer
Correct answer:
unintentional injuries
The leading cause of death in children 12 months to 9 years is unintentional injuries. The leading cause of death in infants
younger than 12 months is congenital anomalies
Question 40
In terms of reimbursement for services for a family nurse practitioner, which of the following statements is accurate?
FNPs are not reimbursed for their services as primary care providers under TRICARE.
There is considerable flux in state and national policy on what services and procedures FNPs may bill for and whether they
will be paid directly.
Private insurance plans only reimburse for services of the FNP if mandated by state law.
Managed care organizations always reimburse FNPs as primary care providers.
Correct answer:
There is considerable flux in state and national policy on what services and procedures FNPs may bill for and whether they
will be paid directly.
This is the only accurate statement of the four choices. FNPs are reimbursed for their services as primary care providers under
TRICARE. Private insurance plans may elect to reimburse for services of the FNP even if not mandated by state law.
Managed care organizations frequently have excluded FNPs from being designated as primary care providers.
Question 41
What is the leading cause of death for all ages and genders in the United States?
cancer
heart disease
accidents
congenital abnormalities
57
Correct answer:
heart disease
Heart disease is the leading cause of death in all ages and genders. The leading cause of death for adolescents is motor vehicle
crashes. The leading cause of death in infants younger than 12 months is congenital anomalies and the leading cause of death
in children 12 months to 9 years is unintentional injuries.
Question 42
A 11-year-old girl is brought into the office with clinical symptoms and a diagnostic history for scabies. The nurse practitioner
knows that she was probably infected:
1-3 days ago
1 week ago
2 weeks ago
3-4 weeks ago
The Correct answer is:
3-4 weeks ago
The incubation period for scabies is about 3-4 weeks after primary infection. Further, the patient with scabies will develop
symptoms in 1-3 days. The classic complaint from patients with scabies is nocturnal pruritus.
Question 43
The nurse practitioner is assessing a 24-year-old female patient with asthma. What is a common clinical manifestation of
asthma?
nocturnal exacerbation
chronic hypoxemia
diffuse crackles
pruritus
The Correct answer is:
Nocturnal exacerbation
Nocturnal exacerbation is a common clinical sign of asthma. Additionally, nocturnal exacerbation is linked to the variation in
circulating catecholamines and vagal tone. Chronic hypoxemia and diffuse crackles are seen in the client with chronic
bronchitis. Pruritus is often seen in clients with contact allergic reactions.
Question 44
When treating a patient who has an extremely high P.T. level from Coumadin (warfarin) usage, the nurse practitioner
understands that which of the following is the antidote:
Vitamin K
Vitamin C
Vitamin A
Vitamin D
The Correct answer is:
Vitamin K
The antidote for Coumadin (warfarin) toxicity is Vitamin K. It is often given in an injection form to patients with extremely
elevated P.T. levels. Dietary sources of Vitamin K include green leafy vegetables.
Question 45
You have a 38-year-old patient seen in the clinic for her 6 week post-partum examination by the nurse practitioner. She is
breast-feeding without difficulty and plans to continue for a year. She wants to begin using a contraceptive and plans no
further pregnancies. Which of the following is an inappropriate choice for this patient?
combination oral contraceptives
progestin-only oral contraceptives
intrauterine device
Depo-Provera 150 mg IM every 3 months
The Correct answer is:
Combination oral contraceptives
Combination oral contraceptives, which include the hormones estrogen and progestin, are not recommended for breast-feeding
mothers due to the potential effect on decreasing milk quantity and quality. Progestin-only oral contraceptives are approved
for nursing mothers because no deleterious effects on milk quantity or quality have been shown. The intrauterine device (IUD)
is also an acceptable contraceptive for breast-feeding mothers, as is Depo-Provera.
Question 46
58
When the FNP is examining a 12-month-old infant, about how much growth should she expect to see in head circumference,
as compared to the infant's head circumference at birth?
3 cm
6 cm
12 cm
18 cm
Correct answer:
12 cm
The typical head growth in the first year of life is 12 cm: 6 cm in the first 3 months; 3 cm in the 4th to 6th months; and 3 cm in
the 6th to 12th months. Subsequent head growth is about 0.5 cm/year for 2 – 7 year olds and 0.3 cm/ year for 8 – 12 year olds.
Question 47
The FNP wants to prescribe a proton pump inhibitor (PPI) for a patient with GERD. Which of the following drugs is NOT a
PPI?
Prilosec
Cyotec
Prevacid
Nexium
Correct answer:
Cyotec
Cyotec is a prostaglandin analogue, a drug specifically designed for gastric protection with NSAIDs. PPIs are a group of drugs
whose main action is a pronounced and long-lasting reduction of gastric acid production.
Question 48
Certain claims are made on advertisements regarding medications, such as Drug X has been known to be in use for 6 years
with over 2 million doses given in North America and Great Britain. Drug X stops heartburn and upset stomach, prevents
esophageal reflux and is the preferred treatment for GERD. The nurse practitioner realizes that this claim is:
Valid; there are enough users mentioned who have had success for it not to be.
Invalid; the level of significance is not mentioned to be at the 0.05 level.
Valid; the cohort and Hawthorne effects are operating.
Invalid; there are no control or comparison groups, and no statistics are stated.
The Correct answer is:
Invalid; there are no control or comparison groups, and no statistics are stated
Even if the claim details extensive use of this drug, Drug X, there must be valid statistics to provide evidence with the use of a
control group that will render a level of significance.
Question 49
What is a CAM modality or medicine?
a complementary or alternative modality or medicine
a consistent and accepted modality or medicine
a generic drug
an untested drug
Correct answer:
a complementary or alternative modality or medicine
Research suggests that 40 – 50% of patients are currently using a form of complementary or alternative therapy. NPs as
providers need to learn about common CAM treatments and particularly how some herbal products interact with prescription
drugs.
Question 50
You are examining a 2-week-old infant in the office. Her mother reports that her pregnancy was fairly unremarkable except
that she took tetracycline during the first trimester before she knew she was pregnant. The nurse practitioner is aware that fetal
exposure to this medication causes:
disruption in bone growth
limb deformities
blindness
hearing loss
The Correct answer is:
Disruption in bone growth
59
Tetracycline is an antibiotic used in many conditions. It is a category D drug classification for pregnancy. Category D
classification means the medication can cause harm to the fetus’s development, such as hinder the development of bone
growth.
Question 51
Which of the following is most commonly reported as the largest single source of income for the elderly population?
asset income
public/private pension earnings
Social Security
family financial support
The Correct answer is:
Social Security
Among the elderly population in the United States, Social Security is mentioned as the largest and often time sole source of
income. Further, less than half the elderly population generate income from private or public pension and income from other
financial investments.
Question 52
When discussing prenatal visits with a pregnant patient, the Family Nurse Practitioner knows that the recommended frequency
of prenatal visits in the patient's 5th week of pregnancy is:
every week
every 4 weeks
every 3 weeks
every 2 weeks
The Correct answer is:
Every 4 weeks
The patient is in her 5th week of pregnancy. Therefore, she will see the provider every 4 weeks until she reaches her 28th
week of pregnancy. Then, she will start seeing the provider every 2 weeks.
The frequency for prenatal visits, after the patient has had her initial examination, is as follows:
Time of Pregnancy Frequency of Visits
up to 28 weeks every 4 weeks
28 to 36 weeks every 2 weeks
> 36 weeks every week
Question 53
The FNP has a 70-year-old female patient who has come in because she began to feel weak and dizzy. After examination the
FNP suspects atrial fibrillation. The FNP’s plan of diagnosis and treatment for this woman would include which of the
following?
doing a 12-lead electrocardiogram (EKG)
prescribing an anticoagulant
monitoring response to anticoagulation with the INR
all of the above
Correct answer:
all of the above
The diagnosis and treatment for this woman would include all of the first three choices. The anticoagulation guidelines are
atrial fibrillation: INR 2.0 – 3.0; synthetic valves: INR 2.5 to 3.5.
Question 54
The FNP is doing a routine physical for a 17-year-old female. When discussing the girl’s social and school life she tells the
FNP that she doesn’t feel that she is smart enough or outgoing enough to have many friends, so she stays mainly to herself.
The FNP understands that, according to Erikson’s developmental stages, this patient may have an unresolved developmental
task of:
early childhood
school-age childhood
adolescence
infancy
Correct answer:
school-age childhood
60
The central task of the school age child is “industry versus inferiority.” Unresolved conflict of this stage has manifested itself
in this patient by withdrawal from peers and a sense of being mediocre.
Question 55
An FNP believes that families develop at a different rate and if one family member is dysfunctional, the rest of the family is
affected negatively. Which of the following health concepts does this reflect?
health belief model
self-efficacy theory
systems theory
family systems theory
Correct answer:
family systems theory
The family systems theory is derived from the systems theory. It holds that families develop at a different rate and if one
family member is dysfunctional, the rest of the family is affected negatively. For example, if a member is an alcoholic, the
entire family becomes dysfunctional.
Question 56
Sadavoy (1987) identified critical geriatric age-related stresses. These include all of the following EXCEPT:
physical disability
loss of youthful appearance and beauty
loss of mental acuity
confrontation with death
The Correct answer is:
Loss of mental acuity
Although some elderly individuals have significant mental decline, this is not always the case. Further, confrontation with
death, physical disability and loss of youthful appearance and beauty are part of the critical geriatric age-related stresses
identified by Sadavoy, as well as interpersonal loss and loss of social support, loss of strength, forced reliance on caregivers,
change in social role and change in living arrangements.
Question 57
The leading cause of death in infants under one year of age is which of the following?
unintentional injuries
congenital anomalies
short gestation
SIDS
Correct answer:
congenital anomalies
The leading cause of death in infants under one year of age is congenital anomalies. Unintentional injuries are the leading
cause of death for those over one year of age to 44 years of age.
Question 58
The FNP has an African-American female patient who is 75 years old. She complains of a gradual loss of her peripheral
vision. Examination indicates increased IOP. Which of the following is the most likely cause of her symptoms?
subconjunctival hemorrhage
primary closed-angle glaucoma
primary open-angle glaucoma
macular degeneration
Correct answer:
primary open-angle glaucoma
Primary open-angle glaucoma has a gradual onset of increased IOP due to blockage of drainage of the aqueous humor. The
optic nerve undergoes ischemic damage resulting in permanent visual loss. It is the most common type of glaucoma and is
most commonly seen in elderly patients, especially those of African background or diabetics.
Question 59
61
Part of the nurse practitioner's role is to understand communication styles. If you are counseling an elderly person, it is
understood that they have an alteration in sensory-perceptual function of hearing. Which of the following would be a good
strategy to implement during a health promotion visit?
You should stand behind the patient when speaking.
You should speak in a tone that does not include shouting.
You should use typical complex sentences to prevent insulting the patient.
You should increase the pitch of your voice.
The Correct answer is:
You should speak in a tone that does not include shouting
Shouting increases the pitch of the voice. Due to presbycusis (hearing loss in the elderly), high-pitched consonant sounds are
the first to be affected. You should always face the client when speaking and use simple understandable language.
Question 60
Treatment options for typical cases of condyloma acuminatum would include all but which of the following?
patient-administered Aldara
cryotherapy
trichloroacetic acid
surgery
Correct answer:
surgery
Treatment options for condyloma acuminatum, the verruciform lesion seen in genital warts include: oliflox, imiquimod,
trichloroacetic acid and cryotherapy. Surgical intervention is reserved for complicated, recalcitrant lesions.
Question 61
You are seeing a 42-year-old female for her initial examination. She has no complaints but needs a Papanicolaou test. Her
blood pressure is 148/96 mm Hg and her body mass index is 32. The rest of her examination is unremarkable. Your BEST
action is to:
Arrange for at least two additional BP measurements during the next 14 days.
Start her on an antihypertensive agent.
Give her subinguinal nitroglycerin immediately.
Advise her to exercise, diet, and reduce her sodium intake.
The Correct answer is:
Arrange for at least two additional BP measurements during the next 14 days
While advising her to exercise, diet, and reduce sodium is good advice, the best action is to monitor her blood pressure to see
if she needs an antihypertensive agent. You would not start one today, since this is her first elevated blood pressure (BP)
reading at your clinic. Subinguinal nitroglycerin is not warranted.
Question 62
The production of sperm usually begins during:
the eighth week of gestation
the beginning of puberty
the end of puberty
around age 21
The Correct answer is:
The end of puberty
Between the ages of 9 and 12 years, the gonads produce more of the sex hormones, which triggers sexual maturation in boys,
or puberty. The approximate age is 11 and it lasts for 2 to 3 years, ending with the first ejaculation that contains mature sperm.
Question 63
Which of the following is NOT true of communication with African Americans?
Prolonged eye contact may be interpreted as rudeness.
Nonverbal communication is important.
Personal questions on the initial visit are views as intrusive.
Head nodding always means agreement.
Correct answer:
Head nodding always means agreement
Head nodding does not always mean understanding or agreement with the African American patient. The other choices are all
true of this population. Large extended family networks are important and older adults are respected with this culture.
Question 64
62
The Scope of Practice for the Nurse Practitioner is an important concept of the professional role. Which of the following is
NOT true regarding it?
Many organizations have completed role delineation studies, which attempt to qualify the core behaviors that all advanced
practice nurses must possess.
The scope of practice for pediatric nurse practitioners (PNP) is the same as that for gerontological nurse practitioners (GNP).
General scope of practice is specified in many published professional documents like the Scope and Standards of Advanced
Practice Registered Nursing, 1996.
Broad variations exist from state to state concerning the Scope of Nurse Practitioner Practice.
The Correct answer is:
The scope of practice for pediatric nurse practitioners (PNP) is the same as that for gerontological nurse practitioner (GNP)
There are role delineation studies which attempt to qualify the core behaviors that all nurse practitioners must possess. The
scope of the PNPs practice is different than that of the GNPs practice.
Question 65
Bony nodules on the distal interphalangeal joints are which of the following?
uric acid crystals
Bouchard’s nodes
Heberden’s nodes
meniscus nodes
Correct answer:
Heberden’s nodes
Heberden’s nodes are on the distal interphalangeal joints (DIP). Bouchard’s nodes are on the proximal interphalangeal joints
(PIP).
Question 66
Which of the following drugs is a drug that is injected directly into the penis for correction of erectile dysfunction?
alprostadil
Muse
taldalafil
vardenafil
Correct answer:
alprostadil
Alprostadil (Caverject) causes vasodilation and is highly effective as a treatment for erectile dysfunction. Muse is a pellet
inserted into the urethra and can achieve the same effect. Taldalafil (Cialis) and vardenafil (Levitra) are oral treatments.
Question 67
When doing an assessment on a 6-year-old male child with a complaint of sore throat, the nurse practitioner notices a finding
of the nasal mucosa and understands that:
the nasal mucosa is redder than the oral mucosa
nasal mucosa is pale and translucent in appearance
nasal mucosa appears pink and boggy without exudate
nasal mucosa appears dark pink with water secretions
The Correct answer is;
The nasal mucosa is redder that oral mucosa
Nasal mucosa is redder than the oral mucosa and increased redness appears with an infection. Pale, boggy turbinates along
with watery secretion often occur with allergic rhinitis. Normal secretion is mucoid whereas purulent, crusty, or bloody
discharge is abnormal.
Question 68
What is the term, defined in federal law, for a process in which a nurse practitioner works with a physician to deliver
healthcare services within the scope of the practitioner’s professional expertise, with medical direction and appropriate
supervision?
partnership
association
collaboration
affiliation
Correct answer:
collaboration
63
The correct term is collaboration. Although state law either does not require collaboration or calls for collaboration and defines
collaboration without using the term supervision, federal law requires, through its definition of collaboration, “supervision.”
Question 69
You are treating an adult male patient who presents with acute onset of unilateral inflammation, pain, and erythema of the first
metatarsophalangeal (MTP) joint. You understand that this could be:
osteoporosis, fibromyalgia, and cellulitis
gout, cellulitis, and osteoporosis
septic arthritis, rheumatoid arthritis, osteoarthritis
cellulitis, rheumatoid arthritis, and gout
Cellulitis, rheumatoid arthritis and gout
Symptoms of erythema, edema, and pain of the first MTP joint are a common presentation for gout and cellulitis present with
warm erythematous, painful areas of the skin. Symptoms present in rheumatoid arthritis (RA) are similar but the inflammation
of RA is usually symmetric. Osteoporosis occurs more in postmenopausal women due to bone loss. Osteoarthritis presents as
pain and stiffness with decreased range of motion, stiffness in the morning, and occasional joint effusions. With septic
arthritis, joint pain and inflammation accompany systemic symptoms of fever and chills.
Question 70
There are bioethical practice dilemmas that are best described as situations in which the proposed treatment alternatives are
which of the following:
not appealing to involved parties
less-than-perfect approaches to the situation
ranked from the most to the least acceptable
lacking acceptance by anyone
The Correct answer is:
Less-than-perfect approaches to the situation
With bioethical dilemmas, the proposed solutions are not perfect and therefore create some aspect of moral conflict.
Question 71
If the FNP is giving the Mini-mental exam (MME) to an elderly patient, which subject area is being tested when the FNP asks
the patient to spell a word backwards?
attention and calculation
orientation
recall
sentence writing
Correct answer:
attention and calculation
The MME is a brief screening exam to assess for dementia. Five subject areas are tested. The area tested by asking the patient
to spell a word backwards is attention and calculation.
Question 72
Which theory holds that a person who believes that he can succeed in performing an action that will result in a positive
outcome is more likely to perform the healthier behavior?
Family Systems Theory
Systems Theory
Health Belief Theory
Self-Efficacy Theory
The Correct answer is:
Self-Efficacy Theory
The Family Systems Theory holds that families develop at different rates and if one member is dysfunctional, the rest are
affected negatively. The Systems Theory holds that all parts of a system are interrelated and dependent on each other. The
Health Belief Theory holds that the person who feels susceptible to the disease and believes that he will benefit from changing
his behavior and is more likely to perform the healthier behavior
Question 73
What is the simplest screen for nutritional adequacy in geriatric patients?
Ask about food choices, likes and dislikes.
Measure their tolerance of dairy products.
64
Measure their weight
Obtain a 72 hour exercise diary
The Correct answer is:
Measure their weight
In older adults, serial weight is the simplest assessment screening for monitoring and tracking an individual’s nutritional
status. With elderly patients, unintentional weight loss, especially when there is not a known cause, is a sign of concern, as
weight loss is typically not a normal part of the aging process. Therefore, further evaluation of a patient who has sustained
weight loss for conditions that may hinder an elderly patient’s ability to thrive, such as depression, cognition, malnutrition,
may be warranted.
Question 74
The nurse practitioner gives a patient a sample package of antibiotics for her to take at home three times a day for the next
seven days. What is this called?
caring
administering
dispensing
integrative medicine
The Correct answer is:
Dispensing
Dispensing is the legal right to select (from stock medications) and label a medication to be self-administered by a patient.
Most states allow nurse practitioners to dispense medications provided that they are prepackaged. Caring is not associated with
medication dispensing but is a part of nursing. Administering refers to the act of giving a single dose of a medication to the
patient while they are in the office.
Question 75
First generation antihistamines readily cross the blood-brain barrier. Which of the following drugs is a first generation
antihistamine?
loratadine
diphenhydramine
desloratadine
levocetirizine
Correct answer:
diphenhydramine
Diphenhydramine (Benadryl) is a first generation antihistamine. Another first generation antihistamine is chlorpheniramine
(Chlor-Trimeton). The other choices are all second generation antihistamines.
Question 76
The FNP has a 25-year-old female patient who presents with a profuse milk-like vaginal discharge and she complains of a
“fishy” vaginal odor. The vulva is not red nor is it itchy. This patient most likely has which of the following vaginal
infections?
Trichomonas vaginitis
bacterial vaginosis
Candidal vaginitis
atrophic vaginitis
Correct answer:
bacterial vaginosis
The most likely diagnosis of these four choices is bacterial vaginosis. Labs will show clue cells (large numbers of bacteria on
the cell surface) and the Whiff test will show a positive pH that is > 4.5 alkaline.
Question 77
The FNP has a 35-year-old male patient who has been smoking since he was 15 years old. She wants to make a brief
intervention using the “5 As.” The “5 As” are which of the following?
ask, advise, assess, assist, arrange
ask, advocate, assess, assist, arrange
ask, advise, assess, aid, arrange
ask, advise, assist, arrange, advocate
65
Correct answer:
ask, advise, assess, assist, arrange
The “5 As” should be employed with all tobacco users, including individuals with no current desire to quit because this can
serve as a motivating factor in future attempts to quit tobacco use. The “5 As” are: Ask about tobacco use; Advise to quit;
Assess willingness to make an attempt to quit; Assist in quit attempt; Arrange follow-up.
Question 78
When discussing dependency with a patient in the clinic, the nurse practitioner understands that:
Treatment is usually simple and requires short-term therapy.
Dependency becomes addiction when there is loss of control (compulsivity) and obsession or preoccupation with the activity.
The action began involuntary and remains so.
Dependency and addiction are easy to treat with medications.
The Correct answer is:
Dependency becomes addiction when there is loss of control (compulsivity) and obsession or preoccupation with the activity
Treatment is complex and require long-term therapy. The action is begun voluntarily but, through repetition, becomes
involuntary. Dependency and addiction are difficult to treat and usually medications alone do not help.
Question 79
Which of the following treatments is recommended for mild persistent asthma?
albuterol metered dose inhaler
albuterol metered dose inhaler plus low-dose steroid metered dose inhaler
albuterol metered dose inhaler plus low-to-medium-dose steroid metered-dose inhaler mixed with salmeterol plus leukotriene
inhibitor.
none of the above
Correct answer:
albuterol metered dose inhaler plus low-dose steroid metered dose inhaler
This would be the recommended treatment for a person with mild persistent asthma. For mild intermittent asthma, the
albuterol metered dose inhaler is all that is recommended, and for moderate persistent asthma, albuterol metered does inhaler
plus low-to-medium-dose steroid metered-dose inhaler mixed with salmeterol plus leukotriene inhibitor is indicated.
Question 80
Which of the following statements about breach of privacy is true?
It is an intentional tort.
It cannot be the basis of a malpractice suit.
It is not a violation of state law.
It is not a violation of federal law.
Correct answer:
It is an intentional tort.
Breach of privacy is an intentional tort. There are many ways to breach a patient’s privacy. It can amount to malpractice in
some cases.
Question 81
Medicaid is a federal program administered by the states. All of the following statements regarding Medicaid are true
EXCEPT:
Medicaid pays NPs 70% to 100% of the fee-for-service rates set for physicians by state Medicaid agencies.
Federal law controls the rates paid by Medicaid.
Medicaid reimbursement generally is lower than the rate paid by commercial insurers.
Many states have applied to the federal government for a Medicaid waiver in order to administer Medicaid in ways that differ
from federal laws and regulations.
Correct answer:
Federal law controls the rates paid by Medicaid.
This is incorrect. Federal law does not control the rates paid by Medicaid. State law controls the rates paid by Medicaid.
Question 82
A 45-year-old man comes to the FNP wanting to have an evaluation for disability under the Medicare program. What should
the FNP tell this patient?
66
I can do an evaluation and determine whether you have a disability under Medicare
The physician must do the evaluation but I can handle the determination of disability under Medicare.
I can do an evaluation but I cannot legally determine whether you have a disability under Medicare.
You must see a physician. I can do nothing for you.
Correct answer:
I can do an evaluation but I cannot legally determine whether you have a disability under Medicare.
The FNP can do a disability evaluation. The FNP does not have the legal authority to determine disability under the Medicare
program.
Question 83
The ecological model of growth and development that is a person-place-process model with microsystems, mesosystems,
exosystems and macrosystems was developed by which of the following theorists?
Arnold Sameroff
Michael Rutter
Emmy Werner
Urie Bronfenbrenner
Correct answer:
Urie Bronfenbrenner
Urie Bronfenbrenner published this theory in a 1979 publication – Ecology of Human Development: Experiments by Nature
and Design. Among other things it holds that understanding how and why a child changes or stays the same over time requires
examination, not only of the child’s emerging capabilities, but also of the quality of the settings where the child spends time.
Question 84
The nurse practitioner (NP), who has practiced for many years, has decided to open her own clinic. She has purchased
equipment and signed a lease on a building. This NP is a(n):
intrapreneur
networker
independent nurse practitioner
entrepreneur
The Correct answer is:
Entrepreneur
Someone who assumes the financial and personal risks of owning and operating their own business is an entrepreneur. An
intrapreneur is someone who is able to carve out a specialty role within an existing organization.
Question 85
In terms of business organization types for nurse practitioner practice, which of the following has its own identity and is a
separate legal entity?
corporation
partnership
LLC
sole proprietorship
Correct answer:
corporation
A corporation is a business entity with its own identity. Although one individual may be the sole stockholder, director and
officer, the corporation is nevertheless a separate legal entity.
Question 86
Which of the following is NOT true in regard to DEA registration for a nurse practitioner?
The DEA oversees NPs’ prescribing of controlled substances.
An NP must register with the DEA to prescribe controlled substances.
The NP must use the DEA number obtained from the DEA on prescriptions for scheduled drugs.
The NP does not need a state controlled substances license.
Correct answer:
The NP does not need a state controlled substances license.
This is not true. Federal registration is based on the applicant’s complying with state and local laws. If a state requires a
separate controlled substances license, an NP must obtain that license and submit a copy with the application for a DEA
67
number.
Question 87
In treating a person with multiple sclerosis, which of the following drugs would be the LEAST likely to be used because it
carries a warning about progressive multifocal leukoencephalopathy, a destructive brain infection?
Betaseron
Tysabri
Avonex
Novantrone
Correct answer:
Tysabri
Natalizumab (Tysabri) is a monoclonal antibody with considerable clinical efficacy in treating MS. However, it carries a
warning about progressive multifocal leukoencephalopathy, a destructive brain infection that is associated with its use.
Question 88
The American Academy of Pediatrics recommends infants to be fed whole cow's milk at what age?
6 months of age
8 months of age
12 months of age
18 months of age
The Correct answer is:
12 months of age
Since cow's milk has a low iron content, it is recommended that infants are not started on it until around age one. Additionally,
formula and breast milk is fortified with protein, linoleic acid, and vitamin E whereas whole cow's milk does not contain
adequate amounts of these substances.
Question 89
A 16-year-old female patient inquires about douching. You are counseling on birth control, safe sexual practices and hygiene.
What instruction regarding douching should the nurse practitioner include?
Hypoallergenic douches include flavored or perfumed types.
Douching during menstruation is a safe effective practice.
Douching removes natural mucus and upset normal vaginal flora.
Daily douching is important if the patient has copious vaginal discharge.
The Correct answer is:
Douching removes natural mucus and upsets normal vaginal flora
Douching is never necessary because it upsets the normal vaginal flora and disrupts pH. Douching is contraindicated during
menstruation due to "retrograde menstruation," a potential precursor to endometriosis. Further, a patient with copious vaginal
discharge needs to be evaluated for an infection.
Question 90
Medical records must be kept safe. It is essential that each healthcare organization establish an effective procedure for
safeguarding them. The following are all true statements regarding medical records EXCEPT:
Medical records should be stored in a secure, restricted-access location.
Competent medical records personnel should review every record before it is examined by the patient or patient representative.
An original medical record should be sent with the patient to specialist appointments and for their attorney to review as
necessary.
Anyone who is not an authorized employee or staff member should not be allowed to examine a medical record.
The Correct answer is:
An original medical record should be sent with the patient to specialist appointments and for their attorney to review as
necessary
An original medical record should never be sent with the patient to a specialist’s appointment. The original medical record
remains on the facility’s premises unless the organization has received a Subpoena Duces Tecum, court order or a subpoena
and a court order to bring the medical record to a court of law. Alternatively, if the medical record has reached the state and
federal law’s identified retention period, the original medical record can be stored offsite or destroyed.
If the patient needs information from the medical record to take to the specialist, the nurse practitioner’s office can make a
copy of what is needed for the referral and send the photocopies of the medical. This process holds true for an attorney. The
attorney will only receive copies of the original medical record. Or, if the patient signs appropriate consent forms, the attorney
can come to the medical office and review the original medical record.
68
Also, the healthcare provider, nurses or any other staff cannot take original medical records home or to another facility to
complete charting or other documentation in the medical record. All charting has to be completed in the facility where the
original medical exists. By removing the original medical record from the facility, it can be lost, accidentally destroyed or
even stolen, for instance. This can lead to breaches of confidentiality and privacy laws, not to mention it is hard to reproduce
an original medical record if it is lost.
In the case of office space restrictions, some physician offices and hospitals may not have sufficient amounts of space to store
all of their medical records. Therefore, storage or retention of the original medical records becomes an issue. There are many
options to storing medical records including storing them offsite, microfilming, scanning the medical records and storing them
on CD ROM’s, disks and more. In this case, the nurse practitioner will need to review state and federal laws, as the laws
govern how medical records can be retained and the compliance guidelines required if the physical, original medical record
has to be stored offsite. Certain medical record documents are permanently kept, such as records of surgical procedures and
master patient indexes. Then, depending on state laws, diagnostic images, for instance x-ray films, can be kept for 5 years
before the images are archived or destroyed. Destruction of medical records are also dictated by federal and state laws. Some
destruction methods include shredding the medical record, for example.
Question 91
You are assessing a 2 week old infant for reflexes. Which of the following is NOT an expected reflex in this age infant?
Moro reflex
Stepping reflex
Tonic Neck reflex
Vertical reflex
The Correct answer is:
Vertical Reflex
The moro reflex (startle reflex) results when a sudden loud noise causes symmetric abduction and extension of the arms
followed by adduction and flexion of the arms over the body. The stepping reflex occurs when the baby is upright and the
dorsal foot is placed on the table causing a simulated stepping motion. The tonic neck reflex (fencing reflex) results when the
infant turns the head to one side with the jaw over the shoulder causing the arm and leg on the side where the head is turned to
extend. There is no vertical reflex.
Question 92
What act states individuals can make decisions about medical care, including the right to accept or refuse any type of medical
or surgical interventions?
The Americans With Disabilities Act
The Patient Self-Determination Act
The Omnibus Budget Reconciliation Act
The Veterans Readjustment Assistance Act
The Correct answer is:
The Patient Self-Determination Act
On December 1, 1991, federal legislation known as the Patient Self-Determination Act (PSDA) took effect in healthcare
organizations. It requires these institutions to develop and maintain written policy and procedures to provide written
information to adults for who care is provided.
Question 93
A 29-year-old Caucasian male calls your office. He tells you he just came in from the woods and discovered a tick on his
upper right thigh. He reports self-removal of the tick and now the area is slightly red. What should you advise him to do?
He should come to the office for a ceftriaxone (Rocephin) injection.
He should be prescribed doxycycline.
He needs no treatment.
He needs a topical scrub to prevent Lyme Disease.
The Correct answer is:
He needs no treatment
To develop Lyme Disease from a tick bite, many factors must be present. The tick must belong to Ixodes species and must
have been attached for at least 48 hours before the disease can spread. There is no need for prophylactic treatment in this case
because the tick has not been present long enough.
Question 94
You are caring for an elderly patient who has had a cerebrovascular accident (CVA) and has urinary incontinence. The family
should be taught to:
insert a Foley catheter
establish a scheduled voiding pattern
69
restrict fluid intake
reposition the patient often to reduce the discomfort of urgency
The Correct answer is:
Establish a scheduled voiding pattern
The patient needs a reestablished pattern for regularity to assist in maintaining bladder control. A Foley catheter would expose
the patient to infection and fluids should not be restricted.
Question 95
The majority of breast cancers occur in which area of the breast?
beneath the nipple and areola
upper outer quadrant
lower outer quadrant
upper inner quadrant
The Correct answer is:
Upper outer quadrant
The upper outer quadrant is the most common site for breast cancer. The second most common site for breast cancer is
beneath the nipple and areola.
Question 96
The theory that a person who believes that he can succeed in performing an action that will result in a positive outcome is
more likely to perform the healthier behavior is which of the following?
Systems Theory
Self-Efficacy Theory
Health Belief Model
Family Systems Theory
Correct answer:
Self-Efficacy Theory
The Self-Efficacy Theory holds that a person who believes that he can succeed in performing an action that will result in a
positive outcome is more likely to perform the healthier behavior. This concept was put forth by Albert Bandura in the 1970s.
Question 97
The nurse practitioner is treating a 42-year-old obese Caucasian male who has an allergy to sulfa. She understands that which
of the following medications is contraindicated in this patient?
ramipril
hydrochlorothiazide
verapamil
trimethoprim
The Correct answer is:
Hydrochlorothiazide
Hydrochlorothiazide has a sulfonamide ring in its chemical structure, generally referred to as "sulfa." This can initiate an
allergic reaction in a patient with a sulfa allergy. The other answer choices, verapamil, ramipril and trimethoprim, are
medications that can be used without concern because they have no sulfonamide component.
Question 98
Which of the following best describes a "co-payment"?
A flexible dollar amount that a person chooses to pay the provider.
A proportion of the total charges for medical services that an insured person must pay the provider.
A fixed dollar amount that a person insured through a health maintenance organization (HMO) or preferred provider
organization (PPO) must pay the provider.
The amount the provider must pay to be allowed to participate with HMOs and PPOs.
The Correct answer is:
A fixed dollar amount that a person insured through an health maintenance organization (HMO) or preferred provider
organization (PPO) must pay the provider
A proportion of the total charges for medical services billed to a patient or paid by them is "co-insurance." The co-payment,
for example $5, $10, or $20, must be paid at the time of the clinic visit in order to receive care.
Question 99
Which of the following ethical concepts for Professional Nurses is the duty to help others?
non-malfeasance
accountability
70
human dignity
beneficence
The Correct answer is:
Beneficence
Beneficence is the duty to help others. Non-malfeasance is the duty to avoid harm to others. Accountability is being
responsible for one's own actions. Human dignity is respect for the patient.
Question 100
According to the American Cancer Society Guidelines for the Early Detection of Cancer, yearly mammograms are
recommended for women who are:
in good health and are 35 years of age or over
in good heath and are 40 years of age or over and continuing for as long as a woman is in good health
in good heath and are 35 years of age or over and continuing every 2 or 3 years if the mammogram is negative
in good heath and are 40 years of age or over and continuing every 2 or 3 years if the mammogram is negative
Correct answer:
in good heath and are 40 years of age or over and continuing for as long as a woman is in good health
The American Cancer Society recommends yearly mammograms starting at age 40 and continuing for as long as a woman is
in good health. The American Cancer Society also recommends that some women -- because of their family history, a genetic
tendency, or certain other factors -- be screened with MRI in addition to mammograms. (The number of women who fall into
this category is small: less than 2% of all the women in the US.)
Test 3 ancc
Question 1
Head circumference should be measured until a child has attained what age?
12 months
18 months
24 months
36 months
The Correct answer is:
24 months
A child's head is usually measured at each well child visit until he or she is 2 years old. During the first three years of life, the
head grows and its growth should be monitored for adequacy. These measurements are recorded on a growth chart so that
changes can be followed and the percentile for age and rate of growth can be determined.
Question 2
A patient is in the office who was bitten by her husband during an assault. There are several bite marks and lacerations on the
forearms. You treat her by suturing the lacerations although it is contraindicated because of the highly infectious nature of
human bites. What is this described as?
This is malpractice.
This is poor judgment and malpractice.
This is negligence.
This is an unfortunate situation.
The Correct answer is:
This is negligence
The suturing of the patient's lacerations is negligence, not malpractice. Since human bites are highly infectious, they should
not be sutured. Negligence occurs when you fail to exercise the care that is reasonable. Injury does not have to occur for
negligence.
Question 3
In a 74-year-old Caucasian male who has had Type II Diabetes for ten years, a hypoglycemic episode is more likely to exhibit:
dizziness and weakness
contusions
symptoms of compulsive reactions
delusional hyperactivity
The Correct answer is:
Dizziness and weakness
Older adults are more likely to have deleterious consequences than younger adults. Younger adults present with tremors,
sweating and adrenergic symptoms. Then, older adults tend to have neurological symptoms, like dizziness and weakness.
71
Question 4
A patient receives another person's medication in error. This was done by an LPN in the office and did not result in a lifethreatening situation for the patient. What is an appropriate way for the nurse practitioner to document this?
Patient received wrong medication. Incident report was filed. LPN was disciplined.
X mg of Y drug was administered to the patient. No adverse effects were noted. Physician notified.
LPN inadvertently administered Y drug to the wrong patient. Supervisor notified. Family threatening litigation.
Patient was given X mg of Y drug in error.
The Correct answer is:
X mg of Y drug was administered to the patient. No adverse effects were noted. Physician notified
Documenting, "X mg of Y drug was administered to the patient. No adverse effects were noted. Physician notified," is the
most accurate note because it is factual. Also, with this noted documentation, the writer is avoiding blame or assuming
liability. The other notes, identified in the remaining answer choices, would be "red flags" mentioning error, litigation, and
incident report.
Question 5
When treating a patient with a topical agent, you anticipate the greatest rate of absorption when it is applied to:
face
ankles
palms of the hands
soles of the feet
The Correct answer is:
face
Certain parts of the body, notably the face, axillae, and genital area, are quite permeable, allowing for greater absorption of
medication. Less permeable areas are the extremities and trunk. The palms of the hands and soles of the feet create a barrier so
little absorption occurs there.
Question 6
You are working in a rural health clinic and have a hunter come into the office. A 26-year-old man reports that while walking
in the woods he was bitten on the leg by a raccoon. Upon examination, you find a 1 cm deep wound on his left lower posterior
leg and the wound is oozing bright red blood. Your next best action is to:
Give rabies immune globulin and rabies vaccine.
Suture wound after proper cleansing.
Administer high-dose parental penicillin.
Initiate antibacterial prophylaxis with amoxicillin.
The Correct answer is:
Give rabies immune globulin and rabies vaccine
All bites should be considered to carry certain infectious risk. Nocturnal wild animals are know to carry rabies. These include
raccoons, foxes, squirrels, skunks, bats, and woodchucks.
Question 7
In diagnosing a toothache (pulpitis), the nurse practitioner has knowledge that all of the following are necessary except:
Systemic manifestations do not occur from dental caries and pulpitis.
You should inquire about fever and chills.
Dental caries are the most frequent type of injury that causes pulpitis.
There is diverse flora involved in the infectious process, including gram-positive anaerobes and bacteroides.
The Correct answer is:
Systemic manifestations do not occur from dental caries and pulpitis
Many patients present in the clinic setting with a toothache or related symptoms. Systemic manifestations often occur and they
include lymphadenopathy, malaise, fever, and pain. The remaining answer choices are true regarding this condition.
Question 8
In terms of cranial nerve testing, which cranial nerve (CN) is associated with facial movements?
CN 1
CN 3
CN 7
CN 11
72
Correct answer:CN 7
Cranial nerve 7 is associated with facial movements. This includes puffing cheeks, raising eyebrows and smiling
Question 9
In terms of the scope of practice for an NP, which of the following statements is incorrect?
The scope of practice defines a specific legal scope determined by state statutes, boards of nursing, educational preparation
and common practice within a community.
General scope of practice is specified in many published professional documents.
Prescriptive authority is recognized as within the scope of practice for nurse practitioners in all 50 states.
Scope of practice is always defined by state statutes enacted by the state legislature.
Correct answer:
Scope of practice is always defined by state statutes enacted by the state legislature.
Some states define scope of practice by state statutes enacted by the state legislature. In other states, the legislature gives the
board of nursing the authority to define the scope of NP practice. Either way is enforceable.
Question 10
Which of the following is an accurate statement regarding the Medicaid health program?
This program is funded with premiums from the participants.
This program is basically the same in every state.
This program offers unlimited number of adult visits to the healthcare provider.
This program is funded by both the state and federal governments.
The Correct answer is:
This program is funded by both the state and federal governments
The Medicaid program is state operated and varies from state to state. Participants generally do not have to pay premiums.
Also, most states have limits on the number of adult visits allowed.
Question 11
Which stage in Erikson's Model of Psychosocial Development is where failure leads to feelings of shame and doubt?
Stage I Trust
Stage II Autonomy
Stage III Initiative
Stage IV Industry
The Correct answer is:
Stage II Autonomy
With Stage I Trust, failure causes mistrust. With Stage II Autonomy, failure leads to feelings of shame and doubt. With Stage
III Initiative, failure leads to feelings of guilt. With Stage IV Industry, failure leads to feelings of inferiority. With Stage V
Identity, failure leads to role confusion. With Stage VI Intimacy, failure leads to additional role confusion.
Question 12
In documenting a patient’s history, which of the following elements is a chronological description of illness from the first sign
and/or symptom?
CC
HPI
ROS
PFSH
Correct answer:
HPI
HPI represents “history of present illness.” It is a chronological description of illness from the first sign and/or symptom or
from the previous encounter to the present. It includes the following elements: location, quality, severity, duration, timing,
context, modifying factors and associated signs and symptoms. A brief HPI consists of one to three of the elements of the HPI.
Question 13
The most common cancer for all ages and genders (not considering mortality) is which of the following?
lung cancer
leukemia
skin cancers
colon cancer
73
Correct answer:
skin cancers
Skin cancers are the most common cancer for all ages and genders. Squamous cell cancer is the most common skin cancer.
Lung cancer is the leading cause of cancer deaths. Leukemia is the most common cancer and cancer death in children ages 0 –
19 years.
Question 14
Office laboratories are subject to federal oversight under the Clinical Laboratory Improvement Amendments (CLIA). These
laboratories are subject to state and federal inspection and approval; however, office labs may obtain exemption from
inspection if they do a limited number of specific laboratory tests. Which of the following is NOT one of the tests that an
office laboratory with an exemption may do?
fecal occult blood (hemoccult)
HIV test
urine pregnancy test
blood glucose
Correct answer:
HIV test
Office laboratories are subject to state and federal inspection and approval. In offices where laboratory tests are limited to
fecal occult blood, urine pregnancy test, blood glucose, urinalysis and office microscopy, practices may obtain exemption
from inspection. HIV testing is not included in the list.
Question 15
A 23-year-old thin Caucasian female presents in the clinic with a new diagnosis of Crohn's disease. She has recently started
medication prescribed by her gastroenterologist and asks you to tell her what else to do. You know that proper
nonpharmacological management of this condition includes all of the following EXCEPT:
Avoid dairy products that contain lactose if they exacerbate symptoms.
A low protein is recommended.
Stress management is important, including relaxation techniques.
A low-roughage diet is recommended during acute exacerbations.
Correct answer:
A low protein diet is recommended.
A diet high in protein and vitamins, particularly B12, is recommended for the patient with Crohn's disease.
Question 16
When lymph nodes are referred to as shotty, the nurse practitioner understands that they are:
small and pellet-like
discrete and cystic
tender, mobile, and >5 mm
irregular, soft, and fixed to surrounding tissue
The correct answer is:
Small and pellet-like
Shotty lymph nodes are small, pellet-like nodes that are movable, cool, nontender and discrete. These lymph nodes range in
size up to 3 mm in diameter.
Question 17
Which of the following questions is NOT related to assessing the psychosocial aspect of a female patient?
When was your last menstrual period?
How frequently do you consume alcohol?
Are you sexually active?
Have you ever smoked?
Correct answer:
When was your last menstrual period?
74
This is not related to psychosocial development or needs. It is a physiological question. The other question relate to the
psychosocial history of the patient.
Question 18
When discussing advance directives with a family member of a terminal patient, it is important to include:
Their provisions go into effect when the patient has become conscious.
Living wills are verbal agreements between the patient and healthcare provider.
Only the patient can make decisions about their care, even when they are incompetent.
The patient can create a healthcare durable power of attorney when he or she is still competent.
The Correct answer is:
The patient can create a healthcare durable power of attorney when he or she is still competent
A competent individual establishes a written advance directive to identify his or her instructions and preferences for medical
care. When the patient is unable to make decisions regarding medical care, such as when the patient has become incompetent
or unconscious, the designated representative or family member and healthcare provider makes decisions based on the advance
directive’s provisions. Types of advance directives are durable power of attorney, a living will or a healthcare proxy. With a
durable power of attorney, the patient identifies the person or representative, a healthcare proxy, who will make medical
decisions on his or her behalf. With a living will, the patient has identified specific instructions for the family or representative
and the medical professional to carry out when he or she is unable to make decisions regarding medical care
Question 19
Which of the following hormone therapy agents provides the LEAST hot flash relief for menopausal women?
oral conjugated equine estrogen: 0.625 mg
oral estradiol - 17β: 2 mg
transdermal estradiol - 17β: 0.05 mg
oral estradiol - 17β: 0.25 mg
Correct answer:
oral estradiol - 17β: 0.25 mg
The three most commonly used prescription hormone therapy agents include oral conjugated equine estrogen and oral and
transdermal estradiol - 17β. The amount of hot flash relief that women get from each form and dose differs. Oral estradiol 17β: 0.25 mg gives the least hot flash relief.
Question 20
Which of the following does not play a significant role in the development of varicose veins?
diabetes mellitus
wearing constricting garments
weakness of the walls of the vein
pregnancy
Correct answer:
diabetes mellitus
Diabetes does not play a significant role in the development of varicose veins. An inherited venous defect of either a valvular
incompetence or a weakness in the walls of the vessel likely plays a significant role. In addition, situations that cause high
venous pressure, such as wearing constricting garments and pregnancy, contribute to their development.
Question 21
With the geriatric client, which assessment tool should the nurse practitioner use to evaluate balance and gait problems?
Index of Independence of Activities of Daily Living Scale
Tinetti Balance and Gait Evaluation
Lawton & Brody Balance and Coordination Scale
Instrumental Activities of Daily Living Scale
The Correct answer is:
Tinetti Balance and Gait Evaluation
The Tinetti scale and gait evaluation is an activity-based test that asks the patient to perform tasks, such as sitting and rising
from a chair, turning, and bending. The Index of Independence o Activities of Daily Living helps to identify daily activities
with which the patient needs assistance. The Instrumental Activities of Daily Living Scale assesses complex tasks.
75
Question 22
The FNP has been asked to prepare a talk for elementary school teachers about playground and classroom safety. Which of the
following methods of presenting this information would be most effective?
videos and lecture
lecture only
presentation of case studies
presentation of information followed by small group discussion
Correct answer:
presentation of information followed by small group discussion
Studies show and theories hold that the older the learner, the more increased is the need for self-direction. Therefore sessions
in which the teachers can actively take part would be the most effective way to deal with the issues in question. Adults like to
see themselves as doers.
Question 23
The nurse practitioner has a patient on verapamil (Calan SR) and is monitoring the patient for therapeutic effects. He
understands that with verapamil you should assess for:
decrease in systemic vascular resistance
increase in heart rate
decrease in ventricular premature beats
increase in blood pressure
The Correct answer is:
Decrease in systemic vascular resistance
Calcium channel blockers depress the rate of discharge from the sinoatrial node and conduction velocity through the
atrioventricular node, causing a decrease in heart rate. In addition, they relax the coronary and systemic arteries, producing
vasodilatation and decreased myocardial contractility.
Question 24
A mother brings her infant into the office and you diagnose oral candidiasis. Which of the following is a pharmacological
treatment option for thrush in the infant:
nystatin oral suspension
clotrimazole troches
systemic ketoconazole
oral Diflucan x 1 dose
The Correct answer is:
Nystatin oral suspension
This is the only recommended treatment option for infants. Clotrimazole troches are used in children 3 and older. Systemic
ketoconazole is not recommended for use in children. Oral Diflucan x 1 dose has been used to treat vaginal candidiasis in
teens and adults.
Question 25
Which of the following statements about Pap smears is incorrect?
If endometrial cells are present a referral for biopsy is indicated.
If endocervical cells are missing the Pap smear is incomplete.
There is a high false-negative rate for Pap smears of 15 – 40%.
The Pap smear is the preferred diagnostic test for cervical cancer.
Correct answer:
The Pap smear is the preferred diagnostic test for cervical cancer.
A pap smear is a screening test only. The diagnostic test is the cervical biopsy, done during a colposcopy.
Question 26
The FNP has a female adult patient who tells him that she has lost a large amount of weight in a short amount of time, has
become irritable and anxious and often experiences insomnia. Upon examination the FNP finds a diffusely enlarged thyroid
gland without nodules, and tachycardia. Of the following which is the most likely diagnosis for these symptoms?
Grave’s disease
76
thyroid cancer
hyperprolactinemia
pheochromocytoma
Correct answer:
Grave’s disease
Grave’s disease is an autoimmune disorder of the thyroid gland causing hyperfunction and production of excess thyroid
hormones. There is a higher incidence in females (8:1 ratio). In later stages of Grave’s disease there can also be exophthalmos
(protruding eyeballs).
Question 27
There are theories of how and why a person changes or stays the same over time. One debate on theory involves nature vs.
nurture. Which of the following theorists is part of the nature position on human growth and development?
John Locke
Arnold Gesell
John B. Watson
B.F. Skinner
Correct answer:
Arnold Gesell
Arnold Gesell promoted the maturational-organismic theory that is a nature position (emphasis on heredity and maturational
process). This theory has a biological basis and the organizing principle of the theory is structure (closed system of
transformational rules governing thought).
Question 28
You are treating a pregnant female and she inquires about HIV testing. The nurse practitioner has the understanding that:
It is better performed in the third trimester
It is an "opt-in" approach.
It produces many false positives.
It is recommended by the ACOG.
The Correct answer is:
It is recommended by the ACOG
The American College of Obstetrics and Gynecology (ACOG) recommends an "opt-out" approach to HIV screening in
pregnant patients. This means that HIV will be routinely preformed unless the patient refuses.
Question 29
In terms of Medicaid, which of the following statements is incorrect?
Medicaid pays for health care.
Medicaid pays for nursing home.
Medicaid pays for prescription drugs.
Providers may collect 10% of the charges for services from the patient.
r:
Providers may collect 10% of the charges for services from the patient.
This statement is incorrect. Providers must agree to accept Medicaid payment in full and not collect from the patient or
beneficiaries
Question 30
Which theorist whose theory of human growth and development puts emphasis on learning and environment (nurture) saw a
child as a “tabula rasa” or “blank slate?”
John B. Watson
B.F. Skinner
Albert Bandura
John Locke
Correct answer:
John Locke
John Locke was a proponent of the nurture position of human growth and development that puts an emphasis on learning and
77
environment. He saw a child as a blank slate upon which the effects of learning and environment would be written.
Question 31
There are many barriers to the electronic medical record. The NP understands that which of the following is NOT one of these
issues?
technical matters: uncertain quality, ease of use, lack of integration
potential for streamlining and automating workflow
financial matters: maintenance, upgrades, replacement
doubts of clinical usefulness
The correct answer is potential for streamlining and automating workflow. Streamlining and automating the workflow is
considered a benefit of the electronic medical record, not a barrier. Other barriers include resource issues (training and retraining), certification, security, and ethical matters.
Question 32
Your friend, a nurse practitioner (NP), has been diverting medications from patients. You and the friend work together at a
clinic, both as NPs. What is your professional responsibility?
Report this to the state board of nursing.
Report this to her husband to get her help.
Report this to the police and clinic owner.
You have no professional responsibility regarding others' actions.
The Correct answer is:
Report this to the state board of nursing
You should report unsafe practice to the state board of nursing. The clinic owner should be made aware of this as well. You
are not obligated to notify the police or her family.
Question 33
A 44-year-old Caucasian female is taking lovastatin for hyperlipidemia and complains of muscle aches in her thighs and right
calf for three days without improvement. She has been on the medication for 3 months. What should the nurse practitioner do
first?
Check bilirubin enzymes.
Order a CPK.
Ask about nighttime muscle cramps.
Stop lovastatin immediately.
The Correct answer is:
Order a CPK
This patient has a complaint of myalgias that could be associated with statin use. This patient should be assessed for
rhabdomyolysis by measuring the creatine phosphokinase (CPK) level. If the CPK value is elevated, lovastatin should be
stopped immediately. Nighttime cramps are not associated with statin usage
Question 34
The FNP faces the task of educating a Native American Navajo female about her newly diagnosed type 2 diabetes. The FNP
understands that which of the following is true in regard to discussing this with the patient?
The FNP should avoid direct eye contact with the patient while educating the patient.
The FNP should include the woman’s husband in the education for the reason that the husband makes all of the decisions in
Native American Navajo families.
Native American Navajo families are not accepting of outside health-care practitioners.
none of the above
Correct answer:
The FNP should avoid direct eye contact with the patient while educating the patient.
This is the true statement. Native Americans often avoid direct eye contact and consider it disrespectful or aggressive.
Question 35
The purpose of an OSHA 200 log is to record:
only work-related deaths
dangerous workplace situations
lost work days
occupational injuries and illnesses
78
The Correct answer is:
Occupational injuries and illnesses
The purpose of an OSHA 200 log is to record occupational injuries and illnesses, including mortality reports and lost
workdays, which could also reveal dangerous working conditions.
Question 36
All pregnant women should undergo screening for HBsAg at the first prenatal visit, regardless of HBV vaccine history. This is
because infants who have been infected perinatally with HBV have what percentage chance of developing hepatocellular
carcinoma or cirrhosis in their lifetime?
25%
20%
10%
5%
Correct answer:
25%
Infants who have been infected perinatally with HBV have an estimated 25% lifetime chance of developing hepatocellular
carcinoma or cirrhosis. The HBV vaccine is not 100% effective and a woman could have carried HBsAg since before
pregnancy.
Question 37
Which of the following should be considered for the first-line therapy for prepatellar bursitis to afford significant pain relief?
intrabursal corticosteroid injection
bursal aspiration
NSAIDs
applying ice to the affected area
Correct answer:
bursal aspiration
With prepatellar bursitis, bursal aspiration should be considered as a first-line therapy because this procedure affords
significant pain relief and allows the bursa to reapproximate. In other sites, first-line therapy includes minimizing or
eliminating the offending activity, applying ice to the affected area for 15 minutes at least four times per day, and the use of
NSAIDs. Intrabursal corticosteroid injections are used if conservative measures have not worked after approximately 4 to 8
weeks.
Question 38
The nurse practitioner understands that there are three groups of behaviors that would improve population health. All of the
following are included EXCEPT:
preventative health services (infant care, immunizations, sexually transmitted disease services
health protection activities (toxic agent control, occupational safety, injury prevention)
health promotion behaviors (smoking cessation, stress reduction, improved fitness)
cost means prevention ( avoiding unnecessary testing, eliminating screenings for malignancy)
The Correct answer is:
Cost means prevention ( avoiding unnecessary testing, eliminating screenings for malignancy)
Health promotion includes cost means prevention with an emphasis on containing costs with early detection through testing
and screenings before the disease occurs. The other answers are important behaviors to improve population health.
Question 39
A woman brings her elderly mother to the office for a check-up. When counseling her, you find a need to discuss falls
prevention. The following are all necessary interventions EXCEPT:
Provide an obstacle free, well-lit environment.
Raise chair heights and seat heights as necessary.
The use of rugs.
Avoid soft sole shoes without proper tread.
The Correct answer is:
Use of rugs
"Throw rugs" have been known to cause falls and slips in the elderly. Therefore, rugs should be avoided.
Question 40
79
What medication and dose can be used for abortive therapy in an adult client with symptoms of migraine headache?
ergotamine (Ergostat) 2 mg SL
ketorolac (Toradol) 100 mg IM
sumatriptan (Imitrex) 1 mg IM
amitriptyline (Elavil) 100 mg PO
The Correct answer is:
Ergotamine (Ergostat) 2 mg SL
The dosing guideline for ergotamine (Ergostat) is 2 mg by mouth, sublingually. Further, additional treatment options for a
migraine headache can also include Ketorolac and Sumatriptan, as well as other medications. Depending on the patient's
severity of symptoms and medical history, the following are dosing guidelines providers can use to administer Ketorolac and
sumatriptan. For Ketorolac, the provider can administer the drug intravenously or intramuscularly before administering orally
as a tablet. When taking Ketorolac orally, the dosing guidelines are 30-60 mg. The patient is given an initial first dose of two
20 mg tablets. After the first dose, the patient is given one 10 mg tablet every 4-6 hours for up to 5 days. The dosing for
Ketorolac should not exceed 40 mg per day. Therefore, the answer choice stating "ketorolac (Toradol) 100 mg IM" is
incorrect, as 100 mg is too high of a dosage. Next, Sumatriptan (Imitrex) dosing guidelines include oral (25-200 mg daily) and
subcutaneous injections (6-12 mg) administration, as well as administration of the medication by a nasal spray (5-20 mg as 1
spray in each nostril and then up to 40 mg a day.) Therefore, the answer choice with the dosing of 1 mg IM for sumatriptan
(Imitrex) is incorrect. Then, amitriptyline is not used for abortive therapy.
Question 41
You have a 2-year-old Caucasian child that attends daycare in the clinic. Her mother tells you she has been rubbing her eye.
You notice that the right eye has reddened ejected conjunctiva and yellow discharge. When treating suppurative conjunctivitis,
the nurse practitioner has knowledge that all of the following are causative organisms for bacterial conjunctivitis except:
Haemophilus influenzae
Pseudomonas aeruginosa
Streptococcus pneumoniae
Staphylococcus aureus
The correct answer is:
Pseudomonas aeruginosa
Therapy in conjunctivitis is aimed at eradicating or eliminating the underlying causes. Therefore, accurate diagnosis is critical.
Further, it is important to understand the causative agents for this condition. Pseudomonas aeruginosa does not cause
conjunctivitis. However, the remaining answer choices are all causative organisms.
Question 42
School age children begin to develop permanent teeth. At what age would the first premolar be likely to erupt?
8 - 9 years
7 - 8 years
11 - 12 years
10 - 11 years
Correct answer:
10 – 11 years
The first premolars are most likely to erupt between 10 and 11 years. Central incisors erupt between 7 and 8 years; lateral
incisors between 8 and 9 years; and cuspids between 11 and 12 years.
Question 43
An elderly client with a recent history of a left hemisphere stroke returns to the clinic for a follow-up. The nurse practitioner
expects to see what symptoms?
bilateral weakness of lower extremities
difficulty with speech
left visual field deficit
left sided weakness
The Correct answer is:
Difficulty with speech
The Broca's area, or speech center, is most often located in the left hemisphere. The patient would probably experience
weakness of the right side of the body and not left sided weakness. Then, the patient will experience right-sided visual deficit,
80
and not left visual field deficit.
Question 44
A middle aged postmenopausal woman is in the office with questions regarding hormone therapy. You explain to her that
hormone therapy users may experience:
an increase in breast cancer rates with long-term use
a 10% increase in bone mass
no change in the occurrence of osteoporosis
reduction in high-density lipoprotein cholesterol
answer is:
An increase in breast cancer rates with long-term use
Postmenopausal hormone therapy can help reduce the risk of postmenopausal fracture by 50% by minimizing further bone
loss. The benefits must be balanced against the noted increased risk of breast cancer and other problems.
Question 45
Principles of structural functional theory adapted from Friedman and others include all of the following EXCEPT:
Families are social systems with instrumental and expressive functions.
In optimally functioning families, members take on predictable roles that meet the needs of its members.
Families are composed of small numbers with characteristics of small-group behavior.
Individuals adopt norms and values solely from outside the family system.
er:
Individuals adopt norms and values solely from outside the family system.
This is incorrect. Individuals adopt norms and values, as well as cultural traditions as part of the process of family
socialization.
Question 46
Which of the following suggests that a patient may be abusing alcohol?
rhinophyma, hypotension, peripheral neuropathy
telangiectasias, flat affect, thyroid dysfunction
hepatosplenomegaly, murmur, osteoarthritis
macrocytosis, tremulousness, hypertension
The Correct answer is:
Macrocytosis, tremulousness, hypertension
Findings that would cue the nurse practitioner that a patient was abusing alcohol are tremors, hypertension, rhinophyma,
peripheral neuropathy, telangiectasias or hepatosplenomegaly. The patient may not exhibit all of these symptoms at one time
however. Further, the patient will have macrocytosis because there is a high rate of B12 and folate deficiency among these
patients
Question 47
Which group has the greatest risk for immune system dysfunction with poor nutritional status?
Frail older adults, aged 65 and over
healthy adults, aged 25-49
healthy adolescents and young adults
thriving infants
The Correct answer is:
Frail older adults, aged 65 and over
Frail geriatric patients are at the greatest risk for altered immune function related to nutrition due to multiple factors such as
altered taste and loss of appetite due to chronic disease. Adequate nutrition has been proven through research to improve
immune status.
Question 48
In terms of provider credentialing by an MCO, all of the following may be part of the credentialing process EXCEPT:
collecting data that includes educational, license, malpractice, employment and certification data on each provider
formal issuance of a certification document to the provider
making a site visit
investigating systems for admitting patients
81
Correct answer:
formally issuing a certification document to the provider
Provider credentialing by an MCO does not include formally certifying the provider. It does include all of the other choices,
however.
Question 49
The ability of a patient and family to understand and act on health information is called:
complementary modality
alternative modality
health literacy
health disparity
Correct answer:
health literacy
Health literacy is the ability of a patient and family to understand and act on health information. It is now recognized as one of
the largest contributors to health outcome.
Question 50
Upon seeing a patient in the office that had a recent cervical whiplash injury, the nurse practitioner knows that which of the
following is NOT true regarding this:
It is identifiable on MRI or CT, but not X-ray.
This occurs after a traumatic event.
Occipital pain and headache can occur.
It may be accompanied by severe pain and spasm.
The Correct answer is:
It is identifiable on MRI or CT, but not x-ray
Diagnostic tests, such as a MRI, CT scan or x-ray, do not show signs of a cervical whiplash injury. This type of injury
commonly follows a car accident when a rear collision is involved. Loss of range of motion and delayed pain sensation are
common.
Question 51
Elizabeth Kübler-Ross has identified five stages of grief. In which stage of grief is the person who tells the FNP that if she
could just live to see her granddaughter’s wedding, she would be ready to die?
denial
bargaining
acceptance
depression
Correct answer:
bargaining
This patient is in the bargaining stage of grief. The five stages are: denial, anger, bargaining, depression and acceptance.
Question 52
You are doing a well-child exam on a male immigrant child and make some concerning findings. He has pectus excavatum
and associated scoliosis. You understand that the nurse practitioner should evaluate for:
cardiac compromise
Marfan syndrome
joint deformities
pulmonary dysfunction
The Correct answer is:
Marfan syndrome
If pectus excavatum is associated with scoliosis, the nurse practitioner should evaluate for Marfan syndrome. Marfan
syndrome is a heritable condition that affects the connective tissue. The primary purpose of connective tissue is to hold the
body together and provide a framework for growth and development. In Marfan syndrome, the connective tissue is defective
and does not act as it should. Because connective tissue is found throughout the body, Marfan syndrome can affect many body
systems, including the skeleton, eyes, heart and blood vessels, nervous system, skin and lungs.
82
Question 53
An option to the injectable influenza vaccination is the nasal spray vaccine. It is important that the nurse practitioner
understands to advise a patient:
Its use is limited to children younger than 6 years.
This is the preferred method of immunization for persons who are allergic to eggs or egg products.
It contains a live, attenuated virus.
It is acceptable to use during pregnancy.
The Correct answer is:
It contains live, attenuated virus
The nasal influenza virus contains influenza viruses that are sufficiently weakened as to be incapable of causing disease.
However, the virus is strong enough to stimulate a protective immune response. The influenza virus vaccine is indicated for
individuals aged 2 to 49 years, if the individual is healthy. The vaccine is contradicted in persons with egg allergy and
pregnant women.
Question 54
When assessing color perception with the preschooler, which test should be given?
Bruckner's test
Hirschberg's test
Ishihara's test
Jaeger's test
The Correct answer is:
Ishihara's test
The Ishihara's test is a test for color perception. The Bruckner's test is used to evaluate for the red reflex presence. The
Hirschberg's test is used for the corneal light reflex. The Jaeger's test assesses near vision.
Question 55
Which of the following organizations is not obligated to reimburse nurse practitioners as primary care providers?
Medicare
MCOs
TRICARE
federally funded school-based clinics
Correct answer:
MCOs
NPs are reimbursed for their services as primary care providers under Medicare, Medicaid, Federal Employees Health Benefits
Program, TRICARE (formerly CHAMPUS), veterans’ and military programs and federally funded school-based clinics.
MCOs have frequently excluded NPs from being designated as primary care providers.
Question 56
When treating the patient with COPD exacerbations, the nurse practitioner is aware that certain organisms are often associated
with this process with advanced disease. Of the following, which is one?
Haemophilus influenzae
Staphylococcus aureus
Pseudomonas aeruginosa
Streptococcus pneumoniae
The Correct answer is:
Pseudomonas aeruginosa
Common pathogens usually found in the presence of advanced disease and repeated exacerbations are Enterobacteriaceae spp.
and Pseudomonas spp. Other organisms, such as Haemophilus influenzae and Streptococcus pneumoniae, are seen in the
disease, but not in advanced cases. Staphylococcus aureus is not commonly seen in COPD cases.
Question 57
An adult patient presents with painful red nodules and pustules under his arm. He tells the FNP that some of the lumps have
started to drain pus. The FNP is most likely to diagnose which of the following?
Hidradenitis Suppurativa
impetigo
meningococcemia
herpes zoster
83
Correct answer:
Hidradenitis Suppurativa
Hidradenitis Suppurativa is a bacterial infection of the sebaceous glands of the axilla (or groin) by Gram-positive
Staphylococcus aureus. It is marked by flare-ups and resolution. It can be confirmed by a C&S of the purulent discharge.
Question 58
The FNP has a patient with a gradual onset of numbness starting in the hands and feet. The CBC on this patient reveals a
macrocytic anemia and a peripheral smear shows multisegmented neutrophils. This patient most likely has which of the
following conditions/diseases?
neutropenia
vitamin B12 deficiency
thrombocytopenia
non-Hodgkin’s lymphoma
Correct answer:
vitamin B12 deficiency
Vitamin B12 deficiency is indicated since the patient has a gradual onset of paresthesias (numbness, tingling or pricking)
starting in the hands and feet with a CBC that reveals a macrocytic anemia and a peripheral smear that shows multisegmented
neutrophils. Pernicious anemia is the most common cause of vitamin B12 deficiency.
Question 59
Which of the following patients does the nurse practitioner need to refer?
a patient who has had persistent low back pain and does not respond to conservative treatment after a week
a patient with a viral upper respiratory infection you have started on symptomatic care
a patient with an anal fissure that has not responded well to treatment after 10 days
a patient with dysphagia without obvious treatable cause
The Correct answer is :
A patient with dysphagia without obvious treatable cause
Low back pain should be evaluated further with diagnostic testing and treatment should be tried longer before referral. A viral
upper respiratory infection does not warrant referral and a patient with an anal fissure should be given time to heal, such as 3060 days before referring the patient.
Question 60
The nurse practitioner understands that in the workplace, sexual harassment may occur. Which of the following is NOT true
concerning sexual harassment?
A hostile environment is a form of sexual harassment.
Sexual harassment must be pervasive and does not need to be longstanding to be considered.
The participants do not have to find the actions objectionable and have work performance be affected.
Sexual harassment is difficult to prove.
The Correct answer is:
The participants do not have to find the actions objectionable and have work performance be affected
Sexual harassment results in a hostile environment where the participants find the actions objectionable and work performance
is affected. It is difficult to prove when there are no witnesses
Question 61
You are treating a patient for sinusitis who has no insurance. The patient's adolescent daughter does have health insurance so
you write a prescription in her name for her mother's sinusitis. This is considered:
adequate care
fraud and abuse
good Samaritan care
none of the above
The Correct answer is:
Fraud and abuse
Using one patient's insurance to fill medications for another person is illegal and considered fraud and abuse. The appropriate
thing to do, if the mother does not have money to buy medications, is offer samples or refer her to social services or a local
agency that helps the uninsured.
84
Question 62
For which of the following diseases/conditions should the FNP refer the patient to an ophthalmologist stat?
cluster headache
temporal arteritis
trigeminal neuralgia
migraine headache (with aura)
Correct answer:
temporal arteritis
Temporal arteritis is a systemic inflammatory process of the medium and large arteries of the body. A patient with temporal
arteritis should be referred to an ophthalmologist stat. Permanent blindness can result if the person is not diagnosed and treated
early.
Question 63
Which of the following findings is often found in a person with stage 2 Lyme disease?
macrocytic anemia
conductive hearing loss
atrioventricular heart block
peripheral neuropathic symptoms
The Correct answer is:
Atrioventricular heart block
Lyme disease has three stages, which are stage 1, stage 2 and stage 3. In stage 2 Lyme disease, also known as early
disseminated infection, the infectious organism spreads or disseminates into the blood. A potential complication for stage 2
Lyme disease is Lyme carditis, which is exhibited by the presence of abnormal heart rhythms, or an atrioventricular block.
Further, in stage 2 Lyme disease, a classic rash may reappear with multiple lesions and arthralgias. Additionally, fatigue,
myalgia, and headaches may occur.
Question 64
The National Practitioner Data Bank is a repository for which of the following types of information?
personal information
professional credentials
licensure information
malpractice payments
Correct answer:
malpractice payments
The National Practitioner Data Bank is a repository for damage award data. It is a data bank that keeps track of payments form
professional liability insurance companies on behalf of the clients to injured parties for successful malpractice claims.
Question 65
Which of the following would NOT be recommended by the FNP as a measure to prevent colonic diverticulosis and
diverticulitis?
regular aerobic exercise
foods low in uric acid
adequate hydration
a high-fiber diet
Correct answer:
foods low in uric acid
Foods low in uric acid are what is recommended for those suffering from gout. The other three choices are recommendations
for preventing colonic diverticulosis and diverticulitis. They help to increase bowel motility and tone.
Question 66
The nurse practitioner knows that the following are all developmental warning signs EXCEPT:
At 6 weeks, the infant has an absence of auditory alertness.
At 18 months, the child does not have a pincher grasp.
At 10 months, the infant does not wave bye.
85
A 2 years, the child does not walk.
The Correct answer is:
At 10 months, the infant does not wave bye
An infant may not wave bye until age 14 months. The remaining answer choices are all warning signs regarding development.
Other warning signs for a 6-week-old infant include lack of visual fixation (focusing) and the infant's head lag when going
from pulling to sitting position. At 18 months, the child does not have a pincer grasp or exhibits the inability to stand without
support.
Question 67
When evaluating the carotids, with correct procedure for auscultating is:
Use the bell of the stethoscope.
Position the client at a 30-degree angle, and press firmly with bell of the stethoscope.
Use the diaphragm of the stethoscope.
Place the stethoscope 1 inch off the area above the sternocleidomastoid muscle.
The correct answer is to use the bell of the stethoscope. The correct procedure is to listen for carotid bruits with the bell of the
stethoscope, which brings out low-frequency sounds and filters out high-frequency sound. The bell should be placed very
lightly on the neck with just enough pressure to seal the edge.
Question 68
A 55-year-old male comes to the clinic complaining of swelling of his left calf which he says is painful to touch. He tells the
FNP that he has just returned from a business trip in Hong Kong. The FNP’s examination reveals a warm, red area that is
painful when palpated and a positive Homan sign. The likely diagnosis for this patient is which of the following?
superficial thrombophlebitis
deep vein thrombosis (DVT)
peripheral vascular disease (PVD)
Raynaud’s phenomenon
Correct answer:
deep vein thrombosis (DVT)
Thrombi develop in the deep venous system of the legs or pelvis secondary to stasis, trauma to vessel walls, inflammation or
increased coagulation. Complications of DVT include pulmonary emboli and stroke.
Question 69
When the FNP examines a patient with suspected hypothyroidism, he would NOT expect to find:
weight gain
coarse, dry skin
tachycardia
intolerance of cold
Correct answer:
tachycardia
The FNP would not expect to find tachycardia in a patient with suspected hypothyroidism; he would be more likely to find
bradycardia, especially in severe cases. He may also find that the person is lethargic and has memory problems, that the person
has thick, coarse hair with a tendency to break, and thick, dry nails.
Question 70
The nurse practitioner (NP) has been asked to resolve a conflict between two medical assistants. One is observed as pleasant
and helpful, while the other is abrasive and angry. What guideline must the NP observe in the resolution of this conflict?
Deal with issues, not personalities.
Weigh the consequences of each possible solution.
Require the medical assistants to reach a compromise.
Encourage ventilation of anger and use humor to minimize the conflict.
The Correct answer is:
Deal with issues, not personalities
86
The conflict must be addressed directly by the nurse practitioner. The personal characteristics of the assistants must not enter
into the conflict resolution process. Compromise is only one method of conflict resolution.
Question 71
A 45-year-old female comes to the clinic reporting that she has had several episodes of a blue color in her fingertips
accompanied by a tingling and numbness of these fingertips. The FNP diagnoses Raynaud’s phenomenon. Which of the
following would the FNP NOT do for the patient suffering from this condition?
Tell her to stop smoking.
Tell her to avoid cold weather.
Prescribe Norvasc.
Prescribe Imitrex.
Correct answer:
Prescribe Imitrex
Imitrex is a vasoconstricting drug. This is why it would not be prescribed for a person with Raynaud’s phenomenon.
Question 72
The descriptors for the levels of evaluation and management (E/M) services recognize seven components that are used in
defining the levels of E/M services. Which of the following is NOT one of these components?
history
counseling
time
expense
Correct answer:
expense
Expense is not one of the seven components used in defining the levels of E/M services. The seven components are: history,
examination, medical decision making, counseling, coordination of care, nature of presenting problem and time.
Question 73
A mother brings a 21-month-old Asian-American female child into the office. The chief complaint is abdominal pain with
flatulence and diarrhea after eating. Up until 3 months ago, she was being breast-fed twice a day. The nurse practitioner
suspects:
lactose intolerance
food allergy
irritable bowel syndrome
Hirschsprung's disease
The Correct answer is:
Lactose intolerance
This condition is common among the Asian population. Primary symptoms include bloating, flatulence, abdominal cramps and
diarrhea after eating foods with lactose. Food allergies are common in children under the age of 3 and they develop
angioedema, flushing, hives and throat itching. Irritable bowel syndrome symptoms are constipation alternating with diarrhea
and more common during late adolescence. Hirschsprung's disease is more common in boys than girls.
Question 74
A female patient is in the office with scoliosis. The nurse practitioner knows that the following are all true regarding the
management of this condition EXCEPT:
The goal of management is to prevent further deformity.
Curves >40° in growing children require surgical rod insertion or spinal fusion.
Curvature of less than 20° is of no concern.
Bracing is required if curves are >25° in growing children.
The Correct answer is:
Curvature of less than 20° are of no concern
Curves < 14° should be observed, documented and evaluated on follow-up in 6 months in children with a Tanner V or less. If
the curvature is 15°, they should be referred to an orthopedist to monitor for progression.
Question 75
In the course of educating a patient who suffers from migraine headaches, the FNP would advise the patient to avoid all but
which of the following foods?
87
chocolate
pickled beets
lima beans
fish
Correct answer:
fish
Certain foods should be avoided by persons who suffer from migraines. Fish is not one of these foods. Foods to be avoided
include: chocolate, broad beans, any pickled, fermented or marinated food, sour cream, ripened cheeses, sausages, pizza,
MSG, nuts, figs, citrus foods, bananas, caffeinated beverages and alcoholic beverages.
Question 76
Which of the following theorists wrote the book Adolescence that first described adolescence as a critical developmental
period?
G. Stanley Hall
Arnold Gesell
Alfred Binet
Theophile Simon
Correct answer:
G. Stanley Hall
G. Stanley Hall wrote the book Adolescence that first described adolescence as a critical developmental period of “storm and
stress” or “Sturm und Drang.” Its three key aspects are conflict with parents, mood disruptions, and risky behavior.
Question 77
Teratogens are agents that can cause structural abnormalities during pregnancy. Which of the following is NOT a teratogen?
Ciprofloxacin
cocaine
aminoglycosides
lithium
Correct answer:
Cipro
Cipro is a Category C drug -- there may be a risk of fetal harm, and the drug should only be used if the potential benefits
outweigh the potential risk. All of the other choices, along with alcohol, cigarettes, Accutane, lithium, and other substances,
are teratogens, meaning they are suspected or known to cause fetal abnormailities.
Question 78
Which of the following statements about revocation of an NPs license is true?
Both the state board of nursing and a court of law can revoke or suspend an NP’s license.
A court can only revoke or suspend an NP’s license if the judge believes the NP to be grossly negligent.
Only the state board of nursing can revoke or suspend an NP’s license.
Only a court can revoke or suspend an NP’s license.
Correct answer:
Only the state board of nursing can revoke or suspend an NP’s license.
A state board of nursing approves an NP’s right to practice in a state and this same board of nursing can suspend or revoke an
NP’s license. A court cannot revoke an NP’s license. If a judge believes an NP to be grossly negligent, the judge may report
the nurse to the state board of nursing.
Question 79
Which of the following is only a presumptive sign of pregnancy?
Goodell’s sign
enlarged uterus
amenorrhea
urine tests
Correct answer:
amenorrhea
Amenorrhea is only a presumptive sign of pregnancy. It could be the result of other factors. Presumptive signs are the softest
88
and least objective signs. The other choices are all probable signs of pregnancy.
Question 80
A sudden and rapid drop in the hemoglobin (< 6 g/dL) along with pallor, clammy skin, tachycardia and hypotension may
indicate:
acute hemorrhage
neutropenia
thrombocytopenia
vitamin B12 deficiency
Correct answer:
acute hemorrhage
Acute hemorrhage is indicated by a sudden and rapid drop in the hemoglobin. Signs and symptoms of shock such as pallor,
clammy skin, tachycardia and hypotension may also be present.
Question 81
Which of the following statements about the Nurse Practice Act of the state is correct?
Only some states have a Nurse Practice Act.
The Nurse Practice Act has no statutory authority.
The Nurse Practice Act is not legally enforceable.
The Nurse Practice Act gives nurses the legal right to practice.
Correct answer:
The Nurse Practice Act gives nurses the legal right to practice.
This is the correct statement. The Nurse Practice Act gives nurses the legal right to practice. All states have a Nurse Practice
Act. It has statutory authority and is legally enforceable.
Question 82
Which of the following terms is used for the process by which an agency of state government grants permission to individuals
(such as FNPs) for the practice of a profession to engage in the practice of that profession and prohibits all others from legally
doing so?
accreditation
certification
endorsement
licensure
Correct answer:
licensure
Licensure is the process by which an agency of state government grants permission to individuals (such as FNPs) for the
practice of a profession to engage in the practice of that profession and prohibits all others from legally doing so. Certification
is the process by which a non-governmental agency or association certifies that an individual licensed to practice as a
professional has met certain predetermined standards specified by that profession for specialty practice.
Question 83
The nurse practitioner must carry professional liability insurance. An occurrence-form type is preferred because:
The carrier will be notified of a potential claim during the policy period.
The coverage is limited and time based.
The amount of insurance money available to pay a claim increases with each renewal of the policy.
The policy proceeds are available to pay claims regardless of when the claim is reported to the carrier.
The Correct answer is:
The policy proceeds are available to pay claims regardless of when the claim is reported to the carrier
The types of professional liability, as well as general liability, insurances include occurrence form and claims-made. With the
occurrence form, the liability insurance plan pays the claim based on when the loss happens and according to the timeframe
the policy was in place. For instance, the nurse practitioner has professional liability insurance, which is occurrence form
based. The nurse practitioner purchased the insurance coverage in 2007. The nurse practitioner has paid the premiums to keep
the coverage up to date with the XYZ Corporation. A patient files a lawsuit against the nurse practitioner in July 2010 for an
incident that occurred in May of 2008. If the patient wins the lawsuit, the XYZ Corporation will pay the claim, under the
occurrence form plan, because the incident occurred during the timeframe when the policy was in effect (May 2008). With a
claims-made policy, this policy pays claims also when the coverage is in effect. However, the claims made coverage policy
89
will pay the claim based on the terms identified in the policy at the current time. Therefore, if an incident occurred in 2008 and
a claim is filed in 2011, the incident has to be covered in the current 2011 and not the 2008 policy coverage terms, before the
insurance company pays the claim. Additionally, the answer choice “the carrier will be notified of a potential claim during the
policy period” is incorrect, as the carrier does not need to be informed during the policy period. This is a requirement with the
claims-made type coverage. Further, the limits do not automatically increase with the occurrence form type of coverage, which
makes the answer choice “amount of insurance money available to pay a claim increases with each renewal of the policy”
incorrect.
Question 84
The most highly infectious stage of pertussis is during which of the following?
convalescent stage
incubation stage
paroxysmal stage
catarrhal stage
Correct answer:
catarrhal stage
The person who has pertussis is most highly infectious during the catarrhal stage of the illness. At this stage, that usually lasts
from one to two weeks, the illness is characterized by complaints related to upper respiratory tract infection.
Question 85
The FNP overhears a staff member talking about a patient who “had a senior moment” because she could not remember
something. The FNP knows that this statement:
could be interpreted as stereotyping older adults and considered ageism
is innocent and has no negative connotation
is a generally accepted comical statement about older adults
none of the above
Correct answer:
could be interpreted as stereotyping older adults and considered ageism
This statement shows a bias against older adults. It can be considered a form of ageism. It is prejudicial and perpetuates
stereotypes of older adults.
Question 86
There is a shortage of pneumococcal vaccine for adults. The nurse practitioner understands that of all of the following, who is
at greatest risk for invasive pneumococcal infection and should be priority?
a 36-year-old woman who underwent a splenectomy recently
a 52-year-old man with a 15-year history of type 2 diabetes
a 78-year-old woman with decreased mobility due to chronic rheumatoid arthritis
a 64-year-old man with chronic obstructive pulmonary disease
The Correct answer is:
A 36-year-old woman who underwent a splenectomy recently
Medical indications for this vaccine include chronic lung disease, chronic renal disease, chronic cardiovascular disease,
chronic liver diseases, diabetes, immunocompromising conditions, and anatomic asplenia (splenectomy). Without the spleen,
the patient's resistance to fight infection is greatly reduced.
Question 87
In terms of liver function tests, which of the following tests has a normal reference range of 5 – 50 u/L?
Alkaline Phosphatase
Serum GGT
Serum AST
Serum ALT
Correct answer:
Serum AST
Serum AST is normal at 5 to 50 u/L. It is present in the liver, heart muscle, skeletal muscle, kidney and lung. It is not specific
for liver injury because it is also elevated in other conditions such as acute MI.
Question 88
90
A patient with a vitamin D deficiency can be helped by all but which of the following?
increased sun exposure
a dose of 50,000 IU once a week for 8 weeks
low-dose (400 – 800 IU) vitamin D supplementation alone
eating a diet rich in vitamin D
Correct answer:
low-dose (400 – 800 IU) vitamin D supplementation alone
This is not enough to correct a vitamin D deficiency. Vitamin D does not have the ability to be stored in fat nor does it have a
long half-life. Daily low-dose supplementation to a high dose once a week for 8 weeks is the norm.
Question 89
A pregnant woman voices her concern with you over the side effects she is having with supplemental iron therapy. The nurse
practitioner knows that maternal iron requirements are the greatest during what part of her pregnancy?
first trimester
second and third trimester
first and second trimester
preconception
The Correct answer is:
Second and third trimester
Maternal iron requirements increase in the second and third trimesters of pregnancy, in part because of the fetus's need to build
iron stores. Iron deficiency anemia is common during pregnancy and the patient should be counseled on ways to tolerate the
side effects of the supplementation.
Question 90
In interacting with a patient, which of the following phrases should NOT be used?
“Tell me about . . .”
“What are your thoughts . . .?”
“What are you feeling?”
“If it were me I would . . .”
Correct answer:
“If it were me I would . . .”
Some phrases should not be used in communicating with patients. “If it were me I would . . .” is one of them. Useful phrases
are those that promote therapeutic interaction and keep it open, genuine and patient-centered.
Question 91
The first-line intervention for anterior epistaxis is which of the following?
nasal packing
simple pressure to the area superior to the nasal alar cartilage
cautery
topical thrombin
Correct answer:
simple pressure to the area superior to the nasal alar cartilage
Anterior epistaxis is usually the result of localized nasal mucosa dryness and trauma. Most episodes can be easily managed
with simple pressure – with firm pressure to the area superior to the nasal alar cartilage. If this action is ineffective, then other
secondary measures such as those mentioned in the other choices may be used.
Question 92
A standard being developed under managed care is credentialing. Which of the following statements about credentialing is
false?
Credentialing for most health plans is being conducted largely by the Council for Affordable Quality Health Care.
There are emerging standards regarding patient satisfaction.
Under managed care, all standards for clinicians are related to the quality of patient care.
none of the above
Correct answer:
Under managed care, all standards for clinicians are related to the quality of patient care.
91
This is false. Under managed care, not all standards for clinicians are related to the quality of patient care. Some standards
look at quantity of care delivered.
Question 93
A mother brings her 3-year-old son into the clinic for a routine checkup. The mother tells the FNP that her child “acts out” a
lot and that she needs help in addressing this behavior. The FNP suggests a “time out” for the child when he misbehaves.
Which statement in regard to a “time out” for children is most accurate?
The “time out” should last for as many minutes as the child has misbehaved.
The child should sit still in the “time out” for as many minutes as his age in years.
The child should be allowed to read or draw during the “time out.”
“Time outs” should not be used until a child is at least old enough to have started school.
Correct answer:
The child should sit still in the “time out” for as many minutes as his age in years.
A “time out” for a child should not last longer than his age in years. Since this child is three years old, his “time out” should be
three minutes. He should sit still during this time, not read, draw or do any other activity.
Question 94
The FNP has a patient who has suffered a cat bite. The FNP knows that the rate of infection from cat bites is:
negligible
5%
50%
80%
Correct answer:
80%
All bites should be considered to carry significant infectious risk. This risk can vary from the relatively low rate of infection
from dog bites (about 5%) to the very high rate from cat bites (about 80%). Initial therapy for all bite wounds should include
vigorous wound cleansing with antimicrobial agents as appropriate and debridement if necessary
Question 95
In the management of Parkinson Disease (PD), surgical intervention such as deep brain stimulation surgery can be helpful to
manage disease-related symptoms:
that are related to memory loss
to ease the tremors and bradykinesia
when medication therapy is not tolerated or helpful
as a last resort when all other options have been exhausted
The Correct answer is:
When medication therapy is not tolerated or helpful
Deep brain stimulation surgery for Parkinson Disease is helpful in making the "off" state more like movement in the "on"
state, and is helpful in the reduction of levodopa-induced dyskinesias. As with other therapies, expert consolation should be
sought, and all options should be thoroughly discussed with the patient before pursuing surgical intervention.
Question 96
The FNP has a 70-year-old female patient who has been diagnosed with giant cell arteritis. The FNP understands that all of the
following are true about this disease and its treatment EXCEPT:
Her symptoms may be accompanied by respiratory tract symptoms.
Approximately 50% of patients with giant cell arteritis experience visual symptoms.
Transient repeated episodes of blurred vision are usually reversible.
Sudden loss of vision is almost always reversible.
Correct answer:
Sudden loss of vision is almost always reversible.
Sudden loss of vision is almost always permanent. Treatment for giant cell arteritis helps minimize the risk of blindness.
Question 97
Health disparities have gained much attention in research and media. The following is NOT a true statement concerning health
disparities:
Nurse practitioners struggle with defining their professional response to the plight of uninsured and underinsured.
92
Remedies to the gaps in healthcare by nurse practitioners include lobbying for incremental increases in coverage to include
services by APNs.
The new economic environment will not affect insurance premiums or the cost of healthcare.
Some of those who suffer health disparities are rural and urban low-income groups, African-Americans and Hispanic people.
The Correct answer is:
The new economic environment will not affect insurance premiums or the cost of healthcare
The new economic environment will affect premiums and healthcare costs.
Question 98
An example of a secondary prevention measure for a 55-year-old healthy and physically active female is which of the
following?
yearly flu shot
recommending the use of bicycle helmets
educating her about skin cancer from sun exposure
doing a mammogram and Pap smear
Correct answer:
doing a mammogram and Pap smear
Secondary prevention measures include activities provided to identify and treat asymptomatic persons who might be at risk for
a particular disease or condition. Doing a mammogram and Pap smear is a secondary prevention measure. All of the other
choices are primary prevention measures.
Question 99
Which of the following treatments would be considered a long-term control medication for asthma sufferers?
Albuterol HFA
Xopenex
Accolate
Ventolin HFA
Correct answer:
Accolate
Accolate (zafirlukast) is a leukotriene receptor antagonist that is used for long-term control of asthma in adults and children 5
– 11 years old. The other choices are all quick-relief (rescue) medications.
Question 100
A 34-year-old female with multiple sclerosis (MS) is in the office. She is now under your care after relocating from another
state to live with relatives who can help her. The nurse practitioner understands that which of the following is NOT part of her
management plan:
Physical and occupational therapy.
Complex treatment regimen must be coordinated with a neurologist.
Acute exacerbations should be treated with prednisone 60-80 mg/day for 1 week, taper over 2-3 weeks.
Corticosteroids are often used for maintenance.
The Correct answer is:
Corticosteroids are often used for maintenance
Corticosteroids are only for acute exacerbations, not for maintenance. The remaining answer choices are treatment options for
this condition
test 4 ancc
Question 1
You are counseling a patient with a new diagnosis of GERD or gastroesophageal reflux disease. Which of the following is
accurate nonpharmacologic management?
lying down and resting after meals and weight reduction.
weight reduction and sleep with the head of the bed elevated on blocks
avoid mint, orange juice, and milk
drink large amounts of fluids with meals and avoid alcohol
93
Correct answer:
Weight reduction and sleep with the head of the bed elevated on blocks
This is the accurate non-pharmacological management. One of the main education interventions is to advise the patient not to
lie down within 3 hours after meals to avoid reflux. Large amounts of fluids encourage GERD, and alcohol, mint, and orange
juice all relax the esophageal sphincter allowing reflux to occur.
Question 2
A patient has anosmia. What is this condition and which cranial nerve is involved?
cranial nerve I - inability to smell
cranial nerve II - inability to taste
cranial nerve III - inability to blink
cranial nerve IV - inability to hear
The Correct answer is:
Cranial nerve I - inability to smell
The cranial nerve I involves the olfactory nerve and it is rarely tested. However, cranial nerve lesions do occur. Anosmia
would be a clinical manifestation of this.
Question 3
What type of headache is typically described as "pressing" in quality?
migraine headache
tension-type headache
cluster headache
simple stress headache
The Correct answer is:
Tension-type headache
Tension-type headache is pressing, nonpulsatile pain, mild to moderate in intensity and usually bilateral in location. Migraine
headache is described as pulsating quality, usually unilateral, with associated nausea and photophobia. Cluster headache is
often behind one eye, steady and intense. Simple stress headache is a fictitious term, which makes it an incorrect answer
choice.
Question 4
You are counseling a young female patient who is trying to get pregnant. She wants to take a home pregnancy test and inquires
about how early she can take this test and see predictable results. Your response is:
within 3 days after conception
within 5 days after conception
1-2 weeks after conception
35 days after the last menses
The Correct answer is:
1-2 weeks after conception
The office urine pregnancy tests and home pregnancy tests are the same and they detect the presence of the beta subunit of
human chorionic gonadotropin (hCG). This is found in the highest quantities in first morning urine.
Question 5
A 28-year-old female patient is in the office and is to undergo radioactive iodine treatment for Graves' disease. The nurse
practitioner is counseling her and understands that the mechanism of action of this treatment therapy is:
alter the thyroid metabolic rate
relieve distress caused by increased thyroid size
destroy the overactive thyroid tissue
reduce production of TSH
The Correct answer is:
Destroy the overactive thyroid tissue
Graves' disease is the most common form of thyrotoxicosis. Radioactive iodine is used for thyroid ablation to destroy the
overactive thyroid tissue that causes the thyroid hormone dysfunction.
Question 6
Bronchiolitis is a common illness in early childhood. Treatment of bronchiolitis in children usually consists of treatment with:
supportive therapy
corticosteroids
94
ribavirin
bronchodilators
Correct answer:
supportive therapy
In most children, bronchiolitis runs a course of 2 to 3 weeks of mild upper respiratory symptoms. Supportive therapy is usually
sufficient. The use of corticosteroids, ribavirin and bronchodilators remains controversial with little evidence of efficacy.
Question 7
The nurse practitioner knows that the influenza vaccination is recommended annually for certain high risk groups. Which of
the following would have the GREATEST need for this vaccine?
dialysis patients
healthcare employees
adults with chronic disease
residents of long-term care facilities
The Correct answer is:
Residents of long-term care facilities
The outbreak of influenza may affect 60% of those in long-term care and the mortality rates related to this are high. All of the
other groups are in need of the vaccine, but not the priority or greatest risk
Question 8
You are treating a 24-year-old man with infectious mononucleosis with tonsillar hypertrophy, exudative pharyngitis, difficulty
swallowing, and a patent airway. You know that you should prescribe:
prednisone
acyclovir
amoxicillin
ibuprofen
The Correct answer is:
Prednisone
Infectious mononucleosis is an acute systemic viral illness usually caused by Epstein-Barr virus, a DNA herpes virus that
typically enters the body via oropharyngeal secretions and infects B lymphocytes. Treatment includes a corticosteroid, such as
prednisone 40 to 60 mg/day for 3 days.
Question 9
You are working in an urgent care clinic within a hospital emergency department. A patient without insurance arrives who has
a puncture wound caused by an unknown sharp object in a trash container. A dirty needle is suspected. The nurse practitioner
should:
administer a tetanus injection only since the patient has no insurance
prescribe appropriate mediations for HIV exposure even though the patient has no resources to buy them
not mention the possibility of HIV from a contaminated needle
offer to buy the HIV medications for the patient and use your employee discount at the pharmacy
The Correct answer is:
Prescribe appropriate mediations for HIV exposure even though the patient has no resources to buy them
The standard of patient care should be followed by the nurse practitioner. This measure should not be altered depending on the
patient's insurance and financial status. It is unethical to not properly inform this patient of the risks involved for being
exposed to a contaminated needle. You should prescribe the medications and refer them to social services or a community
resource agency for assistance. Buying the medications for the patient with an employee discount is unethical and should not
be done.
Question 10
Which of the following factors is least likely to put a woman at a higher for breast cancer?
first degree relative who had breast cancer
shorter exposure to estrogen (as in late menarche with early menopause)
obesity
high-dosed radiation
Correct answer:
shorter exposure to estrogen (as in late menarche with early menopause)
95
Mammography is a secondary prevention measure. It is recommended at age 40 and older. Women at higher risk for breast
cancer include all of the choices except this one. Longer exposure to estrogen (as in early menarche with late menopause) puts
a woman at higher risk.
Question 11
Multiple sclerosis (MS) is usually classified into different forms. The type of MS in which episodes resolve with good
neurologic function between exacerbations would be which of the following?
RRMS
primary progressive MS
secondary progressive MS
none of the above
Correct answer:
RRMS
RRMS is relapsing, remitting MS in which episodes resolve with good neurologic function between exacerbations and it has
minimal to no cumulative defects. This type of MS accounts for approximately 85% of patients with the condition.
Question 12
A 20-year-old college student is in the clinic with complaints of weakness. She is 15% below ideal body weight. She reports
doing well in her classes, drinking alcohol every night and coffee throughout the day. She is bradycardic and gets dizzy when
she stands up. What other findings may be seen in this patient?
sleep apnea
amenorrhea
hypertension
mitral regurgitation
The Correct answer is:
Amenorrhea
This patient has anorexia nervosa and is far below ideal body weight. She exhibits evidence of poor nutrition and health. More
than 90% of patients with anorexia are amenorrheic. Hypotension is more common than hypertension and many patients with
anorexia also exhibit mitral valve prolapse, not regurgitation.
Question 13
The nurse practitioner may put her license in jeopardy if:
The practitioner delegates patient assessment tasks to a licensed practical nurse who is working in the clinic.
The practitioner provides care with the established standards of practice in this jurisdiction.
The practitioner appropriately delegates medication administration to a trusted registered nurse who administers a fatal dose.
A medical assistant in the office exceeds the scope of her authority and the nurse practitioner takes prompt action to correct the
problem.
The Correct answer is:
If the nurse practitioner delegates patient assessment tasks to a licensed practical nurse who is working in the clinic
The licensed practical nurse should not be doing assessments. Assessments are presumed to be within the purview of the
professional nurse, not those with fewer years of nursing education.
Question 14
Certain medications have a direct effect on urinary continence. Which of the following drugs would cause urinary retention as
well as fecal impaction?
diuretics
opioids
alcohol
alpha-adrenergic antagonists
Correct answer:
opioids
Opioids will have the effect of urinary retention, overflow, alteration in sensorium and fecal impaction. Drugs with
anticholinergic activity such as first generation antihistamines, tricyclic antidepressants and antipsychotics have the same
effect.
Question 15
96
A 65-year-old woman has been diagnosed with terminal stomach cancer. She complains to the nurse that her family keeps
telling her to fight and not give up. She gets angry every time that they refuse to acknowledge the prognosis. What is the best
thing that the FNP could say to her?
Your family loves you. You can’t expect them to accept a terminal diagnosis.
You shouldn’t be angry with them. They don’t know any better.
Would you like me to talk to them so that they understand how you feel?
You’re angry because your family won’t acknowledge your prognosis?
Correct answer:
You’re angry because your family won’t acknowledge your prognosis?
This is the best response. It restates what the client has said to ensure understanding. This is part of therapeutic
communication.
Question 16
At what stage of pregnancy is the uterine fundus at the level of the umbilicus?
8 weeks
12 weeks
16 weeks
20 weeks
Correct answer:
20 weeks
At 20 weeks (5 months) of pregnancy the uterine fundus is at the level of the umbilicus. Also the fetal heart tones are heard
with a fetoscope or stethoscope.
Question 17
Which of the following statements about prostate cancer detection is incorrect?
Men at high risk for prostate cancer should begin testing at age 45.
The American Cancer Society recommends routine testing for prostate cancer.
Discussion with a clinician about prostate cancer screening should include an offer for testing with the PSA test.
African-American men are at high-risk for prostate cancer and should begin testing at age 45.
Correct answer:
The American Cancer Society recommends routine testing for prostate cancer.
At this time the American Cancer Society does not recommend routine testing for prostate cancer. The American Cancer
Society believes that men should not be tested without learning about what we know and don’t know about the risks and
possible benefits of testing and treatment.
Question 18
The following are true regarding the pathogenesis of adult abuse and neglect EXCEPT:
The etiology is unclear, but there are a number of recurrent theories emerging.
Financial conditions appear to play a role in the occurrence of abuse.
Very few incidences of abuse to elderly and disabled persons occur.
Alcoholism and drug dependency in a caregiver enhance the likelihood of abuse.
The Correct answer is:
Very few incidences of abuse to elderly and disabled persons occur
It is estimated that almost 2 million older adults and disabled persons are abused annually. The victim is usually a woman and
the perpetrator is most often the spouse or an adult child. The remaining answer choices are true regarding abuse.
Question 19
The FNP sees a patient who has recently completed antibiotic treatment with sulfa drugs. He presents with severe vesicular
lesions all over his body. There is mucosal involvement with blisters on his mouth and nose. What should the FNP suspect
with this history and these symptoms?
Stevens-Johnson syndrome
shingles
meningococcemia
erythema multiforme
97
Correct answer:
Stevens-Johnson syndrome
Stevens-Johnson syndrome is the acute onset of severe vesicular to bullous lesions all over the body. They range from hives to
blisters and hemorrhagic lesions. There is also mucosal involvement with blisters. A history of recent antibiotic treatment with
sulfas, penicillins, phentoin and other drugs can precede the onset. Erythema multiforme is a milder form of Stevens-Johnson
syndrome with no mucosal involvement.
Question 20
In terms of the gastrointestinal system which of the following statements about ulcers is false?
Duodenal ulcers are five times more prevalent than gastric ulcers.
Chronic NSAID use can result in ulcers.
Abdominal exam of a person with an ulcer will reveal normal or mildly tender epigastric area during flare-ups.
Gastric ulcers have a lower risk for malignancy compared to duodenal ulcers which are often malignant.
Correct answer:
Gastric ulcers have a lower risk for malignancy compared to duodenal ulcers which are often malignant.
This statement is false. Gastric ulcers have a higher risk for malignancy compared to duodenal ulcers which are mostly benign.
Duodenal ulcers are seen more in the elderly population.
Question 21
In terms of malpractice insurance, which of the following statements is correct?
NPs should choose a company that is located in the USA and has a stable financial rating.
Individual insurance policies are necessary if an NP works outside of the work setting.
Under a “claims made” policy, an NP is covered only when the insurance policy is active, no matter when the incident
occurred.
All of the above are correct.
Correct answer:
All of the above are correct.
All of the choices are correct statements in terms of malpractice insurance. Also, “occurrence” insurance covers any incident
that occurred while the NP was insured.
Question 22
When counseling the mother of a toddler on lead toxicity and poisoning, what should the nurse practitioner understand that is
most likely to lead to this condition?
If the toddler lives near an electric generating plant, he or she is at risk.
If the toddler is developmentally disabled, he or she is at risk.
If the toddler lives in a 15-year old home with copper plumbing, he or she is at risk.
If the toddler lives in a 100-year old home that is being remodeled, he or she is at risk.
The Correct answer is:
If the toddler lives in a 100-year old home that is being remodeled, he or she is at risk
The toddler is at risk if he or she is exposed to lead-based paint. This paint has not been available in the United States in 30
years. Also, copper pipes are not a risk to the toddler as lead-lined pipes are. A toddler living near a lead factory would be at
risk, not one living near a electric generating plant. A developmental disability does not increase lead toxicity risk.
Question 23
You are educating a patient with Type II diabetes mellitus. The nurse practitioner understands that secondary causes of
hyperglycemia include the use of all of the following medications EXCEPT:
niacin
thiazide diuretics
angiotensin receptor blockers
corticosteroids
The Correct answer is:
Angiotensin receptor blockers
Angiotensin receptor blockers are recommended for use in individuals with concomitant type II diabetes and hypertension.
They do not affect glucose levels. The remaining answer choices are all medications that can cause elevated glucose levels and
should be avoided with a diabetic patient.
Question 24
98
Which of the following statements about tuberculosis is incorrect?
Tuberculosis occurs only in the lungs.
Transmission of tuberculosis is by respiratory droplets.
The majority of cases in the United States are reactivated infections.
A TB infection is controlled by an intact immune system.
Correct answer:
Tuberculosis occurs only in the lungs.
This statement is incorrect. The most common site of infection is the lungs (85%), but infection can also occur in the kidneys,
brain, lymph nodes, adrenals, bone and more.
Question 25
In general, family theory serves as a basis for assessing and coming to understand the structure, development and function of
families through the process of family assessment. There are several family theories including the communication theory.
Which of the following is NOT a part of the communication theory?
The content of messages is time-bound and must be appreciated within context.
Families share common developmental processes with other families.
Lack of clarity in communication may lead to family dysfunction.
Communication conveys values and beliefs between members and the external environment.
Correct answer:
Families share common developmental processes with other families.
Although this statement is true, it is not a part of the communication theory. It is a part of the developmental theory promoted
by Erikson, Piaget and Havinghurst.
Question 26
A patient is in the office with keratosis pilaris. The nurse practitioner knows that all of the following are ways to manage this
EXCEPT:
The use products containing alpha hydroxy acids on the affected areas.
Lactic acid 12% to control this condition.
Continued use of lubricants to the affected areas as needed.
Avoiding moisturizing soaps, which worsen this condition
The Correct answer is to:
Avoiding moisturizing soaps, which worsen this condition
The use of moisturizing soaps, and not avoiding them, are recommended for the treatment of keratosis pilaris. The remaining
answer choices are all adequate management techniques.
Question 27
The nurse practitioner must use critical thinking and decision-making in her practice. The following statements are true
regarding this EXCEPT:
Critical thinking cannot be learned, but is a skill of intelligence.
Critical thinking involves acquisition of knowledge with an attitude of deliberate inquiry.
Decision-making involves making decisions based on an understanding of the different options, and the possible desirability
of the outcomes of each option.
Pattern recognition and common-sense understanding are important aspects that influence decision-making.
The Correct answer is:
Critical thinking cannot be learned, but is a skill of intelligence
Part of critical thinking may be innate, but most people can be taught to think critically. Therefore, this statement is not
accurate.
Question 28
You are treating a 32-year-old female who is in the office with varicose veins. As part of her management, the nurse
practitioner understands that which of the following does not directly contribute to the development of varicose veins?
pregnancy
heredity
99
leg crossing
Raynaud's disease
The Correct answer is:
Raynaud's disease
Varicose veins are seen in 15% of the adult population and are most often found in the lower extremities. Raynaud's disease is
not connected to varicosities. However, pregnancy, heredity and leg crossing are associated with this condition.
Question 29
The most common joint disease in North America is which of the following?
osteoarthritis
bursitis
rheumatoid arthritis
meniscal tear
Correct answer:
osteoarthritis
Osteoarthritis is the most common joint disease in North America. It is a degenerative condition that manifests without
systemic manifestations or acute inflammation. The most problematic joint involvement is in the hip and knee.
Question 30
The FNP has a 67-year-old male patient with type 2 diabetes. In managing this patient’s care which of the following lab tests
should be done every 3 – 6 months?
thyroid assessment
urine microalbumin/creatinine
serum creatinine
A1c
Correct answer:
A1c
The A1c test should be done every 3 – 6 months. The goal for this test is a result of ≤ 6.9% (ADA) or ≤ 6.5% (AACE).
Question 31
A law enacted by a state legislature or Congress is known as which of the following?
regulation
policy
statute
standard
Correct answer:
statute
A statute is a law enacted by a state legislature or Congress. A regulation is law written by a state or federal agency in
accordance with a statute. A policy is a rule made by companies or government agencies, that does not have the force of law
but that dictates day-to-day decisions. A standard is a rule or a principle used as a basis for judgment.
Question 32
Which of the following concepts is NOT a part of the legal theory of negligence?
failure to stop to help victims of an auto accident
failure to follow up
failure to refer when necessary
failure to disclose necessary information to a patient
Correct answer:
failure to stop to help victims of an auto accident
This is not a part of the legal theory of negligence. Negligence involves a duty of care. A duty is established when there is a
provider-patient relationship. There is no legal obligation for an NP to stop at the scene of an accident to help victims.
However, if the NP does stop and help, then negligence can come into play.
Question 33
While doing a 12-month well-child examination, the mother has some questions regarding her child's development. The nurse
practitioner knows that for this age all of the following are expected EXCEPT:
plays social games like peek-a-boo
100
pulls to a stand
waves bye-bye
The child has at least a 25 word vocabulary
The Correct answer is:
The child has at least a 25 word vocabulary
The 12-month-old does not have an advanced vocabulary. A 12-month-old usually speaks 1-5 words, including "mama" and
"dada".
Question 34
There are certain fundamental domains in human growth and development. Temperament and personality would fall into
which domain?
psychological and social domain
cognitive domain
physical domain
maturational domain
Correct answer:
psychological and social domain
Temperament and personality are processes of human development that are part of the psychological and social domain. Other
processes in the psychological and social domain include: interpersonal relationships; moral development; and home
environment and other social contexts.
Question 35
Your 72-year old male patient has many signs and symptoms that are worrisome. You suspect bladder cancer. What sign or
symptoms are most common with this disease process?
gross painless hematuria
fever, worse at night
microscopic proteinuria
nausea and vomiting
The Correct answer is:
Hematuria
Bladder cancer is the second most common urologic malignancy in men, after prostrate cancer. It is usually a disease that
occurs later in life. Gross painless hematuria is the most common presenting sign of bladder cancer. An abdominal mass may
be palpable in advanced disease. Fever, proteinuria, nausea, and vomiting are not indicators of this disease.
Question 36
You are treating a 39-year-old female with an anal fissure. The nurse practitioner knows that all of the following are true
regarding the management of this except:
A stool softener is necessary to prevent stool withholding.
Topical anesthetics are good for comfort measures.
Hydrocortisone cream should be prescribed bid to tid.
There is no need for topical antibiotic ointment.
The Correct answer is:
There is no need for topical antibiotic ointment
A stool softener, topical anesthetics and hydrocortisone cream are used to treat an anal fissure. Additional treatment options
for an anal fissure may include the use of botulinum toxin, nitroglycerin therapy and topical antibiotic ointment.
Question 37
A 35-year-old male is in the clinic after receiving blunt trauma to the chest. He reports that a large piece of wood hit him in the
chest. Which assessment finding would indicate that he may have respiratory complications?
He has an oximetry reading of about 97% consistently.
He has a fever of 102°F and increased sputum production.
He complains of increased pain over the affected area.
He has decreased breath sounds on the affected side.
Correct answer:
He has decreased breath sounds on the affected side
These symptoms are indicative of a pneumothorax, the most common respiratory complication after a traumatic injury to the
chest. Increased pain at the injury site is expected.
Question 38
101
The FNP is treating a patient with recurring severe migraine headaches that progress rapidly and are accompanied by
significant GI upset. This patient also has coronary artery disease. For relief of these headaches the FNP would first prescribe
a(n):
oral product
injectable product
triptan
ergotamine
Correct answer:
oral product
Although injectable products like Imitrex and Migranal have a rapid onset of action, and are best suited for patients with
rapidly progressing migraines accompanied by significant GI upset, they are generally contraindicated in patients with
coronary artery disease and should only be used with great caution and under close supervision. Triptan and ergotamine are
not recommended for patients with coronary artery disease.
Question 39
When should an infant be able to lift his head and hold his head erect?
1 – 2 months
3 - 4 months
5 - 6 months
6 - 8 months
Correct answer:
1 – 2 months
At 1 – 2 months an infant should be able to lift his head and hold it erect. He should also be regarding faces, following objects
through a visual field, smiling spontaneously and recognizing his parents.
Question 40
Professional Standards of Practice exist in the nurse practitioner's workplace. Which of the following best describes these
Standards of Practice?
They focus on the maximum levels of performance.
They are not necessary to legally describe standard of care, but necessary for licensure.
They are authoritative statements by which the quality of practice, service, or education can be judged.
The standards are exact for all levels of nursing.
The Correct answer is:
They are authoritative statements by which the quality of practice, service, or education can be judged
Standards of Practice focus on the minimum levels of acceptable performance as a way of providing consumers with a means
of measuring the quality of care they receive. They may be generic and general, as well as precise and specific. They are used
to legally describe the standard of care that must be met by a provider.
Question 41
The FNP has a patient who she suspects has macular degeneration. The FNP may use which of the following tests to check the
central vision of the patient?
Snellen chart
Amsler grid
amplitude of accommodation
slit lamp examination
Correct answer:
Amsler grid
Macular degeneration is a gradual change in the pigment in the macula (area of central vision) resulting in blindness. The FNP
will check central vision with the Amsler Grid test. If macular degeneration is present, the lines in the center of the grid are
distorted.
Question 42
All of the following are recommended pediatric screening EXCEPT:
hearing screening
anemia screening
metabolic screening
glucose screening
102
The Correct answer is:
Glucose screening
While type II diabetes mellitus in children is on the rise, the Centers for Disease Control (CDC) and the American Academy of
Pediatrics (AAP) does not recommend periodic glucose screening in children. However, the nurse practitioner can further
evaluate a child who exhibits symptoms of diabetes mellitus. Also, the urinalysis, which is part of the routine pediatric
screening, detects evaluated glucose levels. The remaining answer choices are routine types of screenings for the pediatric
population, along with screening of neonatal, genetic, lead, cholesterol and lipid, tuberculin, vision and blood pressure.
Question 43
When counseling a patient with the human herpes virus 2 (HHV-2), the nurse practitioner knows that what percentage of
sexually active adults has serological evidence of this disease?
5%
10%
15%
25%
The Correct answer is:
25%
The human herpes virus 2 (HHV-2) causes painful ulcerated lesions and lymphadenopathy. Approximately 25% of adults who
are sexually active have serological evidence of this disease, although only a small percentage of these persons have
symptoms.
Question 44
In assessing the vision of a toddler which of the following tests might be done?
Snellen test
determine ability to follow an object
Jaeger test
blink reflex
Correct answer:
determine ability to follow an object
The FNP should determine the toddler’s ability to follow an object as part of a vision screening for a toddler or older infant.
The FNP might also perform a corneal light reflex test, perform a cover/uncover test and assess red reflex.
Question 45
A 16-year-old male patient presents in the clinic with abdominal pain, worse with ambulation, nausea, and fever. Next, the
nurse practitioner would test the obturator and iliopsoas muscle to evaluate for which of the following?
cholecystitis
acute appendicitis
inguinal hernia
gastric ulcer
The Correct answer is:
Acute appendicitis
Acute appendicitis is common in this age group and the symptoms are indicative of the condition. The obturator and iliopsoas
muscles are sensitive when the appendix is diseased.
Question 46
Systematically developed statements to assist practitioner and patient about appropriate care for specific clinical outcomes are
termed:
clinical practice guidelines or protocols
standards of practice
scope of practice
advanced practice
Correct answer:
clinical practice guidelines or protocols
Systematically developed statements to assist practitioner and patient about appropriate care for specific clinical outcomes are
termed clinical practice guidelines or protocols. They are variable requirements depending on individual state nurse practice
acts and standards of practice.
Question 47
103
Treatment options for adult seizure patients include al of the following medications EXCEPT:
gabapentin
phenytoin
carbamazepine
aminophylline
The Correct answer is:
Aminophylline
Numerous standard seizure therapies exist including gabapentin, phenytoin, carbamazepine, ethosuximide, lamotrigine,
topiramate, valproic acid and clonazepam. However, aminophylline is not used to treat seizures.
Question 48
Maslow's Hierarchy of Needs includes all of the following EXCEPT:
Narcissistic needs
Safety and security
Self-esteem
Self-actualization
The Correct answer is:
Narcissistic needs
Abraham H. Maslow created a hierarchy that identified the basic needs a human requires for survival. The hierarchy levels
include physiological needs, safety needs, love and belonging needs, esteem or self-esteem needs and self-actualization needs.
The physiological needs are the basic elements a human requires to survive, such as food, shelter and water. Then, safety
needs focuses on security. Love and belonging needs addresses an individual’s desire for relationships with other people and a
need for acceptance by others. Then, esteem needs focuses on self-esteem and an individual’s view of other people. Selfactualization deals with an individual recognizing and achieving his or her potential.
Question 49
Which of the following is NOT a managed care plan?
Health Maintenance Organization
Preferred Provider Organization
Physician Group
Point of Service Plan
Correct answer:
Physician Group
A Physician Group is not a managed care plan. It is defined as two or more physicians legally organized as a partnership,
professional corporation, foundation, not-for-profit-corporation, faculty practice plan, or similar association. All of the other
choices are tpes of managed care plans.
Question 50
Any condition or substance such as hyperthyroidism or caffeine that irritates or overstimulates the heart can cause which of the
following conditions?
hypertension
atrial fibrillation
PVD
DVT
Correct answer:
atrial fibrillation
Atrial fibrillation is caused by any condition or substance that irritates or overstimulates the heart. Examples are
hyperthyroidism, cocaine, caffeine, or alcohol.
Question 51
Which of the following statements about Medicare Part B is incorrect?
It covers all medically necessary services.
It is financed by general federal revenues and by Part B monthly premiums.
It covers 80% of the approved amount after the annual deductible.
Services covered include hospitalization costs.
Correct answer:
Services covered include hospitalization costs.
104
This is incorrect. Hospital costs are covered in Medicare Part A. Payment for hospitalization is based on projected costs of
caring for a patient with a given problem
Question 52
The nurse practitioner has examined a patient who has suffered a bite wound on the chest region from her boyfriend during an
assault. There were numerous lacerations to the patient's forearms as well. The clinician sutured the lacerations even though
this was contraindicated due to the highly infectious nature of human bite wounds. The patient did not suffer ill effects or
complications from this. How can this be described?
This is malpractice.
This is a deviation from standard of care that is appropriate.
This is negligence as well as poor judgment.
This is only poor judgment.
The Correct answer is:
This is negligence as well as poor judgment
This is not malpractice since no ill effects were sustained. This is, however, negligence and poor judgment. Negligence occurs
when one fails to exercise care that a reasonable person would exercise and injury does not have to occur with negligence.
Human bites have a high probability of infection and should not be sutured.
Question 53
According to the Trans-theoretical Model of Change, the stages are:
precontemplation, contemplation, preparation, action, maintenance, and termination
survival needs, safety and security, love and belonging, self-esteem, and self-actualization
primary, secondary, and tertiary
none of the above
The Correct answer is:
Precontemplation, contemplation, preparation, action, maintenance and termination
Prochaska and DiClemente (1984) proposed these six terms, precontemplation, contemplation, preparation, action,
maintenance and termination, as the six predictable stages of change. Survival needs, safety and security, love and belonging,
self-esteem, and self-actualization are the stages of Maslow's hierarchy of needs, proposed in 1954.
Question 54
Which of the following conditions/diseases is a contraindication to bariatric surgery for overweight patients?
untreated or unstable mental health conditions
gastroesophageal reflux
degenerative joint disease
fatty liver
Correct answer:
untreated or unstable mental health conditions
Contraindications to bariatric surgery include untreated or unstable mental health conditions, active drug or alcohol abuse,
poor adherence to advised health regimens and concomitant health conditions that would pose significant operative risk. The
other choices are reasons why bariatric surgery might be recommended for obese persons.
Question 55
A 58-year-old male presents in the office with a glucose of 309 mg/dL and is symptomatic for Type II diabetes mellitus. What
should be done to manage him FIRST?
Start insulin
Start metformin plus pioglitazone
Have him return tomorrow to recheck his blood glucose
Start metformin
The Correct answer is:
Start insulin
The patient can be diagnosed with diabetes if his glucose exceeds 200 mg/dL and he is symptomatic. Most oral agents will
have little effect on his glucose and it should be lowered immediately. Therefore, insulin is the best agent to reduce the blood
sugar so that oral agents will have a chance to work. He should return to the clinic the next day for a recheck of the blood
glucose and medication adjustment.
Question 56
The nurse practitioner recognizes tobacco use and abuse as a major public health concern. All of the following are true
concerning smoking cessation EXCEPT:
Combination therapy with education, support, medications, and nicotine replacement is more effective than one method alone.
105
Nicotine replacement therapy and bupropion double the probability of success.
Smoking cessation therapies such as acupuncture and hypnosis are recommended.
Cessation without pharmacologic interventions should be attempted first.
The Correct answer is:
Smoking cessation therapies such as acupuncture and hypnosis are recommended
Alternative therapies do not have the clinical evidence-based research data to be recommended at this time.
Question 57
Which of the following exercise regimens would you prescribe for a person with osteoarthritis of the knee?
isometric exercise of iliopsoas
quadriceps sets
isometric exercise of the gluteus muscles
stretching exercises of adductors, rotator and gluteus muscles
Correct answer:
quadriceps sets
This is the only exercise regimen of the choices given that would help a person with osteoarthritis of the knee. The other
choices are all exercise regimens for osteoarthritis of the hip.
Question 58
Nursing theory is of two types: nursing grand theories and middle range nursing theories. Which of the following persons
promoted the nursing grand theory of enhancing the body’s reparative processes by manipulation of noise, nutrition, hygiene,
light, comfort and hope?
Nightingale
Benner and Wrubel
Orem
King
Correct answer:
Nightingale
Nursing grand theories are abstract, connect and relate the concepts of person, environment, nursing and health. Nightingale’s
concept was that of enhancing the body’s reparative processes by manipulation of noise, nutrition, hygiene, light, comfort and
hope.
Question 59
The FNP is treating a patient with primary open angle glaucoma (POAG). Which of the following medications is the FNP
least likely to use to treat this patient?
pilocarpine
timolol
dorzolamide
latanoprost
Correct answer:
pilocarpine
Pilocarpine is the least likely medication to be used to treat POAG. Because of its ability to cause pupillary constriction,
pilocarpine and similar medications are now seldom used.
Question 60
A 70-year-old male comes to the FNP with a raised, red lesion in his oral cavity. He tells the FNP that he is afraid that it is
cancer. Besides evaluating the lesion the FNP evaluates the man’s risk for oral cancer. These risks include all of the following
EXCEPT:
poor diet
alcohol abuse
tobacco abuse
advanced age
Correct answer:
poor diet
106
Poor diet is not a risk factor for oral cancer. Alcohol and tobacco use and advanced age are risks.
Question 61
Which of the following classic developmental theorists saw the child as an “untamed savage”?
Charles Darwin
Jacque Rousseau
Arnold Gesell
Alfred Binet
Correct answer:
Jacque Rousseau
Jacque Rousseau saw the child as an “untamed savage.” He wrote Emile – Treatise on Education which served as the
inspiration for what became a new national system of education during the French Revolution.
Question 62
According to the American Diabetes Association, adults, beginning at age 45, should be tested for fasting plasma glucose
every:
3 years
2 years
1 year
none of the above
Correct answer:
3 years
The American Diabetes Association recommends screening adults, beginning at age 45, for diabetes by testing the fasting
plasma glucose every 3 years. There are 79 million people in the United States who have prediabetes. Screening can identify
these people so providers can take action before type 2 diabetes develops
Question 63
Patients have a right to confidentiality and privacy. Which of the following statements about breach of privacy is incorrect?
Breach of privacy is not an intentional tort.
Breach of privacy can be the basis for a lawsuit by a patient.
Breach of privacy can be malpractice.
Breach of privacy can be a violation of both state and federal law.
Correct answer:
Breach of privacy is not an intentional tort.
This statement is incorrect. Breach of privacy is an intentional tort and so, can be the basis of a lawsuit by a patient. Breach of
privacy can also be malpractice, the basis for a disciplinary action by a state’s board of nursing and a violation of state and
federal law.
Question 64
In terms of patient confidentiality, which of the following statements is incorrect?
Healthcare providers must not discuss any information given to them during the healthcare encounter with anyone not directly
involved in providing this care without the patient’s or family’s permission.
Confidentiality of the healthcare encounter is protected under HIPAA.
The individual’s right to privacy does not apply to requests for a patient’s medical records.
Exceptions to guaranteed confidentiality occur when society determines that the need for information outweighs the principle
of confidentiality.
Correct answer:
The individual’s right to privacy does not apply to requests for a patient’s medical records.
This is incorrect. The individual’s right to privacy is respected when requesting or responding to a request for a patient’s
medical record.
Question 65
The theoretical concept that sets forth the idea that the person who feels susceptible to disease and believes that he will benefit
from changing his behavior is more likely to perform the healthier behavior is which of the following?
Health Belief Model
Family Systems Theory
Systems Theory
Self-Efficacy Theory
107
er:
Health Belief Model
The Health Belief Model sets forth the idea that the person who feels susceptible to disease and believes that he will benefit
from changing his behavior is more likely to perform the healthier behavior. It was first developed by social scientists in the
1950s.
Question 66
The average U.S. female gains how about much weight and height between the ages of 10 and 14 years?
38 pounds and 9.5 inches
42 pounds and 9.5 inches
32 pounds and 8.5 inches
35 pounds and 9 inches
Correct answer:
38 pounds and 9.5 inches
The average U.S. female gains about 38 pounds and 9.5 inches between the ages of 10 and 14. The average U.S. male gains
about 42 pounds and 9.5 inches between the ages of 12 and 16.
Question 67
Which of the following physiological changes would NOT be seen during pregnancy?
Plasma volume will increase 50% by the end of the third trimester.
Hemoglobin and hematocrit are decreased.
Cardiac output decreases by 1/3 by the last two trimesters.
The thyroid becomes diffusely enlarged by up to 15%.
Correct answer:
Cardiac output decreases by 1/3 by the last two trimesters.
This is not a physiological change seen during pregnancy. In actuality, cardiac output increases by 1/3 by the last two
trimesters.
Question 68
Most of the drugs used in pregnancy are Category B drugs. The FNP has a patient in the third trimester of pregnancy. Which
of the following drugs is a category C drug that should not be prescribed for this patient?
insulin
Colace
Sulfa drugs
thyroid hormone
Correct answer:
Sulfa drugs
Sulfa drugs are considered Category C drugs in the third trimester because of increased risk of hyperbilirubinemia. NSAIDs
are also contraindicated in the third trimester because they block prostaglandins.
Question 69
You are counseling a 23-year-old with an abnormal pap smear. It is important you discuss the risks of cervical cancer. Which
of the following is NOT a risk factor for cervical cancer?
virginal status
previous high-grade squamous intraepithelial lesion
human papillomavirus
multiple sexual partners
The correct answer is:
Virginal status
A female who has not had sexual intercourse is not at risk for developing cervical cancer. However, the remaining answer
choices are risk factors for cervical cancer.
Question 70
Which of the following gout medications is used for maintenance?
Indocin
Zyloprim
108
Anaprox
colchicine
Correct answer:
Zyloprim
Zyloprim (allopurinol) is a maintenance medication for gout sufferers. Maintenance drugs are prescribed to be used at least 4
to 6 weeks after the acute phase is over. They may be used daily for years to lifetime
Question 71
The following are all accurate statements regarding The National Practitioner Data Bank (NPDB) with the exception of:
The number of NPs who have had a malpractice claim filed against them is decreasing and therefore, they are not in the
NPDB.
Currently, very few NPs are listed in the NPDB.
The Health Care Quality Improvement Act of 1986 established the NPDB.
The purpose of the NPDB is to scrutinize members of the healthcare profession and list those practitioners who have had a
malpractice claim against them.
The correct answer is the number of NPs who have had a malpractice claim filed against them is decreasing and therefore,
they are not in the NPDB. The number of NPs who have had claims against them is increasing as the number of NPs in
practice increases.
Question 72
Which of the following identifies six types of conduct that are considered felonies and may result in fines of up to $25,000
and/or up to 5 years in prison?
The Federal False Claims Act
Stark I Federal Law
The Medicare and Medicaid Patient Protection Act
The Anti-Kickback statute
The Correct answer is:
The Medicare and Medicaid Patient Protection Act
The Medicare and Medicaid Patient Protection Act was enacted in 1987 and prohibits the billing for services that were not
provided. Further, this act prohibits other false statements with regard to billing.
Question 73
Nurse practitioner education is continuous and ongoing. Which of the following statement is NOT accurate regarding a nurse
practitioner's education?
Nurse practitioner programs are evaluated by the National Organization of Nurse Practitioner Faculties (NONPF) and the
American Association of Colleges of Nursing (AACN).
The AACN has developed a practice doctorate in nursing.
The National Organization of Nurse Practitioner Faculties regards the practice doctorate of nursing as an important
evolutionary step for the preparation of nurse practitioners.
The criteria set by the National Organization of Nurse Practitioner Faculties and American Association of Colleges of Nursing
establishes a basis for evaluating medical programs as well as practitioner programs.
The Correct answer is:
The criteria set by the National Organization of Nurse Practitioner Faculties and American Association of Colleges of Nursing
establishes a basis for evaluating medical programs as well as practitioner programs
The criteria set by the National Organization of Nurse Practitioner Faculties and American Association of Colleges of Nursing
are only established to evaluate nurse practitioner programs. The organizations do not establish criteria for the evaluation of
medical programs.
Question 74
You are evaluating a 32-year-old sexually active man in the clinic. You suspect he has a urethral infection due to urethral
discharge and complaints of dysuria. When you obtain testing, you find gram-negative cocci. The nurse practitioner knows
that this gram-negative cocci is more specifically identified as:
N. gonorrhoeae
E. coli
Staphylococcus aureus
U. genitalium
The Correct answer is:
N. gonorrhoeae
Gonorrhea, caused by the gram-negative diplococcus N. gonorrhoeae, is one of the most common sexually transmitted
diseases. This pathogen has a short incubation period, 1 to 5 days. Also, this pathogen is likely to cause infection in 20-30% of
109
men who come in contact with an infected female partner and up to 80% of men who have sex with a male partner
Question 75
In terms of having a laboratory, the NP’s practice must:
get approval by the state
get approval by the federal government
comply with the State Laboratory Administration and the CLIA requirements
all of the above
Correct answer:
all of the above
The laboratory must obtain and fill out the necessary paperwork needed to comply with state (State Laboratory
Administration) and federal (CLIA) requirements. This is done to get the approval of the state and federal government for the
laboratory.
Question 76
You are managing a patient who has irritable bowel syndrome (IBS). Altering the gut pain threshold in IBS is a possible
therapeutic outcome with the use of:
amitriptyline (Elavil)
loperamide (Imodium)
dicyclomine (Bentyl)
Metronidazole (Flagyl)
The Correct answer is:
Amitriptyline (Elavil)
A low dose tricyclic antidepressant or a selective serotonin reuptake inhibitor can be helpful in altering the gut pain threshold,
resulting in less abdominal pain. Imodium and Bentyl are prescribed to treat diarrhea. Flagyl is not used as treatment for
irritable bowel syndrome, but is used to treat certain types of infectious colitis.
Question 77
The part of the joint anatomy that acts as padding and is filled with synovial fluid is which of the following?
articular cartilage
bursae
ligaments
synovial space
Correct answer:
bursae
The bursae are present on the anterior and posterior areas of a joint. They act as padding and are filled with synovial fluid.
Question 78
To eliminate or support the diagnosis of meningitis, lumbar puncture with CSF evaluation is necessary. In a person who has
bacterial meningitis, the FNP might find all of the following EXCEPT:
pleocytosis
elevated CSF opening pressure
increased CSF glucose amount below the normal level of about 40% of the plasma level
elevated CSF protein level
Correct answer:
increased CSF glucose amount below the normal level of about 40% of the plasma level
This is not a typical finding for bacterial meningitis. Evaluation of CSF would show reduced CSF glucose amount below the
normal level of about 40% of the plasma level.
Question 79
The theory of development that sets forth a hierarchy of needs for healthy, creative individuals was put forth by which of the
following?
Abraham Maslow
Carl Rogers
Carol Gilligan
L.S. Vygotsky
110
Correct answer:
Abraham Maslow
Abraham Maslow put forth a theory of basic needs and human potential derived from study of healthy, creative individuals.
These needs include: physiologic; safety, security and stability; affiliation, acceptance and love; ego, self-worth, confidence,
competence and success; and self-actualization.
Question 80
Which of the following types of headache has a higher risk of suicide in males?
cluster headaches
migraine headaches
muscle tension headaches
temporal arteritis
Correct answer:
cluster headache
A cluster headache is a sudden onset of severe “ice-pick” headaches behind one eye that occurs several times a day. The
attacks happen at the same times daily and the cause is unknown. There is a higher risk of suicide in males who have this type
of headache as compared with the other types of chronic headaches.
Question 81
In terms of risk management, which of the following would NOT be a high-risk patient?
a 45-year-old disabled male with multi-system failure
a low-functioning 23-year-old female with Down syndrome
a 65-year-old female who takes two drugs for hypertension
a 19-year-old cocaine addict
Correct answer:
a 65-year-old female who takes two drugs for hypertension
Certain patient characteristics should alert the FNP to be aware of risk management strategies. High-risk patients include those
with the following characteristics: multisystem failure, low intelligence, polypharmacy, noncompliant behavior, positive
review of systems, substance abuse and litigiousness. A 65-year old female who takes only two drugs would not be considered
high risk.
Question 82
Which of the following is not true in regard to a durable power of attorney for health care?
It is limited to the circumstances of terminal illness.
It is legally binding.
It is often accompanied by a power of attorney over financial issues as well.
It is flexible enough to carry out the patient’s wishes throughout the course of an illness.
Correct answer:
It is limited to the circumstances of terminal illness.
This is not true. A durable power of attorney for health care is not limited to the circumstances of terminal illness. A living
will is an advance directive concerning terminal illness only.
Question 83
In regards to hypothyroidism, the nurse practitioner knows that periodic monitoring for this condition is indicated in the
presence of which of the following conditions?
Down syndrome
male gender
alcoholism
antihypertensive use
The Correct answer is:
Down syndrome
Thyroid disease exists in less than 7% of the population but is seen commonly in patients with Down syndrome. Other risk
factors associated with this condition include advancing age, use of certain medications that alter thyroid hormone synthesis,
female gender, postpartum, strong family history of autoimmune diseases, and history of radiation or surgery to the head and
neck.
111
Question 84
When assessing for an urinary tract infection in a geriatric patient, it is important for the nurse practitioner to know:
There is always a fever present.
It is an uncommon finding in catheterized patients.
Elderly people may or may not demonstrate symptoms other than mental status change with this type of infection.
Dysfunctional voiding pattern or infrequent voiding are not contributing factors to the development of a urinary tract infection.
The Correct answer is:
Elderly people may or may not demonstrate symptoms other than mental status change with this type of infection
Geriatric patients with a urinary tract infection usually do not have a fever. It is common in patients with catheters and
dysfunctional voiding pattern and infrequent voiding are contributing factors in the development of a urinary tract infection.
Question 85
What is the name of the agreement between a physician and a nurse practitioner (NP) outlining the NPs role and responsibility
to the clinical practice?
a clinical guideline
a collaborative agreement
a standard of practice contract
a consultation contract
The Correct answer is:
A collaborative agreement
A collaborative agreement must be signed by both the nurse practitioner and the physician who supervises him or her and is
usually submitted to the board of nursing. Agreements must be reviewed periodically. Criteria on how the collaborative
agreement should be written are provided by each state's board.
Question 86
The FNP has a 45-year-old male patient who is disabled due to a work-related injury. He has become withdrawn, does little
else than nap and watch television all day. Which of Erikson’s developmental stages is this patient not meeting?
trust vs. mistrust
generativity vs. stagnation
initiative vs. guilt
industry vs. inferiority
Correct answer:
generativity vs. stagnation
The central task of middle age (40 to 65 years) is “generativity vs. stagnation.” The primary developmental task is one of
contributing to society and helping to guide future generations. This patient has signs of not meeting this stage of
development.
Question 87
Which of the following statements are NOT true concerning entries in the medical record?
Maintaining a complete medical record is important to comply with licensing and accreditation requirements.
Medical record entries should be made in clear and concise language that can be understood by healthcare professionals who
treat the patient.
Allegations of improper care can always be defended without documentation in a medical record if staff witnessed the care.
Maintaining a complete medical record enables the healthcare provider to establish that a patient has received adequate care.
The Correct answer is:
Allegations of improper care can always be defended without documentation in a medical record if staff witnessed the care
Some courts have dismissed claims involving incomplete records. However, some cases have been decided without
documentation of the allegation in the medical record.
Question 88
At which stage of development would a child engage in parallel play?
school age
infant
preschool
toddler
112
Correct answer:
toddler
Parallel play is a common occurrence in the toddler years. When interacting with other toddlers, the child engages in similar
play activity but with minimal interaction.
Question 89
In terms of hazardous waste, which of the following statements is incorrect?
Red plastic containers specially made for sharp instruments and needles are required by law.
Diapers and bloody materials are deposited in red bags in trash cans.
Clinics may dispose of their own hazardous waste.
Personnel who pick up the hazardous waste must be certified in that area.
Correct answer:
Clinics may dispose of their own hazardous waste.
This statement is incorrect. Personnel who pick up the hazardous waste must be certified in that area; a staff person cannot do
the job. Therefore, clinics must hire waste disposal services.
Question 90
You are treating a patient with the diagnosis of bulimia nervosa. The nurse practitioner understands that the following is most
consistent with this diagnosis:
Patients with bulimia nervosa develop hyperkalemia from laxative abuse.
Most patients with bulimia nervosa are significantly obese.
Patients with bulimia nervosa usually present asking for treatment.
Patients with bulimia nervosa have periods of anorexia.
The Correct answer is:
Patients with bulimia nervosa have periods of anorexia
Bulimia nervosa is a secretive disease with relatively few easily noted clinical findings. It is more common in women and
typically is present for many years before the patient presents for treatment or before the disorder is noted by a healthcare
provider. It is estimated that approximately 30-50% of individuals with anorexia will exhibit characteristics of bulimia
nervosa, by eating large amounts of food and then getting rid of the food from the body by using laxatives or even vomiting.
Because of laxative usage, patients with bulimia nervosa are at risk of hypokalemia, and not hyperkalemia. Also, patients with
bulimia nervosa are typically of average weight.
Question 91
Type 2 diabetes mellitus testing should be considered in all adults who are overweight and have additional risk factors. All of
the following are additional risk factors to consider EXCEPT:
history of cardiovascular disease
women with polycystic ovarian syndrome
member of a high risk population
history of hypotension
Correct answer:
history of hypotension
This is not one of the additional risk factors for type II diabetes. All of the other choices are additional risk factors along with:
physical inactivity, first-degree relative with diabetes, women who delivered an infant weighing over 9 pounds, women who
have had gestational diabetes, hypertension, HDL cholesterol level of < 35 mg/dL, triglyceride level > 250 mg/dL, impaired
glucose tolerance and other clinical conditions associated with insulin resistance.
Question 92
A nurse practitioner wants to make a change in the state's law concerning the dispensing of prescription medications by nurse
practitioners. Who would this case need to be taken to?
state board of nursing
state board of pharmacy
nursing specialty organization
state legislature
The Correct answer is:
State legislature
Healthcare policy within the states is codified or enacted into law by the respective state legislature. The boards should have
significant input in the process, such as providing the research data and expert "testimony" that the legislature needs to make
informed decisions. However, boards, such as the state nursing boards, do not have the governing power to pass state laws.
113
Question 93
A 72-year-old woman is diagnosed with giant cell arteritis. Due to the severity of this diagnosis, the nurse practitioner
understands that concomitant disease seen with this condition includes:
acute pancreatitis
psoriatic arthritis
Reiter syndrome
polymyalgia rheumatica
The Correct answer is:
Polymyalgia rheumatica
These two conditions, giant cell arteritis and polymyalgia rheumatica, are autoimmune vasculitis and are often seen together.
Also, polymyalgia rheumatica is a systemic disease affecting medium-sized and large-sized vessels that causes inflammation
of the temporal artery.
Question 94
One of the theories of development is Piaget's (1969) Cognitive Development Theory. This theory focuses on intellectual
changes and the way they relate to the environment. Which of the following is NOT a stage of this continuous interaction
sequential manner?
Abstract Operations
Concrete Operations
Preoperational Thinking
Formal Operations
The Correct answer is:
Abstract Operations
The stages are listed in this order:
1-Sensorimotor (infancy)
2-Preoperational Thinking
3-Concrete Operations
4-Formal Operations
There is no Abstract Operations stage in this theory model.
Question 95
The following component should be included in a history of a patient who is new to your clinic:
interval history, past medical history, family medical history, dietary habits, substance use, sexual practices
past medical and surgical histories, family medical history, psychosocial history, physical activity, tobacco and substance use,
sexual practices
general history on the physical examination for is sufficient; there is no need of interview
past medical and surgical history, family medical and surgical histories, psychosocial history, diet and exercise habits,
chemical use, sexual practices, review of systems
The Correct answer is:
Past medical and surgical histories, family medical history, psychosocial history, physical activity, tobacco and substance use,
sexual practices
Past medical and surgical histories, family medical history, psychosocial history, physical activity, tobacco and substance use,
sexual practices are all areas that are necessary to investigate a new patient's health.
Question 96
Which of the following is NOT a preventative measure for avoiding the ticks associated with Lyme disease?
using insect repellents
wearing long-sleeved shirts and long pants
taking a single 200 mg dose of doxycycline orally
avoiding areas with known or potential tick infestation
This is not a preventative measure. After a tick bite occurs, a single 200 mg dose of doxycycline taken orally seems to be
effective in reducing the Lyme disease risk.
Question 97
The following are all reasons to keep a child out of school or daycare EXCEPT for which of the following?
diarrhea with blood or mucus
purulent conjunctivitis
114
streptococcal pharyngitis, until 3 days after initiation of treatment
impetigo, until 24 hours after initiation of treatment
The child may go back to school 24 hours after initiation of treatment for streptococcal pharyngitis. The remaining answer
choices are all correct. Other childcare exclusions are: (1) pediculosis, until after the first treatment is given, (2) scabies, until
after treatment is given, (3) varicella, until all lesions are crusted over, (4) pertussis, until after 5 days of antibiotics therapy,
(5) hepatitis A, until 1 week after illness began and (6) tuberculosis, until no longer infectious.
Question 98
Which of the following does NOT correctly relate to end-of-life care?
From an insurance and hospice point of view, the end-of-life state begins six months before death.
Pain management is a priority in end-of-life care.
Medical care often shifts from supportive intervention to invasive intervention.
Support for family caregivers is important.
This is not true. Medical care often shifts from invasive intervention aimed at prolonging life to supportive intervention that
focuses on control of symptoms.
Question 99
There is a greater recognition of the use by patients of complementary and alternative modalities and medicines
(CAM) today than ever before. Which of the following is an inaccurate statement regarding the practice environment
involving CAM?
Research suggests that 40-50% of patients are currently utilizing some form of complementary or alternative therapy.
There is a wealth of evidenced based research that supports CAM.
Nurse practitioners, as providers, need to learn about common CAM treatments and about how some herbal products interact
with prescription medications.
The National Center for Complementary and Alternative Medicine is a federal government's lead agency for scientific
research on CAM.
The Correct answer is:
There is a wealth of evidenced based research that supports CAM
There is a dearth of research supporting such treatment modalities.
Question 100
You are evaluating a patient who is having an asthma flare. What would you expect to find upon physical
examination?
inspiratory crackles
a tripod posture
hyperresonance on thoracic percussion
increased vocal fremitus
Physical findings upon examination include hyperresonance on thoracic percussion, decreased tactile fremitus, wheezing,
prolonged expiratory phase, and low diaphragms. Individuals asthma can try to avoid the things that triggers the asthma attack,
if known, in order to help prevent future asthma flare-ups.
test 5 aanp
Question 1
When evaluating a 56-year-old Caucasian female for menopause, the nurse practitioner has knowledge that the primary
function of FSH is:
triggering infiltration
stimulation of maturation of ovarian follicles
inhibiting the release of LH from the pituitary gland
milk secretion
The Correct answer is:
Stimulation of maturation of ovarian follicles
The FSH stimulates the maturation of ovarian follicles and results in a dominant follicle. The production and release of LH is
regulated by estrogen. Milk secretion depends on prolactin
115
Question 2
In terms of cranial nerves, which CN is facial nerve?
CN 7
CN 1
CN 5
CN 4
Correct answer:
CN 7
CN 7 is the facial nerve. It is associated with being able to puff the cheeks, raise eyebrows, and smile.
Question 3
A mother brings her 5-year-old daughter into the office with complaints that she is "acting funny." She states that she has brief
periods when she does not respond to her. Then, she suddenly responds and acts as if nothing has happened. The nurse
practitioner would need to evaluate the child further for:
Horner's syndrome
Avoidance disorder of childhood
Simple partial seizure activity
Petit mal or absence seizures
The Correct answer is:
Petit mal or absence seizures
The description of the patient’s symptoms indicates a petit mal or absence seizure. With petit mal or absence seizure, the
patient does not show symptoms, such as convulsions. Instead, the patient will stare for a short period and fail to response to
anyone or anything around him or her. Although the etiology of the majority of seizures is unclear, the client should be
referred for neurologic evaluation to determine if an underlying abnormality is present.
Question 4
Which of the following effects on urinary continence would be found in a person using opioids?
urinary retention
increase in frequency of voiding
relaxing internal urethral sphincter
none of the above
Correct answer:
urinary retention
For those taking an opioid medication the effect on urinary continence would include urinary retention. Other effects include:
overflow incontinence, alteration in sensorium and fecal impaction.
Question 5
At about what age can the FNP expect to see an infant keep his back straight when pulled to sitting?
1 – 2 months
3 – 4 months
5 months
9 – 11 months
Correct answer:
5 months
At 5 months an infant will typically keep his back straight when he is pulled to a sitting position. At this age infants also bear
weight on their legs when standing, play with their feet and sit with support.
Question 6
When assessing an 80 year old patient, you note that she is less active from chronic pain, she is low income, and she has little
support. What does this place her at risk for?
Hepatitis.
Diabetes.
Carcinoma.
Obesity.
116
Correct answer: Obesity Obesity is linked to a number of illnesses. Even though she may not have as much money for food,
the food is probably empty calories. This increases the risk of obesity.
Question 7
With a giardia lamblia diagnosis, the NP would MOST likely prescribe what medication?
Metronidazole.
Erythromycin.
Ampicillin.
Trimethoprim-sulfamethoxazole.
Correct answer: Metronidazole The preceding drugs are typically used in treating the following: - Campylobacter jejuni:
Erythromycin - Salmonella: Ampicillin - Shigella: Trimethoprim-sulfamethoxazole - Giardia lamblia: Metronidazole
Question 8
Some states require a written protocol for NP practice. Which of the following statements about a protocol is incorrect?
A protocol need not be written; it may be oral.
It is considered a standard.
It consists of mutually agreed-upon medical guidelines between physician and NP.
It defines the individual and shared responsibilities of the physician and NP.
Correct answer:
A protocol need not be written; it may be oral.
A protocol is a written instrument that guides the NP in collecting data from the patient and recommends specific action based
upon the collected data. The protocol provides a guideline for a minimum level of safe practice
Question 9
Which of the following tests would you most likely order for a patient whom you suspect has iron deficiency anemia?
CBC with differential or smear
TIBC
ferritin
all of the above
Correct answer:
all of the above
You might order all of the tests. You might also order a serum iron or special tests to determine underlying bleed, if suspected.
Question 10
Simvastatin is the generic form of which of the following drugs?
Lipitor
Zocor
Tricor
Questran
Correct answer:
Zocor
Zocor (Simvastatin) is a recommended drug therapy for hyperlipidemia. The daily dose range for this drug is 5 – 80 mg.
Question 11
The most common cause of anemia is which of the following?
obesity
iron deficiency
alcohol abuse
drug abuse
Correct answer:
iron deficiency
The most common cause of anemia is iron deficiency. The prevalence of iron deficiency is currently estimated to be
approximately 3% for children 1 to 5 years old.
Question 12
117
When assessing a patient with suspected appendicitis, the nurse practitioner knows that the peak ages for occurrence of this is:
1 to 15 years
10 to 30 years
25 to 55 years
30 to 50 years
The Correct answer is:
10 to 30 years
Acute appendicitis is an inflammatory disease of the vermiform appendix that is caused by an infection or an obstruction. The
peak age of patients is 10 to 30 years. Further, this condition is uncommon in infants and the elderly.
Question 13
You are educating a group of adults about depression. You are NOT likely to include which of the following statements in
your talk?
When adults live in long-term care facilities they are less likely to suffer from depression.
No significant relationship has been found between race and mood disorders.
Once a person has experienced a depressive episode, the incidence of depression increases.
Depression is more common in young women.
Correct answer:
When adults live in long-term care facilities they are less likely to suffer from depression.
This is incorrect. You would tell the group that when adults live in long-term care facilities they are more likely to suffer from
depression. Older adults are at a high risk for depression because of the multiple losses and health problems that frequently
occur at this stage of life.
Question 14
A patient with folliculitis has been prescribed Mupirocin ointment to treat the area. What is the dosage?
Apply tid before antibiotic ointment.
2%, apply bid X 10 days and cover with DSD.
Apply to area bid-tid.
0.5 to 1 mg/kg/day PO in divided doses.
Correct answer: 2%, apply bid X 10 days and cover with DSD The preceding medications should be administered at the
following dosages: Mupirocin ointment - 2%, apply bid X 10 days and cover with DSD. Gentamicin Sulfate cream or
ointment - Apply to area bid-tid. Isoretinoin - 0.5 to 1 mg/kg/day PO in divided doses. Anhydrous ethyl alcohol with 6.25%
aluminum chloride - Apply tid before antibiotic ointment.
Question 15
The nurse practitioner knows that when assessing a burn, the type that results in the surface being raw and moist is:
a first-degree burn
a second-degree burn
a third-degree burn
a fourth-degree burn
The Correct answer is:
A second-degree burn
A first degree burn results in affected skin that blanches with ease. A third-degree burn results in the affected area being white
and leathery. There is no degree level of fourth-degree burn.
Question 16
A patient who has diarrhea as the main symptom of IBS may be prescribed all but which of the following?
Lomotil.
Imodium.
Lotronex.
Amitiza.
:
118
Correct Answer: Amitiza This medication is actually recommended for relief of long-term constipation and may make
diarrhea worse. The other choices are all anti-diarrheals and may be used to help treat IBS. Lotronex is usually prescribed only
when other treatments have not been successful.
Question 17
The following component should be included in a history of a patient who is new to your clinic:
interval history, past medical history, family medical history, dietary habits, substance use, sexual practices
past medical and surgical histories, family medical history, psychosocial history, physical activity, tobacco and substance
use, sexual practices
general history on the physical examination for is sufficient; there is no need of interview
past medical and surgical history, family medical and surgical histories, psychosocial history, diet and exercise habits,
chemical use, sexual practices, review of systems
The Correct answer is:
Past medical and surgical histories, family medical history, psychosocial history, physical activity, tobacco and substance use,
sexual practices
Past medical and surgical histories, family medical history, psychosocial history, physical activity, tobacco and substance use,
sexual practices are all areas that are necessary to investigate a new patient's health.
Question 18
You use the learning theory that appreciates that learners are unique and learn best when treated with positive regard. This
learning theory is which of the following?
cognitive theory
behaviorist theory
humanistic theory
andragogy
Correct answer:
humanistic theory
The humanistic theory appreciates that learners are unique persons. It recognizes that they learn optimally when treated with
positive regard, when their uniqueness is considered, and when they are involved in decision making.
Question 19
The FNP has a 55-year-old male patient who has recently been diagnosed with bladder cancer. The FNP understands that all of
the following are true statements in regard to bladder cancer EXCEPT:
Most patients with newly diagnosed bladder cancer have superficial disease which allows for effective treatment.
Follow-up of bladder cancer patients is crucial because recurrence is often seen.
Long-term survival is rare with the invasive and non-invasive form of the disease.
Treatment is dictated in invasive bladder cancer by the type of tumor and degree of invasion among other things.
Correct answer:
Long-term survival is rare with the invasive and non-invasive form of the disease.
Long-term survival is the norm with the non-invasive form of the disease. With invasive disease, treatment is dictated by the
type of tumor, degree of invasion and presence of metastatic disease, so long-term survival is based on numerous factors.
Question 20
Lead poisoning remains a serious public health problem. Which of the following is the least likely to present a lead hazard?
lead-glazed pottery
lead-soldered vessels used for cooking
fumes from burnt casings of batteries
paint in a home built in 1995
paint in a home built in 1995
This is the least likely source to present a lead hazard. Lead-based paint has not been available in the USA for more than 30
years. Therefore, a home built in 1995 would not contain lead-based paint.
Question 21
Alzheimer’s disease involves what is known as the three A’s. All of the following are one of the components of the three A’s
EXCEPT:
119
aphasia
apraxia
angina
agnosia
Correct answer:
angina
Angina is not one of the three A’s associated with Alzheimer’s disease. The three A’s include: aphasia (difficulty verbalizing);
apraxia (difficulty with gross motor movements such as walking); and agnosia (inability to recognize familiar objects and
people).
Question 22
You are treating a patient with a large red, tender burn to one arm. What is the estimated involved body surface area?
1%
9%
30%
55%
The Correct answer is:
9%
A person's palmar surface represents a body surface area (BSA) of 1% of the total body surface area (TBSA) and is a good
guide in the estimation of BSA involvement. The entire total surface area for one arm is 9%, according the rule of nines.
Question 23
The Occupational and Safety Health Act requires certain situations to be reported. Which of the following would need to be
reported?
a minor closed head injury where no loss of consciousness occurred
a twisted ankle that improved with ice therapy and an Ace wrap
a back strain while lifting a box that resulted in reassignment to office work for a week
a 3 cm abrasion on the forearm
The Correct answer is:
A back strain while lifting a box that resulted in reassignment to office work for a week
The Occupational and Safety Health Act (OSHA) requires the reporting of any injury or illness that requires more than firstaid. Also, the injury has to result in lost work time, limited work status, death, or loss of consciousness.
Question 24
When assessing a patient for dietary habits, what will NOT factor into poor eating habits?
Gender.
Income.
Education.
Medical conditions.
Correct answer: Gender Women typically eat better than men, but that does not always factor into poor eating habits. The
other answers can lead to poor eating habits. This can mean poor nutrition.
Question 25
You are volunteering at a homeless clinic to gain clinical experience. The nurse practitioner knows that which statement is true
regarding this?
Volunteerism negates susceptibility to lawsuits.
Malpractice insurance will be needed.
Malpractice coverage will be provided by the state where the clinic is located.
Malpractice insurance is not necessary when working volunteer status.
The Correct answer is:
Malpractice insurance will be needed
The nurse practitioner should always have malpractice insurance. The "good Samaritan" law protects professional volunteers
in some states, but in the event of being sued, professional liability should be carried.
Question 26
120
The ecological model developed and described by Urie Bronfenbrenner that is a person-place-process model includes all of the
following systems EXCEPT:
Microsystems
Mesosystem
Endosystem
Exosystem
Correct answer:
Endosystem
This is not part of the person-place-process model of Bronfenbrenner. The person-place-process model includes:
microsystems, mesosystem, exosystem and macrosystem. Understanding how and why a child changes or stays the same over
time requires examination, not only of the child’s emerging capabilities but the quality of the settings where children spend
time.
Question 27
Which of the following medications provides the greatest analgesic effect in the shortest amount of time for a tension-type or
migraine headache?
Naprosyn
Aleve
Anaprox
Both Aleve and Anaprox
Correct answer:
Both Aleve and Anaprox
The National Health Foundation Guidelines advise the use of rapid-onset NSAIDs such as ibuprofen in high doses with
booster doses for tension-type and migraine headaches. Plain naproxen (Naprosyn) has a relatively slow onset of analgesic
activity, whereas naproxen sodium (Aleve or Anaprox) use is associated with a significantly more rapid onset of pain relief.
Question 28
You have a child in the office that has had a severe anaphylactic reaction to eggs in the past. The nurse practitioner
understands that she cannot administer the following immunization:
hepatitis B
IPV
varicella
influenza
The Correct answer is:
Influenza
The influenza vaccine is contraindicated for any person who has had anaphylactic reaction to eggs because the vaccine is made
from eggs.
Question 29
Which of the following is federally mandated testing for infants?
hemoglobin
hematocrit
PKU test
lead screening
Correct answer:
PKU test
The PKU test is federally mandated. Severe mental retardation can occur if PKU is not treated early. PKU is the inability to
metabolize phenylalanine to tyrosine because of a defect in the production of the enzyme phenylalanine hydroxylase.
Question 30
You are examining a male infant, age 9 months, and notice an undescended testicle on the right side. When counseling the
mother, the nurse practitioner understands the following to be true of cryptorchidism EXCEPT:
This is where the testis does not descend with massage of the inguinal area.
You cannot exam the infant in a warm room.
Increased risk of testicular cancer occurs if testicles are not removed from the abdomen.
Surgical correction within the first year of life is necessary if it does not spontaneously descend.
121
The Correct answer is:
You cannot exam the child in a warm room
In order to see if the muscle will relax and the testicle will descend, the infant should be sitting and the exam room should be
warm to relax the muscles when massaging the inguinal canal. Another option is to examine the infant during or after a warm
bath.
Question 31
You are discussing Depo-Provera therapy with a 17-year-old adolescent who cannot remember to take her birth control pills.
You must warn her of side effects and long term effects. The Family Nurse Practitioner knows that which of the following is
commonly found after a year of using this agent?
hypermenorrhea
weight gain
cystic acne
insomnia
The Correct answer is:
Weight gain
Depo-Provera is given every 90 days in the form of an intramuscular injection. An unfortunate side effect to this agent is
weight gain. While some patients enjoy the loss of menses (amenorrhea or hypomenorrhea), some find this occurrence
worrisome. Acne is improved with Depo usage and insomnia is not a known side effect.
Question 32
You are treating a patient who presents with a painful, blistering thermal burn on his right hand and digits. What is the best
way to treat this?
Wrap the burn loosely with a nonadherent dressing and prescribe anti-inflammatory agents.
Apply silver sulfadiazine cream to the burn and apply a thick dressing.
Apply topical antibiotic ointment and leave open to air.
Refer to a burn care specialist.
The Correct answer is:
Refer to a burn care specialist
Patients with any burn involving areas of high function such as the hands and feet or ones that have significant cosmetic
consequence such as the face should be referred promptly to specialty care.
Question 33
Which of the following statements about tinnitus is least accurate?
Most people suffer from occasional intermittent tinnitus.
Continuing tinnitus usually means that the person has suffered sensory hearing loss.
The overall prevalence of unexplained tinnitus is 11%.
Approximately 10% of the population in the Untied States suffers from chronic tinnitus that causes severe distress.
Correct answer:
Approximately 10% of the population in the Untied States suffers from chronic tinnitus that causes severe distress.
This is the least accurate statement. Approximately 1% of the population in the United States suffers from chronic tinnitus that
causes severe distress. This type of tinnitus requires some kind of management intervention.
Question 34
You are evaluating a 35-year-old pregnant female who has an indication for genetic counseling. What might this be?
maternal age
increased maternal alpha-fetoprotein
drug exposure during the first trimester
history of previous stillbirth
The Correct answer is:
Maternal age
Women 35 years of age and older are at high risk for pregnancies involving congenital abnormalities. The remaining answer
choices are all reasons for genentic counseling, but to a much lesser degree.
Question 35
A 70-year-old male comes to the FNP with a raised, red lesion in his oral cavity. He tells the FNP that he is afraid that it is
cancer. Besides evaluating the lesion the FNP evaluates the man’s risk for oral cancer. These risks include all of the following
EXCEPT:
122
poor diet
alcohol abuse
tobacco abuse
advanced age
Correct answer:
poor diet
Poor diet is not a risk factor for oral cancer. Alcohol and tobacco use and advanced age are risks.
Question 36
The stethoscope is placed slightly off the chest of an elderly patient who is suspected of having a murmur. The sound
produced is very loud and accompanied by palpable thrill. Which of the following murmur grades does this describe?
III/VI.
IV/VI.
V/VI.
II/VI.
Correct answer: V/VI Each of the preceding murmur grades is defined below: III/VI - Moderately loud IV/VI - Loud with
palpable thrill V/VI - Very loud and heard with the stethoscope partially off the chest; palpable thrill II/VI - Quiet but heard as
soon as the stethoscope is placed on the chest
Question 37
Which of the following is an absolute contraindication for fibrinolysis in ST segment elevation myocardial infarction?
any prior intracranial hemorrhage
history of chronic, severe, poorly controlled hypertension
pregnancy
active peptic ulcer
Correct answer:
any prior intracranial hemorrhage
Any prior intracranial hemorrhage is an absolute contraindication for fibrinolysis in ST segment elevation myocardial
infarction. The other choices are relative contraindications.
Question 38
When assessing a child during a well-child exam, the nurse practitioner (NP) notices that genu valgum is present. The NP
knows that this is normal from:
1-2 years old
3-5 years old
8-10 years old
12-16 years old
The Correct answer is:
3-5 years old
Knock knees or genu valgum is a condition that affects the knees in some children. With knock knees, the knees angle in and
touch one another when the legs are straightened. Genu valgum can be seen in children from ages 3 to 5. Genu valgum is
corrected naturally, without treatment interventions, by the time the child reaches 10 years of age.
Question 39
Which of the following is the correct laboratory norm for Mean Corpuscular Hemoglobin Concentration (MCHC)?
100 – 200 g/dL
50 – 100 g/dL
31.0 – 37.0 g/dL
21.0 – 27.0 g/dL
Correct answer:
31.0 – 37.0 g/dL
MCHC is a measure of the average color of the RBCs in a sample of blood. It is decreased in iron deficiency anemia and
thalassemia and normal in the macrocytic anemias. The normal is 31.0 – 37.0 g/dL.
Question 40
123
Which of the following is NOT one of the Jones criteria for pelvic inflammatory disease (PID)?
lower back pain
cervical motion tenderness
adnexal tenderness
lower abdominal tenderness
Correct answer:
lower back pain
The Jones criteria are the major criteria for PID. Only one of the criteria is necessary to diagnose and treat for PID. All of the
choices are one of the Jones criteria except lower back pain.
Question 41
The program that provides health insurance for uninsured children who qualify is which of the following?
COBRA
SCHIP
OBRA
Medicare
Correct answer:
SCHIP
The State Children’s Health Insurance Program (SCHIP) provides health insurance for uninsured children who qualify. It is
important for the NP to know that a child may be eligible for SCHIP even if the family is not eligible for Medicaid.
Question 42
Of the following laboratory procedures, which one is used as an adjunct for evaluating herpetic infections?
Pap smear
Tzanck smear
Gram stain
KOH slide
Correct answer:
Tzanck smear
The Tzanck smear is used as an adjunct for evaluating herpetic infections (oral, genital or skin). A positive smear will show
large abnormal nuclei on the squamous epithelial cells.
Question 43
An adult male patient reports fatigue and nausea. In the last 30 days, he has travelled extensively in Latin America. He also
reports clay-colored stools and a nearly-constant itching sensation. The patient should be further evaluated for which of the
following?
Hepatitis A.
Intestinal parasites.
Hodgkin' disease.
Lyme disease.
Correct Answer: Hepatitis A This patient's symptoms are consistent with Hepatitis A. The fact that he has recently traveled to
a developing area of the world also puts him at higher risk. His liver functions should be monitored and tests for Hepatitis A
performed.
Question 44
Fraud and abuse by healthcare providers have increased during the last several years. Which of the following is correct
concerning fraud and abuse?
The federal Health Insurance Portability and Accountability Act (HIPAA) allow for broad penalties for defrauding a health
plan.
The Health Insurance Portability and Accountability Act (HIPAA) prohibits knowingly and willfully offering, paying,
soliciting, or receiving anything of value in order to induce furnishing of services under federal and state programs.
The federal False Claims Act maintains a list of "safe harbors," which are arrangements that will not be considered a
violation of the anti-kickback law.
There are no laws that limit physicians from directing patients to their privately owned laboratories or diagnostic centers.
The Correct answer is:
124
The federal Health Insurance Portability and Accountability Act (HIPAA) allow for broad penalties for defrauding a health
plan
The Health Insurance Portability and Accountability Act (HIPAA) provides required guidelines a health plan, provider and
healthcare organization must adhere to in regards to fraud and abuse. Also, HIPAA has privacy rules that outlines protective
measures that healthcare organizations must follow in order to keep patient health information private. Or, in the case of health
insurance plans, HIPAA has mandates the health plans must follow in order to keep member health information private.
Further, the federal anti-kickback statute prohibits knowingly and willfully offering, paying, soliciting, or receiving anything
of value in order to induce furnishing of services under federal and state programs. The federal False Claims Act imposes civil
liability on anyone who submits a false claim. The Stark I and II federal laws prohibit physicians from self-referring patients
and then receiving payment for the referral.
Question 45
The tendency of an individual to expend less effort when working collectively than when working individually is known as
which of the following?
informal behavior
social loafing
cognitive regress
none of the above
Correct answer:
social loafing
Social loafing is the tendency for individuals to expend less effort when working collectively than when working individually.
It directly challenges the logic that the productivity of the group as a whole should at least equal the sum of the productivity of
each individual in that group.
Question 46
When would you expect the uterine fundus in a pregnant woman to be at the level of the umbilicus?
12 weeks
16 weeks
20 weeks
28 weeks
Correct answer:
20 weeks
At 20 weeks (5th month) you would expect the uterine fundus to be at the level of the umbilicus. At this stage fetal heart tones
are heard with the fetoscope or stethoscope.
Question 47
A physician cannot refer a patient covered by Medicare to a clinical laboratory where the physician or an immediate family
member of the physician has a financial relationship. This is part of which of the following laws?
Stark Acts
HIPAA
ADA
Clinical Laboratory Improvement Amendments
Stark Acts
The Stark Acts establish rules regarding “Physicians’ Referrals to Health Care Entities With Which They Have Financial
Relationships.” In addition to clinical laboratories, a physician cannot refer a patient to certain designated health services when
the physician has a financial relationship with the facility offering the services. Even though the Stark Acts are aimed at
physicians, a nurse practitioner can become involved in an activity that violates the Stark Acts.
Question 48
You are doing a complete physical exam on a 25-year-old female patient with sleep apnea. Which of the following are you
most likely to find in this patient?
bruit
paroxysms of sweating
125
increased blood pressure
atrial fibrillation
Correct answer:
increased blood pressure
Increased blood pressure is the only choice that might be found in a patient with sleep apnea. Bruit might be found in renal
artery stenosis. Paroxysms of sweating may be found in a person with pheochromocytoma. Atrial fibrillation might be found
in a person with hyperthyroidism.
Question 49
An insurance company that pays for the medical care of its insured but does not deliver health care is called:
Medicaid
Indemnity Insurer
Managed Care Organization
Private Payer
Correct answer:
Indemnity Insurer
An Indemnity Insurer is an insurance company that pays for the medical care of its insured but does not deliver health care.
Indemnity insurers pay healthcare providers on a per-visit, per-procedure basis.
Question 50
Which of the following statements about characteristics of a theory is least accurate?
Theories interrelate concepts to create more useful ways of looking at particular phenomena.
Theories must be logical in nature.
Theories must differ from other validated theories, laws, and principles.
Theories must be relatively simple.
Correct answer:
Theories must differ from other validated theories, laws, and principles.
This is the least accurate statement. Theories must be consistent with other validated theories, laws, and principles, but they
will leave unanswered questions that need to be investigated.
Question 51
The sudden massive destruction of spirochetes after a Penicillin G injection that causes an immune-mediated reaction is which
of the following?
Reiter’s syndrome
Fitz-Hugh-Curtis syndrome
Jarisch-Herxheimer reaction
Neisseria gonorrhea
Correct answer:
Jarisch-Herxheimer reaction
The Jarisch-Herxheimer reaction is an immune-mediated reaction caused by the sudden massive destruction of spirochetes
after a Penicillin G injection. It usually resolves spontaneously. Treatment is supportive.
Question 52
An asthmatic child is brought in with an attack. The medical history shows that the attacks have become more frequent in the
last few months. What will you want to assess in the family interview?
Exercise.
Diet.
Environment.
Heart disease.
126
Correct answer: Environment The environment of asthmatics is important to maintaining their health. Exposure to secondhand
smoke, chemicals, and pollutants can cause asthma attacks. Determine whether something in the environment is causing the
attacks.
Question 53
Urinary incontinence is defined as “involuntary loss of urine which is sufficient to be a problem.” The type of urinary
incontinence that accounts for 80% in women younger than 60 years is which of the following?
urge incontinence
stress incontinence
overflow incontinence
functional incontinence
Correct answer:
stress incontinence
Stress incontinence is associated with activities that result in increased intraabdominal pressure, such as coughing, sneezing,
laughing, or lifting. It accounts for 80% of urinary incontinence in women younger than 60 years.
Question 54
You are teaching a group of student nurses about gynecological problems. You tell them all of the following about pelvic
inflammatory disease (PID) EXCEPT:
Consistent use of barrier contraceptives increases a woman’s susceptibility to the disease.
Up to 60% of cases are caused by Chlamydia trachomatis or Neisseria gonorrhea.
Bacterial vaginosis and trichomoniasis may also be present.
A new sex partner within 30 days of symptoms is a factor increasing susceptibility.
Correct answer:
Consistent use of barrier contraceptives increases a woman’s susceptibility to the disease.
This is not true. Consistent use of barrier contraceptives decreases a woman’s susceptibility to the disease. It is a protective
factor.
Question 55
Which cranial nerve is the acoustic nerve?
CN 1
CN 4
CN 7
CN 10
Correct answer:
CN 7
Cranial nerve 7 is the facial nerve. It is involved in such things as puffing cheeks, raising eyebrows, and smiling.
Question 56
The domain of learning that relates to values, attitude, feelings, and emotions is which of the following?
affective
psychomotor
cognitive
behaviorist
Correct answer:
affective
The affective domain relates to values, attitude, feelings, and emotions. It begins with an awareness, willing response,
commitment, prioritization, and finally integration into one’s lifestyle. Appropriate teaching methods include role play, role
modeling, discussions, audiovisuals, and printed materials.
Question 57
In terms of human growth and development there are certain fundamental domains. Under which of the following domains
would receptive and expressive language fall?
127
cognitive
physical
psychological
test 6 aanp
Question 1
Olecranon bursitis is located in which of the following places?
shoulder
knee
elbow
heel
Explanation:
Correct answer:
elbow
Olecranon bursitis is located in the elbow. There is pain and swelling behind the elbow. It is often described as a ball or sac
hanging from the elbow. Risk factors include prolonged pressure or trauma to the elbow.
Question 2
Which of the following are you most likely to do to treat a patient who has temporal arteritis?
ice pack on forehead
100% oxygen
administer Tegretol
refer to ER or ophthalmologist
Correct answer:
refer to ER or ophthalmologist
Arteritis is a systemic inflammatory process of the medium and large arteries of the body. If the temporal artery is involved, it
is called temporal arteritis. Permanent blindness may occur if it is not diagnosed and treated early.
Question 3
A 64-year-old Caucasian male has the following lipid level results:
Total cholesterol 240 mg/dL
LDL 140 mg/dL
HDL 35 mg/dL
Triglycerides 201 mg/dL
What class of medications will normalize his lipid elevations and decrease his risk of a cardiac event?
fibric acids
bile acid sequestrants
niacin
statins
The Correct answer is:
Statins
The only medication class that lowers elevated lipid levels and has proven efficacy in lowering the risk of cardiac events, even
for primary prevention, is statin medications. Further, statin therapy has been shown to reduce overall mortality due to
cardiovascular deaths
Question 4
Which of the following statements about herpes zoster (shingles) is least accurate?
More than 66% of those affected are older than 50 years.
Only 5% of cases occur in children younger than 15 years.
Herpes zoster occurs equally in males and females.
Radiotherapy is a protective factor for the disease.
Correct answer:
Radiotherapy is a protective factor for the disease.
All of the statements are accurate except this one. Radiotherapy is a factor increasing susceptibility for the disease.
128
Question 5
The nurse practitioner (NP) understands that the following is NOT true regarding the care of minors:
In most jurisdictions, minors under the age of 18 cannot receive healthcare services without permission of an adult who is a
parent or legal guardian.
If a 17-year-old contracts a sexually transmitted disease, the nurse practitioner must contact the parent or legal guardian.
In case of emergency, when no parent or guardian is available, care may be rendered.
If a 15-year-old becomes pregnant, she is an emancipated minor.
The Correct answer is:
If a 17-year-old contracts a sexually transmitted disease, the nurse practitioner must contact the parent or legal guardian
A teen with a STD is considered an emancipated minor, and the parent or legal guardian should not be contacted. The
remaining answer choices are true regarding the care of minors.
Question 6
While examining a patient you hear a high-pitched diastolic murmur at the second ICS at the right side of the sternum. You
know that this indicates which of the following?
aortic regurgitation
aortic stenosis
mitral stenosis
mitral regurgitation
Correct answer:
aortic regurgitation
A high-pitched diastolic murmur which is best heard at the second ICS at the right side of the sternum indicates aortic
regurgitation. Aortic stenosis produces a harsh and noisy murmur and radiates to the neck.
Question 7
Which of the following statements about end-of-life care is least accurate?
Withholding or withdrawing artificial nutrition or hydration from terminally ill or permanently unconscious patients is illegal.
Oral statements made by a patient may be legally valid advance directives.
Only a few states require “clear and convincing evidence” of a patient without decision-making capabilities that it is his or her
wish to forego life-sustaining treatment.
There is no legal requirement to notify risk management personnel before life-sustaining medical treatment can be stopped.
Correct answer:
Withholding or withdrawing artificial nutrition or hydration from terminally ill or permanently unconscious patients is illegal.
This is the least accurate statement. Just like any other therapy, fluids and nutrition may be withheld if it is the patient’s or
surrogate’s wish.
Question 8
In the latent stage of syphilis which of the following is the most common sign?
painless chancre
maculopapular rash on palms and soles that is not pruritic
no symptoms
valvular damage
Correct answer:
no symptoms
Syphilis has four stages: primary, secondary, latent, and tertiary. The latent stage is characterized by being asymptomatic
Question 9
You are seeing an uncircumcised client that presents with a complaint of not being able to retract the foreskin over the glans
penis. What is the likely diagnosis?
phimosis
129
paraphimosis
lateral phimosis
Peyronie's disease
The Correct answer is:
Phimosis
Phimosis results in the patient's inability to retract the foreskin from behind the glans penis. This condition occurs at any age
and is the result of poor hygiene and chronic infection.
Question 10
When counseling a patient with the human herpes virus 2 (HHV-2), the nurse practitioner knows that what percentage of
sexually active adults has serological evidence of this disease?
5%
10%
15%
25%
The Correct answer is:
25%
The human herpes virus 2 (HHV-2) causes painful ulcerated lesions and lymphadenopathy. Approximately 25% of adults who
are sexually active have serological evidence of this disease, although only a small percentage of these persons have
symptoms.
Question 11
Which of the following is NOT a live attenuated virus vaccine?
MMR
Varicella Zoster
injectable tri-valent influenza vaccine
FluMist
Correct answer:
injectable tri-valent influenza vaccine
The injectable trivalent influenza vaccine (TIV) does not contain live virus and is not shed. There is no risk of transmitting an
infectious agent to household contacts. The other choices are vaccines with live attenuated viruses.
Question 12
Kurt Lewin’s model of change management can best be described as which of the following?
preparation, change and continuity
grounding, amending and permanence
training, exchanging and connection
unfreezing, moving and refreezing
Correct answer:
unfreezing, moving and refreezing
The unfreezing stage involves getting to a point of understanding that change is necessary and getting ready to move away
from the current comfort zone. Moving is moving towards a new way of being. Refreezing is about establishing stability once
the changes have been made.
Question 13
Nurse practitioners are always reimbursed for their services as primary care providers under all but which of the following?
Medicare
Federal Employees Health Benefits Program
Managed Care Organizations
TRICARE
Correct answer:
Managed Care Organizations
130
Managed Care Organizations frequently have excluded NPs from being designated as primary care providers carrying their
own caseloads. Thus, in many MCOs, the only option for NPs is that of being a salaried employee.
Question 14
The knee joint includes four bones. Which of the following is NOT one of these bones?
distal femur
distal tibia
patella
proximal fibula
Correct answer:
distal tibia
The distal tibia is not one of the bones of the knee. The knee joint includes the distal femur, the patella, the proximal fibula,
and the proximal tibia.
Question 15
In terms of etiology of disease, incidence rates can best be described as which of the following?
a group at a certain point in time and the number within a group that has a particular disease or problem
interventions at the clinical stage of disease, directed at treatment and rehabilitation
the rate of involvement of causative agents
the rate of development of a disease in a group over a period of time
Correct answer:
the rate of development of a disease in a group over a period of time
Incidence rates describe the rate of development of a disease in a group over a period of time. They also describe the
continuing occurrence of new cases of disease.
Question 16
Your patient has an extensive infection of several adjacent hair follicles that form a mass with multiple drainage points. You
understand that this patient has which of the following?
folliculitis
carbuncle
furuncle
none of the above
Correct answer:
carbuncle
A carbuncle occurs when there is an extensive infection of several adjacent hair follicles that form a mass with multiple
drainage points. Folliculitis is a minor inflammation of hair follicles with or without pustules. A furuncle (abscess or boil) is a
more extensive infection of a hair follicle.
Question 17
A 23-year-old female who is breastfeeding is inquiring about alcohol use during lactation. The nurse practitioner understands
that:
Because of its high molecular weight, relatively little alcohol is passed into breast milk.
Infant intoxication may be seen with as little as one to two maternal drinks.
Drinking a glass of wine or beer will enhance the let-down reflex.
Maternal alcohol use causes a reduction in the amount of milk ingested by the infant.
The Correct answer is:
Maternal alcohol use causes a reduction in the amount of milk ingested by the infant
Alcohol has a low molecular weight and is highly lipid soluble. These characteristics allow it to have easy passage into breast
milk. Even in small amounts, alcohol ingestion by a nursing mother can cause a smaller amount of milk produced, reduction in
the let-down reflex and less rhythmic and frequent sucking by the infant, resulting in a smaller volume of milk ingested.
Question 18
A 42-year-old Caucasian male has newly diagnosed hypertension and has received a prescription for ramipril. He is otherwise
healthy and on no other medications. Which laboratory test would be important to monitor him?
131
PT with INR
potassium level
ALT/AST
calcium level
The Correct answer is:
Potassium level
Ramipril is an angiotensin-converting enzyme (ACE) inhibitor. Further, this medication causes retention of potassium.
Therefore, the patient's potassium level should be measured about one month after initiating therapy and again after dosage
changes. The remaining answer choices, PT with INR, ALT/AST and calcium level, are not specific laboratory tests for ACE
inhibitor therapy.
Question 19
Which of the following is the gold standard diagnostic test for thalassemia minor?
blood smear
serum ferritin
hemoglobin electrophoresis
none of the above
Correct answer:
hemoglobin electrophoresis
Hemoglobin electrophoresis is the gold standard diagnostic test for thalassemia minor. in beta thalassemia it is abnormal.
There is an elevated Hgb.A2, and Hgb F.
Question 20
What dietary regimen would be appropriate for an elderly client with chronic obstructive pulmonary disease?
high caloric, low protein, high carbohydrate
low caloric, low protein, high carbohydrate
high caloric, high protein, low carbohydrate
low caloric, high protein, low carbohydrate
The Correct answer is:
High caloric, high protein, low carbohydrate
By eating a high protein, high caloric and low carbohydrate diet, the patient with chronic obstructive pulmonary disease is able
to meet his or her nutritional needs. Additionally, a patient with chronic obstructive pulmonary disease, who retains carbon
dioxide, should avoid high carbohydrate foods because carbohydrates metabolize into carbon dioxide (CO2) as a waste
product.
Question 21
The type of nursing research that examines events of the past to enhance self-understanding is called:
phenomenology
historical research
grounded theory
ethnographic research
Correct answer:
historical research
Historical research examines events of the past to enhance self-understanding. It is a type of qualitative research that focuses
on the collection of data through interviews, observations and focus groups to enhance understanding about a particular area of
interest in nursing.
Question 22
Which of the following conditions is most likely to be found in a person with acute cholecystitis?
fever
jaundice
vomiting
palpable internal joint
The Correct answer is:
Vomiting
132
Acute cholecystitis symptoms include vomiting, right upper abdomen pain, and nausea. Fever, jaundice and a palpable internal
joint are not seen in patients with cholelithiasis. Cholelithiasis is defined as a condition in which there is the formation of
calculi or gallstones.
Question 23
A 8-year-old male child is called back into the office because his stool specimen for ova and parasites (O & P) came back
positive. The nurse practitioner knows what regarding enterobiasis?
The parasite causes pruritus around the anus because the females exit at night to lay eggs on the skin.
This parasite is a protozoan and the source is usually contaminated water and it is spread by fecal-oral contamination.
The parasite is in the soil and enters the body through the feet.
Eggs of this parasite enter the body by ingestion of dirt or from unwashed vegetables that contain the eggs in the dirt.
The correct answer is:
The parasite causes pruritus around the anus because the females exit at night to lay eggs on the skin
This infestation is also known as "pinworms" which reside in the intestine. Hookworm larvae reside in the soil and enter the
body through the feet. When dirt is ingested, there is a risk of contacting roundworm. Giardiasis from Giardia lamblia results
from ingesting this organism through contaminated water or fecal-oral transmission.
Question 24
In terms of Medicare, which of the following statements about hospital privileges is incorrect?
Every hospitalized patient covered by Medicare must be under the care of a physician.
The federal government includes licensed doctors of medicine, osteopathy, podiatry and chiropractic in its definition of
physician.
Hospitalized patients may be under the care of a clinical psychologist.
Doctors of medicine or osteopathy may not delegate tasks for hospitalized patients to other qualified health care personnel.
Correct answer:
Doctors of medicine or osteopathy may not delegate tasks for hospitalized patients to other qualified health care personnel.
This is the incorrect statement. Doctors of medicine or osteopathy may delegate tasks for hospitalized patients to other
qualified health care personnel “to the extent recognized under State law or a State’s regulatory mechanism.” NPs who deliver
care to hospitalized patients presumably fall under the delegation rule.
Question 25
The FNP calculates a patient’s BMI to be 23. Which category would this patient fall into?
normal weight
underweight
overweight
obese
Correct answer:
normal weight
This patient would fall into the normal weight category. Persons with a BMI of 18.5 to 24.9 are considered of normal weight.
Question 26
Which of the following is NOT a risk factor for osteoporosis?
alcohol abuse
active lifestyle
advancing age
history of bone fracture
Correct answer:
active lifestyle
An inactive lifestyle, not an active lifestyle would be a risk factor for osteoporosis. All of the other choices are risk factors
along with: female gender, family history, and low estrogen levels in women, among other things.
Question 27
133
Which of the following is a system of controlled oversight and authorization of services and benefits provided to patients?
clinical ladder
case management
triage
clinical guidelines
The Correct answer is:
Case management
Case management is a special type of communication that is a necessity for healthcare providers. It is the process of
interviewing and teaching patients and for sharing or clarifying information with others involved in the patient's care
Question 28
A patient with hypertension is in the office and you have prescribed Cozaar. Cozaar, if taken with which of the following
drugs, can put a patient at risk for developing a renal calculi?
Tagament
Nizoral
Diflucan
Oxipurinol
Correct answer:
Oxipurinol
Cozaar is an Angiotensin II Receptor Antagonist prescribed for the treatment of hypertension. This drug blocks the material in
the body that constricts or narrows the blood vessels, then relaxes the blood vessels, allowing the blood to flow more freely
and results in a lowered blood pressure. The common side effects of Cozaar are dizziness, diarrhea, muscle cramps, heartburn,
stuffy nose, pain in the leg, knee or the back. Some serious side effects include chest pain, difficulty breathing and swelling in
the face, throat or tongue. Cozaar, if taken with certain other drugs, can cause a potential drug interaction. For example, when
Cozaar is taken with Tagament, Cozaar increases the serum level of Tagament. Then, if Cozaar is taken with Diflucan,
Diflucan reduces the efficacy of Cozaar. When Cozaar is taken with Oxipurinol, the patient has an increased risk of
developing renal calculi. Further, if the patient drinks grapefruit juice with Cozaar, grapefruit juice causes a delay in the
absorption and decreases the serum levels of the Cozaar.
Question 29
You are treating a 48-year-old male who is suffering with alcoholism. The first most helpful approach for him is:
Inform the patient of the long-term health consequences of alcohol abuse.
Tell the patient to stop drinking.
Refer the patient to Alcoholics Anonymous.
Counsel the patient that alcohol abuse is a treatable disease.
The correct answer is:
Counsel the patient that alcohol abuse is a treatable disease
The first approach for the nurse practitioner is to counsel the patient and family that alcoholism is a lifelong but treatable
disease. In addition, asking about current drinking habits and associated consequences to health with each visit is important
and are appropriate next steps. Then, the remaining answer choices are also important interventions after the nurse practitioner
counsels the patient and the family regarding alcoholism as a treatable disease.
Question 30
You are discussing medication therapy in a 39-year-old man who has just been diagnosed with gastroesophageal reflux disease
(GERD). Which medication on his medication list do you know exacerbates the symptoms of GERD?
metformin
ferrous sulfate
verapamil
viagra
The Correct answer is:
Verapamil
The lower esophageal sphincter is opened and closed by muscles. Verapamil is a calcium channel blocker and calcium is
needed for muscle contractions. Gastroesophageal reflux disease can be exacerbated in this case so verapamil should be
avoided in this patient.
134
Question 31
You suspect anemia in an 88-year-old female patient, particularly vitamin B12 deficiency. Which of the following lab indices
is more indicative of a B12 deficiency?
macrocytosis
leukocytosis
thrombocytosis
microcytosis
The Correct answer is:
Macrocytosis
A vitamin B12 deficiency produces a form of anemia known as pernicious anemia. It is more common in older adults and
characterized by macrocytosis, where red blood cells are larger than expected. Leukocytosis describes large numbers of white
blood cells. Thrombocytosis refers to an increased number of platelets. Microcytosis, small red blood cells, is seen in iron
deficiency anemia or thalassemia.
Question 32
A 36-year-old mentally challenged and disabled man has been diagnosed with prostate cancer and it is determined he needs
surgery. Consent for this surgery should be obtained from:
the patient's court-appointed guardian
the administrator of the group home where the patient lives
the patient himself
the client's mental health physician
The Correct answer is:
The patient's court-appointed guardian
Since this patient's mental capacity to consent is questionable, alternatives must be sought. If this patient has a court-appointed
guardian, that person is the decision maker. If no guardian exists, the nurse practitioner would have to evaluate the patient's
family dynamics and state and federal laws to determine if a close relative is appropriate to consent for this surgical procedure.
Also, a group home employee cannot consent for this as they have no legal authority.
Question 33
NSAID and corticosteroid use are potent risk factors in which of the following types of peptic ulcer disease?
gastric ulcer
duodenal ulcer
nonerosive gastritis
chronic type B gastritis
Correct answer:
gastric ulcer
NSAID and corticosteroid use are a potent risk factors for gastric ulcers. Cigarette smoking is also a risk factor. Nearly all
gastric ulcers found in patients without H. Pylori infection are caused by NSAID use.
Question 34
When a patient tells you that she has no family anymore since her sister and her husband died last year and you reply, “That
must make you very sad,” you are using which of the following types of therapeutic communication?
trust
honesty
validation
empathy
Correct answer:
empathy
Empathy is an emotional linkage between two or more people through which feelings are communicated. It involves trying to
imagine what it must be like to be in another person’s situation.
Question 35
The amount of cream or ointment needed to treat a dermatologic condition on one leg for one time is which of the following?
6g
3g
135
10 g
12 g
Correct answer:
6g
The amount of cream or ointment needed to treat a dermatologic condition on one leg for one time is 6 g. Prescribers often
write prescriptions for an inadequate amount of topical medication creating a situation in which treatment fails because of an
inadequate length of therapy.
Question 36
As part of a nonpharmacologic treatment for a second-degree burn, you would do all of the following EXCEPT:
Remove all rings to avoid a tourniquet effect.
Apply ice to the site.
Gently cleanse with a mild detergent.
Cover the burn area with a thin layer of silver sulfadiazine cream.
Correct answer:
Apply ice to the site.
Do NOT apply ice to the site. In addition to the other three choices you may flush a chemical burn copiously with water and
debride any broken blisters and dead skin.
Question 37
Which of the following injuries shows no specific radiographic findings?
tennis elbow
finger fractures
Jones fracture
Boxer’s fractures
Correct answer:
tennis elbow
Tennis elbow as well as nursemaid’s elbow show no specific radiographic findings. Finger fractures may reveal crush injuries.
Jones fractures reveal a fracture to the fifth metatarsal. Boxer’s fractures reveal a fracture of the fifth metacarpal, which
frequently is medially angulated.
Question 38
A 67-year-old man is in the clinic with frequent dizziness while standing. Orthostatic hypotension can be diagnosed in an
older adult if the systolic blood pressure decreases:
more than 20 points any time after rising
more than 20 points within one minute after rising
any degree drop if the patient becomes weak or dizzy
more than 20 points within three minutes after rising
The Correct answer is:
More than 20 points within three minutes after rising
Orthostatic hypotension, also called postural hypotension, is diagnosed in older adults when the systolic blood pressure drops
20 mm Hg or more within 3 minutes of moving to an upright position. It can be evaluated from lying to sitting or from sitting
to standing. Also, if the systolic reading does not drop but the diastolic drops by 10 mm Hg or more, postural hypotension can
be diagnosed.
Question 39
You have a 50-year-old male patient complaining of hearing loss. One of the tests you do involves placing the tuning fork on
the forehead and asking the patient where it is heard best. This is known as which of the following?
Rinne test
Weber test
tympanogram
audiogram
136
Correct answer:
Weber test
The Weber test involves placing the tuning fork on the forehead or front teeth. The patient then indicates where it is heard
best. The Rinne test involves placing the tuning fork alternately on the mastoid bone and in front of the ear canal. The other
choices are tests that do not involve a tuning fork.
Question 40
Which of the following would NOT be considered one of the general principles of medical record documentation?
The rationale for ordering diagnostic and other ancillary services should be documented or easily inferred.
The patient’s progress, response to and changes in treatment, and revision of diagnosis should be documented.
Health risk factors do not appear in documentation, only in practice.
The medical record should be complete and legible.
Correct answer:
Health risk factors do not appear in documentation, only in practice.
Appropriate health risk factors should be identified in documentation. Past and present diagnoses should also be accessible to
the treating and/or consulting physician
Question 41
A physician or nurse practitioner who specializes in the care of hospitalized patients is called a(n):
specialist
primary care provider
hospitalist
adjunct provider
Correct answer:
hospitalist
Traditionally, a patient who needed to be admitted to a hospital was admitted through the patient’s primary care provider, who
coordinated the care of the patient. This tradition has been challenged by the realization that this model is not too efficient.
More hospitalists (physicians and nurse practitioners who specialize in the care of hospitalized patients) are taking over this
aspect of practice.
Question 42
An 18-year-old female comes to the clinic complaining of a sore throat and fatigue that has persisted for several months. She
also has some upper abdominal pain. The FNP’s examination reveals enlarged cervical nodes, erythema of the pharynx and
red tonsils. The most likely diagnosis is:
rhinitis medicamentosa
acute mononucleosis
acute sinusitis
allergic rhinitis
Correct answer:
acute mononucleosis
Mononucleosis is an infection by the Epstein-Barr virus that is transmitted through saliva by intimate oral contact. The classic
triad to look for is pharyngitis, cervical lymphadenopathy and fatigue.
Question 43
You are examining a male infant, age 9 months, and notice an undescended testicle on the right side. When counseling the
mother, the nurse practitioner understands the following to be true of cryptorchidism EXCEPT:
This is where the testis does not descend with massage of the inguinal area.
You cannot exam the infant in a warm room.
Increased risk of testicular cancer occurs if testicles are not removed from the abdomen.
Surgical correction within the first year of life is necessary if it does not spontaneously descend.
The Correct answer is:
You cannot exam the child in a warm room
137
In order to see if the muscle will relax and the testicle will descend, the infant should be sitting and the exam room should be
warm to relax the muscles when massaging the inguinal canal. Another option is to examine the infant during or after a warm
bath.
Question 44
Which of the following is NOT one of the Jones criteria for pelvic inflammatory disease (PID)?
lower back pain
cervical motion tenderness
adnexal tenderness
lower abdominal tenderness
Correct answer:
lower back pain
The Jones criteria are the major criteria for PID. Only one of the criteria is necessary to diagnose and treat for PID. All of the
choices are one of the Jones criteria except lower back pain
Question 45
Which of the following would NOT be a breach of patient confidentiality?
discussing a patient’s condition with the nurse assistant assigned to the patient
discussing a patient’s condition with family members
giving a patient’s name and address to a vendor
talking about a patient within earshot of others
Correct answer:
discussing a patient’s condition with the nurse assistant assigned to the patient
All of the choices would be a breach of patient confidentiality except this one. Other breaches include: releasing medical
information about a patient without prior written permission, leaving a telephone message on a patient’s answering machine,
leaving patient records within view of others, and discarding unshredded duplicate records.
Question 46
Osteoporosis is indicated by a bone density score of which of the following?
less than -2.5
between -1 and -2.5
greater than -1
none of the above
Correct answer:
less than -2.5
Interpretation of bone density testing is based on a T score, which refers to the number of standard deviations above or below
the average value in young adults of the same gender. Osteoporosis is a bone density indicated by a T score less than -2.5. It is
also indicated by the presence of fragility fractures, irrespective of the bone density test results.
Question 47
A 16-year-old male is in the office. He has a insect bite on his left forearm and you suspect a brown recluse spider bite. What
medical management would you provide?
ice pack and elevation of the area
active and passive range of motion (ROM) to the area
avoidance of antihistamines
warm moist soaks to the affected area
The Correct answer is:
Ice pack and elevation of the area
Ice packs are preferred to heat to decrease the edema. The area should be immobilized and tetanus toxoid given.
Antihistamines may reduce the swelling and relieve site itching
Question 48
You have a 58-year-old female patient who is post-menopausal and whom you have diagnosed as having an ovarian cyst. The
cyst is 3 cm and the client has no other risk factors. Which of the following actions would you take for this patient?
138
Refer for emergency care.
Recheck her at 6 weeks.
Refer to a surgeon.
Do nothing.
Correct answer:
Recheck her at 6 weeks.
If the patient is post-menopausal, the cyst is smaller than 5 cm, and the patient does not have any other risk factors, you would
recheck her in 6 weeks, and then again at 3 months, and then 6 months. Refer the patient if there is any increase in size of the
mass.
Question 49
A 70-year-old male comes to the FNP with a raised, red lesion in his oral cavity. He tells the FNP that he is afraid that it is
cancer. Besides evaluating the lesion the FNP evaluates the man’s risk for oral cancer. These risks include all of the following
EXCEPT:
poor diet
alcohol abuse
tobacco abuse
advanced age
Correct answer:
poor diet
Poor diet is not a risk factor for oral cancer. Alcohol and tobacco use and advanced age are risks.
Question 50
The Health Insurance Portability and Accountability Act of 1996 (HIPPA) protects the privacy of patients and their health care
records. All of the following are basic requirements of the privacy rule EXCEPT:
Providers and their staff are restricted to conveying the minimum necessary information about patients.
If a provider wants to release patient information for marketing purposes, the patient must authorize the use of the information
in writing.
Providers may not disclose health information to oversight agencies, such as CMS without patient authorization.
Individuals have no right to psychotherapy notes, information compiled in anticipation of civil or criminal litigation and
certain clinical laboratory information.
Correct answer:
Providers may not disclose health information to oversight agencies, such as CMS without patient authorization.
This is not a requirement of the privacy rule. Providers may disclose health information to oversight agencies, such as CMS
without patient authorization.
Question 51
A 38-year-old male patient complains of a sudden onset of severe “ice pick” pains behind one eye. The headache is
accompanied by tearing, a clear runny nasal discharge and a drooping eyelid. What type of headache does he have?
muscle tension headache
migraine headache
cluster headache
temporal arteritis
Correct answer:
cluster headache
Cluster headaches are severe headaches with lancinating pain behind one eye that occurs several times a day, usually at the
same times every day. Their cause is unknown and they are more common in adult males in their 30s and 40s.
Question 52
Human papillomavirus (HPV) is considered the causal agent in what percentage of cervical cancers worldwide?
75.5%
89.2%
90.9%
139
99.8%
Correct answer:
99.8%
Human papillomavirus (HPV) is now considered the causal agent in 99.8% of cervical cancers worldwide. HPV strains 16, 18,
31, 33, 39, and 42 are strongly associated with cervical cancer.
Question 53
Psoriasis is an inherited skin disorder. All of the following are typical treatments for psoriasis EXCEPT:
Psoralen drugs
methotrexate
Goeckerman regimen
systemic penicillin
Correct answer:
systemic penicillin
Topical steroids and tar preparations (Psoralen drugs) are used for psoriasis. Systemic drugs such as methotrexate are used for
severe forms of the disease as is the Goeckerman regimen. Systemic penicillin would not be used as this is a drug that is used
for infections, not for psoriasis, a condition where the squamous epithelial cells undergo rapid mitotic division producing
psoriatic plaque.
Question 54
When assessing a rash near the eye and side of the temple, the nurse practitioner understands that herpes keratitis has what
signs and/or symptoms?
blindness, painful swollen hair follicle
small nodule that moves under the skin of the eyelid
acute onset of eye pain, photophobia and blurred vision in the affected eye
a yellow triangular thickening of the conjunctiva
The Correct answer is:
Acute onset of eye pain, photophobia and blurred vision in the affected eye
Herpes keratitis is caused by a herpes virus infection, usually shingles, and it causes these signs and symptoms. A stye or
hordeolum is a painful acute bacterial infection of a hair follicle on the eyelid that does not result in blindness. A chalazion
results in a movable nodule. Then, pterygium presents with a yellow triangular thickening of the conjunctiva.
Question 55
Medications may produce anxiety as a side effect. Of the following, which medication is least likely to have this effect?
corticosteroids
bronchodilators
antihistamines
antibiotics
Correct answer:
antibiotics
Antibiotics are the least likely of the choices to produce anxiety as a side effect. Besides the other three choices,
anticholinergics, antihypertensives, antipsychotics, antidepressants, amphetamines, and anesthetics may also produce anxiety.
Question 56
Which of the following best defines a hospitalist?
a physician or nurse practitioner who specializes in the care of hospitalized patients
a member of a hospital accreditation board
any physician or nurse practitioner who has hospital privileges
none of the above
Correct answer:
a physician or nurse practitioner who specializes in the care of hospitalized patients
140
To increase efficiency, physicians and nurse practitioners can specialize in the care of hospitalized patients. These clinicians
are called hospitalists.
Question 57
A 45-year-old African-American male reports chest pain that is sudden and severe. Also, the male patient describes the pain as
"tearing." You notice that the pain is accompanied by a decrease in peripheral pulses. You understand that this may indicate a
diagnosis of:
pericarditis
aortic dissection
acute MI
angina
The Correct answer is:
Aortic dissection
Aortic dissection almost invariably begins with sudden onset of severe chest pain that is tearing or ripping in quality and is
accompanied by absent or decreased peripheral pulses and neurologic deficits. The pain of angina and acute myocardial
infarction (MI) is usually described as "pressure" by the patient. Pericarditis produces pain that is more gradual in its onset.
Question 58
A woman who has been on hormone replacement therapy (HRT) is likely to have some possible improvement. Which of the
following is one of these?
A decrease in the symptoms associated with menopause.
A reduction in the risk of dementia.
A reduction in the risk of osteoarthritis.
A reduction in the risk of endometrial cancer risk.
The Correct answer is:
A decrease in the symptoms associated with menopause
As with all medication, the use of HRT comes with the possibility of adverse effects. There is no decrease in the risk of
dementia or osteoarthritis. There is a reduction of osteoporosis risk. Endometrial cancer risk, and not a reduction in the risk,
with unopposed estrogen exists.
Question 59
Which of the following vaccines is NOT a live attenuated vaccine?
MMR
varicella
Salk polio
varicella zoster
Correct answer:
Salk polio
An attenuated vaccine is a vaccine created by reducing the virulence of a pathogen, but still keeping it viable (or "live").
Attenuation takes an infectious agent and alters it so that it becomes harmless or less virulent. The Salk polio vaccine is not a
live attenuated vaccine.
Question 60
A positive Homan’s sign is associated with which of the following?
deep vein thrombosis
superficial thrombophlebitis
peripheral vascular disease
Raynaud’s phenomenon
Correct answer:
deep vein thrombosis
Deep vein thrombosis (DVT) occurs when a thrombus develops in the deep venous system of the legs or pelvis. On physical
exam of a patient with DVT, a positive Homan’s sign (lower leg pain on dorsiflexion of the foot) is noted
Question 61
You are advising your patient with diabetes about exercise and diet. You would tell this patient all of the following EXCEPT:
Exercising heavily will decrease the risk of hypoglycemia in the evening and at bedtime.
141
Skipping a meal will increase the risk of hypoglycemia in the evening and at bedtime.
Eat simple carbohydrates before or during exercise and complex carbohydrates after exercise.
Exercise increases glucose utilization.
Correct answer:
Exercising heavily will decrease the risk of hypoglycemia in the evening and at bedtime.
This is not correct. Exercising heavily will increase the risk of hypoglycemia in the evening and at bedtime and should be
avoided. In older diabetics preexisting CHD should be ruled out before starting an exercise program by administering a stress
test.
Question 62
Some employers offer the NP the opportunity for bonuses. If a bonus is based on the number of patient visits per year it is
which of the following types of bonus formulas?
quality-based
profit-based
patient satisfaction-based
productivity-based
Correct answer:
productivity-based
A formula based on the number of patient visits per year is a productivity-based formula. This type of formula makes good
business sense under a fee-for-service arrangement.
Question 63
What type of headache is typically described as "pressing" in quality?
migraine headache
tension-type headache
cluster headache
simple stress headache
The Correct answer is:
Tension-type headache
Tension-type headache is pressing, nonpulsatile pain, mild to moderate in intensity and usually bilateral in location. Migraine
headache is described as pulsating quality, usually unilateral, with associated nausea and photophobia. Cluster headache is
often behind one eye, steady and intense. Simple stress headache is a fictitious term, which makes it an incorrect answer
choice
Question 64
Medical decision making refers to the complexity of establishing a diagnosis and/or selecting a management option. The four
levels of evaluation and management services include all of the following EXCEPT:
high complexity
low complexity
straightforward
up-front
Correct answer:
up-front
There is no up-front level of E/M services. The four types of medical decision making are: straightforward, low complexity,
moderate complexity and high complexity
Question 65
Which of the following is NOT one of the basic requirements/allowances of the HIPAA privacy rule?
Providers releasing patient information for marketing purposes must explain to the patient how the information will be used, to
whom it will be disclosed, and the time frame.
Providers and their staff are restricted to conveying the “minimum necessary information” about patients.
Providers may disclose health information to oversight agencies.
In general, providers may disclose psychotherapy notes without patient consent.
142
Correct answer:
In general, providers may disclose psychotherapy notes without patient consent.
There are special rules for psychotherapy notes. In general, patient authorization is required in order to disclose psychotherapy
notes to carry out treatment, payment or healthcare operations.
Question 66
Lead is primarily absorbed through which of the following body systems?
neurologic
integumentary
respiratory and gastrointestinal
integumentary and lymphatic
Correct answer:
respiratory and gastrointestinal
Lead is absorbed primarily through the respiratory and gastrointestinal systems. After lead is absorbed into the bloodstream
most of it is bound to red blood cells.
Question 67
Which of the following statements about collaborative agreements is incorrect?
It is a written document between a physician and a nurse practitioner.
It is never submitted to the board of nursing.
It outlines the nurse practitioner’s role and responsibility to the clinical practice.
It must be signed by both the physician and the nurse practitioner.
Correct answer:
It is never submitted to the board of nursing.
All of the choices are true statements except this one. The collaborative agreement is usually submitted to the board of
nursing.
Question 68
A 16-year-old female patient inquires about douching. You are counseling on birth control, safe sexual practices and hygiene.
What instruction regarding douching should the nurse practitioner include?
Hypoallergenic douches include flavored or perfumed types.
Douching during menstruation is a safe effective practice.
Douching removes natural mucus and upset normal vaginal flora.
Daily douching is important if the patient has copious vaginal discharge.
The Correct answer is:
Douching removes natural mucus and upsets normal vaginal flora
Douching is never necessary because it upsets the normal vaginal flora and disrupts pH. Douching is contraindicated during
menstruation due to "retrograde menstruation," a potential precursor to endometriosis. Further, a patient with copious vaginal
discharge needs to be evaluated for an infection.
Question 69
The ability to conduct a cultural assessment to collect relevant cultural data regarding the patient’s presenting problem as well
as accurately conducting a culturally-based physical assessment is called:
cultural diversity
cultural awareness
cultural skill
cultural desire
Correct answer:
cultural skill
Cultural skill is the ability to conduct a cultural assessment to collect relevant cultural data regarding the patient’s presenting
problem as well as accurately conducting a culturally-based physical assessment. It is part of the “ASKED” mnemonic to
promote becoming culturally competent.
143
Question 70
You have a patient complaining of severe pain in the great toe in which you observe erythema. He tells you that this happened
quite quickly and got to the point of being severe in just a few hours. You suspect acute gout. Which of the following tests is
the only confirmatory test for acute gout?
joint aspiration
serum uric acid
creatinine, BUN
24-hour urine for urate and creatinine
Correct answer:
joint aspiration
Joint aspiration is the only confirmatory test for acute gout. It can find MSU crystal in phagocytes or free in tophi, seen under
a polarized microscope. The other tests are of limited value in acute gout.
Question 71
Of the following prostaglandin inhibitors, which one is a proprionic acid?
Naproxen
Mefenamic acid
Indomethacin
Diflunisal
Correct answer:
Naproxen
Naproxen is a proprionic acid along with Ibuprofen and naproxen sodium. Mefenamic acid is a fenamate and the other two
choices are acetic acid/salicylic acids.
Question 72
Which of the following types of research is most likely to be considered a philosophy as well as a research method?
phenomenology
quasi experimental research
longitudinal research
cross-sectional research
Correct answer:
phenomenology
Phenomenology is both a philosophy and a research method. The purpose is to describe experiences as they are lived and to
capture the “lived experience” of study participants
Question 73
Which of the following is a secondary prevention measure?
educating a diabetic about exercise and diet
immunizations
bicycle helmets
mammogram
Correct answer:
mammogram
All screening tests are secondary prevention measures. A mammogram and breast exam are secondary prevention measures.
Question 74
A mother brings her 8-year-old son to your office. She tells you that he has had no immunizations but she would like him to
have all of the necessary immunizations now. Which of the following immunizations would NOT be necessary for this child?
PCV (3 doses)
Td (3 doses primary, then every 10 years)
144
IPV (3 doses)
MMR (2 doses)
Correct answer:
PCV (3 doses)
PCV (pneumococcal conjugate vaccine) is not given after age 7 years. All of the other immunizations should be given along
with Hepatitis B (3 doses).
Question 75
When you say something to a patient such as, “Let me be sure that I understand what you are saying,” this is which of the
following forms of therapeutic communication?
validation
caring
active listening
honesty
Correct answer:
validation
Validation is listening to the patient and responding congruently in order to be sure that you have the same understanding as
the patient. Active listening is hearing and interpreting language, noticing nonverbal and paraverbal enhancements, and
identifying underlying feelings.
Question 76
Severe cough, fever, conjunctivitis, photophobia, and Koplik’s spots are most related to which of the following?
scarlet fever
measles
rubella
roseola
Correct answer:
measles
A person with measles will present with severe cough, fever, conjunctivitis, photophobia, and/or Koplik’s spots. The rash will
be bright red to purple macules and papules ofent becoming confluent
Question 77
There are two approaches to nursing research. They are which of the following?
academic and practical
direct and indirect
professional and non-professional
quantitative and qualitative
Correct answer:
quantitative and qualitative
There are two approaches to nursing research: quantitative and qualitative. Both involve rigor that produces credible data for
evidence-based practice, and both make significant contributions to nursing science
Question 78
A male patient has asked you for a prescription of Viagra. After a careful history, you make the determination that he has
erectile dysfunction (ED). The nurse practitioner understands that ED is:
Not a condition associated with men over age 70
Primarily psychological in origin
The persistent inability to achieve and maintain an erection adequate for sexual intercourse.
The physiologic dysfunction when smooth muscles contract, causing a lack of adequate amounts of blood in the penis to
render a rigid, larger penis.
The Correct answer is:
The persistent inability to achieve and maintain an erection adequate for sexual intercourse
145
Erectile dysfunction is experienced by 40% of men over 40 years of age and 70% of men over 70. It involves a dysfunction in
the hemodynamic mechanism of smooth muscle relaxation that increases blood flow in the penis.
Question 79
Which of the following is another term for statutory authority?
regulation
legal authority
contract authority
administrative rule
:
Correct answer:
legal authority
Statutory authority is legal authority. Elected officials (the legislature) vote on a bill such as the Nurse Practice Act. Bills that
pass become law and have statutory authority.
Question 80
Which of the following is NOT true regarding healthcare issues in the United States?
Most of Americans have access to healthcare and are being adequately treated.
Of all children, 13% are uninsured.
It is estimated that 45+ million Americans are underserved because of limited access to healthcare.
Many of the working poor cannot obtain healthcare because of the high costs of insurance, fees and medications.
The Correct answer is:
Most of Americans have access to healthcare and are being adequately treated
There is a large portion of Americans who do not have access to healthcare.
Question 81
Which of the following is an orthopedic maneuver of the knee to test for damage to the meniscus?
Drawer sign
McMurray sign
Lachman sign
Goodell’s sign
Correct answer:
McMurray sign
The McMurray sign is an orthopedic maneuver of the knee to test for damage to the meniscus. If the knee locks up and the
patient is unable to fully extend the affected knee, the test is positive.
Question 82
Your pregnant patient is approaching her estimated delivery date. She asks you for some information about how big her baby
will be. You might tell her all of the following EXCEPT:
95% of newborns weight 7 – 10 lb.
95% of newborns are between 18 and 22 in. long.
Approximately 5% - 10% of body weight is lost in the first few days.
The weekly gain from birth to 6 months is 5 – 7 ounces per week.
Correct answer:
95% of newborns weight 7 – 10 lb.
This is not correct. 95% of newborns weight 5 – 10 lb. Approximately 5% - 10% of body weight is lost in the first few days;
then birthweight is regained in 7 – 10 days.
Question 83
You have a patient who is a 22 month old male. He presents with a low-grade fever. Your initial treatment for this patient
would be which of the following?
Do nothing.
146
Give antipyretics and observe.
Obtain a CBC.
Give prophylactic antibiotics for 3 days pending blood culture results.
Correct answer:
Give antipyretics and observe.
For children over 3 months of age who present with a low-grade fever, you would give antipyretics and observe. If fever is
over 102° F, then you would obtain a CBC. If the WBC is greater than 15,000/mm3, then you would obtain a blood culture
and administer IM Ceftriaxone.
Question 84
What is the reason the Standards of Nurse Practitioner Practice were established?
They are established to promote autonomous practice.
They are established to limit the liability of nurse practitioners.
They are established to regulate and control nurse practitioner practice.
They are established to protect nurse practitioners from frivolous lawsuits.
The Correct answer is:
They are established to regulate and control nurse practitioner practice.
The reason Standards of Practice exist in all healthcare related specialties (nursing, medicine, dental, etc.) is that they regulate
and control practice. The purpose of Standards of Practice is to provide accountability for professionals and to help protect the
public from unethical behavior and unsafe practice. The remaining answer choices are not reasons for the establishment of
Standards of Nurse Practitioner Practice.
Question 85
The FNP has a menopausal woman patient who is troubled by hot flashes. The FNP tells her that they can often be reduced in
number and minimized in severity with simple lifestyle changes. All of the following would be a hot flash trigger EXCEPT:
chocolate consumption
alcohol use
tight, restrictive clothing
cold baths or showers
Correct answer:
cold baths or showers
Hot baths or showers are hot flash triggers, not cold baths or showers. In addition to the other choices which are all hot flash
triggers, other triggers include: spicy foods, elevated ambient temperature and humidity, and cigarette smoking.
Question 86
For which of the following would the FNP use the bell of the stethoscope?
mid-to high-pitched tones such as lung sounds
low tones such as the extra heart sounds
mitral regurgitation
aortic stenosis
Correct answer:
low tones such as the extra heart sounds
The bell of the stethoscope is used for low tones such as the extra heart sounds (S3 or S4) as well as for mitral stenosis. The
diaphragm of the stethoscope is used for all of the other choices given.
Question 87
Which of the following diseases/conditions is least likely to increase susceptibility to meningitis?
hypertension
sickle cell anemia
diabetes
alcoholism
Correct answer:
hypertension
147
Of the choices hypertension is the least likely to increase susceptibility to meningitis. Other factors increasing susceptibility
include: basilar skull fracture; indwelling CSF shunting device; debilitation or institutionalization; and contact with others who
have had meningitis.
Question 88
The most likely associated symptoms of dementia include all of the following EXCEPT:
insidious onset over months or years
loss of intellectual or cognitive function
mental status changes
acute onset over hours to a few days
The Correct answer is:
Acute onset over hours to a few days
Delirium has an acute onset over hours to a few days and is a condition in which the patient exhibits reduced ability to
maintain attention to external stimuli and shift attention appropriately to new stimuli. The other answer choices are all true of
dementia, making them incorrect answers in this case. Dementia has an insidious onset over months or years.
Question 89
Which of the following is NOT a conceptual model for advanced practice nursing?
Benner’s model
Shuler’s model
Calkin’s model
Ford model
Correct answer:
Ford model
There is no Ford model of NP practice; however, Loretta Ford established the first pediatric nurse practitioner program.
Question 90
A good quality assurance program should do which of the following?
identify educational needs
improve the documentation of care
reduce the clinician’s overall exposure to liability
all of the above
Correct answer:
all of the above
A good quality assurance program would include all of the first three choices. These programs also look at organizational
effectiveness, efficiency, and client and provider interactions
Question 91
Which of the following medications might you prescribe for a patient with acute bronchitis?
Robitussin
Guaifenesin
Ventolin
any of the above
Correct answer:
any of the above
All of the medications listed in the choices might be prescribed for a patient with acute bronchitis. You would also increase
fluids and if the patient smokes, have them stop smoking, if possible.
Question 92
Which of the following drugs would NOT be a prophylactic treatment for migraine headaches?
beta-blockers
tricyclic antidepressants
prednisone
amitriptyline
148
Correct answer:
prednisone
Prednisone would not be a prophylactic measure for migraine headaches. It would be used, however, if the FNP suspects
temporal arteritis.
Question 93
The FNP has an adult male patient in the clinic who woke up with one side of his face paralyzed. He cannot fully close his
eyelid and has difficulty chewing and swallowing food on that side of his face. Which of the following is the most likely
diagnosis?
Bell’s Palsy
trigeminal neuralgia
focal migraine
acute bacterial meningitis
Correct answer:
Bell’s Palsy
Bell’s Palsy is the abrupt onset of unilateral facial paralysis that is due to dysfunction of the motor branch of the facial nerve
(CN 7). Facial paralysis can progress rapidly within 24 hours. Skin sensation remains intact but tear production on the affected
side may stop. Most cases spontaneously resolve.
Question 94
One part of a spectrum of disease that is often found together with giant cell arteritis is which of the following?
polymyalgia rheumatica
migraine headache
blindness
brain tumor
Correct answer:
polymyalgia rheumatica
Giant cell arteritis and polymyalgia rheumatica are thought to represent two parts of a spectrum of disease and are often found
together. Blindness is a serious complication of the disease.
Question 95
A 32-year-old female with interstitial cystitis (IC) is in the office. She has been newly diagnosed and you are counseling her.
All of the following are important to discuss EXCEPT:
Discuss that interstitial cystitis is not a malignancy or a risk factor for a more serious disease.
Teach the patient the importance of adequate nonirritating fluid intake.
Remind her that cigarette smoking can irritate the bladder.
Discuss the measures for curative treatment.
The Correct answer is:
Discuss the measures for curative treatment
Interstitial cystitis is an incurable condition that involves painful urination, frequency, urgency and nocturia. It is chronic and
can be controlled, not cured. Treatment aims are directed at relief of symptoms. The remaining answers are important as
patient education information.
Question 96
Which of the following is NOT one of the stages in Erikson’s theory of psychosocial development?
trust
autonomy
initiative
loyalty
Correct answer:
loyalty
Loyalty is not one of Erikson’s stages of psychosocial development. The eight stages are: trust, autonomy, initiative, industry,
149
identity, intimacy, generativity, and ego identity.
Question 97
Which of the following patient is most likely to qualify and be using Medicaid as their insurance?
the single mother of four dependent children, age 26
a middle aged female who works for the government, age 47
a young male factory worker, age 23
an elderly retired high school teacher, age 78
The Correct answer is:
The single mother of four dependent children, age 26
Medicaid is a federally mandated program that guarantees healthcare services to low-income families with dependent children
and to low-income and disabled individuals. Government employees and factory workers are most likely insured through a
commercial carrier or uninsured. An elderly teacher would have Medicare since they have paid into the federal retirement
system and are over 65 years of age
Question 98
Which of the following tests is federally mandated for infants?
TSH
Lead screening
PKU
all of the above
Correct answer:
PKU
PKU (phenylketonuria) is a federally mandated test. Severe mental retardation may develop if this is not treated early.
Question 99
The continuing occurrence of new cases of a disease is known as which of the following?
prevalence rate
colonization rate
infectivity rate
incidence rate
:
Correct answer:
incidence rate
The incidence rates describe the rate of development of a disease in a group over a period of time or the continuing occurrence
of new cases of disease. It differs from the prevalence rate which describes a group at a certain point in time and the number
within the group that has a particular disease or problem.
Question 100
A mother brings her infant to the clinic to say that he has tearing in both eyes. The FNP also sees a mucoid discharge. Which
of the following conditions/diseases might this indicate?
congenital lacrimal duct obstruction
Wilms tumor
Fifth disease
Kawasaki disease
Correct answer:
congenital lacrimal duct obstruction
Congenital lacrimal duct obstruction is a failure of the tear duct to open at birth. A thin membrane blocks the entrance of the
nasolacrimal duct causing tear blockage. This is a condition that spontaneously resolves.
Quiz 6 ancc
Question 1
150
The nurse practitioner (NP) understands that the following is NOT true regarding the care of minors:
In most jurisdictions, minors under the age of 18 cannot receive healthcare services without permission of an adult who is a
parent or legal guardian.
If a 17-year-old contracts a sexually transmitted disease, the nurse practitioner must contact the parent or legal guardian.
In case of emergency, when no parent or guardian is available, care may be rendered.
If a 15-year-old becomes pregnant, she is an emancipated minor.
The Correct answer is:
If a 17-year-old contracts a sexually transmitted disease, the nurse practitioner must contact the parent or legal guardian
A teen with a STD is considered an emancipated minor, and the parent or legal guardian should not be contacted. The
remaining answer choices are true regarding the care of minors.
Question 2
You are conducting a health assessment with a patient. She tells you that she does not understand what will happen during the
tests she will undergo the next day. Which of the following is an appropriate nursing diagnosis for this patient?
Low health literacy.
Knowledge deficit.
Patient needs further education.
Patient does not understand care plan.
Correct Answer: Knowledge deficit It is key to note that the patient does not fully understand the tests or procedures
prescribed. The next step should be to provide patient education. Patients who are educated about tests and care protocols have
been shown to have much higher rates of compliance, and proper education also empowers patients.
Question 3
Which of the following tests is used to measure lower esophageal sphincter (LES) tone?
Bernstein test
barium swallow radiograph
Barrett test
none of the above
Correct answer:
Bernstein test
The Bernstein test measures the LES competence. The Bernstein test is a method to reproduce symptoms of heartburn. It is
usually done with other tests to measure esophageal function.
Question 4
You are treating a 48-year-old male who is suffering with alcoholism. The first most helpful approach for him is:
Inform the patient of the long-term health consequences of alcohol abuse.
Tell the patient to stop drinking.
Refer the patient to Alcoholics Anonymous.
Counsel the patient that alcohol abuse is a treatable disease.
The correct answer is:
Counsel the patient that alcohol abuse is a treatable disease
The first approach for the nurse practitioner is to counsel the patient and family that alcoholism is a lifelong but treatable
disease. In addition, asking about current drinking habits and associated consequences to health with each visit is important
and are appropriate next steps. Then, the remaining answer choices are also important interventions after the nurse practitioner
counsels the patient and the family regarding alcoholism as a treatable disease.
Question 5
A 44-year-old Caucasian female is taking lovastatin for hyperlipidemia and complains of muscle aches in her thighs and right
calf for three days without improvement. She has been on the medication for 3 months. What should the nurse practitioner do
first?
Check bilirubin enzymes.
Order a CPK.
Ask about nighttime muscle cramps.
Stop lovastatin immediately.
151
The Correct answer is:
Order a CPK
This patient has a complaint of myalgias that could be associated with statin use. This patient should be assessed for
rhabdomyolysis by measuring the creatine phosphokinase (CPK) level. If the CPK value is elevated, lovastatin should be
stopped immediately. Nighttime cramps are not associated with statin usage.
Question 6
You are providing care to a female patient who is 6 months pregnant. She is suffering from hemorrhoids. Which of the
following may help alleviate her discomfort?
Sleeping on her stomach.
Sleeping on her side.
Sleeping on her back.
Sleeping with the head slightly elevated.
Correct Answer: Sleeping on her side For patients who are not pregnant, sleeping on the stomach can alleviate pressure on
inflamed blood vessels that lead to hemorrhoids. Pregnant patients should instead sleep on their side, as sleeping on the
stomach can be uncomfortable especially in later stages of pregnancy. Warm sitz baths may also help alleviate discomfort
caused by hemorrhoids.
Question 7
When assessing a client for systemic lupus erythematosus (SLE), what ophthalmologic finding would the nurse practitioner
find that would be consistent with this diagnosis?
cotton-wool spots
arteriovenous (AV) nicking
retinal hemorrhages
conjunctivitis
The Correct answer is:
Cotton-wool spots
Cotton-wool spots are the most common ophthalmological problems associated with systemic lupus erythematosus (SLE).
Retinal hemorrhages and AV nicking are seen in patients with hypertension. Additionally, conjunctivitis is an infection of the
conjunctiva.
Question 8
According the CDC classification of overweight and obesity, a person with a BMI (kg/m2) of 27.0 would be classified as:
healthy weight
overweight
obese
extremely obese
Correct answer:
overweight
Persons with a BMI (kg/ m2) of 25 – 29.9 are classified as overweight. A person of healthy weight would have a BMI of 18.5
– 24.9.
Question 9
Which of the following statements is TRUE concerning managed care today?
Managed care is declining in favor in the healthcare system of the United States.
The proportion of managed care revenue declined over the last 10 years.
More consumers are switching from HMOs to plans where there is more choice, less control by a gate keeping provider, and
no prior approval for referrals.
The monitoring of managed care plans by the American Nurses Association and the American Medical Association has
contributed to their popularity.
The Correct answer is:
More consumers are switching from HMOs to plans where there is more choice, less control by a gate keeping provider, and
no prior approval for referrals
Managed care now dominates healthcare in the U.S. and over 50% of revenue received by all physicians is from managed
care. Managed care plans are monitored by state and federal government, not the ANA and AMA.
Question 10
152
An obese child's health is in danger from the weight. What will have an effect on the nutritional assessment?
Age.
Family and culture.
Medical history.
Chief complaint.
Correct answer: Family and culture Eating habits are influenced by family and culture. This can be determined in the social
assessment and family interviews.
Question 11
Which of the following types of headache has a higher risk of suicide in males?
cluster headaches
migraine headaches
muscle tension headaches
temporal arteritis
Correct answer:
cluster headache
A cluster headache is a sudden onset of severe “ice-pick” headaches behind one eye that occurs several times a day. The
attacks happen at the same times daily and the cause is unknown. There is a higher risk of suicide in males who have this type
of headache as compared with the other types of chronic headaches
Question 12
What medication and dose can be used for abortive therapy in an adult client with symptoms of migraine headache?
ergotamine (Ergostat) 2 mg SL
ketorolac (Toradol) 100 mg IM
sumatriptan (Imitrex) 1 mg IM
amitriptyline (Elavil) 100 mg PO
The Correct answer is:
Ergotamine (Ergostat) 2 mg SL
The dosing guideline for ergotamine (Ergostat) is 2 mg by mouth, sublingually. Further, additional treatment options for a
migraine headache can also include Ketorolac and Sumatriptan, as well as other medications. Depending on the patient's
severity of symptoms and medical history, the following are dosing guidelines providers can use to administer Ketorolac and
sumatriptan. For Ketorolac, the provider can administer the drug intravenously or intramuscularly before administering orally
as a tablet. When taking Ketorolac orally, the dosing guidelines are 30-60 mg. The patient is given an initial first dose of two
20 mg tablets. After the first dose, the patient is given one 10 mg tablet every 4-6 hours for up to 5 days. The dosing for
Ketorolac should not exceed 40 mg per day. Therefore, the answer choice stating "ketorolac (Toradol) 100 mg IM" is
incorrect, as 100 mg is too high of a dosage. Next, Sumatriptan (Imitrex) dosing guidelines include oral (25-200 mg daily) and
subcutaneous injections (6-12 mg) administration, as well as administration of the medication by a nasal spray (5-20 mg as 1
spray in each nostril and then up to 40 mg a day.) Therefore, the answer choice with the dosing of 1 mg IM for sumatriptan
(Imitrex) is incorrect. Then, amitriptyline is not used for abortive therapy.
Question 13
For which of the following CPT codes is one descriptor and nothing further required for taking the history of an established
patient?
99211
99212
99213
99214
Correct answer:
99212
Of the five levels of visit for an established patient, the 99212 level has the requirement of 1 descriptor for patient history.
Question 14
An elderly male has a diagnosis of left-sided congestive heart failure (CHF). The nurse practitioner would identify which of
the following as a common condition associated with CHF?
ventricular dysrhythmias
peripheral vascular disease
153
untreated hypertension
chronic obstructive pulmonary disease
The Correct answer is:
Untreated hypertension
Untreated hypertension causes significant increased work of the left ventricle. Further, the years of untreated high blood
pressure eventually causes congestive heart failure (CHF).
Question 15
Which of the following tests would you order to confirm a diagnosis of strep throat?
Throat culture.
Sputum culture.
CBC.
Mono test.
Correct Answer: Throat culture Throat culture is the classic diagnostic tool for strep throat. This test involves a swab of the
throat. If the throat culture does not reveal strep, other tests such as the mono test may be in order
Question 16
The FNP is seeing a college-age female who participated in strenuous sports activities in high school. The FNP understands
that all of the following may be a risk for this patient EXCEPT:
menorrhagia
eating disorders
delayed menses
osteoporosis
Correct answer:
menorrhagia
Menorrhagia (heavy menstrual bleeding) is not a common concern for women who take part in strenuous physical activity.
Females who participate in strenuous physical activities are at risk for eating disorders, delayed menses and osteoporosis.
Question 17
You are dealing with a pregnant adolescent who is drinking alcohol. What should the nurse practitioner understand concerning
this?
There is no level or time of exposure that is considered to be safe.
The risk to the fetus from alcohol exposure is greatest during the third trimester.
This is not problematic since there is not an increased risk of miscarriage.
The risk of fetal alcohol syndrome exists only if the patient consumes alcohol during the entire pregnancy.
The Correct answer is:
There is no level or time of exposure that is considered to be safe.
Fetal alcohol syndrome (FAS) occurs more commonly in infants whose mother drinks during pregnancy. This condition is
preventable and no safe level of maternal alcohol ingestion during pregnancy exists. FAS can occur if alcohol is ingested at
any time during the pregnancy and miscarriage is a risk. Therefore, the pregnant female does not need to drink during the
entire pregnancy for FAS to occur.
Question 18
A 65 year old female patient's records indicate that her pap smears have been negative for the past 15 years. How may you
alter her physical exam?
Perform a pap smear every year.
Do not perform a pap smear.
Begin testing for Chlamydia.
Perform a breast exam.
Correct answer: Do not perform a pap smear It may be possible to suspend pap smears after a patient turns 65. She must have
at least 10 years of negative results to cease the screening for cervical cancer.
Question 19
You are working in an urgent care clinic within a hospital emergency department. A patient without insurance arrives who has
a puncture wound caused by an unknown sharp object in a trash container. A dirty needle is suspected. The nurse practitioner
should:
administer a tetanus injection only since the patient has no insurance
154
prescribe appropriate mediations for HIV exposure even though the patient has no resources to buy them
not mention the possibility of HIV from a contaminated needle
offer to buy the HIV medications for the patient and use your employee discount at the pharmacy
The Correct answer is:
Prescribe appropriate mediations for HIV exposure even though the patient has no resources to buy them
The standard of patient care should be followed by the nurse practitioner. This measure should not be altered depending on the
patient's insurance and financial status. It is unethical to not properly inform this patient of the risks involved for being
exposed to a contaminated needle. You should prescribe the medications and refer them to social services or a community
resource agency for assistance. Buying the medications for the patient with an employee discount is unethical and should not
be done.
Question 20
Which of the following drugs that might be prescribed for a patient with asthma and chronic obstructive pulmonary disease
(COPD) is not used specifically for treatment and prevention of bronchospasm?
theophylline
Singulair
Atrovent
Spiriva
Correct answer:
Singulair
All of the medications might be used for treatment and prevention of bronchospasm except Singulair. This drug is a
leukotriene receptor antagonist that inhibits the action of the inflammatory mediator (leukotriene) by blocking select receptor
sites. it is a controller drug used to prevent inflammation.
Question 21
The FNP is communicating with a patient about controlling her blood sugar. The patient tells the FNP that she is afraid that
she will have to take insulin shots soon. Which of the following statements would demonstrate using the therapeutic
communication technique of reflection?
What would it be like for you to have to take insulin shots every day?
If that is necessary a specialist will show you just what to do.
Maybe you should be more vigilant about your diet.
Don’t worry. That’s not likely to happen.
Correct answer:
What would it be like for you to have to take insulin shots every day?
In using reflection, the FNP takes one or two words said by the patient to reflect back to the patient for consideration. By
asking her, “What would it be like for you to have to take insulin shots every day?” the FNP is using reflection.
Question 22
There are many barriers to the electronic medical record. The NP understands that which of the following is NOT one of these
issues?
technical matters: uncertain quality, ease of use, lack of integration
potential for streamlining and automating workflow
financial matters: maintenance, upgrades, replacement
doubts of clinical usefulness
The correct answer is potential for streamlining and automating workflow. Streamlining and automating the workflow is
considered a benefit of the electronic medical record, not a barrier. Other barriers include resource issues (training and retraining), certification, security, and ethical matters.
Question 23
The mother of an infant calls and says that her son has a fever. He is also sleeping more and difficult to wake up. What is the
appropriate advice?
Give the child baby aspirin.
Monitor the child overnight.
Bring the child to the emergency room.
Call in a prescription.
155
Correct answer: Bring the child to the emergency room A high fever and lack of responsiveness can indicate a serious
condition in a infant. He should be seen as soon as possible.
Question 24
You evaluate a 52-year-old female with rheumatoid arthritis who has an abnormal hemogram. Her results include:
Hgb = 9.8 g
Hct = 32%
MCV = 86 fL
These results are most consistent with which type of anemia?
folate-deficiency anemia
iron-deficiency anemia
sickle cell anemia
anemia of chronic disease
The Correct answer is:
Anemia of chronic disease
Anemia is defined as a decrease in the oxygen-carrying capability of the blood. This condition is not a 'disease' but rather a
sign or symptom of an underlying process. With anemia of chronic disease, you will find normal size red blood cells (RBCs)
but low hemoglobin (Hgb) and hematocrit (Hct) levels. Mean corpuscular volume (MCV) is often normal. With folatedeficiency anemia, MCV is often elevated. With iron-deficiency anemia, MCV is often low. Sickle cell anemia results in
abnormal shape of cells
Question 25
The earliest age that an average child would be able to copy a triangle, know his colors, and count on his fingers is:
age 2 years
age 3 years
age 4 years
age 5 years
The Correct answer is:
Age 5 years
Common developmental tasks for the 5-year-old child would include the ability to draw a person with a body, head, and arms
and recognize most letters and print some. Further, the child knows how to skip, copy a triangle, count on fingers and knows
colors.
Question 26
If a mother asks the FNP when her toddler should be toilet trained, which of the following would be the best response?
Most children should be toilet trained at about two year of age.
Do not start toilet training until the child is over three years of age.
When the child has sphincter control, the ability to delay immediate gratification, and an understanding of the task of toilet
training, then he is ready.
none of the above
Correct answer:
When the child has sphincter control, the ability to delay immediate gratification, and an understanding of the task of toilet
training, then he is ready.
A child is ready for toilet training on his own readiness scale. When he has sphincter control, the ability to delay immediate
gratification, and an understanding of the task of toilet training, then he will be ready. To do it any sooner will be frustrating to
the child and parent.
Question 27
Which of the following suggests that a patient may be abusing alcohol?
rhinophyma, hypotension, peripheral neuropathy
telangiectasias, flat affect, thyroid dysfunction
hepatosplenomegaly, murmur, osteoarthritis
macrocytosis, tremulousness, hypertension
The Correct answer is:
Macrocytosis, tremulousness, hypertension
156
Findings that would cue the nurse practitioner that a patient was abusing alcohol are tremors, hypertension, rhinophyma,
peripheral neuropathy, telangiectasias or hepatosplenomegaly. The patient may not exhibit all of these symptoms at one time
however. Further, the patient will have macrocytosis because there is a high rate of B12 and folate deficiency among these
patients.
Question 28
When choosing a blood pressure cuff for a young child, what size should it be?
2/3 upper arm.
All of the upper arm.
1/2 upper arm.
You do not use a blood pressure cuff.
Correct answer: 2/3 upper arm Children do need their blood pressure taken occasionally. The cuff should not be any larger
than 2/3 the size of the child's arm. Because this varies with age, and exact size is not given
Question 29
An elderly patient presents with fever, left lower quadrant abdominal pain and diarrhea. Which of the following BEST
describes the symptoms the patient is experiencing?
Acute hepatitis.
Iron deficiency anemia.
Testicular cancer.
Diverticulitis.
Correct answer: Diverticulitis Diverticulitis is when the colon secondary consists of puchlike hernias. These hernias are caused
by the lack of dietary fiber.
Question 30
Communication would be a part of which of the following fundamental domains?
cognitive
psychological and social
physical
none of the above
Correct answer:
cognitive
The cognitive domain includes communication, the receptive and expressive language. It also includes perception, thinking,
information processing and memory.
Question 31
A 54-year-old man presents with a 4-month history of upper abdominal pain, which he describes as intermittent and centrally
located. He complains of a burning sensation in the epigastric area, most often with meals and often accompanied by mild
nausea. The use of over-the-counter antacids, like Maalox and Tums, have been ineffective. The patient's clinical presentation
is most consistent with:
acute colonic diverticulitis
duodenal ulcer
gastric ulcer
acute gastroenteritis
The Correct answer is:
Gastric ulcer
The clinical presentation of gastric ulceration includes pain often reported with or immediately after a meal. The associated
symptoms for gastric ulceration are nausea, vomiting and weight loss. The male to female ratio for individuals with a gastric
ulcer is equal. Further, the peak incidence for patients with a gastric ulcer is in the fifth and sixth decades of life.
Question 32
Of the following drugs prescribed, which is a cholinesterase inhibitor that comes in the form of a pill or syrup only?
Galantamine.
Hydrocodone.
Rivastigmine.
Donepezil.
157
Correct answer: Galantamine There are three cholinesterase inhibitors commonly prescribed by doctors. These are Donepezil,
Galantamine and Rivastigmine. Hydrocodone is a type of medication that treats moderate to severe pain as well as cough.
Question 33
Of the following choices, which describes the Kernig sign?
Elicited with the patient lying supine and the hip flexed 90 degrees, pain presents when extension of the knee occurs.
The headache worsens when lowering the patient from sitting to supine positioning.
Neck pain occurs with passive flexion of one hip and knee, which causes flexion of the contralateral leg.
Passive neck flexion in a supine position where the patient progresses to a flexion of the knees and hips.
The Correct answer is:
Elicited with the patient lying supine and the hip flexed 90 degrees, pain presents when extension of the knee occurs
To eliminate or support the diagnosis of meningitis, the practitioner may want to assess for Kernig sign. Kernig sign is
suggestive of nuchal rigidity and meningeal irritation. The Kernig sign is elicited when the patient is in a supine lying position
and the hip is flexed at 90 degrees. A positive sign is present when extension of the knee results in resistance or pain is
experienced in the lower back or posterior thigh.
Question 34
A patient comes to the clinic for a skin examination because of a “mole” that has been concerning her. She asks the FNP if this
could be a melanoma. The FNP understands that all of the following are signs that the lesion could be a melanoma EXCEPT:
It is asymmetric with non-matching sides.
Its borders are irregular.
It is of a uniform black or brown color.
Its diameter is larger than 6 mm.
Correct answer:
It is of a uniform black or brown color.
All of the choices are correct except this one. One of the signs that a lesion may be a malignant melanoma is that the color is
not uniform brown, black, red, white, or blue. Another sign is that it is an evolving lesion, either new or changing.
Question 35
In which of the following cases would the FNP refer a patient to a specialist for low back pain? A patient with:
acute neck pain
bladder dysfunction
reduced range of movement
stiffness
Correct answer:
bladder dysfunction
If the patient has a bladder dysfunction, or a limb or bowel dysfunction, prompt referral to a specialist is needed. Surgery is
usually considered only if severe symptoms persist beyond 3 months.
Question 36
In males, puberty is considered precocious if it occurs before:
12 years of age
9 years of age
14 years of age
10 years of age
Correct answer:
9 years of age
In boys, puberty starts at 9 years of age. It is considered precocious puberty if it begins before 9 years of age. In girls puberty
starts at 8 years of age and is considered precocious if it begins before 8 years of age.
Question 37
When the FNP is examining a 12-month-old infant, about how much growth should she expect to see in head circumference,
as compared to the infant's head circumference at birth?
3 cm
6 cm
158
12 cm
18 cm
Correct answer:
12 cm
The typical head growth in the first year of life is 12 cm: 6 cm in the first 3 months; 3 cm in the 4th to 6th months; and 3 cm in
the 6th to 12th months. Subsequent head growth is about 0.5 cm/year for 2 – 7 year olds and 0.3 cm/ year for 8 – 12 year olds.
Question 38
A 58-year-old male presents in the office with a glucose of 309 mg/dL and is symptomatic for Type II diabetes mellitus. What
should be done to manage him FIRST?
Start insulin
Start metformin plus pioglitazone
Have him return tomorrow to recheck his blood glucose
Start metformin
The Correct answer is:
Start insulin
The patient can be diagnosed with diabetes if his glucose exceeds 200 mg/dL and he is symptomatic. Most oral agents will
have little effect on his glucose and it should be lowered immediately. Therefore, insulin is the best agent to reduce the blood
sugar so that oral agents will have a chance to work. He should return to the clinic the next day for a recheck of the blood
glucose and medication adjustment.
Question 39
A patient who has been prescribed Amitiza for IBS should be advised to seek immediate medical attention if she experiences
which of the following?
Difficulty breathing.
Nausea.
Dry mouth.
Dizziness.
Correct Answer: Difficulty breathing Patients taking Amitiza may experience difficulty breathing within 30 to 60 minutes of
taking a dose. This is often not serious. However, it is difficult to tell this from a more serious reaction. Therefore, immediate
medical attention should be sought.
Question 40
You are managing a 74-year-old patient who has osteoarthritis and chronic pain. Which of the following medications increases
the risk of a gastrointestinal related ulceration?
warfarin
celecoxib
pravastatin
thiazide diuretic
The Correct answer is:
Celecoxib
Celecoxib is a nonsteroidal anti-inflammatory drug (NSAID). Further, NSAIDs increase the risk of upper and lower
gastrointestinal ulcerations. Warfarin does not increase the risk of ulceration, but if one occurs, the risk of bleeding is
increased. Statins and diuretics are not considered ulcerogenic.
Question 41
When managing the respiratory patient, the nurse practitioner knows that all of the following antimicrobial strategies help
facilitate the development of resistant pathogens EXCEPT:
prescribing a broader spectrum agent
higher antimicrobial dosage
lower antimicrobial dosage
longer course of therapy
159
The Correct answer is:
Higher antimicrobial dosage
Factors that facilitate the development of resistant microbes include repeated exposure to a given agent, underdosing, and
unnecessarily prolonged period of treatment. Shorter course high-dose therapy maximizes and exploits concentrationdependent killing by achieving higher maximum concentration and area under the curve/minimal inhibitory concentration
values.
Question 42
You are counseling a mother whose child is receiving the Varicella vaccine (VZV). Which of the following statements would
be appropriate?
"Your child will never have a breakout of the chickenpox."
"Mild cases of chickenpox have been reported in immunized patients."
"Children must have a varicella titer drawn before they receive this vaccine."
"The vaccine delivers killed varicella virus to the recipient."
The Correct answer is:
Mild cases of chickenpox have been reported in immunized patients
The varicella vaccine contains attenuated virus and is administered in two doses, one at age one and the other at age 4 to 6
years. The vaccine is highly protective against severe, invasive varicella, however, mild forms of chickenpox are occasionally
reported after immunization. Children do not need a titer prior to receiving this vaccine.
Question 43
The FNP has a 64-year-old female patient who inquires about the Zoster vaccine for shingles. The FNP might tell her all of the
following EXCEPT:
The vaccine is prepared from a live, attenuated strain of varicella-zoster virus.
Reactivation of the varicella virus is not related to a decline in varicella-zoster virus-specific immunity.
The use of zoster vaccine significantly reduces the risk of shingles.
The vaccine should be used even with a history of shingles.
Correct answer:
Reactivation of the varicella virus is not related to a decline in varicella-zoster virus-specific immunity.
This statement is incorrect. Reactivation of the varicella virus seems to be related to a decline in varicella-zoster virus-specific
immunity. Therefore, the use of zoster vaccine significantly reduces shingles risk.
Question 44
In a pregnant woman if there is vaginal bleeding and cramping and the cervix remains closed it is which of the following types
of spontaneous abortions?
inevitable abortion
threatened abortion
complete abortion
incomplete abortion
Correct answer:
threatened abortion
In a threatened abortion there is vaginal bleeding and cramping but the cervix remains closed. The pregnancy may be able to
be salvaged.
Question 45
An 81-year-old female is in the office. She has been taking ibuprofen over-the-counter in large dosages for a long time. The
nurse practitioner comprehends which side effect is likely regarding this situation?
impairment of renal function
liver failure
neuropathy
none of the above
The Correct answer is:
Impairment of renal function
160
The elderly patient has impaired renal function already and ibuprofen can further impair the kidneys. This in turn can result in
nephrosis, cirrhosis, and congestive heart failure.
Question 46
Which of the following cognitive development factors would be the least likely to be associated with preschooler growth and
development?
preconceptual thinking
conservation
animism
egocentrism
Correct answer:
conservation
Conservation of number, mass and volume is understanding that a certain aspect or quality of an object can change in
appearance without changing the object itself. It is a factor in the cognitive development of school-aged children, not
preschool children.
Question 47
A teenage patient comes in with a cut to the hand that requires stitches. When you check his medical records, you find that he
had his immunizations for school. The immunizations include DTaP. What action should you take?
Treat the hand and give a tetanus booster.
Treat the hand and give a diphtheria booster.
Treat the hand.
Give a tetanus and diphtheria booster.
Correct answer: Treat the hand The DTaP booster is the diphtheria-tetanus-acellular pertussis vaccine. The patient is up to date
on his vaccinations. It is only necessary to treat the hand.
Question 48
The FNP is advising the mother of a 2-year-old child about the child’s intake of fat in her diet. The FNP knows that the
percentage of the daily total intake of fat for this child should be no more than which of the following?
10%
25%
30 - 35%
40 - 50%
Correct answer:
30 - 35%
Toddlers can have a higher fat level in their diet than older children. Thirty to thirty-five percent is acceptable for nutrition and
digestive needs.
Question 49
You are providing care to a female patient who reports having 8 to 10 migraine headaches per month. Which of the following
would you prescribe to help prevent migraines in this patient?
Luvox.
Demerol.
Maxalt.
Mobic.
:
Correct Answer: Luvox Some antidepressants or anti-anxiety drugs can also help prevent migraines. Luvox is one that is
commonly prescribed. The other choices listed may help treat the pain associated with migraines, but will not prevent them.
Given the frequency of the patient's migraines, some type of preventative medication may be appropriate.
Question 50
A 25-year-old obese Caucasian male had an acute onset of pain that descends down to the lower leg and foot. The nurse
practitioner understands that this is most likely:
osteomyelitis
herniated intervertebral disk injury
lumbosacral strain
osteoporosis
161
The Correct answer is:
Herniated intervertebral disk injury
A herniated intervertebral disk injury results in pain that descends to the lower leg and foot. Lumbosacral strain causes pain in
the back buttock, and sometimes the thigh. Osteomyelitis is usually preceded by an event that permits an infectious agent to
enter the bone. Osteoporosis occurs most often in postmenopausal women.
Question 51
For alcohol withdrawal symptoms, the preferred benzodiazepine is lorazepam. This is used when there is concomitant history
of:
folate-deficiency anemia
hepatic disease
seizure disorder
multiple substance abuse
The Correct answer is:
Hepatic disease
Librium or Valium are good therapeutic agents for patients with adequate hepatic function. However, lorazepam is the
preferred agent for those with impaired hepatic function because of its short half life.
Question 52
Which of the following is the correct laboratory norm for Mean Corpuscular Hemoglobin Concentration (MCHC)?
100 – 200 g/dL
50 – 100 g/dL
31.0 – 37.0 g/dL
21.0 – 27.0 g/dL
Correct answer:
31.0 – 37.0 g/dL
MCHC is a measure of the average color of the RBCs in a sample of blood. It is decreased in iron deficiency anemia and
thalassemia and normal in the macrocytic anemias. The normal is 31.0 – 37.0 g/dL.
Question 53
Piaget’s cognitive development theory includes all of the following EXCEPT:
sensorimotor
postoperational thinking
concrete operations
formal operations
Correct answer:
postoperational thinking
Piaget’s cognitive development theory focuses on intellectual changes which occur in a sequential manner as a result of
continuous interaction with the environment. The stages are: sensorimotor (infancy); preoperational thinking; concrete
operations; and formal operations (involves logic, determines possibilities, problem-solves and makes decisions).
Question 54
Which of the following osteoporosis treatments stops bone loss but does not rebuild bone?
Fosamax
Evista
Teriparatide
Miacalcin
Correct answer:
Miacalcin
Miacalcin (calcitonin salmon) stops bone loss but does not rebuild bone. It is a good choice for relief of bone pain from
vertebral fractures.
Question 55
162
Attachment is the enduring and specific affective bond that develops over the first year of life. The underlying emotion in an
insecure-avoidant attachment is which of the following?
anger
anxiety
love
confusion
Correct answer:
anger
The underlying emotion in an insecure – avoidant attachment is anger. Love is the underlying emotion in a secure attachment.
Anxiety/ambivalence is the underlying emotion in an insecure – anxious attachment. Confusion/dysfunction is the underlying
process in an insecure – disorganized attachment.
Question 56
The NP is discussing medications for insomnia with her elderly patient. The patient says she saw a commercial on TV about a
drug that is commonly used in adults but is known to cause amnesia and oversedation in the elderly. Which drug is the patient
MOST likely referring to?
Barbiturates.
Benzodiazepines.
Zolpidem.
Zaleplon.
Correct answer: Benzodiazepines Benzodiazepines have proven effective in treating insomnia in adults. However, elderly
adults are not advised to take this medication as it can cause disorientation and nausea, among other things.
Question 57
Severe hypoglycemia occurs when a patient’s blood glucose is:
< 100 mg/dL
< 50 mg/dL
> 50 mg/dL
< 80 mg/dL
Correct answer:
< 50 mg/dL
Severe hypoglycemia occurs when the blood glucose is < 50 mg/dL. A patient will feel weak and may have a headache with
clammy hands and anxiety. The patient may also have difficulty in concentration and thinking. Severe hypoglycemia must be
corrected immediately so that it does not progress to coma.
Question 58
A patient who has been prescribed Flomax should be advised to seek immediate medical attention if he experiences which of
the following?
Fast heartbeat.
Runny nose.
Weakness.
Ejaculatory problems.
Correct Answer: Fast heartbeat This is a potentially serious side effect of Flomax. Patients experiencing this should seek
immediate medical attention. Less serious side effects include runny nose, drowsiness, weakness, problems ejaculating, and
blurred vision.
Question 59
Which of the following would you prescribe for a male patient who has difficulty urinating due to BPH?
Luvox.
Uroxatral.
Cialis.
Flagyl.
Correct Answer: Uroxatral Uroxatral, or alfuzosin, is prescribed for men who have difficulty urinating due to BPH. Most
patients experience relief of symptoms within 2-3 weeks. Alfuzosin is approved only for use in men.
163
Question 60
The NP is trying to determine whether her elderly patient's insominia is long term or short term. Of the following, which is
NOT typically associated with short term elderly insomnia?
Depression and anxiety.
Acute stress.
Medications.
Withdrawal of sedatives.
Correct answer: Depression and anxiety There are various causes of insomnia in the elderly. While some may be the cause of
short term insomnia and others long term insomnia, long term insomnia is more common in the elderly than short term.
Question 61
The FNP has a patient who desires to quit smoking. She asks the FNP if there is anything that can help her in trying to quit.
The FNP understands that all of the following are true in regard to smoking cessation EXCEPT:
Bupropion, an atypical antidepressant, is used for smoking cessation.
Bupropion can be used with nicotine products such as gum or nasal sprays.
Bupropion is safe to use for persons with hypertension or seizures.
Nicotine patches should not be used with other nicotine products.
Correct answer:
Bupropion is safe to use for persons with hypertension or seizures.
This is incorrect. Bupropion (Zyban) is contraindicated in persons with hypertension or a history of seizures.
Question 62
In terms of the musculoskeletal system, which of the following joint variants is characterized by “knock-knees”?
genu valgum
genu varum
genu recurvatum
none of the above
Correct answer:
genu valgum
Genu valgum is characterized by “knock knees.” Genu varum is characterized by bowlegs. Genu recurvatum is characterized
by a hyperextension or backward curvature of the knees.
Question 63
The physiologic occurrence represented by the rise of plasma glucose in the early morning hours is known as:
Somogyi effect
hypoglycemia
Dawn phenomenon
hyperglycemia
Correct answer:
Dawn phenomenon
The Dawn phenomenon is a normal physiologic event in which the plasma glucose rises in the early morning. This is because
of the reduction of tissue sensitivity to insulin between 5 a.m. and 8 a.m. from physiologic spike of growth hormone.
Question 64
Connor’s Abbreviated Parent-Teacher Questionnaire is used for which of the following?
behaviors
temperament
language
global development
Correct answer:
behaviors
The Connor’s Abbreviated Parent-Teacher Questionnaire is used for behaviors. Another test of behaviors is the Achenbach’s
164
Child Behavior Checklist (ACBCL).
Question 65
Which of the following vaccines would NOT be routinely recommended for patients with HIV?
live, attenuated oral polio vaccine
hepatitis B vaccine x 3
hepatitis A
Td every 10 years
Correct answer:
live, attenuated oral polio vaccine
The live, attenuated oral polio vaccine should be avoided because of an increased risk of paralytic polio in
immunocompromised vaccine recipients. If a patient requires polio vaccination, clinicians should use the inactivated vaccine
to avoid the risks of a live vaccine.
Question 66
Certain claims are made on advertisements regarding medications, such as Drug X has been known to be in use for 6 years
with over 2 million doses given in North America and Great Britain. Drug X stops heartburn and upset stomach, prevents
esophageal reflux and is the preferred treatment for GERD. The nurse practitioner realizes that this claim is:
Valid; there are enough users mentioned who have had success for it not to be.
Invalid; the level of significance is not mentioned to be at the 0.05 level.
Valid; the cohort and Hawthorne effects are operating.
Invalid; there are no control or comparison groups, and no statistics are stated.
The Correct answer is:
Invalid; there are no control or comparison groups, and no statistics are stated
Even if the claim details extensive use of this drug, Drug X, there must be valid statistics to provide evidence with the use of a
control group that will render a level of significance.
Question 67
All of the following persons were involved in the ethologic theory of human growth and development EXCEPT:
Konrad Lorenz
Carl Rogers
Harry Harlow
Mary Ainsworth
Correct answer:
Carl Rogers
Carl Rogers was a promoter of the humanistic theory of human growth and development rather than the ethologic theory. The
other choices are those who were involved in ethologic theories along with John Bowlby, Marshall Klaus and John Kennel.
Question 68
In advising a new mother who is not able to breast feed about bottle feeding the FNP would tell her all but which of the
following?
The amount of formula the infant needs at 0 to 1 month is about 2 – 4 oz. every 3 to 4 hours.
The amount of formula the infant needs at 2 – 4 months is about 5 – 7 oz. every 4 to 5 hours.
Iron-fortified formulas are best.
Suggest partial hydrolysate formula if the infant is at risk for cognitive deficiencies.
Correct answer:
Suggest partial hydrolysate formula if the infant is at risk for cognitive deficiencies.
The FNP would not tell the bottle-feeding mother this information. It is incorrect. The FNP would only suggest partial
hydrolysate formula if the infant is at risk for atopic diseases, for instance a strong family history of atopic disease.
Question 69
Konrad Lorenz is best known for the theory that:
there are biologically-programmed periods (sensitive periods) predisposed for particular learning.
maternal separation and social isolation resulted in dramatic impairment of social-emotional development.
promoted rooming-in and father participation in the birth of a child.
neutral stimulus associated with a meaningful one over time leads to a “conditioned” response that can be elicited by neutral
stimulus alone as though it were the meaningful one.
165
Correct answer:
there are biologically-programmed periods (sensitive periods) predisposed for particular learning.
Konrad Lorenz is best known for the theory that there are biologically-programmed periods (sensitive periods) predisposed for
particular learning. It is one of the many ethologic theories developed in the 20th century.
Question 70
A patient has a family history of ovarian cancer and a genetic predisposition. What screening assessment is necessary?
Biopsy.
Pelvic exam.
Abdominal exam.
Breast exam.
Correct answer: Pelvic exam There is not a specific screen for ovarian cancer. A pelvic exam may be required each year. A
transvaginal ultrasound is also effective.
Question 71
In the treatment of the patient with polycystic ovary syndrome (PCOS), you know that the following are measures to manage
this condition EXCEPT:
Exercise and weight loss will improve this condition.
Metformin (Glucophage) improves insulin sensitivity, lowers LH, lowers androgen, and improves fertility.
Estrogen excess leads to hyperinsulinemia in these patients.
Monthly Provera 10 mg daily for 10-14 days at the end of each month is recommended.
The Correct answer is:
Estrogen excess leads to hyperinsulinemia in these patients
It is the androgen excess, and not estrogen excess, that leads to insulin resistance. In turn, the androgen excess spurs
hyperinsulinemia and increases the patient’s risk for diabetes mellitus. Polycystic ovary syndrome occurs when the menstrual
cycle is interrupted. Also, patients with polycystic ovary syndrome have elevated levels of androgen, which is a male
hormone. With polycystic ovary syndrome, cysts emerge in the ovaries that contain fluid. The fluid causes the sacs to get
bigger. Further, polycystic ovary syndrome is triggered by too much luteinizing hormone (LH) produced by the pituitary
gland. The increased LH triggers the production of androgen, which in turns can cause metabolic syndrome to occur.
Polycystic ovary syndrome is diagnosed based on the patient’s symptoms, blood tests to evaluate the hormone levels and in
some cases ultrasonography. For females who do not want to become pregnant, combination oral contraceptive containing
estrogen and progestin may be prescribed to reduce the androgen levels.
Question 72
Organized creation of beneficial change to attain unprecedented levels of performance is which of the following?
risk management
quality improvement
quality assurance
none of the above
Correct answer:
quality improvement
Quality improvement is the organized creation of beneficial change to attain unprecedented levels of performance. It differs
from quality assurance in that it is continuous rather than episodic.
Question 73
A patient presents in the office 3 weeks post-myocardial infarction (MI) complaining of pericardial pain and elevated
temperature. Physical examination reveals a pericardial friction rub. The nurse practitioner determines which of the following
diagnostic studies are necessary?
cardiac enzymes with myoglobin
complete blood count (CBC) with differential
echocardiogram
24-hour holter monitoring
The Correct answer is:
Complete blood count (CBC) with differential
166
Post-myocardial infarction, also known as Dressler's syndrome, may develop 1 to 4 weeks after a myocardial infarction (MI).
A post-myocardial infarction is characterized by pericarditis with effusion and fever. Also, post-myocardial infarction or
Dressler's syndrome is due to antigen-antibody reactions. Laboratory findings for this condition include elevated white blood
cell count and erythrocyte sedimentation rate.
Question 74
What is the significance of an S4 heart sound?
It is a marker of ventricular overload or systolic dysfunction or both.
It is a marker of poor diastolic function, most often found in poorly controlled hypertension or recurrent myocardial ischemia.
It marks the end of systole and is produced by events surrounding closure of aortic and pulmonic valves.
It marks the beginning of systole and is produced by events surrounding closure of the mitral and tricuspid valves.
Correct answer:
It is a marker of poor diastolic function, most often found in poorly controlled hypertension or recurrent myocardial ischemia.
The S4 heart sound is a marker of poor diastolic function, most often found in poorly controlled hypertension or recurrent
myocardial ischemia. It is heard late in diastole, and can sound like it is “hooked on” to the front of S1.
Question 75
A 15-year-old Caucasian male presents in the clinic with a reddened, pruritic rash. The nurse practitioner understands that the
most common sites for adolescent atopic dermatitis are:
forehead, scalp, and cheeks
wrists, ankles, and cubital core
face, neck, back, and antecubital fossae
palmar creases and extensor surface of legs
The Correct answer is:
Face, neck, back and antecubital fossae
In the adolescent and the young adult population, common sites for a pruritic rash are the popliteal and antecubital fossae,
face, neck, upper arms and back, dorsa of the hand, feet, fingers, and toes.
Question 76
A patient in his 50s complains about a blurred vision when reading. What is this most likely to indicate?
Miagraines.
Age.
Tumor.
Poor diet.
Correct answer: Age Eyesight typically decreases with age. Age is the most likely cause of poor nearsighted. The other
answers are less likely than the aging process.
Question 77
If a clinician is doing a research study that measures the impact of a fall prevention program on the number of falls among
older adults it would be which of the following types of research?
experimental research
descriptive research
historical research
correlational research
Correct answer:
experimental research
Experimental research is research aimed at determining the effect of one or more variables. The fall prevention research falls
into this category. For a true experimental design to exist, the sample must be randomly selected; there must be manipulation
of the independent variable by the researcher as is usually seen in intervention studies; and there must be a control group.
Question 78
An option to the injectable influenza vaccination is the nasal spray vaccine. It is important that the nurse practitioner
understands to advise a patient:
Its use is limited to children younger than 6 years.
This is the preferred method of immunization for persons who are allergic to eggs or egg products.
167
It contains a live, attenuated virus.
It is acceptable to use during pregnancy.
The Correct answer is:
It contains live, attenuated virus
The nasal influenza virus contains influenza viruses that are sufficiently weakened as to be incapable of causing disease.
However, the virus is strong enough to stimulate a protective immune response. The influenza virus vaccine is indicated for
individuals aged 2 to 49 years, if the individual is healthy. The vaccine is contradicted in persons with egg allergy and
pregnant women.
Question 79
Quality assurance is a system designed to evaluate and monitor quality of care. Which statement is NOT an accurate
concerning quality assurance?
These programs identify components of structure, process and outcomes of care.
These programs look at organization effectiveness, efficiency and client and provider interactions.
These programs audit financial records and payroll of the organization to evaluate appropriateness of monetary distribution.
These programs promote responsibility and accountability to deliver high-quality care and assist in the evaluation and
improvement of patient care.
The Correct answer is:
These programs audit financial records and payroll of the organization to evaluate appropriateness of monetary distribution
Payroll and financial records of an organization are not evaluated and viewed in a quality assurance program. These programs
are set up to evaluate and monitor the care of patients by providers, ancillary staff, personnel and the facility in general.
Question 80
A woman who has been on hormone replacement therapy (HRT) is likely to have some possible improvement. Which of the
following is one of these?
A decrease in the symptoms associated with menopause.
A reduction in the risk of dementia.
A reduction in the risk of osteoarthritis.
A reduction in the risk of endometrial cancer risk.
The Correct answer is:
A decrease in the symptoms associated with menopause
As with all medication, the use of HRT comes with the possibility of adverse effects. There is no decrease in the risk of
dementia or osteoarthritis. There is a reduction of osteoporosis risk. Endometrial cancer risk, and not a reduction in the risk,
with unopposed estrogen exists.
Question 81
Which of the following is likely to be an age-related issue for school-age children?
magical thinking
egocentrism
concern about being different
modesty
Correct answer:
modesty
Modesty emerges in older school-age children. Magical thinking and egocentrism are related to the toddler years. Concern
about being different relates to adolescence.
Question 82
Assessing an elderly male you note an abnormality. Which of the following is not a common age-related change?
Arcus senilis
presbyopia
sustained nystagmus
sensitivity to glare
The Correct answer is:
Sustained nystagmus
Sustained nystagmus is indicative of a neurologic complication. The other answer choices are associated with normal agerelated changes.
168
Question 83
A 42-year-old male is in the clinic and has been diagnosed with peptic ulcer disease (PUD). The nurse practitioner (NP)
understands that which of the following would be prescribed as the initial treatment for this disease in an uncomplicated state.
doxepin (Sinequan) 25 mg at hs
pirenzepine 50 mg tid
clarithromycin 500 mg bid
omeprazole (Prilosec) 20 mg qd
The Correct answer is:
Omeprazole (Prilosec) 20 mg qd
The goal of treatment for peptic ulcer disease (PUD) includes relief of pain, healing of the ulcer, and cost-effectiveness. A
proton pump inhibitor will heal 90% of duodenal ulcers after 4 weeks and 90% of gastric ulcers after 8 weeks. Doxepin and
pirenzepine are anticholinergic drugs and are limited by side effects and not used in the U.S. at this time. Clarithromycin is
part of the eradication therapy to treat Helicobacter pylori associated problems
Question 84
The World Health Organization has put forth some precautions for use of combined oral contraceptive pills. According to
WHO which of the following medical situations would have NO restriction on use?
BMI ≥ 30
HIV
acute hepatitis
hypertension adequately controlled without vascular disease
Correct answer:
HIV
A person with HIV would fall into Category 1 which has no restriction. The other choices fall either into Category 2, the
advantages outweigh the risk; Category 3, exercise caution; or Category 4, refrain from use.
Question 85
The nurse practitioner understands that the pain experienced with angina pectoris or myocardial infarction is related to severe
irritation of the myocardial nerve fibers by an increase in which of the following:
lactic acid
serum potassium
serum magnesium
blood glucose
The correct answer is:
Lactic acid
When the myocardial cells are deprived of glucose from occlusion of the coronary arteries, aerobic metabolism does not occur.
Therefore, lactic acid accumulates and irritates the myocardial nerve fibers. This sends pain messages to the cardiac nerves
and upper thoracic posterior roots located in the left shoulder and arm
Question 86
You are treating a 48-year-old male with recurring gout. The nurse practitioner knows that all of the following are treatment
options during the acute phase except:
indomethacin (Indocin) BID
colchicine 0.5mg, every hour until relief or until diarrhea occurs
naproxen sodium (Anaprox) BID PRN
probenecid
The Correct answer is:
Probenecid
Indomethacin (Indocin), colchicine and naproxen sodium (Anaprox) are all treatments for the acute phase of therapy for gout.
Probenecid and allopurinol are two medications used for maintenance, control and prevention of gout, not treatment during the
acute phase.
Question 87
When treating a female patient with an uncomplicated urinary tract infection but otherwise healthy, the nurse practitioner
knows that the preferred therapy is:
169
amoxicillin
azithromycin
TMP-SMX
cephalexin
The Correct answer is:
TMP-SMX
The first line medication therapy choice for an urinary tract infection is trimethoprim-sulfamethoxazole or TMP-SMX. TMPSMX is a combination of drugs such as Bactrim, Cotrim, or Septra.
Question 88
The FNP has a 45-year-old male patient who presents with a sudden onset of facial asymmetry and she is unable to make any
facial expression on the affected side. If nothing else is found upon examination of this woman, which of the following cranial
nerves is likely paralyzed?
CN II
CN III
CN VI
CN VII
Correct answer:
CN VII
Cranial nerve VII is related to facial function. Dysfunction of this nerve gives the characteristic findings of Bell’s palsy (facial
asymmetry, droop of mouth, absent nasolabial fold, and impaired eyelid movement).
Question 89
Which of the following is a precursor lesion of squamous cell carcinoma?
actinic keratoses
Stevens-Johnson syndrome
erythrema multiforme
erythema migrans
Correct answer:
actinic keratoses
Actinic keratoses are precursor lesions of squamous cell carcinoma. They appear as dry, red lesions with a rough texture and
are usually located in sun-exposed areas of skin such as the cheeks, nose, face, neck, arms and back. They are more common
in light-skinned individuals
Question 90
The levels of E/M services recognize four types of medical decision making. Which of the following is NOT one of these four
types?
low complexity
high complexity
minimal
straightforward
Correct answer:
minimal
The four types of medical decision making recognized by the levels of E/M services are: straightforward, low complexity,
moderate complexity and high complexity. Medical decision making refers to the complexity of establishing a diagnosis
and/or selecting a management option.
Question 91
The nurse practitioner knows the following to be true of an advance directive EXCEPT:
Living wills are written documents prepared in advance in case of terminal illness or nonreversible loss of consciousness.
An advance directive must be created by an attorney for the patient in order for the advance directive to be a legal document.
The provisions of advance directives go into effect when the patient has become incompetent.
The provisions of an advance directives go into effect when the patient is declared terminally ill.
The Correct answer is:
An advance directive must be created by an attorney for the patient in order for the advance directive to be a legal document
170
An advanced directive is a document that states the patient’s wishes or instructions associated with medical preferences if he
or she is unable to make competent decisions or is unconscious and cannot speak to consent to or refuse treatment. An advance
directive can be a durable power of attorney, living will or a healthcare proxy. An attorney does not have to create an advance
directive for the patient. However, the advance directive has to be notarized in some states or witnessed by one to two other
competent individuals. The remaining answer choices are accurate statements regarding an advance directive.
Question 92
There is suspicious S3 heart sound. Where is this heard?
Aortic area.
Pulmonic area.
Tricuspid area.
Bicuspid area.
Correct answer: Pulmonic area The S3 heart sound is heard at the pulmonic area. It can be benign. It may also be a sign of a
condition.
Question 93
In terms of the scope of practice for an NP, which of the following statements is incorrect?
The scope of practice defines a specific legal scope determined by state statutes, boards of nursing, educational preparation
and common practice within a community.
General scope of practice is specified in many published professional documents.
Prescriptive authority is recognized as within the scope of practice for nurse practitioners in all 50 states.
Scope of practice is always defined by state statutes enacted by the state legislature.
Correct answer:
Scope of practice is always defined by state statutes enacted by the state legislature.
Some states define scope of practice by state statutes enacted by the state legislature. In other states, the legislature gives the
board of nursing the authority to define the scope of NP practice. Either way is enforceable.
Question 94
Of the following topical corticosteroids, which one has the highest potency?
Temovate
Topicort
Elocon
Westcort
Correct answer:
Temovate
Temovate is considered a topical corticosteroid with super-high potency. Topicort is considered high potency. Elocon has
midrange potency. Westcort has a low potency.
Question 95
The blink reflex in an infant will typically disappear after:
3 months
6 months
9 months
12 months
Correct answer:
12 months
The blink reflex is when the eyelids close in response to bright light. It disappears after 12 months.
Question 96
Which of the following statements about Managed Care Organizations (MCOs) is false?
MCO is an umbrella term that may include HMOs and other forms of health plans.
MCOs reimburse primary care providers on a fee-for-service basis, a capitated basis or a combination of both.
MCOs do not credential providers; that is done by the federal government.
MCOs sell a priced package of health services to their clients.
Correct answer:
MCOs do not credential providers; that is done by the federal government.
This is the false statement. MCOs do credential providers. They collect educational, license, malpractice, employment and
171
certification data on each provider and make a judgment that a provider is or is not adequately prepared to care for the MCO’s
patients.
Question 97
What is the lifetime risk to the average American man of having latent prostate cancer?
10%
3%
67%
40%
Correct answer:
40%
The average American man has a 40% lifetime risk of latent prostate cancer. He also has an approximate 10% risk of clinically
significant disease and an approximate 3% risk of dying of prostate cancer.
Question 98
A mother asks if her child can come out of the car seat. You are counseling the mother on the use of seat belts for her child.
You anticipate that the adult car seat belts fit correctly when the child is _____ tall and is _____ old.
40 inches
6 to 8 years
50 inches
5 to 6 years
59 inches
8 to 12 years
60 inches
13 to 15 years
The Correct answer is:
59 inches
8 to 12 years
Children should be in a booster seat until they are 59 inches tall or 8 to 12 years of age. Children should remain in the back
seat of the car until they are age 13.
Question 99
Professional Standards of Practice exist in the nurse practitioner's workplace. Which of the following best describes these
Standards of Practice?
They focus on the maximum levels of performance.
They are not necessary to legally describe standard of care, but necessary for licensure.
They are authoritative statements by which the quality of practice, service, or education can be judged.
The standards are exact for all levels of nursing.
The Correct answer is:
They are authoritative statements by which the quality of practice, service, or education can be judged
Standards of Practice focus on the minimum levels of acceptable performance as a way of providing consumers with a means
of measuring the quality of care they receive. They may be generic and general, as well as precise and specific. They are used
to legally describe the standard of care that must be met by a provider.
Question 100
The following abbreviation does not have a substitute meaning for the electronic medical record (EMR):
EHR
CPR
EPR
EBR
Correct answer:
EBR
The electronic medical record (EMR) is often referred to as the electronic health record (EHR), computer-based patient record
(CPR) and the electronic patient record (EPR). These terms are often used interchangeably in the healthcare community. EBR
is not a known abbreviation related to electronic records.
172
aanp test 1
Question 1
A 24-year-old graduate student is in the clinic and newly diagnosed with iron deficiency anemia. She reports she is a strict
vegetarian and has strong beliefs about avoiding meat. What should you suggest as part of her dietary measures that could help
with this condition?
mushrooms, oatmeal, and whole grain bread
dark green leafy vegetables and dried peas and beans
baked potatoes, beets, and broccoli
beets, broccoli, and beef
-------------------------------------------------------------------------------Incorrect. The answer is
dark green leafy vegetables and dried peas and beans
-------------------------------------------------------------------------------Explanation:
The Correct answer is:
Dark green leafy vegetables and dried peas and beans
The patient has iron deficiency anemia because she does not get enough dietary iron. In addition to an iron supplement, she
should be encouraged to eat leafy green vegetables, like spinach, and dried peas and beans, like lentils, black beans, red beans,
and white beans. It is important to instruct the patient on the importance of Vitamin C along with these foods for adequate
absorption.
Question 2
Part of the well-child examination is a lead risk assessment. A positive response to any risk questions indicates risk of lead
exposure. A positive response to which of the following questions would be an indicator of a risk for lead poisoning?
Does your child live in or regularly visit a house that was built before 2005?
Does your child live in a home built before 1978 that is undergoing renovations?
Does your child have a playmate that has had lead poisoning?
All of the above
Correct answer:
"Does your child live in a home built before 1978 that is undergoing renovations?"
A positive answer to this questions would be a risk indicator for lead exposure. If the home was built before 1978 and is
undergoing renovations, then the child may be at risk for lead exposure.
Question 3
Which of the following statements about the role of the family nurse practitioner is incorrect?
The FNP assesses family structure and dynamics to help individuals maximize their health.
The FNP role is a combination of pediatric NP, adult NP and gerontological NP roles.
The FNP teaches family members to recognize the influences of their family health patterns.
FNPs provide a comprehensive psychosocial approach to caring for individuals.
Correct answer:
The FNP role is a combination of pediatric NP, adult NP and gerontological NP roles.
This is incorrect. The FNP role is a unique role. It is not a combination of other NP roles. Being an FNP requires mastery of a
unique body of knowledge and tasks for the care of an individual within a family context
Question 4
Which of the following medications is contraindicated as a treatment for acute gouty arthritis?
aspirin
naproxen
indomethacin
naproxen sodium
173
Correct answer:
aspirin
Aspirin is contraindicated as a treatment for gout because it can precipitate gout. A loading dose of an NSAID, such as
naproxen, 750 mg or indomethacin, 50 mg, followed by lower doses, can be helpful.
Question 5
The nurse practitioner must carry professional liability insurance. An occurrence-form type is preferred because:
The carrier will be notified of a potential claim during the policy period.
The coverage is limited and time based.
The amount of insurance money available to pay a claim increases with each renewal of the policy.
The policy proceeds are available to pay claims regardless of when the claim is reported to the carrier.
The Correct answer is:
The policy proceeds are available to pay claims regardless of when the claim is reported to the carrier
The types of professional liability, as well as general liability, insurances include occurrence form and claims-made. With the
occurrence form, the liability insurance plan pays the claim based on when the loss happens and according to the timeframe
the policy was in place. For instance, the nurse practitioner has professional liability insurance, which is occurrence form
based. The nurse practitioner purchased the insurance coverage in 2007. The nurse practitioner has paid the premiums to keep
the coverage up to date with the XYZ Corporation. A patient files a lawsuit against the nurse practitioner in July 2010 for an
incident that occurred in May of 2008. If the patient wins the lawsuit, the XYZ Corporation will pay the claim, under the
occurrence form plan, because the incident occurred during the timeframe when the policy was in effect (May 2008). With a
claims-made policy, this policy pays claims also when the coverage is in effect. However, the claims made coverage policy
will pay the claim based on the terms identified in the policy at the current time. Therefore, if an incident occurred in 2008 and
a claim is filed in 2011, the incident has to be covered in the current 2011 and not the 2008 policy coverage terms, before the
insurance company pays the claim. Additionally, the answer choice “the carrier will be notified of a potential claim during the
policy period” is incorrect, as the carrier does not need to be informed during the policy period. This is a requirement with the
claims-made type coverage. Further, the limits do not automatically increase with the occurrence form type of coverage, which
makes the answer choice “amount of insurance money available to pay a claim increases with each renewal of the policy”
incorrect.
Question 6
When discussing prenatal visits with a pregnant patient, the Family Nurse Practitioner knows that the recommended frequency
of prenatal visits in the patient's 5th week of pregnancy is:
every week
every 4 weeks
every 3 weeks
every 2 weeks
The Correct answer is:
Every 4 weeks
The patient is in her 5th week of pregnancy. Therefore, she will see the provider every 4 weeks until she reaches her 28th
week of pregnancy. Then, she will start seeing the provider every 2 weeks.
The frequency for prenatal visits, after the patient has had her initial examination, is as follows:
Time of Pregnancy Frequency of Visits
up to 28 weeks every 4 weeks
28 to 36 weeks every 2 weeks
> 36 weeks every week
Question 7
In terms of inflammatory bowel disease, which of the following statements is incorrect?
A person with ulcerative colitis has approximately a 10% chance of having a flare in 2 years after achieving disease remission.
A person with Chrohn’s disease has approximately a 40% chance of having a flare in 2 years after achieving disease
remission.
With ulcerative colitis, colorectal cancer risk is greatly increased afer about a decade of disease.
With Chrohn’s disease there is an increased risk for small bowel malignancy.
Correct answer:
A person with ulcerative colitis has approximately a 10% chance of having a flare in 2 years after achieving disease remission.
174
This statement is incorrect. A person with ulcerative colitis has approximately a 50% chance of having a flare in 2 years after
achieving disease remission. The incidence of a flare within 2 years in someone with Chrohn’s disease is less (40%).
Question 8
Which of the following is NOT a preventative measure for avoiding the ticks associated with Lyme disease?
using insect repellents
wearing long-sleeved shirts and long pants
taking a single 200 mg dose of doxycycline orally
avoiding areas with known or potential tick infestation
Correct answer:
taking a single 200 mg dose of doxycycline orally
This is not a preventative measure. After a tick bite occurs, a single 200 mg dose of doxycycline taken orally seems to be
effective in reducing the Lyme disease risk.
Question 9
A Family Nurse Practitioner working in a primary care facility is treating a patient with atopic dermatitis. She knows that the
most important aspect of skin care for this individual is:
frequent lubricant application
frequent bathing with antibacterial soap
year-round topical high-potency steroid application
frequent oatmeal baths
The Correct answer is:
Frequent lubricant application
Atopic dermatitis, or eczema, is one manifestation of a type I hypersensitivity reaction. Treatment options include avoiding
offending agents, minimizing skin dryness by limiting soap and water exposure and the frequent consistent use of lubricants.
After control of acute symptoms, a low potency topical steroid can be applied.
Question 10
Bony nodules on the distal interphalangeal joints are which of the following?
uric acid crystals
Bouchard’s nodes
Heberden’s nodes
meniscus nodes
Correct answer:
Heberden’s nodes
Heberden’s nodes are on the distal interphalangeal joints (DIP). Bouchard’s nodes are on the proximal interphalangeal joints
(PIP).
Question 11
A 72 year old patient explains to the NP that she thinks she has diarrhea. Her stools are bloody and watery with mucus.
Testing shows the stools are positive for blood and leukocytes. What will MOST likely be the diagnosis?
Shigella.
Adenovirus.
Campylobacter jejuni.
Salmonella.
Correct answer: Campylobacter jejuni is a bacterial diarrhea that is very common in children ages 1 - 5 but can occur at any
age. Vomiting and fever are noticeable symptoms of the illness.
Question 12
A patient who has been prescribed Levsin for IBS asks when she should take her medication. Which of the following is an
appropriate response to this patient?
The patient should take her medication with full meals.
The patient should take her medication on an empty stomach.
The patient should take her medication 30 to 60 minutes before a meal.
The patient should take her medication with a small snack such as crackers or milk.
175
Correct Answer: The patient should take her medication 30 to 60 minutes before a meal Levsin appears to work best with
food, but should be taken 30 to 60 minutes before eating to give it time to enter the body. Unless otherwise directed, the
patient should not take the medication on an empty stomach
Question 13
What is the main purpose of certification?
It is a requirement in all 50 states.
It is a process that measures and validates competence.
The federal government requires this for reimbursement.
all of the above
The Correct answer is:
It is a process that measures and validates competence.
Certification is a process used to measure and validate competence of a Family Nurse Practitioner in his or her specialty area.
Certification is not required by all states for a Family Nurse Practitioner to practice. While Medicare requires certification in
order for the Family Nurse Practitioner to independently bill, all federal organizations do not require it. Then, the answer "all
of the above" is incorrect because there is one correct answer.
Question 14
In Ecology of Human Development: Experiments b Nature and Design, Urie Bronfenbrenner identified a person-place-process
model. Which of the four systems that he identified has a broad-based historical, cultural, demographic and institutional
context?
mesosystem
microsystem
macrosystem
exosystem
Correct answer:
macrosystem
Bronfenbrenner’s macrosystem has a broad-based historical, cultural, demographic and institutional context. Examples would
include managed care and welfare reform initiatives.
Question 15
A patient you diagnosed with hypothyroidism was started on levothyroxine. At what interval should the nurse practitioner
reassess her TSH?
1 to 2 weeks
2 to 4 weeks
4 to 6 weeks
6 to 8 weeks
The Correct answer is:
6 to 8 weeks
In the treatment of hypothyroidism, T4 replacement is needed in the form of levothyroxine (Synthroid or Levoxyl). The initial
dosage for an adult is 75 to 125 mcg. For an elderly person, the dose is 75% less than the adult dosage. Because of the long
half-life of levothyroxine, the effects of a dosage adjustment or initiation would not cause a change in TSH for approximately
five to six drug half-lives, or about 6 to 8 weeks.
Question 16
The nurse practitioner (NP) has been asked to resolve a conflict between two medical assistants. One is observed as pleasant
and helpful, while the other is abrasive and angry. What guideline must the NP observe in the resolution of this conflict?
Deal with issues, not personalities.
Weigh the consequences of each possible solution.
Require the medical assistants to reach a compromise.
Encourage ventilation of anger and use humor to minimize the conflict.
The Correct answer is:
Deal with issues, not personalities
The conflict must be addressed directly by the nurse practitioner. The personal characteristics of the assistants must not enter
into the conflict resolution process. Compromise is only one method of conflict resolution.
176
Question 17
When treating a person with scabies, permethrin (Elimite) is the preferred method of treatment. Which of the following
statements about this treatment is false?
The lotion must be left on for 8 to 14 hours for it to be effective.
Even with effective therapy, pruritis may still be a significant problem.
Dead mites and their waste are eliminated from the body immediately with treatment.
Lindane (Kwell) should not be used by pregnant women, children or the elderly.
Correct answer:
Dead mites and their waste are eliminated from the body immediately with treatment.
Individuals with scabies often have a significant problem with pruritis after permethrin treatment because of the presence of
dead mites and their waste trapped in the skin. This debris is eliminated from the body over a few weeks.
Question 18
Bowel sounds in a healthy adult patient should sound like all but the following?
Gurgling.
Bubbling.
High pitch.
Loud splashing.
Correct Answer: Loud splashing Normal bowel sounds are high-pitched, bubbling, gurgling sounds. They may vary in
intensity and volume. Loud splashing may indicate an obstruction or other condition.
Question 19
The general family systems theory explains the dynamic structure and function of the family within the context of a unified
whole. All of the following statements about this theory are correct EXCEPT:
Family systems change over time.
Families have no need for homeostasis.
A change affecting one part of a family manifests itself as change in the whole family system.
All parts of the system are dependent on one another, even though each part has its own role.
Correct answer:
Families have no need for homeostasis.
This is not a correct statement in regard to the general family systems theory. Families strive for homeostasis or a predictable
steady state that reflects a balance between change and stability.
Question 20
A male patient is in the office with a deep cough, which worsens at night, as well as wheezing and mucoid sputum production.
A diagnosis of bronchitis is made. Which of the following is NOT a good management technique?
Tell the patient to avoid antihistamines.
Cough suppressants are encouraged in all cases.
Tell the patient to stop smoking.
Antibiotic treatment is not recommended in uncomplicated acute bronchitis.
The Correct answer is:
Cough suppressants are encouraged in all cases
Cough suppressants should be avoided except if the patient is unable to sleep due to irritating cough.
Question 21
You have a patient in the office with symptoms of aortic regurgitation. The nurse practitioner understands which of the
following is characteristic of this?
an acute onset of shortness of breath in the fifth or sixth decade
a long asymptomatic period with sudden death usually during exercise
long asymptomatic period followed by exercise intolerance, then dyspnea at rest
dyspnea on exertion for a long period of time before sudden cardiac death
177
The Correct answer is:
Long asymptomatic period followed by exercise intolerance, then dyspnea at rest
Aortic regurgitation (AR) is characterized by the patient having a long asymptomatic period with slowing activities.
Eventually, the patient develops shortness of breath. Then, left ventricle failure occurs
Question 22
Types of special communication that the nurse practitioner my engage in include all of the following EXCEPT:
triage
case management
written documentation review
opinions stated by staff
The Correct answer is:
Opinions stated by staff
Special communication is privileged and confidential and certain standards must be met. Opinions regarding a patient that are
stated by staff members is not a form of special communication.
Question 23
An expected finding in a 57-year-old female with estrogen deficiency atrophic vaginitis include:
an odorous vaginal discharge
an increased number of lactobacilli
a reduced number of white blood cells
a pH greater than 5.0
The Correct answer is:
A pH greater than 5.0
During menopause, the pH shifts as the vaginal flora changes due to decreased estrogen. An odorous vaginal discharge and
increased number of lactobacilli is expected in bacterial vaginosis.
Question 24
Your patient expresses concern that she will not be able to afford the medication you prescribe because she is not insured.
Which of the following is an appropriate next step?
Give the patient samples of the medication.
Refer the patient to a medical social worker who can explore payment options with her.
Prescribe the patient a cheaper medication.
Reinforce that it is important that the patient take the medication.
Correct Answer: Refer the patient to a medical social worker who can explore payment options with her Patient
noncompliance can be the result of financial constraints. A medical social worker can explore community resources and other
options for patients who have trouble paying for their medication.
Question 25
When treating an infant with colic and instructing the mother, the nurse practitioner knows that all of the following are
nonpharmacologic treatments EXCEPT:
Ensuring adequate burping during feeding.
Encouraging and provide support for the mother.
Adding cow's milk in the diet of the breastfeeding mother is helpful.
Switching to hypoallergenic formula may be helpful.
The Correct answer is:
Adding cow's milk in the diet of the breastfeeding mother is helpful
Cow's milk should be avoided. The remaining answer choices are all nonpharmacologic treatments along with reassuring
parents that no physical problem is present, educating the patient regarding normal infant crying and soothing the infant by
providing motion.
Question 26
The majority of breast cancers occur in which area of the breast?
beneath the nipple and areola
upper outer quadrant
178
lower outer quadrant
upper inner quadrant
The Correct answer is:
Upper outer quadrant
The upper outer quadrant is the most common site for breast cancer. The second most common site for breast cancer is
beneath the nipple and areola.
Question 27
You are providing care to a patient who will undergo an oral glucose tolerance test in the morning. She asks if she needs to
restrict her food prior to the test. Which of the following is an appropriate response to this patient?
Tell the patient that she can eat normally up until the time of the test.
Tell the patient that she should not eat or drink anything for 24 hours prior to the test.
Tell the patient that she should not eat anything or drink anything but water for 10 hours prior to the test.
Tell the patient she can eat normally until 4 hours before the test.
Correct Answer: Tell the patient that she should not eat any food or drink anything but water for at least 10 hours prior to the
test Prior to an oral glucose tolerance test, the patient should not eat or drink anything but water for at least 10 hours. Because
the test seeks to measure the patient's tolerance of the glucose solution, any other food or drink other than water may interfere
with the test results.
Question 28
Which of the following tests is a diagnostic test?
hemoglobin test
Mantoux test
MRI scan
fasting blood glucose test
Correct answer:
MRI scan
Diagnostic tests are more specific and/or sensitive than screening tests. They give objective proof that a disease process or
abnormal condition is present. Other diagnostic tests include: tissue biopsies, cultures and CT scans.
Question 29
Which of the following screening tests for colorectal cancer is used primarily to find cancer as opposed to finding polyps and
cancer?
fecal immunochemical test
flexible sigmoidoscopy
double-contrast barium enema
CT colonography
Correct answer:
fecal immunochemical test
Other similar tests are the fecal occult blood test and the stool DNA test. The other choices for this question are tests that are
used to find polyps and cancer.
Question 30
A nurse practitioner will likely be doing pap smears in practice. It is important to understand that the following statements are
all accurate EXCEPT:
The first cervical cancer screening should begin no later than 21 years of age.
There is no reason to screen women annually if the test has been normal in the past.
Women who are 70 years old or older with three or more normal Pap tests and no history of abnormal Pap tests may choose to
stop cervical cancer screening.
Screening should be done every 2 years for liquid-based Pap tests.
Correct answer:
There is no reason to screen women annually if the test has been normal in the past
This statement is incorrect. At or after age 30 years, women who have had three normal test results in a row may get screened
every 2 - 3 years. However, women with certain risk factors such as DES exposure, HIV infection, or a weakened immune
system should continue to be screened annually regardless of normalcy of previous tests. If a woman is overy 70 with three or
more normal Pap tests and no history of abnormal Pap tests, she may choose to stop cervical cancer screening.
179
Question 31
The prevention measures used as a part of the management of a person with an established disease would fall under which of
the following classifications?
primary
secondary
tertiary
none of the above
Correct answer:
tertiary
Tertiary prevention measures are part of the management of a person with an established disease. The goal is to minimize
disease-associated complications and the negative health effects of the conditions. Medication and lifestyle modification for a
person with Type 2 diabetes would be an example of a tertiary prevention measure.
Question 32
Which of the following statements is incorrect in regard to treatment of rheumatoid arthritis?
The goal of treatment of patients with RA is to reduce inflammation and pain, while preserving function and preventing
deformity.
Water exercise is helpful.
Behavioral management is important.
Traditional DMARDS to minimize the risk of joint damage include both methotrexate and NSAIDs.
Correct answer:
Traditional DMARDS to minimize the risk of joint damage include both methotrexate and NSAIDs.
As helpful as NSAIDs are in symptom control, these products do not alter the underlying disease process. They are not one of
the traditional disease-modifying anti-rheumatic drugs (DMARDs).
Question 33
In terms of teaching a patient about postoperative care, which of the following would be the most effective when time is
limited?
providing written instructions and letting the patient and family use these as a guide
providing a list of outside sources for the patient and family to contact for information
establish the patient’s highest priority learning needs and teaching them with patient and family
ask the patient and family whether they have any questions and answer those questions
Correct answer:
establish the patient’s highest priority learning needs and teaching them with patient and family
In a limited time frame, priorities must be set. The most important teaching should be done in person with the patient and
family. This does not mean that other measures such as written instructions are not helpful in addition to establishing the
patient's highest priority learning needs and teaching them with patient and family.
Question 34
The CMS has developed “Documentation Guidelines for Evaluation and Management” that FNPs and other Medicare
providers are expected to follow in coding patient visits. Which of the following is incorrect in terms of these guidelines?
Medical record documentation must support the level of care billed.
An FNP must distinguish between a new patient and an established patient.
History taking, examination and medical decision making are the key components in determining code selection.
A billable visit may not occur in a patient’s home.
Correct answer:
A billable visit may not occur in a patient’s home.
This is incorrect. A billable visit is a face-to-face contact between the patient and an FNP, physician assistant or physician. An
encounter may occur in the provider’s office, an inpatient setting or the patient’s home.
Question 35
180
Which of the following lesions is a precursor of squamous cell carcinoma?
senile purpurae
lentigines
seborrheic keratoses
actinic keratoses
Correct answer:
actinic keratoses
Actinic keratoses are small rough pin-to-reddish lesions that do not heal. They are usually located in sun-exposed areas. They
are squamous cell precancer skin lesions.
Question 36
Which of the following statements concerning administrative agencies are correct?
The Internal Revenue Service (IRS) promulgates and applies regulations to individual disputes concerning federal taxation.
The Food and Drug Administration (FDA) decides the costs of each medication.
The National Labor Relations Board (NLRB) decides which federal labor laws go before Congress.
The American Medical Association (AMA) decides how national labor law applies to physician offices.
The Correct answer is:
The Internal Revenue Service (IRS) promulgates and applies regulations to individual disputes concerning federal taxation
The Food and Drug Administration (FDA) promulgates regulations and applies them to individual determinations involving
the manufacture, marketing, and advertising of drugs, medical devices, cosmetics, and foods. The National Labor Relations
Board (NLRB) decides how national labor laws apply to individual disputes. The American Medical Association (AMA) does
not involve itself with national labor laws.
Question 37
A 17-year-old female patient is being managed with Depo-Provera for the purpose of birth control. The nurse practitioner
understands that the recommended length of use is usually:
no more than 2 years
as determined by her lipid response to the medication
as long as the patient desires this form of contraception
less than 1 year
The Correct answer is:
No more than 2 years
Depo-Provera is a highly reliable form of contraception and a good choice for young women and adolescents who do not wish
to become pregnant for at least 18 months after discontinuing usage. Due to the U.S. Food and Drug Administration warning,
prolonged use of Depo-Provera can result in loss of bone density. Therefore, it is not recommended that individuals take DepProvera for more than 2 years
Question 38
The patient has the right to confidentiality. The following are all accurate statements regarding patient confidentiality
EXCEPT:
The individual's right to privacy is respected when requesting or responding to a request for a medical record.
HIPAA was set up by the Department of Health and Human Services and stands for Health and Insurance Portability Advisory
Association.
The statute requires that the provider discuss confidentiality issues with the patient and establish consent.
The provider shall not discuss any information with anyone unless the patient has given consent to do so.
The Correct answer is:
HIPAA was set up by the Department of Health and Human Services and stands for Health and Insurance Portability Advisory
Association
HIPAA stands for Health and Insurance Portability and Accountability Act, and not Advisory Association. HIPAA emerged in
1996 and the Department of Health and Human Services plays a role in governing this act. Further, HIPAA establishes
national standards for electronic healthcare transactions and national identifiers for providers, health plans, and employers
Question 39
The nurse practitioner knows that intervention for patients with heatstroke includes:
total body ice packing
rehydration
withholding fluids until nausea subside
181
sodium and potassium supplements
The Correct answer is:
Rehydration
Heatstroke is a life-threatening emergency caused by a failure of the body's thermoregulatory system, usually in response to
extreme environmental and personal factors. Interventions include cooling the patient with the use of tepid sprays, fanning the
body, and aggressive rehydration.
Question 40
In terms of infant growth and development from birth to 2 years, major tasks in the cognitive development domain include all
of the following EXCEPT:
presence of blink reflex in newborns
preconceptual thinking in older infants
a greater auditory acuity for high rather than low frequency sounds as infants
binocular vision
Correct answer:
preconceptual thinking in older infants
Preconceptual thinking does not take place as a cognitive development until the ages of 2 to 4. All of the other choices are
cognitive developments that take place from birth to 2 years.
Question 41
A 30-year-old female comes to the clinic complaining of a small raised area on the left eyelid. The FNP diagnoses a
hordeolum. The treatment for this condition would include all of the following EXCEPT:
warm compresses to the eye for 10 minutes three to four times a day
application of erythromycin ointment to the affected lid
incision and drainage if needed
intralesion corticosteroid injection if needed
Correct answer:
intralesion corticosteroid injection if needed
A hordeolum is also known as a stye and is usually caused by a staphylococcal infection of a hair follicle on the eyelid. The
intralesion corticosteroid injection would not be used for a stye. It is used for a chalazion if other treatment is not successful. A
chalazion is an inflammatory eyelid condition that may not involve infection but can follow hordeolum.
Question 42
Which of the following treatments would be considered a long-term control medication for asthma sufferers?
Albuterol HFA
Xopenex
Accolate
Ventolin HFA
Correct answer:
Accolate
Accolate (zafirlukast) is a leukotriene receptor antagonist that is used for long-term control of asthma in adults and children 5
– 11 years old. The other choices are all quick-relief (rescue) medications.
Question 43
When assessing cardiac status, the nurse practitioner knows that the correct ausculatory site for the aortic area is the:
left fourth interspace close to the sternum
right second interspace close to the sternum
midclavicular line, second interspace, left side
midclavicular line, fifth interspace, left side
The Correct answer is:
Right second interspace close to the sternum
The right side of the chest close to the sternal boarder at the second intercostal space is the correct area to auscultate the aortic
valve. The mitral valve is heard at the fifth left intercostal space at the midclavicular line. The tricuspid valve is auscultated at
182
the fourth left intercostal space. Then, the pulmonic valve is heard
Question 44
Prophylactic treatment for migraine headache includes the use of:
naproxen sodium
propranolol
acetaminophen
nonsteroidal anti-inflammatory drugs (NSAIDs)
The Correct answer is:
Propranolol
Migraine headaches can be prevented with propranolol. Prophylactic therapy is aimed at limiting the number and severity of
future headaches. Abortive therapy is used to stop a headache.
Question 45
The standard billing form for billing third-party providers is which of the following?
E&M 1200
E&M 1500
CMS 1200
CMS 1500
Correct answer:
CMS 1500
The standard billing form is the CMS 1500. It can be purchased from the American Medical Association or from other
commercial suppliers. The CMS 1500 form asks for ICD codes, CPT codes, date of service, patient identifying information
and provider identifying information. A bill submitted without a CPT or ICD code will be rejected.
Question 46
A patient who has been given a nursing diagnosis of vesicular breath sounds would be expected to display which of the
following?
Difficulty breathing.
High-pitched, wheezy breathing.
Low-pitched, breezy breathing.
Low-pitched, hollow breathing.
Correct Answer: Low-pitched, breezy breathing Vesicular breath sounds are low-pitched, breezy, and soft. They are associated
with normal breathing and are heard when ausculating over the lung area. Though they are considered normal, it is key that
these breath sounds be documented as part of an overall assessment, as they are valuable information for diagnosis.
Question 47
A 11-year-old girl is brought into the office with clinical symptoms and a diagnostic history for scabies. The nurse practitioner
knows that she was probably infected:
1-3 days ago
1 week ago
2 weeks ago
3-4 weeks ago
The Correct answer is:
3-4 weeks ago
The incubation period for scabies is about 3-4 weeks after primary infection. Further, the patient with scabies will develop
symptoms in 1-3 days. The classic complaint from patients with scabies is nocturnal pruritus.
Question 48
Which of the following would not be considered a body change related to menopause?
atrophied ovaries
labia and vagina become atrophic and thinner
urinary incontinence
palpable ovaries
Correct answer:
palpable ovaries
Palpable ovaries are considered an abnormal finding in menopausal women. After several years of menopause, the ovaries are
183
atrophied and a smaller size. Ovarian cancer must be ruled out when a palpable ovary is found.
Question 49
You are counseling a 23-year-old with an abnormal pap smear. It is important you discuss the risks of cervical cancer. Which
of the following is NOT a risk factor for cervical cancer?
virginal status
previous high-grade squamous intraepithelial lesion
human papillomavirus
multiple sexual partners
The correct answer is:
Virginal status
A female who has not had sexual intercourse is not at risk for developing cervical cancer. However, the remaining answer
choices are risk factors for cervical cancer.
Question 50
A 25-year-old female patient comes to the clinic with a “rash” on the inner wrist area. She tells the FNP that it is extremely
itchy, especially at night. The FNP tells her that she has scabies. The FNP understands that all of the following are true in
regard to this disease EXCEPT:
Kwell (lindane) is the preferred treatment.
Permethrin 5% can be used as a treatment by applying to the entire body and head and washing off after 8 to 14 hours.
The entire household must be treated.
Linens and clothes must be washed in very hot water.
Correct answer:
Kwell (lindane) is the preferred treatment.
All of the choices are true except this one. Kwell may be used as an alternative treatment but it is out of favor as a treatment
due to neurotoxicity.
Question 51
Participants in a research study who do not have the disease or condition that is being studied, but who are included for
comparison are:
case series
case subjects
cross sectionals
controls
The Correct answer is:
Controls
Controls are commonly used in many different types of research studies. Additionally, case series refer to an observational
study where patients with interesting characteristics are studied. Case subject is another term for participants. Cross sectionals
are types of observational studies where a particular characteristic is studied at one time rather than over a period of time.
Question 52
With prepatellar bursitis, which of the following should be considered as a first-line-therapy?
minimizing the offending activity
applying ice to the affected area for 15 minutes at least 4 times a day
NSAIDs
bursal aspiration
Correct answer:
bursal aspiration
With prepatellar bursitis, bursal aspiration should be considered as a first-line therapy. This is because this procedure affords
significant pain relief and allows the bursa to reapproximate. The therapies in the other three choices are appropriate for
bursitis in other sites.
Question 53
What is the most common type of anemia worldwide?
anemia of chronic disease
184
pernicious anemia
iron-deficiency anemia
sickle cell anemia
The Correct answer is:
Iron-deficiency anemia
Iron deficiency anemia is the most common type of anemia in the world. It is estimated that 8 years of poor iron intake is
needed in adults before chronic iron-deficiency anemia occurs. Causes of iron-deficiency anemia include menstrual blood loss,
blood loss from malignancy, and other gastrointestinal related blood loss. Diet is rarely the etiology.
Question 54
Which of the following would be considered a prophylactic treatment for migraine headaches?
Imitrex
Cafergot
Tigan
Inderal
Correct answer:
Inderal
Inderal is a prophylactic treatment for migraine headaches. The other drugs are abortive treatment. Tricyclic antidepressants
such as Elavil may also be used as a prophylactic treatement.
Question 55
An infant is brought in with a temperature of 99.7 F. The parents insist that this is higher than average. What is the appropriate
response?
Run a CBC.
Call a consult.
Continue the assessment.
Tell the parents they are overreacting.
Correct answer: Continue the assessment An infant's body temperature typically runs a little high. 104 degrees F is considered
a fever. Temperature can fluctuate with individuals, so it is important to complete the assessment.
Question 56
The FNP is educating a group of women about prevention of osteoporosis. In this class, the FNP would tell the group all but
which of the following?
Primary prevention of osteoporosis includes ensuring the development of maximal adult bone density.
Calcium intake and weight-bearing exercises throughout the teen and adult years is important in achieving maximal adult bone
density.
The daily calcium intake goal should be the equivalent of 1000 mg/d for premenopausal women.
The recommended minimal dose of vitamin D is 3000 IU/d daily.
Correct answer:
The recommended minimal dose of vitamin D is 3000 IU/d daily.
This statement would not be included in the talk. It is incorrect. The recommended minimal dose of vitamin D is 600 – 900
IU/d daily. Daily doses of up to 2000 IU/d are likely safe, but 3000 IU/d is too much.
Question 57
How far away should you hold the ophthalmoscope to check the red reflex?
12 inches.
6 inches.
24 inches.
8 inches.
Correct answer: 12 inches The red reflex monitors the pupil reactions. They should be observed from 12 to 16 inches away.
Question 58
The FNP is advising an older patient who has unintended weight loss on what energy-dense protein foods to eat. The FNP
would recommend which of the following as part of this energy-dense protein diet?
peanut butter and eggs
carrots and other crunchy vegetables
185
breads and other grain products
fruits
Correct answer:
peanut butter and eggs
Peanut butter and eggs are good sources of complete proteins and are energy and nutrient dense. The other choices are not
good sources of energy-dense protein.
Question 59
The kidneys are located in the retroperitoneal area. The right kidney is lower than the left, the lower half falling below the rib
cage. The reason for this is:
better functioning of the kidneys
displacement by the liver
displacement by the stomach
none of the above
Correct answer:
displacement by the liver
The lower half of the right kidney falls below the rib cage because it is displaced by the liver. The kidneys are the body’s
regulators of electrolytes, fluids and bicarbonate.
Question 60
Which of the following are you NOT likely to see in geriatric patients?
a decrease in the GFR
less elasticity and cilia in the lungs
less hydrochloric acid in the gastrointestinal system
decreased levels of insulin
Correct answer:
decreased levels of insulin
This is not likely to be seen in geriatric patients. You are more likely to see increased levels of insulin along with mild
peripheral insulin resistance.
Question 61
An otherwise healthy adult female patient presents with a series of dry, plaque-like skin lesions. She reports that they
occasionally bleed when bumped but do not otherwise itch or suppurate. Which of the following is the MOST likely diagnosis
for this patient?
Psoriasis.
Hyperhydrensis supperitiva.
Keratosis pilaris.
Skin cancer.
Correct Answer: Psoriasis This patient's lesions are consistent with psoriasis. Since she is otherwise healthy, do they do not
indicate an underlying health condition.
Question 62
In relation to uterine size during pregnancy, the nurse practitioner knows that the uterus is approximately tennis-ball sized at
what stage of pregnancy?
6 weeks
8 weeks
12 weeks
20 weeks
The Correct answer is:
8 weeks
The uterus is the size of a lemon nongravid and is mobile, firm and nontender. By 8 weeks gestation, it has grown to the size
of a tennis ball or orange. At 12 weeks, the uterus is the size of a softball or grapefruit. By 20 weeks, the uterine fundus is at
the umbilicus.
186
Question 63
At about what age can the FNP expect to see an infant keep his back straight when pulled to sitting?
1 – 2 months
3 – 4 months
5 months
9 – 11 months
Correct answer:
5 months
At 5 months an infant will typically keep his back straight when he is pulled to a sitting position. At this age infants also bear
weight on their legs when standing, play with their feet and sit with support.
Question 64
Which of the following patients has the highest risk factor for peptic ulcer disease?
a 65-year-old female who drinks 4 cups of coffee per day
a 45-year-old male who has one or two alcoholic drinks per day
a 55-year-old male who has cirrhosis as a result of alcohol abuse
none of the above
Correct answer:
a 55-year-old male who has cirrhosis as a result of alcohol abuse
This is the patient who has the greatest risk factor for peptic ulcer disease. Coffee drinking and occasional alcohol use are not
risk factors for peptic ulcer disease. H. pylori is also found in individuals with asymptomatic gastritis and dyspepsia without
ulceration.
Question 65
If an adult presents with the classic triad of fever, headache and stiff neck, which of the following diseases/conditions would
these symptoms most likely indicate?
migraine headache
bacterial meningitis
mononucleosis
chronic epidural hematoma
Correct answer:
bacterial meningitis
Of the choices given, the most likely diagnosis would be bacterial meningitis. As with most forms of infectious disease,
however, atypical presentation in older adults is common. In particular, stiff neck and fever are often absent.
Question 66
A mother brings her 3-year-old son into the clinic for a routine checkup. The mother tells the FNP that her child “acts out” a
lot and that she needs help in addressing this behavior. The FNP suggests a “time out” for the child when he misbehaves.
Which statement in regard to a “time out” for children is most accurate?
The “time out” should last for as many minutes as the child has misbehaved.
The child should sit still in the “time out” for as many minutes as his age in years.
The child should be allowed to read or draw during the “time out.”
“Time outs” should not be used until a child is at least old enough to have started school.
:
The child should sit still in the “time out” for as many minutes as his age in years.
A “time out” for a child should not last longer than his age in years. Since this child is three years old, his “time out” should be
three minutes. He should sit still during this time, not read, draw or do any other activity
Question 67
An elderly patient presents with diarrhea. She explains to the NP that her stool is large and liquid. Testing shows that her stool
shows no signs of blood or leukocytes. The NP will MOST likely give a diagnosis of what type?
Shigella.
187
Adenovirus.
Campylobacter jejuni.
Salmonella.
Correct answer: Adenovirus Adenovirus is a viral type of diarrhea. Symptoms include nausea, vomiting and low-grade fever,
among other things
Question 68
Which of the following parts of the eye are used for color perception?
cones
rods
fundus
macula
Correct answer:
cones
The cones are for color perception. The fundus is the interior surface of the eye opposite the lens. The rods are for detecting
light and for depth perception. The macula is for central vision.
Question 69
Which of the following is not a typical complication of anorexia nervosa?
amenorrhea
weight loss of > 5% of body weight
cardiomyopathy
lanugo
Correct answer:
weight loss of > 5% of body weight
This is not a typical complication because the weight loss in a person with anorexia would amount to at least 15% of body
weight. Osteoporosis from prolonged estrogen depletion and stress fractures are also complications.
Question 70
A 45-year-old woman complains to the FNP that her 80-year-old mother is too demanding and that it has created a great stress
in her life. According to Havighurst’s developmental tasks, this complaint reflects which of the following?
a need for independence
a need to adjust to an aging parent
establishing an explicit affiliation with one’s own age group
a need to achieve emotional independence
Correct answer:
a need to adjust to an aging parent
One of Havighurst’s developmental tasks for 30 – 60 year old persons is adjusting to an aging parent. Other tasks in this age
group include: achieving adult social and civic responsibility, developing adult leisure time activities and relating oneself to
one’s spouse as a person.
Question 71
A middle-aged female presents with abnormal uterine bleeding. A hormonal profile reveals increased FSH and LH levels.
What is the most likely cause for these findings?
onset of climacteric
premature ovarian failure
anterior pituitary disorder
hypothalamic disorder
The Correct answer is:
Onset of climacteric
The amount of circulating estrogen begins to fall and the ovaries decline to function. The middle-aged woman may begin to
experience symptoms typically associated with menopause. The body's feedback system will attempt to stimulate the ovaries
and increase estrogen level. FSH and LH levels rise in response to these efforts.
188
Question 72
Medical decision making refers to the complexity of establishing a diagnosis and/or selecting a management option. The four
levels of evaluation and management services include all of the following EXCEPT:
high complexity
low complexity
straightforward
up-front
Correct answer:
up-front
There is no up-front level of E/M services. The four types of medical decision making are: straightforward, low complexity,
moderate complexity and high complexity
Question 73
Title XIX of the Social Security Act is also known as which of the following?
Americans with Disabilities Act
Medicare
OSHA
Medicaid
Correct answer:
Medicaid
Title XIX of the Social Security Act is also known as Medicaid which is a federal and state matching program that offers
medical assistance to low income persons, the disabled, blind or members of families with dependent children. It pays for
health care, nursing home, and prescription drugs.
Question 74
You are treating a 64-year-old man who has the following PSA levels. What can be concluded about the following annual
readings?
Year 1: 3.8 ng/mL Year 2: 4.5 ng/mL Year 3: 5.5 ng/mL
There is a steady increase that is worrisome.
There is a steady increase but it is not worrisome.
They are all within normal range.
None are within normal range.
The Correct answer is:
There is a steady increase that is worrisome
The prostate gland produces an antigen called prostate-specific antigen (PSA). The PSA test detects the amount of PSA in the
patient’s blood. Typically, the normal range for the PSA level is below 4 ng/mL in an elderly male and below 2.5 ng/mL in a
younger male. In the case of the 64-year-old patient, his PSA level during year 1 was 3.8 ng/mL, which is in the normal range.
However, starting at year 2, the patient’s PSA level increased too 4.5 ng/mL, which is outside of the normal range. Then, it
elevated to 5.5 ng/mL in year 3, demonstrating his PSA level is steadily increasing outside of the normal range of 4 ng/mL.
This elevation in PSA levels is a sign that some condition is causing the patient’s PSA level to rise, such as prostatitis, benign
prostatic hyperplasia (BPH) or even prostate cancer, for instance. Therefore, the patient should be referred for further testing,
including a referral to an urologist for a prostate biopsy to rule out prostate cancer. Statistics reveal that about two thirds of
elderly male individual’s develop prostate cancer, when their PSA levels are elevated more than 10 ng/ mL. Further, statistics
suggest that about 25% of males who have elevated PSA levels between 4 to 10 ng/mL have a condition contributing to the
increase in the PSA.
Question 75
Which of the following would NOT be a direct mechanism of transmission of infection?
touching
sexual intercourse
food
childbearing
Correct answer:
food
189
Transmission through food is an indirect way to transmit infection. Direct mechanisms of transmission of infection include
(besides the other three choices): kissing, breastfeeding and transfusions.
Question 76
A teenage patient comes in and his chief complaint is painful urination. What will you want to determine in the interview?
Exercise.
Diet.
Family history.
Sexual activity.
Correct answer: Sexual activity Sexual activity is more likely to begin during the teen years. Different sexually transmitted
diseases present with painful urination. Sexual activity will determine the tests needed to determine the condition.
Question 77
What is a normal reaction after tapping a patient's Achilles tendon in an exam?
Plantar flexion at the ankle.
Extension of leg at the knee.
Extension of the ankle.
Flexion of leg at the knee.
Correct answer: Plantar flexion at the ankle All of the answers are normal reflexes. The Achilles' tendon, however, should
cause plantar flexion at the ankle. An abnormal reflex may be a sign of damaged nerves.
Question 78
A patient with sensitive skin had a TB skin test but has no symptoms. He returns with redness and swelling that appears
positive. What is the next step?
Prescribe medication.
Schedule a chest x-ray.
Repeat the test.
Quarantine the patient.
Correct answer: Schedule a chest x-ray A chest x-ray shows signs of TB and is more accurate than the skin test. The skin test
may appear positive if there is exposure to the illness or allergic reaction. Because the patient has no symptoms, the chest xray is recommended.
Question 79
When treating the patient with heartburn symptoms, the nurse practitioner understand that the drug that is most likely to
produce rapid relief for a patient is:
H2 blockers
proton pump inhibitors
sucralfate
antacids
The Correct answer is:
Antacids
Antacids produce the most rapid change in gastric pH and the most rapid relief of symptoms. H2 blockers and proton pump
inhibitors may take many hours before relief is realized. Therefore, H2 blockers are not adequate for immediate relief. Further,
sucralfate does not affect gastric pH.
Question 80
Which of the following is a pansystolic murmur that is heard best at the apex or the apical area, radiates to the axilla and is a
loud-blowing and high-pitched murmur?
tricuspid regurgitation
aortic stenosis
mitral regurgitation
none of the above
Correct answer:
mitral regurgitation
Mitral regurgitation is a pansystolic murmur that is heard best at the apex or the apical area, radiates to the axilla and is a loudblowing and high-pitched murmur. The diaphragm of the stethoscope is used to detect mitral regurgitation.
Question 81
190
You are seeing a 3-year-old female child in the office. Her mother is concerned about her not being able to elevate her left arm
without crying out. A clavicular fracture is suspected. Which of the following is NOT true regarding a fracture of the clavicle?
This is one of the most common fractures that occur in childhood.
The patient may avoid moving the arm on the injured side or may angle the head toward the injured side to relax the trapezius
muscle.
There is usually no visible or palpable deformity upon examination and it is rarely seen on an x-ray.
The mechanisms of injury is usually a fall on the outstretched hand or direct blow to the shoulder area.
The Correct answer is:
There is usually no visible or palpable deformity upon examination and it is rarely seen on an x-ray
There is most always a visible and a palpable deformity seen and the x-ray findings will show a visible separation of the
clavicular bone. Further, the remaining answer choices are all true of this type of fracture.
Question 82
What agency decides who may be called a Family Nurse Practitioner?
The American Medical Association (AMA)
The American Nurses Association (ANA)
State Boards of Nursing
The American Nurses Credentialing Center (AACN)
The Correct answer is:
State Boards of Nursing
The authority for a Family Nurse Practitioner to practice is found in state legislative statutes and in rules and regulations. The
Nurse Practice Act of every state authorizes the State Boards of Nursing to establish statutory authority to define who may be
called a Family Nurse Practitioner, and therefore, provide title protection. The American Medical Association (AMA) and the
American Nurses Association (ANA) are professional organizations that qualified healthcare providers may become a member
of. The American Nurses Credentialing Center (AACN) is an accrediting certification agency for professional nurses. These
three agencies do not have the authority to decide who is called a Family Nurse Practitioner. Therefore, they are incorrect
answers.
Question 83
Nurse practitioners are certified by what governing body?
state boards of nursing
an entity such as ANCC or AANP
The American Nurses Association
The state where they practice
The Correct answer is:
An entity such as ANCC or AANP
The two certifying bodies for adult and family nurse practitioners in the U.S. are the American Nurses Credentialing Center
(ANCC) and American Academy of Nurse Practitioners (AANP). The state boards license the nurse practitioners in the state
where they practice and most require certification in order to become licensed.
Question 84
A 33-year-old Caucasian woman is in the clinic inquiring about sexual activity during pregnancy. She is 7 months pregnant
and concerned that this will place her fetus at an increased risk. The nurse practitioner has knowledge that:
This may increase the risk of cardiovascular abnormalities in the fetus.
This may stimulate labor and, therefore, should be avoided.
This may increase the risk of pre-term labor.
There is absolutely no increased risk to the fetus.
Correct answer:
This may increase the risk of pre-term labor
The lower uterine segment may be physically stimulated, therefore resulting in pre-term labor. Oxytocin is released during an
orgasm and this is known to stimulate the uterus into pre-term labor as well. With the absence of complications associated
with pregnancy, sexual activity is not contraindicated. Should vaginal discharge, vaginal bleeding, or membrane rupture occur,
the patient should avoid sexual activity.
Question 85
Which of the following drugs is NOT a selective serotonin reuptake inhibitor (SSRI)?
Prozac
Paxil
191
Elavil
zoloft
Correct answer:
Elavil
Elavil is not an SSRI. It is a tricyclic antidepressant and unlike SSRIs it is not used as a first line of treatment for depression
Question 86
This extended release formula of a narcotic pain reliever was prescribed to an elderly patient. Which medication is it?
ER Oxycodone.
Ibuprofen.
Naproxen.
Morphine 30 mg.
Correct answer: ER Oxycodone ER Oxycodone and Morphine 30 mg are the only two narcotic pain relievers listed. Morphine
30 mg is a short-acting drug and works like ER Oxycodone in that it treats moderate to severe pain.
Question 87
You are treating a pregnant female and she inquires about HIV testing. The nurse practitioner has the understanding that:
It is better performed in the third trimester
It is an "opt-in" approach.
It produces many false positives.
It is recommended by the ACOG.
The Correct answer is:
It is recommended by the ACOG
The American College of Obstetrics and Gynecology (ACOG) recommends an "opt-out" approach to HIV screening in
pregnant patients. This means that HIV will be routinely preformed unless the patient refuses.
Question 88
You are treating a patient who suspects she was given something by her date because she feels "funny" now and "doesn't
remember much about last night." You suspect flunitrazepam (Rohypnol). You know that it has been associated with:
amnesia
agitation
increased appetite
hallucination
The Correct answer is:
Amnesia
Flunitrazepam is a benzodiazepine known as the date rape drug. Because it can result in amnesia, sexual assault can often
occur without the victim's knowledge. While not available for prescription use in North America, it is commonly prescribed as
a sleep aid in other countries and is known as a street drug in the U.S.
Question 89
When counseling a patient with allergic rhinitis, you prescribe Cromolyn. The nurse practitioner understands that its
mechanism of action is a (an):
mast cell stabilizer
vasoconstrictor
leukotriene modifier
anti-immunoglobulin E antibody
The Correct answer is:
Mast cell stabilizer
Cromolyn is a mast cell stabilizer in a nasal spray that is effective at preventing symptoms of allergic rhinitis. This drug does
not acutely relieve symptoms. Instead, it takes around a week of use to achieve symptom relief.
Question 90
In doing a neurological examination, the Romberg test would be associated with which of the following?
mental status
cerebellar system
sensory system
motor exam
192
Correct answer:
cerebellar system
The Romberg test is based on the premise that a person requires at least two of the three following senses to maintain balance
while standing: proprioception (the ability to know one's body in space); vestibular function (the ability to know one's head
position in space); and vision (which can be used to monitor changes in body position).
Question 91
Which of the following drugs is a selective serotonin reuptake inhibitor (SSRI)?
buproprion
sertraline
alprazolam
Dexedrine
Correct answer:
sertraline
Sertraline (Zoloft) is an SSRI. The others are not. Other SSRIs include: fluoxetine (Prozac), and paroxetine (Paxil).
Question 92
Which of the following developmental milestones is not properly matched with its average age or age range?
unfists: 3 – 4 months
babbling vowels: 5 – 6 months
points to named pictures when asked “show me:” 30 months
transfers objects: 4 – 7 months
Correct answer:
points to named pictures when asked “show me:” 30 months
This developmental milestone is not matched correctly with its average age of accomplishment. The average age for pointing
to named pictures when asked “show me” is 18 – 24 months.
Question 93
A patient has a history of a GI bleed. She comes in complains of abdominal pain in the interview. What test should be run?
Blood glucose.
CBC.
CEA test.
Stool hematest.
Correct answer: Stool hematest GI bleeding After a physical exam, order a stool hematest. A stool hematest will reveal if there
is blood in the GI tract.
Question 94
According to guidelines, which of the following might be considered least important in determining the level of visit to bill for
evaluation and management of a patient (not counseling)?
examination
time
history taking
medical decision making
Correct answer:
time
Time is a minor consideration in determining the level of visit to bill, according to guidelines, if a clinician is billing an office
visit for evaluation and management. If a visit is primarily counseling, however, time matters and should be documented.
Question 95
A family member brings her elderly mother into the office with a new onset of fecal incontinence. The nurse practitioner
knows that the most common cause is:
193
poor fluid consumption
constipation
medication related
inactivity
The Correct answer is:
Constipation
The most common cause of fecal incontinence in elderly adults is underlying constipation. Risk factors include age greater
than 80 years, impaired mobility, and neurologic disorders including dementia. While inactivity and poor fluid consumption
may contribute to constipation, they are not the cause of fecal incontinence.
Question 96
A patient who has recently been prescribed Savella for fibromyalgia should be made aware of all but which of the following
potential side effects?
Risk of abnormal bleeding.
Constipation.
Diarrhea.
Insomnia.
Correct Answer: Diarrhea Diarrhea is not among the common reported side effects of Savella. Constipation and insomnia are
commonly reported. Savella does elevate the risk of abnormal bleeding, which is a potentially more serious side effect.
Question 97
The nurse practitioner knows that it is developmentally important to do what during the physical examination of an adolescent
female?
Discuss only the major areas of abnormality.
Verbally affirm normalcy.
Verbally address problems of sexually transmitted diseases.
Maintain a comfortable silence.
The Correct answer is:
Verbally affirm normalcy
During the adolescent years there is a need to discuss the physical changes that the patient is undergoing. Further, verbal
affirmation of normalcy is necessary to decrease anxiety.
Question 98
The nurse practitioner knows that all of the following are nonpharmacologic treatments for a patient with acute
glomerulonephritis EXCEPT:
Avoid high-potassium foods.
Treat them as inpatient until edema and hypertension are under control.
Restrict protein in presence of azotemia and metabolic acidosis.
Encourage increased fluid intake.
Encourage increased fluid intake
The patient’s fluid intake should be restricted to only the amount the patient requires to replace lost fluids. The remaining
answer choices are included in the treatment of acute glomerulonephritis.
Question 99
Which of the following drugs is a cholinesterase inhibitor that comes in the form of a pill, syrup or drug-releasing skin patch?
Galantamine.
Hydrocodone.
Rivastigmine.
Donepezil.
Correct answer: Rivastigmine There are three cholinesterase inhibitors commonly prescribed by doctors. These are Donepezil,
Galantamine and Rivastigmine. Hydrocodone is a type of medication that treats moderate to severe pain as well as cough.
Question 100
Asking the patient to slide the tongue from side to side in an exam tests which nerve?
Vagus.
Trigeminal.
Hypoglossal.
194
Trochlear.
Correct answer: Hypoglossal The hypoglossal nerve controls the tongue. Listening to annunciation, looking at the tongue, and
having the patient move it from side to side is how it is assessed. It is important to examine to tongue for any lesions that may
cause the patient to hold the tongue in a certain way.
Test 6 aanp
Question 1
Olecranon bursitis is located in which of the following places?
shoulder
knee
elbow
heel
Explanation:
Correct answer:
elbow
Olecranon bursitis is located in the elbow. There is pain and swelling behind the elbow. It is often described as a ball or sac
hanging from the elbow. Risk factors include prolonged pressure or trauma to the elbow.
Question 2
Which of the following are you most likely to do to treat a patient who has temporal arteritis?
ice pack on forehead
100% oxygen
administer Tegretol
refer to ER or ophthalmologist
Correct answer:
refer to ER or ophthalmologist
Arteritis is a systemic inflammatory process of the medium and large arteries of the body. If the temporal artery is involved, it
is called temporal arteritis. Permanent blindness may occur if it is not diagnosed and treated early.
Question 3
A 64-year-old Caucasian male has the following lipid level results:
Total cholesterol 240 mg/dL
LDL
140 mg/dL
HDL
35 mg/dL
Triglycerides
201 mg/dL
What class of medications will normalize his lipid elevations and decrease his risk of a cardiac event?
fibric acids
bile acid sequestrants
niacin
statins
The Correct answer is:
Statins
The only medication class that lowers elevated lipid levels and has proven efficacy in lowering the risk of cardiac events, even
for primary prevention, is statin medications. Further, statin therapy has been shown to reduce overall mortality due to
cardiovascular deaths
Question 4
Which of the following statements about herpes zoster (shingles) is least accurate?
More than 66% of those affected are older than 50 years.
Only 5% of cases occur in children younger than 15 years.
Herpes zoster occurs equally in males and females.
Radiotherapy is a protective factor for the disease.
Correct answer:
Radiotherapy is a protective factor for the disease.
195
All of the statements are accurate except this one. Radiotherapy is a factor increasing susceptibility for the disease.
Question 5
The nurse practitioner (NP) understands that the following is NOT true regarding the care of minors:
In most jurisdictions, minors under the age of 18 cannot receive healthcare services without permission of an adult who is a
parent or legal guardian.
If a 17-year-old contracts a sexually transmitted disease, the nurse practitioner must contact the parent or legal guardian.
In case of emergency, when no parent or guardian is available, care may be rendered.
If a 15-year-old becomes pregnant, she is an emancipated minor.
The Correct answer is:
If a 17-year-old contracts a sexually transmitted disease, the nurse practitioner must contact the parent or legal guardian
A teen with a STD is considered an emancipated minor, and the parent or legal guardian should not be contacted. The
remaining answer choices are true regarding the care of minors.
Question 6
While examining a patient you hear a high-pitched diastolic murmur at the second ICS at the right side of the sternum. You
know that this indicates which of the following?
aortic regurgitation
aortic stenosis
mitral stenosis
mitral regurgitation
Correct answer:
aortic regurgitation
A high-pitched diastolic murmur which is best heard at the second ICS at the right side of the sternum indicates aortic
regurgitation. Aortic stenosis produces a harsh and noisy murmur and radiates to the neck.
Question 7
Which of the following statements about end-of-life care is least accurate?
Withholding or withdrawing artificial nutrition or hydration from terminally ill or permanently unconscious patients is
illegal.
Oral statements made by a patient may be legally valid advance directives.
Only a few states require “clear and convincing evidence” of a patient without decision-making capabilities that it is his or
her wish to forego life-sustaining treatment.
There is no legal requirement to notify risk management personnel before life-sustaining medical treatment can be stopped.
Correct answer:
Withholding or withdrawing artificial nutrition or hydration from terminally ill or permanently unconscious patients is illegal.
This is the least accurate statement. Just like any other therapy, fluids and nutrition may be withheld if it is the patient’s or
surrogate’s wish.
Question 8
In the latent stage of syphilis which of the following is the most common sign?
painless chancre
maculopapular rash on palms and soles that is not pruritic
no symptoms
valvular damage
Correct answer:
no symptoms
Syphilis has four stages: primary, secondary, latent, and tertiary. The latent stage is characterized by being asymptomatic
Question 9
196
You are seeing an uncircumcised client that presents with a complaint of not being able to retract the foreskin over the glans
penis. What is the likely diagnosis?
phimosis
paraphimosis
lateral phimosis
Peyronie's disease
The Correct answer is:
Phimosis
Phimosis results in the patient's inability to retract the foreskin from behind the glans penis. This condition occurs at any age
and is the result of poor hygiene and chronic infection.
Question 10
When counseling a patient with the human herpes virus 2 (HHV-2), the nurse practitioner knows that what percentage of
sexually active adults has serological evidence of this disease?
5%
10%
15%
25%
The Correct answer is:
25%
The human herpes virus 2 (HHV-2) causes painful ulcerated lesions and lymphadenopathy. Approximately 25% of adults
who are sexually active have serological evidence of this disease, although only a small percentage of these persons have
symptoms.
Question 11
Which of the following is NOT a live attenuated virus vaccine?
MMR
Varicella Zoster
injectable tri-valent influenza vaccine
FluMist
Correct answer:
injectable tri-valent influenza vaccine
The injectable trivalent influenza vaccine (TIV) does not contain live virus and is not shed. There is no risk of transmitting an
infectious agent to household contacts. The other choices are vaccines with live attenuated viruses.
Question 12
Kurt Lewin’s model of change management can best be described as which of the following?
preparation, change and continuity
grounding, amending and permanence
training, exchanging and connection
unfreezing, moving and refreezing
Correct answer:
unfreezing, moving and refreezing
The unfreezing stage involves getting to a point of understanding that change is necessary and getting ready to move away
from the current comfort zone. Moving is moving towards a new way of being. Refreezing is about establishing stability once
the changes have been made.
Question 13
Nurse practitioners are always reimbursed for their services as primary care providers under all but which of the following?
Medicare
Federal Employees Health Benefits Program
Managed Care Organizations
TRICARE
197
Correct answer:
Managed Care Organizations
Managed Care Organizations frequently have excluded NPs from being designated as primary care providers carrying their
own caseloads. Thus, in many MCOs, the only option for NPs is that of being a salaried employee.
Question 14
The knee joint includes four bones. Which of the following is NOT one of these bones?
distal femur
distal tibia
patella
proximal fibula
Correct answer:
distal tibia
The distal tibia is not one of the bones of the knee. The knee joint includes the distal femur, the patella, the proximal fibula,
and the proximal tibia.
Question 15
In terms of etiology of disease, incidence rates can best be described as which of the following?
a group at a certain point in time and the number within a group that has a particular disease or problem
interventions at the clinical stage of disease, directed at treatment and rehabilitation
the rate of involvement of causative agents
the rate of development of a disease in a group over a period of time
Correct answer:
the rate of development of a disease in a group over a period of time
Incidence rates describe the rate of development of a disease in a group over a period of time. They also describe the
continuing occurrence of new cases of disease.
Question 16
Your patient has an extensive infection of several adjacent hair follicles that form a mass with multiple drainage points. You
understand that this patient has which of the following?
folliculitis
carbuncle
furuncle
none of the above
Correct answer:
carbuncle
A carbuncle occurs when there is an extensive infection of several adjacent hair follicles that form a mass with multiple
drainage points. Folliculitis is a minor inflammation of hair follicles with or without pustules. A furuncle (abscess or boil) is a
more extensive infection of a hair follicle.
Question 17
A 23-year-old female who is breastfeeding is inquiring about alcohol use during lactation. The nurse practitioner understands
that:
Because of its high molecular weight, relatively little alcohol is passed into breast milk.
Infant intoxication may be seen with as little as one to two maternal drinks.
Drinking a glass of wine or beer will enhance the let-down reflex.
Maternal alcohol use causes a reduction in the amount of milk ingested by the infant.
The Correct answer is:
Maternal alcohol use causes a reduction in the amount of milk ingested by the infant
Alcohol has a low molecular weight and is highly lipid soluble. These characteristics allow it to have easy passage into breast
milk. Even in small amounts, alcohol ingestion by a nursing mother can cause a smaller amount of milk produced, reduction in
the let-down reflex and less rhythmic and frequent sucking by the infant, resulting in a smaller volume of milk ingested.
198
Question 18
A 42-year-old Caucasian male has newly diagnosed hypertension and has received a prescription for ramipril. He is otherwise
healthy and on no other medications. Which laboratory test would be important to monitor him?
PT with INR
potassium level
ALT/AST
calcium level
The Correct answer is:
Potassium level
Ramipril is an angiotensin-converting enzyme (ACE) inhibitor. Further, this medication causes retention of potassium.
Therefore, the patient's potassium level should be measured about one month after initiating therapy and again after dosage
changes. The remaining answer choices, PT with INR, ALT/AST and calcium level, are not specific laboratory tests for ACE
inhibitor therapy.
Question 19
Which of the following is the gold standard diagnostic test for thalassemia minor?
blood smear
serum ferritin
hemoglobin electrophoresis
none of the above
Correct answer:
hemoglobin electrophoresis
Hemoglobin electrophoresis is the gold standard diagnostic test for thalassemia minor. in beta thalassemia it is abnormal.
There is an elevated Hgb.A2, and Hgb F.
Question 20
What dietary regimen would be appropriate for an elderly client with chronic obstructive pulmonary disease?
high caloric, low protein, high carbohydrate
low caloric, low protein, high carbohydrate
high caloric, high protein, low carbohydrate
low caloric, high protein, low carbohydrate
The Correct answer is:
High caloric, high protein, low carbohydrate
By eating a high protein, high caloric and low carbohydrate diet, the patient with chronic obstructive pulmonary disease is able
to meet his or her nutritional needs. Additionally, a patient with chronic obstructive pulmonary disease, who retains carbon
dioxide, should avoid high carbohydrate foods because carbohydrates metabolize into carbon dioxide (CO2) as a waste
product.
Question 21
The type of nursing research that examines events of the past to enhance self-understanding is called:
phenomenology
historical research
grounded theory
ethnographic research
Correct answer:
historical research
Historical research examines events of the past to enhance self-understanding. It is a type of qualitative research that focuses
on the collection of data through interviews, observations and focus groups to enhance understanding about a particular area of
interest in nursing.
Question 22
Which of the following conditions is most likely to be found in a person with acute cholecystitis?
fever
jaundice
199
vomiting
palpable internal joint
The Correct answer is:
Vomiting
Acute cholecystitis symptoms include vomiting, right upper abdomen pain, and nausea. Fever, jaundice and a palpable internal
joint are not seen in patients with cholelithiasis. Cholelithiasis is defined as a condition in which there is the formation of
calculi or gallstones.
Question 23
A 8-year-old male child is called back into the office because his stool specimen for ova and parasites (O & P) came back
positive. The nurse practitioner knows what regarding enterobiasis?
The parasite causes pruritus around the anus because the females exit at night to lay eggs on the skin.
This parasite is a protozoan and the source is usually contaminated water and it is spread by fecal-oral contamination.
The parasite is in the soil and enters the body through the feet.
Eggs of this parasite enter the body by ingestion of dirt or from unwashed vegetables that contain the eggs in the dirt.
The correct answer is:
The parasite causes pruritus around the anus because the females exit at night to lay eggs on the skin
This infestation is also known as "pinworms" which reside in the intestine. Hookworm larvae reside in the soil and enter the
body through the feet. When dirt is ingested, there is a risk of contacting roundworm. Giardiasis from Giardia lamblia results
from ingesting this organism through contaminated water or fecal-oral transmission.
Question 24
In terms of Medicare, which of the following statements about hospital privileges is incorrect?
Every hospitalized patient covered by Medicare must be under the care of a physician.
The federal government includes licensed doctors of medicine, osteopathy, podiatry and chiropractic in its definition of
physician.
Hospitalized patients may be under the care of a clinical psychologist.
Doctors of medicine or osteopathy may not delegate tasks for hospitalized patients to other qualified health care personnel.
Correct answer:
Doctors of medicine or osteopathy may not delegate tasks for hospitalized patients to other qualified health care personnel.
This is the incorrect statement. Doctors of medicine or osteopathy may delegate tasks for hospitalized patients to other
qualified health care personnel “to the extent recognized under State law or a State’s regulatory mechanism.” NPs who deliver
care to hospitalized patients presumably fall under the delegation rule.
Question 25
The FNP calculates a patient’s BMI to be 23. Which category would this patient fall into?
normal weight
underweight
overweight
obese
Correct answer:
normal weight
This patient would fall into the normal weight category. Persons with a BMI of 18.5 to 24.9 are considered of normal weight.
Question 26
Which of the following is NOT a risk factor for osteoporosis?
alcohol abuse
active lifestyle
advancing age
history of bone fracture
Correct answer:
active lifestyle
An inactive lifestyle, not an active lifestyle would be a risk factor for osteoporosis. All of the other choices are risk factors
200
along with: female gender, family history, and low estrogen levels in women, among other things.
Question 27
Which of the following is a system of controlled oversight and authorization of services and benefits provided to patients?
clinical ladder
case management
triage
clinical guidelines
The Correct answer is:
Case management
Case management is a special type of communication that is a necessity for healthcare providers. It is the process of
interviewing and teaching patients and for sharing or clarifying information with others involved in the patient's care
Question 28
A patient with hypertension is in the office and you have prescribed Cozaar. Cozaar, if taken with which of the following
drugs, can put a patient at risk for developing a renal calculi?
Tagament
Nizoral
Diflucan
Oxipurinol
Correct answer:
Oxipurinol
Cozaar is an Angiotensin II Receptor Antagonist prescribed for the treatment of hypertension. This drug blocks the material in
the body that constricts or narrows the blood vessels, then relaxes the blood vessels, allowing the blood to flow more freely
and results in a lowered blood pressure. The common side effects of Cozaar are dizziness, diarrhea, muscle cramps, heartburn,
stuffy nose, pain in the leg, knee or the back. Some serious side effects include chest pain, difficulty breathing and swelling in
the face, throat or tongue. Cozaar, if taken with certain other drugs, can cause a potential drug interaction. For example, when
Cozaar is taken with Tagament, Cozaar increases the serum level of Tagament. Then, if Cozaar is taken with Diflucan,
Diflucan reduces the efficacy of Cozaar. When Cozaar is taken with Oxipurinol, the patient has an increased risk of
developing renal calculi. Further, if the patient drinks grapefruit juice with Cozaar, grapefruit juice causes a delay in the
absorption and decreases the serum levels of the Cozaar.
Question 29
You are treating a 48-year-old male who is suffering with alcoholism. The first most helpful approach for him is:
Inform the patient of the long-term health consequences of alcohol abuse.
Tell the patient to stop drinking.
Refer the patient to Alcoholics Anonymous.
Counsel the patient that alcohol abuse is a treatable disease.
The correct answer is:
Counsel the patient that alcohol abuse is a treatable disease
The first approach for the nurse practitioner is to counsel the patient and family that alcoholism is a lifelong but treatable
disease. In addition, asking about current drinking habits and associated consequences to health with each visit is important
and are appropriate next steps. Then, the remaining answer choices are also important interventions after the nurse practitioner
counsels the patient and the family regarding alcoholism as a treatable disease.
Question 30
You are discussing medication therapy in a 39-year-old man who has just been diagnosed with gastroesophageal reflux disease
(GERD). Which medication on his medication list do you know exacerbates the symptoms of GERD?
metformin
ferrous sulfate
verapamil
viagra
The Correct answer is:
Verapamil
201
The lower esophageal sphincter is opened and closed by muscles. Verapamil is a calcium channel blocker and calcium is
needed for muscle contractions. Gastroesophageal reflux disease can be exacerbated in this case so verapamil should be
avoided in this patient.
Question 31
You suspect anemia in an 88-year-old female patient, particularly vitamin B12 deficiency. Which of the following lab indices
is more indicative of a B12 deficiency?
macrocytosis
leukocytosis
thrombocytosis
microcytosis
The Correct answer is:
Macrocytosis
A vitamin B12 deficiency produces a form of anemia known as pernicious anemia. It is more common in older adults and
characterized by macrocytosis, where red blood cells are larger than expected. Leukocytosis describes large numbers of white
blood cells. Thrombocytosis refers to an increased number of platelets. Microcytosis, small red blood cells, is seen in iron
deficiency anemia or thalassemia.
Question 32
A 36-year-old mentally challenged and disabled man has been diagnosed with prostate cancer and it is determined he needs
surgery. Consent for this surgery should be obtained from:
the patient's court-appointed guardian
the administrator of the group home where the patient lives
the patient himself
the client's mental health physician
The Correct answer is:
The patient's court-appointed guardian
Since this patient's mental capacity to consent is questionable, alternatives must be sought. If this patient has a court-appointed
guardian, that person is the decision maker. If no guardian exists, the nurse practitioner would have to evaluate the patient's
family dynamics and state and federal laws to determine if a close relative is appropriate to consent for this surgical procedure.
Also, a group home employee cannot consent for this as they have no legal authority.
Question 33
NSAID and corticosteroid use are potent risk factors in which of the following types of peptic ulcer disease?
gastric ulcer
duodenal ulcer
nonerosive gastritis
chronic type B gastritis
Correct answer:
gastric ulcer
NSAID and corticosteroid use are a potent risk factors for gastric ulcers. Cigarette smoking is also a risk factor. Nearly all
gastric ulcers found in patients without H. Pylori infection are caused by NSAID use.
Question 34
When a patient tells you that she has no family anymore since her sister and her husband died last year and you reply, “That
must make you very sad,” you are using which of the following types of therapeutic communication?
trust
honesty
validation
empathy
Correct answer:
empathy
Empathy is an emotional linkage between two or more people through which feelings are communicated. It involves trying to
imagine what it must be like to be in another person’s situation.
Question 35
202
The amount of cream or ointment needed to treat a dermatologic condition on one leg for one time is which of the following?
6g
3g
10 g
12 g
Correct answer:
6g
The amount of cream or ointment needed to treat a dermatologic condition on one leg for one time is 6 g. Prescribers often
write prescriptions for an inadequate amount of topical medication creating a situation in which treatment fails because of an
inadequate length of therapy.
Question 36
As part of a nonpharmacologic treatment for a second-degree burn, you would do all of the following EXCEPT:
Remove all rings to avoid a tourniquet effect.
Apply ice to the site.
Gently cleanse with a mild detergent.
Cover the burn area with a thin layer of silver sulfadiazine cream.
Correct answer:
Apply ice to the site.
Do NOT apply ice to the site. In addition to the other three choices you may flush a chemical burn copiously with water and
debride any broken blisters and dead skin.
Question 37
Which of the following injuries shows no specific radiographic findings?
tennis elbow
finger fractures
Jones fracture
Boxer’s fractures
Correct answer:
tennis elbow
Tennis elbow as well as nursemaid’s elbow show no specific radiographic findings. Finger fractures may reveal crush injuries.
Jones fractures reveal a fracture to the fifth metatarsal. Boxer’s fractures reveal a fracture of the fifth metacarpal, which
frequently is medially angulated.
Question 38
A 67-year-old man is in the clinic with frequent dizziness while standing. Orthostatic hypotension can be diagnosed in an
older adult if the systolic blood pressure decreases:
more than 20 points any time after rising
more than 20 points within one minute after rising
any degree drop if the patient becomes weak or dizzy
more than 20 points within three minutes after rising
The Correct answer is:
More than 20 points within three minutes after rising
Orthostatic hypotension, also called postural hypotension, is diagnosed in older adults when the systolic blood pressure drops
20 mm Hg or more within 3 minutes of moving to an upright position. It can be evaluated from lying to sitting or from sitting
to standing. Also, if the systolic reading does not drop but the diastolic drops by 10 mm Hg or more, postural hypotension can
be diagnosed.
Question 39
You have a 50-year-old male patient complaining of hearing loss. One of the tests you do involves placing the tuning fork on
the forehead and asking the patient where it is heard best. This is known as which of the following?
Rinne test
Weber test
203
tympanogram
audiogram
Correct answer:
Weber test
The Weber test involves placing the tuning fork on the forehead or front teeth. The patient then indicates where it is heard
best. The Rinne test involves placing the tuning fork alternately on the mastoid bone and in front of the ear canal. The other
choices are tests that do not involve a tuning fork.
Question 40
Which of the following would NOT be considered one of the general principles of medical record documentation?
The rationale for ordering diagnostic and other ancillary services should be documented or easily inferred.
The patient’s progress, response to and changes in treatment, and revision of diagnosis should be documented.
Health risk factors do not appear in documentation, only in practice.
The medical record should be complete and legible.
Correct answer:
Health risk factors do not appear in documentation, only in practice.
Appropriate health risk factors should be identified in documentation. Past and present diagnoses should also be accessible to
the treating and/or consulting physician
Question 41
A physician or nurse practitioner who specializes in the care of hospitalized patients is called a(n):
specialist
primary care provider
hospitalist
adjunct provider
Correct answer:
hospitalist
Traditionally, a patient who needed to be admitted to a hospital was admitted through the patient’s primary care provider, who
coordinated the care of the patient. This tradition has been challenged by the realization that this model is not too efficient.
More hospitalists (physicians and nurse practitioners who specialize in the care of hospitalized patients) are taking over this
aspect of practice.
Question 42
An 18-year-old female comes to the clinic complaining of a sore throat and fatigue that has persisted for several months. She
also has some upper abdominal pain. The FNP’s examination reveals enlarged cervical nodes, erythema of the pharynx and
red tonsils. The most likely diagnosis is:
rhinitis medicamentosa
acute mononucleosis
acute sinusitis
allergic rhinitis
Correct answer:
acute mononucleosis
Mononucleosis is an infection by the Epstein-Barr virus that is transmitted through saliva by intimate oral contact. The classic
triad to look for is pharyngitis, cervical lymphadenopathy and fatigue.
Question 43
You are examining a male infant, age 9 months, and notice an undescended testicle on the right side. When counseling the
mother, the nurse practitioner understands the following to be true of cryptorchidism EXCEPT:
This is where the testis does not descend with massage of the inguinal area.
You cannot exam the infant in a warm room.
Increased risk of testicular cancer occurs if testicles are not removed from the abdomen.
Surgical correction within the first year of life is necessary if it does not spontaneously descend.
204
The Correct answer is:
You cannot exam the child in a warm room
In order to see if the muscle will relax and the testicle will descend, the infant should be sitting and the exam room should be
warm to relax the muscles when massaging the inguinal canal. Another option is to examine the infant during or after a warm
bath.
Question 44
Which of the following is NOT one of the Jones criteria for pelvic inflammatory disease (PID)?
lower back pain
cervical motion tenderness
adnexal tenderness
lower abdominal tenderness
Correct answer:
lower back pain
The Jones criteria are the major criteria for PID. Only one of the criteria is necessary to diagnose and treat for PID. All of the
choices are one of the Jones criteria except lower back pain
Question 45
Which of the following would NOT be a breach of patient confidentiality?
discussing a patient’s condition with the nurse assistant assigned to the patient
discussing a patient’s condition with family members
giving a patient’s name and address to a vendor
talking about a patient within earshot of others
Correct answer:
discussing a patient’s condition with the nurse assistant assigned to the patient
All of the choices would be a breach of patient confidentiality except this one. Other breaches include: releasing medical
information about a patient without prior written permission, leaving a telephone message on a patient’s answering machine,
leaving patient records within view of others, and discarding unshredded duplicate records.
Question 46
Osteoporosis is indicated by a bone density score of which of the following?
less than -2.5
between -1 and -2.5
greater than -1
none of the above
Correct answer:
less than -2.5
Interpretation of bone density testing is based on a T score, which refers to the number of standard deviations above or below
the average value in young adults of the same gender. Osteoporosis is a bone density indicated by a T score less than -2.5. It is
also indicated by the presence of fragility fractures, irrespective of the bone density test results.
Question 47
A 16-year-old male is in the office. He has a insect bite on his left forearm and you suspect a brown recluse spider bite. What
medical management would you provide?
ice pack and elevation of the area
active and passive range of motion (ROM) to the area
avoidance of antihistamines
warm moist soaks to the affected area
The Correct answer is:
Ice pack and elevation of the area
Ice packs are preferred to heat to decrease the edema. The area should be immobilized and tetanus toxoid given.
Antihistamines may reduce the swelling and relieve site itching
205
Question 48
You have a 58-year-old female patient who is post-menopausal and whom you have diagnosed as having an ovarian cyst. The
cyst is 3 cm and the client has no other risk factors. Which of the following actions would you take for this patient?
Refer for emergency care.
Recheck her at 6 weeks.
Refer to a surgeon.
Do nothing.
Correct answer:
Recheck her at 6 weeks.
If the patient is post-menopausal, the cyst is smaller than 5 cm, and the patient does not have any other risk factors, you would
recheck her in 6 weeks, and then again at 3 months, and then 6 months. Refer the patient if there is any increase in size of the
mass.
Question 49
A 70-year-old male comes to the FNP with a raised, red lesion in his oral cavity. He tells the FNP that he is afraid that it is
cancer. Besides evaluating the lesion the FNP evaluates the man’s risk for oral cancer. These risks include all of the following
EXCEPT:
poor diet
alcohol abuse
tobacco abuse
advanced age
Correct answer:
poor diet
Poor diet is not a risk factor for oral cancer. Alcohol and tobacco use and advanced age are risks.
Question 50
The Health Insurance Portability and Accountability Act of 1996 (HIPPA) protects the privacy of patients and their health care
records. All of the following are basic requirements of the privacy rule EXCEPT:
Providers and their staff are restricted to conveying the minimum necessary information about patients.
If a provider wants to release patient information for marketing purposes, the patient must authorize the use of the
information in writing.
Providers may not disclose health information to oversight agencies, such as CMS without patient authorization.
Individuals have no right to psychotherapy notes, information compiled in anticipation of civil or criminal litigation and
certain clinical laboratory information.
Correct answer:
Providers may not disclose health information to oversight agencies, such as CMS without patient authorization.
This is not a requirement of the privacy rule. Providers may disclose health information to oversight agencies, such as CMS
without patient authorization.
Question 51
A 38-year-old male patient complains of a sudden onset of severe “ice pick” pains behind one eye. The headache is
accompanied by tearing, a clear runny nasal discharge and a drooping eyelid. What type of headache does he have?
muscle tension headache
migraine headache
cluster headache
temporal arteritis
Correct answer:
cluster headache
Cluster headaches are severe headaches with lancinating pain behind one eye that occurs several times a day, usually at the
same times every day. Their cause is unknown and they are more common in adult males in their 30s and 40s.
Question 52
206
Human papillomavirus (HPV) is considered the causal agent in what percentage of cervical cancers worldwide?
75.5%
89.2%
90.9%
99.8%
Correct answer:
99.8%
Human papillomavirus (HPV) is now considered the causal agent in 99.8% of cervical cancers worldwide. HPV strains 16, 18,
31, 33, 39, and 42 are strongly associated with cervical cancer.
Question 53
Psoriasis is an inherited skin disorder. All of the following are typical treatments for psoriasis EXCEPT:
Psoralen drugs
methotrexate
Goeckerman regimen
systemic penicillin
Correct answer:
systemic penicillin
Topical steroids and tar preparations (Psoralen drugs) are used for psoriasis. Systemic drugs such as methotrexate are used for
severe forms of the disease as is the Goeckerman regimen. Systemic penicillin would not be used as this is a drug that is used
for infections, not for psoriasis, a condition where the squamous epithelial cells undergo rapid mitotic division producing
psoriatic plaque.
Question 54
When assessing a rash near the eye and side of the temple, the nurse practitioner understands that herpes keratitis has what
signs and/or symptoms?
blindness, painful swollen hair follicle
small nodule that moves under the skin of the eyelid
acute onset of eye pain, photophobia and blurred vision in the affected eye
a yellow triangular thickening of the conjunctiva
The Correct answer is:
Acute onset of eye pain, photophobia and blurred vision in the affected eye
Herpes keratitis is caused by a herpes virus infection, usually shingles, and it causes these signs and symptoms. A stye or
hordeolum is a painful acute bacterial infection of a hair follicle on the eyelid that does not result in blindness. A chalazion
results in a movable nodule. Then, pterygium presents with a yellow triangular thickening of the conjunctiva.
Question 55
Medications may produce anxiety as a side effect. Of the following, which medication is least likely to have this effect?
corticosteroids
bronchodilators
antihistamines
antibiotics
Correct answer:
antibiotics
Antibiotics are the least likely of the choices to produce anxiety as a side effect. Besides the other three choices,
anticholinergics, antihypertensives, antipsychotics, antidepressants, amphetamines, and anesthetics may also produce anxiety.
Question 56
Which of the following best defines a hospitalist?
a physician or nurse practitioner who specializes in the care of hospitalized patients
a member of a hospital accreditation board
any physician or nurse practitioner who has hospital privileges
none of the above
207
Correct answer:
a physician or nurse practitioner who specializes in the care of hospitalized patients
To increase efficiency, physicians and nurse practitioners can specialize in the care of hospitalized patients. These clinicians
are called hospitalists.
Question 57
A 45-year-old African-American male reports chest pain that is sudden and severe. Also, the male patient describes the pain as
"tearing." You notice that the pain is accompanied by a decrease in peripheral pulses. You understand that this may indicate a
diagnosis of:
pericarditis
aortic dissection
acute MI
angina
The Correct answer is:
Aortic dissection
Aortic dissection almost invariably begins with sudden onset of severe chest pain that is tearing or ripping in quality and is
accompanied by absent or decreased peripheral pulses and neurologic deficits. The pain of angina and acute myocardial
infarction (MI) is usually described as "pressure" by the patient. Pericarditis produces pain that is more gradual in its onset.
Question 58
A woman who has been on hormone replacement therapy (HRT) is likely to have some possible improvement. Which of the
following is one of these?
A decrease in the symptoms associated with menopause.
A reduction in the risk of dementia.
A reduction in the risk of osteoarthritis.
A reduction in the risk of endometrial cancer risk.
The Correct answer is:
A decrease in the symptoms associated with menopause
As with all medication, the use of HRT comes with the possibility of adverse effects. There is no decrease in the risk of
dementia or osteoarthritis. There is a reduction of osteoporosis risk. Endometrial cancer risk, and not a reduction in the risk,
with unopposed estrogen exists.
Question 59
Which of the following vaccines is NOT a live attenuated vaccine?
MMR
varicella
Salk polio
varicella zoster
Correct answer:
Salk polio
An attenuated vaccine is a vaccine created by reducing the virulence of a pathogen, but still keeping it viable (or "live").
Attenuation takes an infectious agent and alters it so that it becomes harmless or less virulent. The Salk polio vaccine is not a
live attenuated vaccine.
Question 60
A positive Homan’s sign is associated with which of the following?
deep vein thrombosis
superficial thrombophlebitis
peripheral vascular disease
Raynaud’s phenomenon
Correct answer:
deep vein thrombosis
Deep vein thrombosis (DVT) occurs when a thrombus develops in the deep venous system of the legs or pelvis. On physical
exam of a patient with DVT, a positive Homan’s sign (lower leg pain on dorsiflexion of the foot) is noted
208
Question 61
You are advising your patient with diabetes about exercise and diet. You would tell this patient all of the following EXCEPT:
Exercising heavily will decrease the risk of hypoglycemia in the evening and at bedtime.
Skipping a meal will increase the risk of hypoglycemia in the evening and at bedtime.
Eat simple carbohydrates before or during exercise and complex carbohydrates after exercise.
Exercise increases glucose utilization.
Correct answer:
Exercising heavily will decrease the risk of hypoglycemia in the evening and at bedtime.
This is not correct. Exercising heavily will increase the risk of hypoglycemia in the evening and at bedtime and should be
avoided. In older diabetics preexisting CHD should be ruled out before starting an exercise program by administering a stress
test.
Question 62
Some employers offer the NP the opportunity for bonuses. If a bonus is based on the number of patient visits per year it is
which of the following types of bonus formulas?
quality-based
profit-based
patient satisfaction-based
productivity-based
Correct answer:
productivity-based
A formula based on the number of patient visits per year is a productivity-based formula. This type of formula makes good
business sense under a fee-for-service arrangement.
Question 63
What type of headache is typically described as "pressing" in quality?
migraine headache
tension-type headache
cluster headache
simple stress headache
The Correct answer is:
Tension-type headache
Tension-type headache is pressing, nonpulsatile pain, mild to moderate in intensity and usually bilateral in location. Migraine
headache is described as pulsating quality, usually unilateral, with associated nausea and photophobia. Cluster headache is
often behind one eye, steady and intense. Simple stress headache is a fictitious term, which makes it an incorrect answer
choice
Question 64
Medical decision making refers to the complexity of establishing a diagnosis and/or selecting a management option. The four
levels of evaluation and management services include all of the following EXCEPT:
high complexity
low complexity
straightforward
up-front
Correct answer:
up-front
There is no up-front level of E/M services. The four types of medical decision making are: straightforward, low complexity,
moderate complexity and high complexity
Question 65
Which of the following is NOT one of the basic requirements/allowances of the HIPAA privacy rule?
Providers releasing patient information for marketing purposes must explain to the patient how the information will be used,
to whom it will be disclosed, and the time frame.
209
Providers and their staff are restricted to conveying the “minimum necessary information” about patients.
Providers may disclose health information to oversight agencies.
In general, providers may disclose psychotherapy notes without patient consent.
Correct answer:
In general, providers may disclose psychotherapy notes without patient consent.
There are special rules for psychotherapy notes. In general, patient authorization is required in order to disclose psychotherapy
notes to carry out treatment, payment or healthcare operations.
Question 66
Lead is primarily absorbed through which of the following body systems?
neurologic
integumentary
respiratory and gastrointestinal
integumentary and lymphatic
Correct answer:
respiratory and gastrointestinal
Lead is absorbed primarily through the respiratory and gastrointestinal systems. After lead is absorbed into the bloodstream
most of it is bound to red blood cells.
Question 67
Which of the following statements about collaborative agreements is incorrect?
It is a written document between a physician and a nurse practitioner.
It is never submitted to the board of nursing.
It outlines the nurse practitioner’s role and responsibility to the clinical practice.
It must be signed by both the physician and the nurse practitioner.
Correct answer:
It is never submitted to the board of nursing.
All of the choices are true statements except this one. The collaborative agreement is usually submitted to the board of
nursing.
Question 68
A 16-year-old female patient inquires about douching. You are counseling on birth control, safe sexual practices and hygiene.
What instruction regarding douching should the nurse practitioner include?
Hypoallergenic douches include flavored or perfumed types.
Douching during menstruation is a safe effective practice.
Douching removes natural mucus and upset normal vaginal flora.
Daily douching is important if the patient has copious vaginal discharge.
The Correct answer is:
Douching removes natural mucus and upsets normal vaginal flora
Douching is never necessary because it upsets the normal vaginal flora and disrupts pH. Douching is contraindicated during
menstruation due to "retrograde menstruation," a potential precursor to endometriosis. Further, a patient with copious vaginal
discharge needs to be evaluated for an infection.
Question 69
The ability to conduct a cultural assessment to collect relevant cultural data regarding the patient’s presenting problem as well
as accurately conducting a culturally-based physical assessment is called:
cultural diversity
cultural awareness
cultural skill
cultural desire
Correct answer:
cultural skill
210
Cultural skill is the ability to conduct a cultural assessment to collect relevant cultural data regarding the patient’s presenting
problem as well as accurately conducting a culturally-based physical assessment. It is part of the “ASKED” mnemonic to
promote becoming culturally competent.
Question 70
You have a patient complaining of severe pain in the great toe in which you observe erythema. He tells you that this happened
quite quickly and got to the point of being severe in just a few hours. You suspect acute gout. Which of the following tests is
the only confirmatory test for acute gout?
joint aspiration
serum uric acid
creatinine, BUN
24-hour urine for urate and creatinine
Correct answer:
joint aspiration
Joint aspiration is the only confirmatory test for acute gout. It can find MSU crystal in phagocytes or free in tophi, seen under
a polarized microscope. The other tests are of limited value in acute gout.
Question 71
Of the following prostaglandin inhibitors, which one is a proprionic acid?
Naproxen
Mefenamic acid
Indomethacin
Diflunisal
Correct answer:
Naproxen
Naproxen is a proprionic acid along with Ibuprofen and naproxen sodium. Mefenamic acid is a fenamate and the other two
choices are acetic acid/salicylic acids.
Question 72
Which of the following types of research is most likely to be considered a philosophy as well as a research method?
phenomenology
quasi experimental research
longitudinal research
cross-sectional research
Correct answer:
phenomenology
Phenomenology is both a philosophy and a research method. The purpose is to describe experiences as they are lived and to
capture the “lived experience” of study participants
Question 73
Which of the following is a secondary prevention measure?
educating a diabetic about exercise and diet
immunizations
bicycle helmets
mammogram
Correct answer:
mammogram
All screening tests are secondary prevention measures. A mammogram and breast exam are secondary prevention measures.
Question 74
A mother brings her 8-year-old son to your office. She tells you that he has had no immunizations but she would like him to
have all of the necessary immunizations now. Which of the following immunizations would NOT be necessary for this child?
211
PCV (3 doses)
Td (3 doses primary, then every 10 years)
IPV (3 doses)
MMR (2 doses)
Correct answer:
PCV (3 doses)
PCV (pneumococcal conjugate vaccine) is not given after age 7 years. All of the other immunizations should be given along
with Hepatitis B (3 doses).
Question 75
When you say something to a patient such as, “Let me be sure that I understand what you are saying,” this is which of the
following forms of therapeutic communication?
validation
caring
active listening
honesty
Correct answer:
validation
Validation is listening to the patient and responding congruently in order to be sure that you have the same understanding as
the patient. Active listening is hearing and interpreting language, noticing nonverbal and paraverbal enhancements, and
identifying underlying feelings.
Question 76
Severe cough, fever, conjunctivitis, photophobia, and Koplik’s spots are most related to which of the following?
scarlet fever
measles
rubella
roseola
Correct answer:
measles
A person with measles will present with severe cough, fever, conjunctivitis, photophobia, and/or Koplik’s spots. The rash will
be bright red to purple macules and papules ofent becoming confluent
Question 77
There are two approaches to nursing research. They are which of the following?
academic and practical
direct and indirect
professional and non-professional
quantitative and qualitative
Correct answer:
quantitative and qualitative
There are two approaches to nursing research: quantitative and qualitative. Both involve rigor that produces credible data for
evidence-based practice, and both make significant contributions to nursing science
Question 78
A male patient has asked you for a prescription of Viagra. After a careful history, you make the determination that he has
erectile dysfunction (ED). The nurse practitioner understands that ED is:
Not a condition associated with men over age 70
Primarily psychological in origin
The persistent inability to achieve and maintain an erection adequate for sexual intercourse.
The physiologic dysfunction when smooth muscles contract, causing a lack of adequate amounts of blood in the penis to
render a rigid, larger penis.
212
The Correct answer is:
The persistent inability to achieve and maintain an erection adequate for sexual intercourse
Erectile dysfunction is experienced by 40% of men over 40 years of age and 70% of men over 70. It involves a dysfunction in
the hemodynamic mechanism of smooth muscle relaxation that increases blood flow in the penis.
Question 79
Which of the following is another term for statutory authority?
regulation
legal authority
contract authority
administrative rule
:
Correct answer:
legal authority
Statutory authority is legal authority. Elected officials (the legislature) vote on a bill such as the Nurse Practice Act. Bills that
pass become law and have statutory authority.
Question 80
Which of the following is NOT true regarding healthcare issues in the United States?
Most of Americans have access to healthcare and are being adequately treated.
Of all children, 13% are uninsured.
It is estimated that 45+ million Americans are underserved because of limited access to healthcare.
Many of the working poor cannot obtain healthcare because of the high costs of insurance, fees and medications.
The Correct answer is:
Most of Americans have access to healthcare and are being adequately treated
There is a large portion of Americans who do not have access to healthcare.
Question 81
Which of the following is an orthopedic maneuver of the knee to test for damage to the meniscus?
Drawer sign
McMurray sign
Lachman sign
Goodell’s sign
Correct answer:
McMurray sign
The McMurray sign is an orthopedic maneuver of the knee to test for damage to the meniscus. If the knee locks up and the
patient is unable to fully extend the affected knee, the test is positive.
Question 82
Your pregnant patient is approaching her estimated delivery date. She asks you for some information about how big her baby
will be. You might tell her all of the following EXCEPT:
95% of newborns weight 7 – 10 lb.
95% of newborns are between 18 and 22 in. long.
Approximately 5% - 10% of body weight is lost in the first few days.
The weekly gain from birth to 6 months is 5 – 7 ounces per week.
Correct answer:
95% of newborns weight 7 – 10 lb.
This is not correct. 95% of newborns weight 5 – 10 lb. Approximately 5% - 10% of body weight is lost in the first few days;
then birthweight is regained in 7 – 10 days.
Question 83
You have a patient who is a 22 month old male. He presents with a low-grade fever. Your initial treatment for this patient
would be which of the following?
213
Do nothing.
Give antipyretics and observe.
Obtain a CBC.
Give prophylactic antibiotics for 3 days pending blood culture results.
Correct answer:
Give antipyretics and observe.
For children over 3 months of age who present with a low-grade fever, you would give antipyretics and observe. If fever is
over 102° F, then you would obtain a CBC. If the WBC is greater than 15,000/mm3, then you would obtain a blood culture
and administer IM Ceftriaxone.
Question 84
What is the reason the Standards of Nurse Practitioner Practice were established?
They are established to promote autonomous practice.
They are established to limit the liability of nurse practitioners.
They are established to regulate and control nurse practitioner practice.
They are established to protect nurse practitioners from frivolous lawsuits.
The Correct answer is:
They are established to regulate and control nurse practitioner practice.
The reason Standards of Practice exist in all healthcare related specialties (nursing, medicine, dental, etc.) is that they regulate
and control practice. The purpose of Standards of Practice is to provide accountability for professionals and to help protect the
public from unethical behavior and unsafe practice. The remaining answer choices are not reasons for the establishment of
Standards of Nurse Practitioner Practice.
Question 85
The FNP has a menopausal woman patient who is troubled by hot flashes. The FNP tells her that they can often be reduced in
number and minimized in severity with simple lifestyle changes. All of the following would be a hot flash trigger EXCEPT:
chocolate consumption
alcohol use
tight, restrictive clothing
cold baths or showers
Correct answer:
cold baths or showers
Hot baths or showers are hot flash triggers, not cold baths or showers. In addition to the other choices which are all hot flash
triggers, other triggers include: spicy foods, elevated ambient temperature and humidity, and cigarette smoking.
Question 86
For which of the following would the FNP use the bell of the stethoscope?
mid-to high-pitched tones such as lung sounds
low tones such as the extra heart sounds
mitral regurgitation
aortic stenosis
Correct answer:
low tones such as the extra heart sounds
The bell of the stethoscope is used for low tones such as the extra heart sounds (S3 or S4) as well as for mitral stenosis. The
diaphragm of the stethoscope is used for all of the other choices given.
Question 87
Which of the following diseases/conditions is least likely to increase susceptibility to meningitis?
hypertension
sickle cell anemia
diabetes
alcoholism
214
Correct answer:
hypertension
Of the choices hypertension is the least likely to increase susceptibility to meningitis. Other factors increasing susceptibility
include: basilar skull fracture; indwelling CSF shunting device; debilitation or institutionalization; and contact with others who
have had meningitis.
Question 88
The most likely associated symptoms of dementia include all of the following EXCEPT:
insidious onset over months or years
loss of intellectual or cognitive function
mental status changes
acute onset over hours to a few days
The Correct answer is:
Acute onset over hours to a few days
Delirium has an acute onset over hours to a few days and is a condition in which the patient exhibits reduced ability to
maintain attention to external stimuli and shift attention appropriately to new stimuli. The other answer choices are all true of
dementia, making them incorrect answers in this case. Dementia has an insidious onset over months or years.
Question 89
Which of the following is NOT a conceptual model for advanced practice nursing?
Benner’s model
Shuler’s model
Calkin’s model
Ford model
Correct answer:
Ford model
There is no Ford model of NP practice; however, Loretta Ford established the first pediatric nurse practitioner program.
Question 90
A good quality assurance program should do which of the following?
identify educational needs
improve the documentation of care
reduce the clinician’s overall exposure to liability
all of the above
Correct answer:
all of the above
A good quality assurance program would include all of the first three choices. These programs also look at organizational
effectiveness, efficiency, and client and provider interactions
Question 91
Which of the following medications might you prescribe for a patient with acute bronchitis?
Robitussin
Guaifenesin
Ventolin
any of the above
Correct answer:
any of the above
All of the medications listed in the choices might be prescribed for a patient with acute bronchitis. You would also increase
fluids and if the patient smokes, have them stop smoking, if possible.
Question 92
Which of the following drugs would NOT be a prophylactic treatment for migraine headaches?
beta-blockers
tricyclic antidepressants
215
prednisone
amitriptyline
Correct answer:
prednisone
Prednisone would not be a prophylactic measure for migraine headaches. It would be used, however, if the FNP suspects
temporal arteritis.
Question 93
The FNP has an adult male patient in the clinic who woke up with one side of his face paralyzed. He cannot fully close his
eyelid and has difficulty chewing and swallowing food on that side of his face. Which of the following is the most likely
diagnosis?
Bell’s Palsy
trigeminal neuralgia
focal migraine
acute bacterial meningitis
Correct answer:
Bell’s Palsy
Bell’s Palsy is the abrupt onset of unilateral facial paralysis that is due to dysfunction of the motor branch of the facial nerve
(CN 7). Facial paralysis can progress rapidly within 24 hours. Skin sensation remains intact but tear production on the affected
side may stop. Most cases spontaneously resolve.
Question 94
One part of a spectrum of disease that is often found together with giant cell arteritis is which of the following?
polymyalgia rheumatica
migraine headache
blindness
brain tumor
Correct answer:
polymyalgia rheumatica
Giant cell arteritis and polymyalgia rheumatica are thought to represent two parts of a spectrum of disease and are often found
together. Blindness is a serious complication of the disease.
Question 95
A 32-year-old female with interstitial cystitis (IC) is in the office. She has been newly diagnosed and you are counseling her.
All of the following are important to discuss EXCEPT:
Discuss that interstitial cystitis is not a malignancy or a risk factor for a more serious disease.
Teach the patient the importance of adequate nonirritating fluid intake.
Remind her that cigarette smoking can irritate the bladder.
Discuss the measures for curative treatment.
The Correct answer is:
Discuss the measures for curative treatment
Interstitial cystitis is an incurable condition that involves painful urination, frequency, urgency and nocturia. It is chronic and
can be controlled, not cured. Treatment aims are directed at relief of symptoms. The remaining answers are important as
patient education information.
Question 96
Which of the following is NOT one of the stages in Erikson’s theory of psychosocial development?
trust
autonomy
initiative
loyalty
Correct answer:
loyalty
216
Loyalty is not one of Erikson’s stages of psychosocial development. The eight stages are: trust, autonomy, initiative, industry,
identity, intimacy, generativity, and ego identity.
Question 97
Which of the following patient is most likely to qualify and be using Medicaid as their insurance?
the single mother of four dependent children, age 26
a middle aged female who works for the government, age 47
a young male factory worker, age 23
an elderly retired high school teacher, age 78
The Correct answer is:
The single mother of four dependent children, age 26
Medicaid is a federally mandated program that guarantees healthcare services to low-income families with dependent children
and to low-income and disabled individuals. Government employees and factory workers are most likely insured through a
commercial carrier or uninsured. An elderly teacher would have Medicare since they have paid into the federal retirement
system and are over 65 years of age
Question 98
Which of the following tests is federally mandated for infants?
TSH
Lead screening
PKU
all of the above
Correct answer:
PKU
PKU (phenylketonuria) is a federally mandated test. Severe mental retardation may develop if this is not treated early.
Question 99
The continuing occurrence of new cases of a disease is known as which of the following?
prevalence rate
colonization rate
infectivity rate
incidence rate
:
Correct answer:
incidence rate
The incidence rates describe the rate of development of a disease in a group over a period of time or the continuing occurrence
of new cases of disease. It differs from the prevalence rate which describes a group at a certain point in time and the number
within the group that has a particular disease or problem.
Question 100
A mother brings her infant to the clinic to say that he has tearing in both eyes. The FNP also sees a mucoid discharge. Which
of the following conditions/diseases might this indicate?
congenital lacrimal duct obstruction
Wilms tumor
Fifth disease
Kawasaki disease
Correct answer:
congenital lacrimal duct obstruction
Congenital lacrimal duct obstruction is a failure of the tear duct to open at birth. A thin membrane blocks the entrance of the
nasolacrimal duct causing tear blockage. This is a condition that spontaneously resolves.
quiz 9 aanp
Question 1
217
Which of the following statements about folic acid deficiency anemia is incorrect?
There is a decreased physiologic need for folate in pregnancy.
The most common cause is inadequate dietary intake and overcooking vegetables.
A deficiency in folate results in RBC changes that manifest as macrocytic anemia.
Drugs that interfere with folate absorption include phenytoin, and methotrexate.
Correct answer:
There is a decreased physiologic need for folate in pregnancy.
This statement is incorrect. The opposite is true. There is an increased physiologic need for folate in pregnancy as well as in
chronic illness.
Question 2
Lead poisoning remains a serious public health problem. Which of the following is the least likely to present a lead hazard?
lead-glazed pottery
lead-soldered vessels used for cooking
fumes from burnt casings of batteries
paint in a home built in 1995
Correct answer:
paint in a home built in 1995
This is the least likely source to present a lead hazard. Lead-based paint has not been available in the USA for more than 30
years. Therefore, a home built in 1995 would not contain lead-based paint.
Question 3
An elderly patient presents with a flat, nonpalpable colored spot, 4 mm in size. What is the medical term for this spot?
Bulla.
Tumor.
Pustule.
Macule.
Correct answer: Macule Bulla: fluid-filled, elevated, circumscribed lesion larger than 5 mm Tumor: solid, elevated mass larger
than 1 cm Pustule: pus-filled, elevated, circumscribed lesion up to 5 mm in size Macule: flat, nonpalpable colored spot up to 5
mm in size
Question 4
When counseling a menopausal woman with many risk factors for osteoporosis, the nurse practitioner knows that which of the
following are preventative measures she should be instructed on regarding osteoporosis?
smoking cessation and weight bearing exercise
optimal caloric intake and Vitamin E supplements
exercise, weight gain and a high fat diet
organic supplements and adequate Vitamin B12 intake
The Correct answer is:
Smoking cessation and weight bearing exercise
Although the mechanism is not known, research has shown that cigarette smoking accelerates skeletal bone loss. Therefore,
smoking cessation is important in the prevention of osteoporosis as well as other diseases and conditions. Exercise is
necessary at least 30 minutes, three times a week, to maintain bone density and it has been shown to decrease the incidence of
hip fractures. Walking increases hip and spine density. Further, Vitamin D and calcium intake along with smoking cessation
prevents osteoporosis.
Question 5
In terms of elder abuse, which of the following statements is false?
Only infants are more likely to sustain serious injury from physical abuse than the elderly.
Only one in fourteen elder abuse cases is reported.
90% of abusers of older adults are reported to be family members.
Approximately 5% of older adults are neglected or abused in the United States.
Correct answer:
Approximately 5% of older adults are neglected or abused in the United States.
218
This statement is false. Approximately 10% of older adults are neglected or abused in the United States. Health care providers
are responsible for reporting any suspected abuse or neglect to the appropriate state protective agency.
Question 6
What percentage of the usual and customary fee paid to physicians is reimbursed to the FNP by Medicare?
100%
95%
90%
85%
Correct answer:
85%
Today, nurse practitioners can be reimbursed directly by Medicare. Medicare reimburses NPs 85% of the usual and customary
fee that is paid to a physician for the same visit.
Question 7
What is palpated when examining the thyroid?
Carotid.
Greater trochanter.
Sternocleidomastoid.
Isthmus.
Correct answer: Isthmus A physical exam of the thyroid usually requires palpation to determine size and hardness. Palpate the
Isthmus. It should not palpate if the thyroid is not enlarged.
Question 8
Which of the following drugs for treating anxiety has the most rapid onset of action?
diazepam
clonazepam
alprazolam
oxazepam
Correct answer:
diazepam
Diazepam (Valium) has a rapid onset of action and a relatively sustained effect. The other choices all have a slow onset of
action and a relatively or highly sustained effect.
Question 9
A 16-year-old male comes to the clinic and tells the FNP that he awoke with severe scrotal pain in the left testicle. He has felt
nauseated and has had abdominal pain since this happened. What is the likely cause of this teenager’s pain and nausea?
testicular cancer
priapism
testicular torsion
chronic prostatitis
Correct answer:
testicular torsion
The likely cause of this patient’s symptoms is testicular torsion. This condition may be spontaneous or associated with
physical activity or trauma. The salvage rate is 100% if corrected within the first six hours and 0% if done after 24 hours.
Question 10
There are different psychosocial periods in adolescence. In which period would achievement of identity and intimacy in
relationships take place?
pre-adolescence
early adolescence
middle adolescence
late adolescence
Correct answer:
late adolescence
219
Late adolescence takes place between 18 and 21 years. During this period vocation and career choices become important.
Achievement of identity and intimacy in relationships are hallmarks of this period.
Question 11
Which of the following factors is least likely to put a woman at a higher for breast cancer?
first degree relative who had breast cancer
shorter exposure to estrogen (as in late menarche with early menopause)
obesity
high-dosed radiation
Correct answer:
shorter exposure to estrogen (as in late menarche with early menopause)
Mammography is a secondary prevention measure. It is recommended at age 40 and older. Women at higher risk for breast
cancer include all of the choices except this one. Longer exposure to estrogen (as in early menarche with late menopause) puts
a woman at higher risk.
Question 12
Which of the following is NOT a factor in putting a woman at a higher risk of breast cancer?
genetics
first degree relative with breast cancer
longer exposure to estrogen
thin stature
Correct answer:
thin stature
This is not a factor that puts a woman at a higher risk of breast cancer. Obesity is a factor, however, since adipose tissue can
synthesize small amounts of estrogen.
Question 13
A 79 year old patient with HTN was prescribed NSAIDs for pain by her previous doctor. The NP states that she cannot
prescribe NSAIDs due to the patient's HTN. What would be the risk of prescribing an NSAID?
Decreased effectiveness of diuretics.
Fluid retention.
Hypoxia.
Ulcer and bleeding risk.
Correct answer: Decreased effectiveness of diuretics Prescribing medications to patients with various diseases could put the
patient at risk for additional conditions. For example, a patient with CHF cannot be prescribed NSAIDs because they may
cause fluid retention.
Question 14
In patients with depression, the maximum recommended dose of Cymbalta is which of the following?
30 mg/day.
60 mg/day.
120 mg/day.
40 mg/day.
Correct Answer: 60 mg/day The maximum recommended dose of Cymbalta for depression is 60 mg/day. This may be given
either as one 60 mg dose or as two 30 mg doses (taken at different times during the day). It may be advisable to start patients
on a smaller dose for the first week, with gradual increase to the full dosage.
Question 15
A 35-year-old Caucasian female is in the office with rosacea. She is asking about treatment options. What is the recommended
treatment for her?
topical 5-fluorouracil
low-dose tetracycline
oral ketoconazole
Dilantin
220
The Correct answer is:
Low-dose tetracycline
Treatment with systemic low-dose tetracycline is a very effective measure for rosacea. Further, topical treatment with
metronidazole or a low-dose steroid cream may also be helpful. In some cases, ketoconazole, the antifungal cream and not an
oral form, is used to treat rosacea. Topical 5-fluorouracil is used to treat actinic keratosis.
Question 16
Nursing theory is of two types: nursing grand theories and middle range nursing theories. Which of the following persons
promoted the nursing grand theory of enhancing the body’s reparative processes by manipulation of noise, nutrition, hygiene,
light, comfort and hope?
Nightingale
Benner and Wrubel
Orem
King
Correct answer:
Nightingale
Nursing grand theories are abstract, connect and relate the concepts of person, environment, nursing and health. Nightingale’s
concept was that of enhancing the body’s reparative processes by manipulation of noise, nutrition, hygiene, light, comfort and
hope.
Question 17
Which of the following tests would you order to confirm a diagnosis of Hepatitis B?
Albumin level.
Liver function.
Prothombin time.
Hepatitis B surface antigen.
Correct Answer: Hepatitis B surface antigen This test can confirm a diagnosis of Hepatitis B. A positive result indicates an
active infection. The other tests listed are used to monitor liver damage caused by Hepatitis B
Question 18
According to the Tanner staging, when a female has breast buds with areolar enlargement she is in which Tanner stage?
stage 1
stage 2
stage 3
stage 4
Correct answer:
stage 2
According to the stages of breast development in females (Tanner staging) when a girl has breast buds with areolar
enlargement, she is in Tanner stage 2. There are five Tanner stages in all.
Question 19
The Trans-theoretical Model of Change sets forth six predictable stages of change. Which of the following is NOT one of
these?
pre-contemplation
preparation
maintenance
post-contemplation
Correct answer:
post-contemplation
The Trans-theoretical Model of Change assesses an individual's readiness to act on a new healthier behavior, and provides
strategies, or processes of change to guide the individual through the stages of change to action and maintenance. The six
stages are: pre-contemplation, contemplation, preparation, action, maintenance and termination.
Question 20
The FNP has a 35-year-old male patient who has been smoking since he was 15 years old. She wants to make a brief
intervention using the “5 As.” The “5 As” are which of the following?
ask, advise, assess, assist, arrange
ask, advocate, assess, assist, arrange
221
ask, advise, assess, aid, arrange
ask, advise, assist, arrange, advocate
Correct answer:
ask, advise, assess, assist, arrange
The “5 As” should be employed with all tobacco users, including individuals with no current desire to quit because this can
serve as a motivating factor in future attempts to quit tobacco use. The “5 As” are: Ask about tobacco use; Advise to quit;
Assess willingness to make an attempt to quit; Assist in quit attempt; Arrange follow-up.
Question 21
Of the fundamental domains of human growth and development, which one includes fine and gross motor abilities?
psychological and social domain
cognitive domain
physical domain
none of the above
Correct answer:
physical domain
Fine and gross motor abilities fall into the physical domain. Also parts of the physical domain are: genetic factors, physical
stature and appearance, nutritional status, and physical health and well- being.
Question 22
A patient comes in with a consistent cough. What about his background makes the cough suspicious?
Osteoporosis.
Obesity.
Exercise.
Asbestos exposure.
Correct answer: Asbestos exposure Asbestos exposure is linked to lung cancer. A patient with this history should be examined
carefully to rule out lung cancer.
Question 23
A 10-year-old boy has a tick embedded in his scalp. The mother brings him to you for removal. What is the correct technique
to safely remove the tick and help prevent Lyme Disease?
use petroleum jelly
use a hot match
pull it off with tweezers
use isopropyl alcohol
The Correct answer is:
Pull it off with tweezers
The person removing the tick with tweezers should take care and not crush the tick because it may contain infectious
organisms. After the removal of the tick, the skin should be cleansed with antibacterial soap and water. This area should be
monitored for 30 days for erythema migrans.
Question 24
You are providing care to a female patient who reports having 1 to 2 migraines per month. She asks if there are any dietary
changes she can make to help prevent her migraines. Which of the following is an appropriate response to this patient.
The patient should eat a vegan diet.
The patient should eat a well-balanced diet and note if any foods trigger her headaches.
The patient should eat a very low-fat diet.
The patient should avoid all dairy.
Correct Answer: The patient should eat a well-balanced diet and note if any foods trigger her headaches There is no "migraine
diet." However, many patients have success in preventing or alleviating their migraines by eating a well balanced diet and
exercising regularly. Because some foods may trigger migraines, the patient should also note if she experiences migraines after
eating specific foods, and then avoid those foods or limit her consumption.
Question 25
222
A mother comes into the office with her 2-year-old child who attends daycare. The child is pulling at her ears, running a fever,
and has bright red bulging tympanic membranes bilaterally. The nurse practitioner knows that with acute otitis media (AOM)
the risk factors include:
Chinese race, previous otitis media, many siblings
Summer season, full-time day care, premature at birth
higher socioeconomic level, full-time day care, allergies
second-hand smoke, attending day care, American Indians and Eskimos
Second-hand smoke, attending day care, American Indians and Eskimos
The key factor that contributes to acute otitis media (AOM) is a dysfunctional eustachian tube that allow bacteria from the
nasopharynx into the middle ear. American Indians and Eskimos have more repetitive and severe otitis media than members of
other races. Further, this condition occurs more often during the fall, winter, and spring months than summer. Children who
attend day care centers have a greater risk of acquiring acute otitis media as do members of lower socioeconomic levels. Other
risks for the development of acute otitis media are second-hand smoke, having a large number of siblings, male gender, and
developmental abnormalities
Question 26
Which of the following statements about DEA registration for prescribing controlled substances is incorrect?
The NP must use a DEA number on prescriptions for scheduled drugs.
The state cannot require a separate controlled substances license.
The DEA number is a method of minimizing unauthorized prescribing.
Federal registration is based on the applicant’s complying with state and local laws.
Correct answer:
The state cannot require a separate controlled substances license.
This is incorrect. The state can require a separate controlled substances license. If the state requires a separate controlled
substances license, an NP must obtain that license and submit a copy with the application for a DEA number.
Question 27
Which of the following characteristics would be unusual in a patient with uncomplicated gallbladder disease?
guarding
positive Murphy's sign
nausea
fever
The Correct answer is:
Fever
Fever is an unusual symptom in a patient with uncomplicated gallbladder disease. This may be seen in an episode of acute
cholecystitis. A positive Murphy's sign is usually elicited when the gallbladder is inflamed.
Question 28
The nurse practitioner understands that after starting vitamin B12 therapy for a patient, what would be expected at a 2 week
follow-up visit?
reduced RBCs, WBCs, and platelets
increased macrocytosis and anisocytosis
increased hemoglobin/hematocrit and reticulocyte count
ferritin level of 40 ng/ml
The Correct answer is:
Increased hemoglobin/hematocrit and reticulocyte count
With vitamin B12 therapy, the nurse practitioner will see an increase in the hemoglobin/hematocrit and reticulocyte count.
Other improvements will include improved appetite and decrease in neurological symptoms, such as gait disturbances,
weakness, and paresthesias.
Question 29
A 35-year-old patient with hypertension has taken 25 mg of hydrochlorothiazide daily for the past 4 weeks. His blood pressure
has decreased from 155/95 to 145/90. How should the nurse practitioner proceed?
Increase the hydrochlorothiazide to 50 mg daily.
Add a drug from another class to the daily 25 mg hydrochlorothiazide.
Stop the hydrochlorothiazide and start a drug from a different class.
Continue the hydrochlorothiazide at 25 mg daily.
223
The Correct answer is:
Add a drug from another class to the daily 25 mg hydrochlorothiazide
This patient exceeds the target blood pressure and the minimum threshold of 140/90 mm Hg. It is not acceptable to continue
the current dose. Increasing the hydrochlorothiazide to 50 mg daily will not result in a decrease in blood pressure, only an
increase in potassium loss. Adding a medication from a different class allows for combined effects.
Question 30
What is the intent of the National Practitioner Data Bank (NPDB)?
To investigate adverse actions that involves licensure, clinical privileges, and professional society membership.
To improve the quality of healthcare by encouraging state licensing boards, professional societies, and facilities to identify
and discipline those who engage in unprofessional behavior.
To notify the state board of nursing and medicine of any unprofessional behavior by a provider that is reported to them.
To list doctors with outstanding care.
The Correct answer is:
To improve the quality of healthcare by encouraging state licensing boards, professional societies, and facilities to identify and
discipline those who engage in unprofessional behavior
The National Practitioner Data Bank (NPDB) is an electronic database that retains information regarding providers, such as
physicians, nurse practitioners and other health care professionals. The NPDB has information relating to the payments a
provider has made associated with judgments and medical liabilities. Further, the NPDB contains information regarding
negative actions associated with licenses and medical privileges. The NPDB is an application that only stores information
about providers. The NPDB does not investigate adverse actions that involve licensure, clinical privileges, and professional
society membership nor does it notify the state board of nursing and medicine of any unprofessional provider behavior.
Question 31
Which of the following drugs is a calcium channel blocker?
Capoten
Lopressor
Norvasc
Inderal
Correct answer:
Norvasc
Calcium channel blockers block the calcium channels in the arterioles resulting in systemic vasodilation, which results in
decreasing PVR. Calcium channel blockers include: Norvasc, Procardia SL, Calan and Cardizem CD.
Question 32
In terms of informed consent, which of the following statements is incorrect?
A 17-year-old married female does not have the right to accept or reject treatment by a healthcare provider; this must be
done by the parent or legal guardian.
The clinician has the duty to explain relevant information to the patient so that the patient can make an appropriate decision.
It must be documented in the medical records that the relevant information has been provided.
Informed consent does not absolve the NP from allegations of malpractice should it occur.
Correct answer:
A 17-year-old married female does not have the right to accept or reject treatment by a healthcare provider; this must be done
by the parent or legal guardian.
All of the statements are correct except this one. A 17-year-old married female is an emancipated minor and has the right to
accept or reject treatment by a healthcare provider.
Question 33
Treatment options in cluster headache include the use of:
the triptans
nonsteroidal anti-inflammatory drugs (NSAIDs)
oxygen
all of the above
The Correct answer is:
All of the above
Cluster headaches is also known as migrainous neuralgia. Cluster headaches are most common in middle-aged men. Treatment
includes reduction of triggers, such as tobacco and alcohol use, and initiation of prophylactic therapy and appropriate abortive
224
therapy, such as the triptans, nonsteroidal anti-inflammatory drugs (NSAIDs) and oxygen.
Question 34
Which of the following facts about immunizations for children is NOT correct?
MMR is not recommended before the age of 12 months.
Give the influenza vaccine before 6 months so that it is entirely effective.
DTaP is the only form of DTP vaccine used in this country.
DT is for infants/children younger than 7 years of age who are unable to tolerate the pertussis component.
Correct answer:
Give the influenza vaccine before 6 months so that it is entirely effective.
This is not correct. A child under the age of 6 months has an immature immune system. The influenza vaccine would not be
effective before the age of 6 months.
Question 35
What type of insurance coverage does the clinic need that is purchased by an organization to address employee job-related
injuries?
business interruption insurance
directors and officers insurance
workers' compensation insurance
professional liability insurance
The Correct answer is:
Workers' compensation insurance
Professional liability insurance is acquired to protect the organization from lawsuits by clients arising from negligent acts of
employees. Business interruption coverage is usually purchased in tandem with fire insurance and it will reimburse an
organization for losses sustained from a catastrophic event. Workers' compensation is the line of coverage that protects
employees after on-the-job injuries.
Question 36
Into which domain of human growth and development does home environment fall?
physical domain
physiological domain
cognitive domain
psychological and social domain
Correct answer:
psychological and social domain
The psychological and social domain includes home environment and other social contexts. It also includes temperament and
personality, interpersonal relationships and moral development.
Question 37
The Family Nurse Practitioner knows that when she prescribes a topical medication, the greatest rate of absorption is expected
when it is applied to the:
chest region or back
forearms
face and neck
palms of the hands and soles of the feet
The Correct answer is:
Face and neck
Certain body parts, notably the face, neck, axillae, and genital area, are quite permeable, and allow greater absorption of the
medication. Less permeable areas are the extremities, trunk and the palms of the hands and soles of the feet.
Question 38
A geriatric nurse practitioner is dealing with the physiologic changes that would affect responses to pharmacologic agents she
is prescribing to her elderly patient. As the individual ages, this nurse practitioner knows that what is happening to affect this
process?
There is decreased percentage of body fat.
There is decreased albumin concentration necessary for metabolism.
225
There is increased gastric emptying.
There is increased glomerular filtration rate.
There is decreased albumin concentration necessary for metabolism
Albumin, a form of protein, is necessary for metabolism of medications because they are protein bound. Albumin decreases
with age and these low levels decrease the number of protein-binding sites, therefore, increasing the amount of free drug in the
plasma. This could result in a drug overdose in elderly patients. Fat content does not affect medication elimination or
processing. Gastric emptying and glomerular filtration decrease with age
Question 39
A 50 year old patient comes in for a check up. His wife mentions that he does not see cars in his periphery while driving. He
has not had an eye exam in three years. What test may you choose to run?
Biopsy.
Tonometry.
Blinking reflex.
CT scan.
Correct answer: Tonometry The patient is at an age where glaucoma may appear. Tonometry measure eye pressure and will
show if it is high. A full visual exam is necessary to diagnose glaucoma.
Question 40
A 78-year-old woman who presents with incontinence, is prescribed Detrol. She has the most common type of incontinence
among the elderly population. What type of incontinence is it?
Urge.
Stress.
Functional.
Transient.
Correct answer: Urge There are pharmalogical and non-pharmacological aids that assist with alleviating this type of
incontinence. Behavioral therapy such as gentle bladder stretching and pharmacological care such as being prescribed
VESIcare and Detrol have proven effective.
Question 41
Which of the following is NOT a risk factor for osteoporosis?
alcohol abuse
active lifestyle
advancing age
history of bone fracture
Correct answer:
active lifestyle
An inactive lifestyle, not an active lifestyle would be a risk factor for osteoporosis. All of the other choices are risk factors
along with: female gender, family history, and low estrogen levels in women, among other things.
Question 42
Enuresis is involuntary urination after a child has reached the age when bladder control is usually attained. Management of
primary nocturnal enuresis includes all of the following EXCEPT:
limiting fluid intake before bedtime
double voiding before bedtime
avoiding punishment
withholding verbal praise so as not to call attention to the problem
Correct answer:
withholding verbal praise so as not to call attention to the problem
Verbal praise is an excellent means of motivational therapy for a child with primary nocturnal enuresis. Other means of
motivational therapy include a reward system and keeping a dryness calendar.
Question 43
Medical records must be kept safe. It is essential that each healthcare organization establish an effective procedure for
safeguarding them. The following are all true statements regarding medical records EXCEPT:
Medical records should be stored in a secure, restricted-access location.
Competent medical records personnel should review every record before it is examined by the patient or patient
226
representative.
An original medical record should be sent with the patient to specialist appointments and for their attorney to review as
necessary.
Anyone who is not an authorized employee or staff member should not be allowed to examine a medical record.
The Correct answer is:
An original medical record should be sent with the patient to specialist appointments and for their attorney to review as
necessary
An original medical record should never be sent with the patient to a specialist’s appointment. The original medical record
remains on the facility’s premises unless the organization has received a Subpoena Duces Tecum, court order or a subpoena
and a court order to bring the medical record to a court of law. Alternatively, if the medical record has reached the state and
federal law’s identified retention period, the original medical record can be stored offsite or destroyed.
If the patient needs information from the medical record to take to the specialist, the nurse practitioner’s office can make a
copy of what is needed for the referral and send the photocopies of the medical. This process holds true for an attorney. The
attorney will only receive copies of the original medical record. Or, if the patient signs appropriate consent forms, the attorney
can come to the medical office and review the original medical record.
Also, the healthcare provider, nurses or any other staff cannot take original medical records home or to another facility to
complete charting or other documentation in the medical record. All charting has to be completed in the facility where the
original medical exists. By removing the original medical record from the facility, it can be lost, accidentally destroyed or
even stolen, for instance. This can lead to breaches of confidentiality and privacy laws, not to mention it is hard to reproduce
an original medical record if it is lost.
In the case of office space restrictions, some physician offices and hospitals may not have sufficient amounts of space to store
all of their medical records. Therefore, storage or retention of the original medical records becomes an issue. There are many
options to storing medical records including storing them offsite, microfilming, scanning the medical records and storing them
on CD ROM’s, disks and more. In this case, the nurse practitioner will need to review state and federal laws, as the laws
govern how medical records can be retained and the compliance guidelines required if the physical, original medical record
has to be stored offsite. Certain medical record documents are permanently kept, such as records of surgical procedures and
master patient indexes. Then, depending on state laws, diagnostic images, for instance x-ray films, can be kept for 5 years
before the images are archived or destroyed. Destruction of medical records are also dictated by federal and state laws. Some
destruction methods include shredding the medical record, for example.
Question 44
Those systems that form outside the family and reflect functional needs not met within the family are called:
exosystems
endosystems
suprasystems
econosystems
Correct answer:
suprasystems
Suprasystems form outside the family and reflect functional needs not met within the family. Relationships with teachers,
schools, religious and civic organizations, the healthcare system and friends are examples of suprasystems that meet needs not
met by interactions within the family system.
Question 45
A mother brings her 7-year-old daughter to the clinic after the child received a bad sunburn. The treatment for this condition
will include all of the following EXCEPT:
a hot shower or bath
acetaminophen or ibuprofen for pain
topical emollients
removal from sunlight exposure
Correct answer:
a hot shower or bath
The FNP should not recommend a hot shower or bath. On the contrary cool water or saline compresses to the affected areas is
227
typical treatment. The mother should also be educated as to measures to prevent it from happening again.
Question 46
The nurse practitioner's standards of practice are established for what purpose?
They are meant to regulate and control nurse practitioner practice.
They limit liability of nurse practitioners.
They protect nurse practitioners from frivolous lawsuits.
They are set up to promote autonomous practice.
The Correct answer is:
They are meant to regulate and control nurse practitioner practice
For all professionals (nurses, physicians, dentists, etc.) the standards of practice are set up to regulate and control practice.
Their purpose is to provide accountability for the professionals and to help protect the public from unethical and unsafe
practice.
Question 47
Which of the following sexually transmitted male genitourinary infections is most likely to be treated with patient-applied
podofilox 0.5% solution or imiquimod 5% cream?
balanitis
gonococcal urethritis
genital warts
lymphogranuloma venereum
Correct answer:
genital warts
Genital warts are verruca-form lesions caused by the human papillomavirus. Treatment options include patient-applied
podofilox 0.5% solution or imiquimod 5% cream. They can also be treated with liquid nitrogen or cryoprobe, trichloroacetic
acid, podophyllin resin or surgical removal.
Question 48
In relation to structural functional theory, which of the following is NOT an example of a suprasystem?
relationships with family
relationships with friends
relationships with religious organizations
relationships with the healthcare system
The Correct answer is:
Relationships with family
A suprasystem forms OUTSIDE of the family and reflect functional needs that are not met within the family. Examples of a
suprasystem include relationships with all of the following: friends, religious organizations, the healthcare system, schools,
teachers, civic organizations and other groups.
Question 49
A pregnant patient presents with abdominal pain. What test should you use to determine the cause?
Ultrasound.
CBC.
MRI
CT scan.
Correct answer: Ultrasound An ultrasound will not damage a fetus. It is part of prenatal care. The other imaging answers may
harm the fetus and are not recommended for pregnant women.
Question 50
The stethoscope is placed slightly off the chest of an elderly patient who is suspected of having a murmur. The sound
produced is very loud and accompanied by palpable thrill. Which of the following murmur grades does this describe?
III/VI.
IV/VI.
V/VI.
II/VI.
Correct answer: V/VI Each of the preceding murmur grades is defined below: III/VI - Moderately loud IV/VI - Loud with
palpable thrill V/VI - Very loud and heard with the stethoscope partially off the chest; palpable thrill II/VI - Quiet but heard as
soon as the stethoscope is placed on the chest
Question 51
228
All of the following are accurate regarding documentation EXCEPT:
You should document your criticism of a fellow healthcare provider's clinical decision in the patient's medical record. This
will protect you if your treatment needs to be defended later.
You should use standard abbreviations in the medical record so that subsequent readers will have no doubt as to your
meaning and intent.
You should document a patient's noncompliant medical behavior in the medical record.
You should document telephone conversations with the patient and/or family in the medical record and be particularly
specific about recording medication changes.
The Correct answer is:
You should document your criticism of a fellow healthcare provider's clinical decision in the patient's medical record. This
will protect you if your treatment needs to be defended later.
Ridiculing another provider in the patient's medical record is never appropriate. It will allow the plaintiffs' lawyers an avenue
to find wrongdoing. The nurse practitioner should deal with the provider directly, preferably in person.
Question 52
When creating a plan of care for a patient diagnosed with fibromyalgia, which of the following is NOT among the medications
you would consider?
Duloxetine.
Pregabalin.
Gabapentin.
Levetiracetam.
Correct Answer: Levetiracetam Levetiracetam, or LEV, is an antiepileptic drug (AED) and is not indicated for fibromyalgia. It
may in fact pose serious risk of drug interaction with pregabalin (Lyrica), which is a common treatment for fibromyalgia.
Gabapentin and Duloxetine may also be prescribed for patients with fibromyalgia
Question 53
You are treating a female patient with pyelonephritis with fluoroquinolone. The nurse practitioner knows that with
fluoroquinolone use, length of antimicrobial therapy during uncomplicated pyelonephritis is typically:
3 days
7 days
14 days
20 days
The Correct answer is:
7 days
First line therapy includes TMP-SMX or fluoroquinolones. The dosage should be at least 7 days to eradicate the uropathogen.
Short-course therapy is not recommended.
Question 54
The FNP has a pregnant patient who is diagnosed with chlamydia trachomatis. Which of the following treatments is preferred
for this woman?
azithromycin 1 gm PO single dose
doxycycline 100 mg BID x 7 days
Cipro 500 mg x one dose
erythromycin base QID x 7 days
Correct answer:
azithromycin 1 gm PO single dose
For pregnant women erythromycin base QID x 7 days was the preferred treatment. However the CDC recommendation at the
present time for a pregnant patient who is diagnosed with chlamydia trachomatis is azithromycin 1 gm PO single dose.
Question 55
Which of the following is NOT consistent with the description of benign prostatic hyperplasia (BPH)?
boggy gland on palpation
gland has a rubbery consistency
gland size is larger than 2.5 cm X 3 cm
chronic incomplete bladder emptying
229
The Correct answer is:
Boggy gland on palpation
The diagnosis of benign prostatic hyperplasia (BPH) is based on numerous components of the evaluation. On rectal
examination, the prostate has a rubbery consistency, enlarged, greater than 2.5 cm X 3 cm and in many cases has lost the
median sulcus or furrow. The patient usually complains of symptoms of chronic incomplete bladder emptying, as well.
Question 56
Which of the following symptoms would the FNP see in a child with moderate dehydration?
2 – 4 second capillary refill
markedly decreased skin turgor with tenting possible
low to normal pulse
parched mucous membranes
Correct answer:
2 – 4 second capillary refill
A 2 – 4 second capillary refill is what the FNP would find in a child with moderate dehydration. He would also find slightly to
moderately decreased skin turgor, slightly decreased tears, slightly increased pulse, normal blood pressure, dry mucous
membranes and decreased urine output.
Question 57
The most common form of urinary incontinence in elderly persons is which of the following?
stress incontinence
transient incontinence
urge incontinence
urethral obstruction
Correct answer:
urge incontinence
Urge incontinence is the most common form of urinary incontinence in elderly persons. Behavioral therapy, including a
voiding schedule and gentle bladder stretching are helpful.
Question 58
An adult patient presents with painful red nodules and pustules under his arm. He tells the FNP that some of the lumps have
started to drain pus. The FNP is most likely to diagnose which of the following?
Hidradenitis Suppurativa
impetigo
meningococcemia
herpes zoster
Correct answer:
Hidradenitis Suppurativa
Hidradenitis Suppurativa is a bacterial infection of the sebaceous glands of the axilla (or groin) by Gram-positive
Staphylococcus aureus. It is marked by flare-ups and resolution. It can be confirmed by a C&S of the purulent discharge.
Question 59
What is the leading cause of death for adolescents?
motor vehicle crashes
leukemia
heart disease
home accidents
Correct answer:
motor vehicle crashes
The leading cause of death for adolescents is motor vehicle crashes. The leading cause of death for all ages and genders is
heart disease. The most common cause of cancer deaths is lung cancer.
Question 60
Which of the following drugs has been demonstrated to have use in preventing dementia?
Aricept
230
Exelon
Cognex
none of the above
Correct answer:
none of the above
All of the drugs mentioned in the first three choices can be used either alone or together to have a minor and time-limited
effect in dementia care. The use of these products and others to prevent dementia are currently not supported.
Question 61
When assessing a patient with a headache, the nurse practitioner knows that there are certain "red flags". The mnemonic
SNOOP is often used for assessing these red flags. What is the "N" in SNOOP?
Never shine the light in the patient's eyes.
Neurologic signs and symptoms
New onset of a different headache
Numbness and tingling
The Correct answer is:
Neurologic signs and symptoms
In SNOOP ,"S" is for Systemic symptoms. Then, the "N" is for Neurologic signs and symptoms. Next, the "O" is for Onset
that is sudden and abrupt. Further, "O" is for Onset age >50. Finally, the "P" is for Previous headache history.
Question 62
A patient with a history of which of the following has a high risk of developing colon cancer?
Polyps.
Auras.
Ulcers.
GERD.
Correct answer: Polyps A history of polyps places a patient at a higher risk of developing colon cancer. Ulcers and GERD are
not related to colon cancer. Auras accompany migraines.
Question 63
Regarding workforce policy, which of the following is an accurate statement?
There are no options for the advanced practice nurses to work outside the primary care role.
Employment patterns of nurse practitioners are of special interest to health workforce analysts because of the presumed
connection between the employment opportunities for nurse practitioners and the supply of primary care physicians.
There is a decrease in the number of nurses with educational preparation as advanced practice nurses.
Only half of the nurse practitioners are employed in healthcare or nursing.
The Correct answer is:
Employment patterns of nurse practitioners are of special interest to health workforce analysts because of the presumed
connection between the employment opportunities for nurse practitioners and the supply of primary care physicians
There are many options for the nurse practitioner (NP) outside of primary care. There is an increase in the number of nurses
educated as NPs now than ever and most are employed in nursing or healthcare workplaces.
Question 64
Which of the following drugs is a Category C drug in terms of pregnancy?
Accutane
ciprofloxacin
amoxicillin
insulin
Correct answer:
ciprofloxacin
Ciprofloxacin (Cipro) is a Category C drug. This means that there may be evidence of fetal risk in animals, and there is
insufficient human evidence of the drug's safety in pregnancy. The drug should only be prescribed if the benefits outweigh
the potential risk of using the drug. Accutane is a Category X drug (proven fetal risk outweighs the benefits); amoxicillin is a
Category B drug (animal studies show no risk but no human data available OR adequate human studies show no risk, although
some adverse effects have been demonstrated in animals); and insulin is a Category A drug (animal and human data show no
risk to pregnant women).
231
Question 65
It is important that the nurse remembers patients' specific care. What is the standard procedure?
Inform patient, let them remember.
Keep detail on private sticky notes at nurses' station.
Document everything in patients' files.
Not necessary to note patients' care.
Correct answer: Document everything in patients' file It is a requirement that every practitioner carefully note patients' files.
Malpractice is the result of improperly documented files.
Question 66
Which of the following statements about Medicare Part B is incorrect?
It covers all medically necessary services.
It is financed by general federal revenues and by Part B monthly premiums.
It covers 80% of the approved amount after the annual deductible.
Services covered include hospitalization costs.
Correct answer:
Services covered include hospitalization costs.
This is incorrect. Hospital costs are covered in Medicare Part A. Payment for hospitalization is based on projected costs of
caring for a patient with a given problem.
Question 67
A sexually active female complains of embarrassing vaginal odor that is worse after intercourse. She also has a vaginal
discharge that is thin and of milk-like consistency. Examination shows no redness or irritation. The FNP might suspect which
of the following?
Candida vaginitis
polycystic ovary syndrome
bacterial vaginitis
Trichomonas vaginitis
Correct answer:
bacterial vaginitis
Bacterial vaginitis is caused by anaerobic bacterial overgrowth that is due to unknown reasons or develops secondary to
pregnancy terminations or PID. It is not an STD. Vaginal discharge has a fish-like odor, is off-white to gray, and of milk-like
consistency.
Question 68
All of the following are recommended nutritional supplements EXCEPT:
fluoride supplements to reduce susceptibility to dental caries
Vitamin K, 1 mg IM, given at birth to all newborns to prevent hemorrhagic disease of the newborn
iron from fortified cereal, started by 6 months of age in all infants to replace iron stores
oral Vitamin A 10,000 units/day, recommended for mothers who are breastfeeding their infants until they are ingesting
whole milk
The Correct answer is:
Oral Vitamin A 10,000 units/day, recommended for breastfed infants until they are ingesting whole milk
Vitamin D, not A, is recommended for mothers who are breastfeeding their infants at 200units/day until they are ingesting
whole milk. The remaining answer choices are recommended nutritional supplements.
Question 69
Which of the following is NOT a risk factor in heat stroke?
lack of exercise
obesity
age (very young or very old)
alcohol intake
232
Correct answer:
lack of exercise
This is not a factor in heat stroke. Obesity, age, alcohol use, certain medications that alter adrenergic activity and oral
decongestants are all risk factors for heat stroke. Good hydration with non-alcoholic fluids, particularly water, along with
dressing lightly, helps to minimize heat stroke risk.
Question 70
How often would you screen a sexually active 21 year old patient for Chlamydia?
Every two years.
Every three years.
Every four years.
Every year.
Correct answer: Every year Sexually active patients should be screened regularly for sexually transmitted diseases. Chlamydia
is often without symptoms and should be regularly screened at least once a year.
Question 71
When treating a patient who has an extremely high P.T. level from Coumadin (warfarin) usage, the nurse practitioner
understands that which of the following is the antidote:
Vitamin K
Vitamin C
Vitamin A
Vitamin D
The Correct answer is:
Vitamin K
The antidote for Coumadin (warfarin) toxicity is Vitamin K. It is often given in an injection form to patients with extremely
elevated P.T. levels. Dietary sources of Vitamin K include green leafy vegetables.
Question 72
There are theories of how and why a person changes or stays the same over time. One debate on theory involves nature vs.
nurture. Which of the following theorists is part of the nature position on human growth and development?
John Locke
Arnold Gesell
John B. Watson
B.F. Skinner
Correct answer:
Arnold Gesell
Arnold Gesell promoted the maturational-organismic theory that is a nature position (emphasis on heredity and maturational
process). This theory has a biological basis and the organizing principle of the theory is structure (closed system of
transformational rules governing thought).
Question 73
During a health assessment with a male patient, he tells you that he has not had a bowel movement in three days, although he
usually has one each day. He says that the new medication he is taking makes it more difficult to pass feces. Which of the
following is an appropriately worded nursing diagnosis for this patient?
Patient is constipated.
Elimination, altered.
Elimination, fecal, altered.
Elimination, fecal, altered, r/t change in medication.
Correct Answer: Elimination, fecal, altered, r/t change of medication It is key to indicated not only that the patient's
elimination pattern is altered, but that the patient notes that this change has occurred along with the change in his medication.
You must also specify that it is fecal elimination that is altered so that appropriate remedies, and evaluation of the patient's
medication, can be undertaken.
Question 74
In males, which Tanner stage is characterized by the penis thickening?
Stage II
233
Stage III
Stage IV
Stage V
Correct answer:
Stage IV
Tanner Stage IV is when the penis thickens. Stage I is prepuberty; Stage II is when the testis begins to enlarge; Stage III is
when the penis elongates; and Stage V is the adult pattern.
Question 75
You are counseling a patient who is on metformin for diabetes. You know that metformin has all of the following effects
EXCEPT:
enhanced fibrinolysis
increased LDL cholesterol production
improved insulin-mediated glucose uptake
modest weight loss with initial use
The Correct answer is:
Increased LDL cholesterol production
Metformin is a biguanide that helps stabilize diabetes where insulin resistance is a factor. It does not increase LDL production.
The remaining choices are benefits of metformin usage.
remaining choices are benefits of metformin usage.
Question 76
In treating the diabetic patient, the nurse practitioner (NP) sees it necessary to prescribe a pioglitazone (Actos) or rosiglitazone
(Avandia). When managing the patient on these medications, the NP knows that the primary action of these medications is to:
increase glucose uptake into the muscle and fat
increase postprandial uptake of glucose into the intestine
decrease hepatic glucose output
increase secretion of insulin from the pancreas
Correct answer:
increase glucose uptake into the muscle and fat
These medications are insulin sensitizers, which increase the glucose uptake in the muscle and fat. The alpha-glucosidase
inhibitors increase postprandial glucose uptake in the intestine. Metformin is used to decease hepatic glucose output. Oral
sulfonylureas increase insulin secretion in the pancreas.
Question 77
The following component should be included in a history of a patient who is new to your clinic:
interval history, past medical history, family medical history, dietary habits, substance use, sexual practices
past medical and surgical histories, family medical history, psychosocial history, physical activity, tobacco and substance
use, sexual practices
general history on the physical examination for is sufficient; there is no need of interview
past medical and surgical history, family medical and surgical histories, psychosocial history, diet and exercise habits,
chemical use, sexual practices, review of systems
The Correct answer is:
Past medical and surgical histories, family medical history, psychosocial history, physical activity, tobacco and substance use,
sexual practices
Past medical and surgical histories, family medical history, psychosocial history, physical activity, tobacco and substance use,
sexual practices are all areas that are necessary to investigate a new patient's health.
Question 78
When evaluating the carotids, with correct procedure for auscultating is:
Use the bell of the stethoscope.
Position the client at a 30-degree angle, and press firmly with bell of the stethoscope.
Use the diaphragm of the stethoscope.
Place the stethoscope 1 inch off the area above the sternocleidomastoid muscle.
The correct answer is to use the bell of the stethoscope. The correct procedure is to listen for carotid bruits with the bell of the
stethoscope, which brings out low-frequency sounds and filters out high-frequency sound. The bell should be placed very
lightly on the neck with just enough pressure to seal the edge.
Question 79
234
In treating von Willebrand Disease (VWD) desmopressin acetate is used to treat bleeding complications in Type I of the
disease; however, it is contraindicated in which other type of this disease?
Type 2M
Type 2B
Type 2N
Type 3
Correct answer:
Type 2B
Desmopressin acetate is used to treat bleeding complications or as preoperative preparation for Type I; however it is
contraindicated in Type 2B. Type 2B was previously known as Type II B.
Question 80
Which of the following is NOT part of the standard plan of care for an adult patient with Fifth Disease?
Rest.
Fluids.
Antibiotics.
OTC analgesics.
Correct Answer: Antibiotics Fifth Disease is not treated with antibiotics. Most patients do fine with home care. This care
should include rest and plenty of fluids. Over the counter analgesics can be used for associated fever and pain.
Question 81
The nurse practitioner will often have to treat systemic anaphylaxis. What is the most common clinical manifestation of this?
persistent vertigo
nausea and vomiting
headache
urticaria
The Correct answer is:
Urticaria
Anaphylaxis is typically manifested as systemic IGE-mediated reaction in response to exposure to an allergen. It is
characterized by urticaria, angioedema, widespread vasodilation and bronchodilation.
Question 82
You are subpoenaed to appear for a deposition in a negligence case concerning a nurse. Which statement of advice from an
attorney would NOT be appropriate?
"Be sure to be truthful and remember, it is OK to say 'I don't know' or 'I don't remember' if you have to."
"Take as much time as you need to think about what you are going to say before you respond. Don't let the other lawyer put
words in your mouth."
"Chew gum discreetly to calm your nerves and dress for dinner because depositions usually take all day and you may not
have time to change."
"Review the medical record of the patient in question and other pertinent material before appearing."
The Correct answer is:
"Chew gum discreetly to calm your nerves and dress for dinner because depositions usually take all day and you may not have
time to change"
A professional appearance boosts credibility of a witness and it is not appropriate to chew gum. The nurse practitioner should
review documents and be prepared prior to the deposition. Questions about the nurse practitioner's professional work history
are usually asked, so a copy of the curriculum vitae is a useful tool to bring
Question 83
Which of the following would NOT be considered one of the general principles of medical record documentation?
The rationale for ordering diagnostic and other ancillary services should be documented or easily inferred.
The patient’s progress, response to and changes in treatment, and revision of diagnosis should be documented.
Health risk factors do not appear in documentation, only in practice.
The medical record should be complete and legible.
Correct answer:
Health risk factors do not appear in documentation, only in practice.
Appropriate health risk factors should be identified in documentation. Past and present diagnoses should also be accessible to
the treating and/or consulting physician
235
Question 84
You are seeing an 18-year-old woman in the office with a "pimple" on her left eyelid. Upon examination, you find a 2.5 mm
pustule on the lateral boarder of the eyelid. This is most consistent with:
cellulitis
chalazion
blepharitis
hordeolum
The Correct answer is:
Hordeolum
A hordeolum, or stye, is caused by staphylococcal infection of a hair follicle. Cellulitis is a serious complication of a
hordeolum and severe edema and redness would be present. A chalazion is characterized by a hard, nontender swelling of the
upper or lower lid.
Question 85
Which of the following statements about tendonitis and its treatment is false?
With rotator cuff involvement, the likelihood of concurrent bursitis is low.
When the hand or wrist is affected, splinting and NSAIDs are reasonable first-line therapies.
Achilles tendonitis may necessitate treatment with a posterior splint.
There is a 10% risk of tendon rupture with recurrent Achilles tendonitis.
Correct answer:
With rotator cuff involvement, the likelihood of concurrent bursitis is low.
This statement is false. With rotator cuff involvement, the likelihood of concurrent bursitis is high. Treatment includes limiting
overhead movement and intrabursal corticosteroid injection.
Question 86
The cranial nerve(s) associated with the gag reflex and the soft palate is(are) which of the following?
CN 3, 4 & 6
CN 9 & 10
CN 12
CN 2
Correct answer:
CN 9 & 10
CN 9 & 10 are the Glossopharyngeal Vagus nerves. They are associated with the gag reflex, the symmetrical soft palate, uvula
and voice quality.
Question 87
A 33-year-old female has been previously diagnosed with primary dysmenorrhea and now presents in the office with
symptoms including pain. She has tried over the counter ibuprofen and naproxen with no results. How should the nurse
practitioner treat or manage her?
oral contraceptives
B vitamins
encourage regular exercise
prescription strength naproxen
The Correct answer is:
Oral contraceptives
The primary treatments for dysmenorrhea include NSAIDs and hormonal contraceptives. The general recommendation is to
use one agent and add the other if one does not work alone. Both should be considered for women who are symptomatic with
one agent only
Question 88
You are working with a dermatologist and treating patients with acne. When prescribing tretinoin (Retin-A), a nurse
practitioner should advise the patient to:
Use a sunscreen because the drug is photosensitizing.
Add a sulfa-based cream to enhance anti-acne effects.
236
Expect much improvement in the acne lesions after 1 week of use.
Use it with benzoyl peroxide to minimize irritating effects.
The Correct answer is:
Use a sunscreen because the drug is photosensitizing
Tretinoin (retinoic acid) takes 6 weeks of therapy before noting improvement. It should not be used with benzoyl peroxide or
sulfa-based products. It can cause a severe photosensitive reaction, therefore, the sun should be avoided or sunscreen used.
Question 89
The most common cancer death in females is which of the following?
lung cancer
leukemia
breast cancer
colon cancer
Correct answer:
lung cancer
Lung cancer is the most common type of cancer death in females as well as males.
Question 90
Which of the following findings is often found in a person with stage 2 Lyme disease?
macrocytic anemia
conductive hearing loss
atrioventricular heart block
peripheral neuropathic symptoms
The Correct answer is:
Atrioventricular heart block
Lyme disease has three stages, which are stage 1, stage 2 and stage 3. In stage 2 Lyme disease, also known as early
disseminated infection, the infectious organism spreads or disseminates into the blood. A potential complication for stage 2
Lyme disease is Lyme carditis, which is exhibited by the presence of abnormal heart rhythms, or an atrioventricular block.
Further, in stage 2 Lyme disease, a classic rash may reappear with multiple lesions and arthralgias. Additionally, fatigue,
myalgia, and headaches may occur.
Question 91
The nurse practitioner in a small physician owned office wants to evaluate a patient with pernicious anemia who is receiving
treatment. The best laboratory test he could order after one month of therapy is:
a Schilling test
a reticulocyte count
a serum ferritin
hemoglobin and hematocrit
The Correct answer is:
Hemoglobin and hematocrit
With pernicious anemia treatment, the hematocrit rises 4% to 5% per week and is usually normal within one month.
Hemoglobin values usually stabilize at this time as well. A Schilling test is unnecessary and will not show significant findings.
Reticulocytosis occurs rapidly, with a peak at 7 to 10 days into therapy. Serum ferritin is usually not affected with pernicious
anemia.
Question 92
You are assessing a 2 week old infant for reflexes. Which of the following is NOT an expected reflex in this age infant?
Moro reflex
Stepping reflex
Tonic Neck reflex
Vertical reflex
The Correct answer is:
Vertical Reflex
237
The moro reflex (startle reflex) results when a sudden loud noise causes symmetric abduction and extension of the arms
followed by adduction and flexion of the arms over the body. The stepping reflex occurs when the baby is upright and the
dorsal foot is placed on the table causing a simulated stepping motion. The tonic neck reflex (fencing reflex) results when the
infant turns the head to one side with the jaw over the shoulder causing the arm and leg on the side where the head is turned to
extend. There is no vertical reflex.
Question 93
What is the best way to understand the chief complaint?
Ask direct questions.
Ask closed questions.
Ask indirect questions.
Ask open-ended questions.
Correct answer: Ask open-ended questions Open ended questions allow patients to describe symptoms in their own words.
This will give a better idea of what the patient is experiencing and prevent any leading.
Question 94
Which of the following persons developed the first standardized measurement of intelligence?
G. Stanley Hall
Alfred Binet
Arnold Gesell
Sigmund Freud
Correct answer:
Alfred Binet
Alfred Binet would be considered a proponent of the nature side of the nature vs. nurture debate. He developed the first
standardized measurement of intelligence. The intention was to identify “mentally defective” children needing specialized
education.
Question 95
There is a greater recognition of the use by patients of complementary and alternative modalities and medicines (CAM) today
than ever before. Which of the following is an inaccurate statement regarding the practice environment involving CAM?
Research suggests that 40-50% of patients are currently utilizing some form of complementary or alternative therapy.
There is a wealth of evidenced based research that supports CAM.
Nurse practitioners, as providers, need to learn about common CAM treatments and about how some herbal products interact
with prescription medications.
The National Center for Complementary and Alternative Medicine is a federal government's lead agency for scientific
research on CAM.
The Correct answer is:
There is a wealth of evidenced based research that supports CAM
There is a dearth of research supporting such treatment modalities.
Question 96
Which of the following herbal over-the-counter medicines is NOT used for anxiety and/or depression?
Kava kava
St. John’s Wort
Valerian root
none of the above
Correct answer:
none of the above
All of the choices except this one are herbal remedies for anxiety and/or depression. St. John’s Wort acts like MAOI, SSRI, or
TCA. Kava kava has action at GABA receptors similar to benzodiazepines. Valerian root also has action similar to
benzodiazepines
Question 97
Which of the following medications is NOT appropriate to prescribe for the management of genital herpes?
Zovirax.
Diflucan.
Valtrex.
238
Famvir.
Correct Answer: Diflucan Diflucan, or flucanazole, is an oral medication prescribed for the treatment of vaginal yeast
infections. The other choices are suppressive medications prescribed for the management of genital herpes.
Question 98
Which of the following standards of the Standards of Professional Nursing Practice has the competency for the nurse
practitioner to participate in peer review as appropriate?
Standard 13, Collaboration
Standard 14, Professional Practice Evaluation
Standard 15, Resource Utilization
Standard 16, Environmental Health
The Correct answer is:
Standard 14, Professional Practice Evaluation
Standard 14 reads: "The registered nurse evaluates her or his own nursing practice in relation to professional practice standards
and guidelines, relevant statutes, rules and regulations."
Question 99
A 32-year-old healthy pregnant female has asymptomatic bacteriuria. How should this be managed?
prescribe amoxicillin
prescribe ciprofloxin
prescribe nitrofurantoin
order force fluids only
The Correct answer is:
Prescribe nitrofurantoin
Since the patient is at risk for developing pyelonephritis, a pregnant patient with asymptomatic bacteriuria should be treated
with nitrofurantoin. Ciprofloxacin is a quinolone and this class should be avoided during pregnancy. Amoxicillin, while safe,
does not cover E.coli, the most common urinary pathogen.
Question 100
The nurse practitioner (NP) understands that the research process is similar to the processes that the NP use to provide patient
care in that both are decision-making processes. The steps of these processes in the correct order are:
Notification, subject selection, consent verification and study implementation
Study scope, literature review, study protocol and study design
Research construction, research compilation, research conclusion, research implementation
Assessing, planning, implementing and evaluating
The Correct answer is:
Assessing, planning, implementing and evaluating
When conducting research, the nurse uses the process steps of assessing, planning, implementing and evaluating. During the
assessing stage of the research, the nurse identifies the problem and performs a literature review. The nurse practitioner also
identifies any variables that may need to be later measured in the study. With planning, the nurse performs research to create a
hypothesis, determine measurements for the variables of the study and then decide how the sample for the study is selected.
The nurse practitioner then collects the data. During the evaluation or evaluating stage, the nurse practitioner analyzes or
evaluates the data, makes a conclusion about the findings and determines how to communicate the findings.
quiz 10 aanp
Question 1
A young female, who breastfeeds her infant, is in the office with a painful, tender, warm and reddened area on her right breast.
Mild axillary lymphadenopathy exists as well. You diagnose mastitis. What is an appropriate treatment option?
Stop breastfeeding immediately.
Antibiotic therapy with Macrodantin 100 mg QID.
Antibiotic therapy with Augmentin 875/125 mg BID or IV therapy.
Avoid analgesics such as ibuprofen and acetaminophen that could cause breastfeeding issues.
Antibiotic therapy with Augmentin 875/125 mg BID or IV therapy
Breastfeeding should be continued or milk should be expressed to shorten the duration of symptoms. Macrodantin will not
cover the organisms usually present with mastitis, such as S. aureus. With mastitis, analgesic such as ibuprofen and
acetaminophen are recommended that are compatible with breastfeeding.
239
Question 2
The nurse practitioner (NP) is dealing with a patient who needs to have a durable power of attorney for healthcare. The NP
knows that all of the following are true of this document EXCEPT:
The document must be flexible enough to carry out the patient's wishes throughout the course of an illness.
They are not limited to the circumstances of terminal illness.
The durable power of attorney is not binding and legal.
The document is often accompanied by a durable power of attorney over financial issues as well.
The Correct answer is:
The durable power of attorney is not binding and legal
A durable power of attorney for healthcare is a legal and binding document that allows the patient to determine who will make
decisions when the patient is unable to make them. The remaining answer choices are true of the durable power of attorney.
Question 3
An obese elderly patient with Type II diabetes is prescribed a "first line of defense" treatment to regulate the disease. Which of
the following is the patient MOST likely prescribed?
Sulphonylureas
Metformin
Thiazolidinedions
Insulin.
Correct answer: Metformin Metformin is ideal for patients who are elderly and obese. It is also considered a "first line of
defense" drug.
Question 4
Which of the following would a NP prescribe to her 67 year old patient suffering from insomnia?
Only nap once a week.
Avoid napping.
Only nap twice a week.
Only nap three times a week.
Correct answer: Avoid napping When treating a patient suffering from insomnia non-pharmacologically, the NP encourages
the patient to stay on a regular routine that includes going to bed and waking up at the same time each day. Napping during the
day may prevent the patient from being able to do this
Question 5
Medicaid is a federal program administered by the states. All of the following statements regarding Medicaid are true
EXCEPT:
Medicaid pays NPs 70% to 100% of the fee-for-service rates set for physicians by state Medicaid agencies.
Federal law controls the rates paid by Medicaid.
Medicaid reimbursement generally is lower than the rate paid by commercial insurers.
Many states have applied to the federal government for a Medicaid waiver in order to administer Medicaid in ways that differ
from federal laws and regulations.
Correct answer:
Federal law controls the rates paid by Medicaid.
This is incorrect. Federal law does not control the rates paid by Medicaid. State law controls the rates paid by Medicaid
Question 6
The theory that explains current family situations in terms of past relationships and family histories is which of the following?
Maslow’s hierarchy of needs
Piaget’s cognitive development theory
Erikson’s stages of psychosocial development
Bowen’s family systems theory
Correct answer:
Bowen’s family systems theory
Bowen’s family systems theory explains current family situations in terms of past relationships and family histories. It
connects one’s past family experiences with current behaviors and suggests that multiple factors interacting across time
240
influence family functioning.
Question 7
A 52-year-old female patient who wears a right lower leg brace weighs 100 lb, is 65 inches tall, and her vital signs are normal.
She has developed post-polio symptoms. The nurse practitioner understands she needs to:
Avoid exposure to cold or chilling, which may cause a loss of strength in the affected muscle.
Reduce the amount of time using the brace for joint support to prevent further loss of strength.
Exercise all muscle groups vigorously to prevent disuse syndrome.
Gain weight to prevent further disability.
The Correct answer is:
Avoid exposure to cold or chilling, which may cause a loss of strength in the affected muscle
In addition to avoiding gaining weight and exercising to the point of muscle pain, regular health maintenance visits are
necessary. Cold temperatures can cause a loss of muscle strength in the affected muscle groups and should be avoided. The
brace should be used along with periodic evaluation of muscle strength and function.
Question 8
An adult patient admits to drinking alcohol on the weekends. He does, however, say in the interview that he is thinking about
cutting back on his drinking. What other statement would indicate that the patient is possibly an alcoholic?
His friend tell him he drinks too much.
He wants to cut out drinking on the weekends to reduce calories.
He drinks one beer a night.
He does not drink to alleviate stress.
Correct answer: His friend tell him he drinks too much A patient who feels guilty about drinking and is told to cut down by his
friends or family may be an alcoholic. Using the CAGE strategy will help determine the risk of alcoholism.
Question 9
An elderly patient presents with a fever, chills, myalgia and erythema migrans. The patient is exhibiting symptoms of what
stage of Lyme Disease?
1
2
3
None of these.
Correct answer: 1 There are three stages of Lyme Disease. Stage 1 is known for: fever, chills, myalgia and erythema migrans
Stage 2 is known for: fatigue, migratory arthralgia and cranial nerve palsies Stage 3 is known for: memory, sleep and mood
problems
Question 10
What reflex of the eyes would you test on a two month old infant?
Step reflex.
Plantar flexion reflex.
Babinski reflex.
Red reflex.
Correct answer: Red reflex The red reflex is the only reflex of the eye listed. This should be done within the first two months
of a child's life. It will determine if there are any problems with the back of the eyes.
Question 11
A patient admits to fatigue in an interview. The exam reveals that his blood pressure is 145/90. What might you screen for?
Hepatitis.
Diabetes.
STD.
Anemia.
Correct answer: Diabetes High blood pressure should always be investigated. Fatigue and high blood pressure could indicate
diabetes. A glucose test should be ordered.
Question 12
Which of the following nurse practitioner roles provided the model for other emerging nurse practitioner specialties?
pediatric nurse practitioner
family nurse practitioner
241
geriatric nurse practitioner
adult nurse practitioner
Correct answer:
pediatric nurse practitioner
The first NP program established in 1964 through the collaborative effort of Loretta Ford and Henry K. Silver provided the
model for other emerging nurse practitioner specialties. The role of the NP continues to expand.
Question 13
The healthcare delivery system has made a shift in the provision of non acute care from the hospital to the ambulatory care
setting and home. Nursing research previously conducted in hospitals:
remains relevant in the new settings because the care patients receive is the same
should be applied directly to patient care provided in the new settings
may no longer be applicable to the healthcare delivery
now becomes applicable to all in-patient hospital care providers
The Correct answer is:
May no longer be applicable to the healthcare delivery
Such a drastic change in the healthcare delivery has occurred and this makes studies no longer applicable to nursing care.
Question 14
Which of the following is one of the most common causes of failure to thrive in an infant?
improper feeding
hypothyroidism
cystic fibrosis
celiac disease
Correct answer:
improper feeding
Improper feeding is one of the most common causes of failure to thrive. It can be for economic reasons, lack of education,
psychologic, or feeding intolerance.
Question 15
Which of the following is a relative contraindication for oral contraceptives?
coronary artery disease
smoking, if younger than 35 years
undiagnosed genital bleeding
migraine with focal aura
Correct answer:
smoking, if younger than 35 years
Smoking, if younger than 35 years, is a relative contraindication for oral contraceptives. If over the age of 35, it is an absolute
contraindication.
Question 16
Seizures in newborns almost always reflect significant nervous system pathology. When the newborn has rigid posturing of
the extremities and trunk during the seizure, he is having which of the following types of seizure?
myoclonic
tonic
multifocal
focal
Correct answer:
tonic
A tonic seizure is indicated by rigid posturing of the extremities and trunk. A myoclonic seizure is characterized by focal or
generalized jerking of the extremities. A focal seizure is twitching of muscle groups, including the face. A multifocal seizure is
similar to focal, but involves multiple muscle groups.
242
Question 17
There are a number of healthcare issues that should concern the FNP. All of the following are true in terms of current
healthcare issues EXCEPT:
There is increasing emphasis on culturally competent care and evidence-based practice.
New emphasis on genetic research makes medical ethics a growing concern.
Causes of infant mortality have changed from primarily noninfectious causes to primarily infectious and nutritional causes.
There is an increasing concern over drug-resistant organisms and pandemic flu.
Correct answer:
Causes of infant mortality have changed from primarily noninfectious causes to primarily infectious and nutritional causes.
This statement is not true. The opposite is true. Causes of infant mortality have changed from primarily infectious and
nutritional causes to primarily noninfectious causes such as congenital anomalies and perinatal events.
Question 18
You have a 45-year-old male patient who presents with fever and chills. He has splinter hemorrhages on his nailbeds and
violet-colored painful nodes on his fingers. When you do a cardiac exam you find that a heart murmur is present. Which of the
following is most likely to be a diagnosis for this patient?
congestive heart failure
myocardial infarction
Raynaud’s phenomenon
bacterial endocarditis
Correct answer:
bacterial endocarditis
Bacterial endocarditis (BE) is an infection of the valves and inner lining of the heart (called the endocardium). It happens
when bacteria from the skin, mouth, intestines, or urinary tract enter the bloodstream (usually during a dental or medical
procedure) and infect the heart.
Question 19
How often must hospitals check the National Practitioner Data Bank before granting clinical privileges to nurse practitioners?
every year
every two years
every three years
none of the above
Correct answer:
every two years
Hospitals must check the NPDB data ever two years before granting clinical privileges. Certain agencies may also check the
NPDB data. The general public does not have access to the NPDB data.
Question 20
A 70 year old patient has tremors. What is the likely cause?
Age.
Wear and tear.
Depression.
Pathology.
Correct answer: Pathology Age does take its toll on the body. It is important to know the difference between normal aging and
abnormal symptoms. Tremors are not a natural part of aging and indicate a pathology related cause.
Question 21
You have a 40-year-old male patient who is a heavy smoker. You recommend a program for him to help him stop smoking.
He says that it would be nice to be able to quit, but right now he is not ready. He is in which stage of the trans-theoretical
model of change?
preparation
pre-contemplation
243
termination
action
Correct answer:
pre-contemplation
This patient is in the pre-contemplation stage of change. He is not yet able to even think about changing.
Question 22
In terms of neural tube defects, which of the following statements is least accurate?
Spina bifida and anencephaly are the most common of the neural tube defects.
Myelomeningocele results from failure of the neural tube to close spontaneously.
At least 70% of people with a myelomeningocele have normal intelligence.
The mortality rate for myelomeningocele is 50% with most deaths occurring before 2 years of age.
Correct answer:
The mortality rate for myelomeningocele is 50% with most deaths occurring before 2 years of age.
This is the least accurate statement. Myelomeningocele accounts for most congenital abnormalities of the central nervous
system. The mortality rate for myelomeningocele is 10% to 15% with most deaths occurring before 4 years of age.
Question 23
You are providing patient education to a female patient who has been diagnosed with genital herpes. Which of the following is
NOT part of appropriate patient education for this patient?
The patient should abstain from all sexual activity during an outbreak.
The patient should use condoms or other barriers even when she does not have an outbreak.
The patient should permanently abstain from all sexual activity.
The patient should disclose her diagnosis to sexual partners.
:
Correct Answer: The patient should permanently abstain from all sexual activity People with genital herpes can have normal
sex lives. The patient should be educated to use barriers to prevent infecting her partners and to avoid sex during outbreaks.
She should also be encouraged to disclose her diagnosis to sex partners.
Question 24
A young female patient is in the clinic complaining of tenderness and burning of her vulva. A pelvic exam is performed and
she is notably reddened, edematous, and excoriated. The nurse practitioner does a wet prep with saline and KOH and finds
pseudohyphae and spores. The diagnosis for this client is:
gonorrhea
bacterial vaginosis
chlamydial infection
vulvovaginal candidiasis
The Correct answer is:
Vulvovaginal candidiasis
The yeast and spores on the wet prep with KOH are diagnostic for candidal infection. Bacterial vaginosis would reveal clue
cells and a positive amine odor. Then, chlamydia and gonorrhea requires a cervical culture to be diagnosed
Question 25
The term which is defined as more than one abstract idea or term symbolizing the association or interaction of two or more
concepts is which of the following?
model
phenomenon
proposition
construct
Correct answer:
construct
A concept is intended to represent an abstract ides. A construct is similar to a concept; however, a construct is not just an
abstraction. A construct consists of more than one abstract idea or term and symbolizes the association or interaction of two or
244
more concepts.
Question 26
Which of the following is a tertiary prevention measure?
cardiac rehab
speech therapy
education for pre-existing disease
all of the above
Correct answer:
all of the above
All of the choices are tertiary prevention measures. Tertiary prevention involves rehabilitation and avoidance of further bodily
damage.
Question 27
There is a nurse practitioner (NP) that works in an HIV practice. When talking to a patient, she learns that the patient's sister
lives next door to the NP. She sees this lady on her way to the mailbox and states that she met her brother today. The neighbor
replies, "Don't you work in an HIV clinic?" How is this situation described?
This is not a breach of confidentiality.
This is malpractice.
The nurse practitioner has no liability.
This is breach of confidentiality.
The Correct answer is:
This is breach of confidentiality
There has definitely been a breach of confidentiality with this situation. The neighbor's brother is a patient of the nurse
practitioner (NP) and the NP is bound by law not to disclose health information to anyone without first obtaining permission.
Even though the NP did not acknowledge that the brother was her patient, it was implied.
Question 28
An FNP is preparing a campaign for teachers in an inner city middle school. The purpose is to decrease and eventually
eliminate aggressive behavior. Which of the following would be the best method for presenting the information to the
teachers?
preparing a manual
lecturing to the teachers
panel presentation and small discussion groups
preparing videos for presentation during lecture
Correct answer:
panel presentation and small discussion groups
Adult learning theory says that the older the learner, the more self-direction is needed. Since adults are “doers” an interactive
session is most effective.
Question 29
Principles of structural functional theory adapted from Friedman and others include all of the following EXCEPT:
Families are social systems with instrumental and expressive functions.
In optimally functioning families, members take on predictable roles that meet the needs of its members.
Families are composed of small numbers with characteristics of small-group behavior.
Individuals adopt norms and values solely from outside the family system.
Correct answer:
Individuals adopt norms and values solely from outside the family system.
This is incorrect. Individuals adopt norms and values, as well as cultural traditions as part of the process of family
socialization.
Question 30
245
Which of the following tests would NOT be used to test for hearing?
Weber test
Rinne test
Snellen test
all of the tests
Correct answer:
Snellen test
The Snellen test is used to measure central distance vision. The Weber test and the Rinne test are used to measure hearing
Question 31
In doing anthropometric measurements for an obese child, the FNP would likely find all of the following EXCEPT:
BMI is equal or greater than the 95th percentile for age and gender.
Skin-fold thickness per calibrated caliper measurements is at or above the 95th percentile for age, sex and race.
Percent of ideal body weight is greater or equal to 120%.
Weight for height ratio is greater than the 95th percentile on CDC growth charts.
Correct answer:
Skin-fold thickness per calibrated caliper measurements is at or above the 95th percentile for age, sex and race.
This is incorrect. The skin-fold thickness per calibrated caliper measurements would be at or above the 85th percentile for age,
sex and race.
Question 32
Which of the following laboratory procedures is most useful for helping with diagnosis of fungal infections?
KOH slide
Pap smear
Tzanck smear
Gram stain
Correct answer:
KOH slide
The KOH (potassium hydroxide) slide is useful for helping with the diagnosis of fungal infections (hair, skin, nails). KOH is
also used for the “Whiff Test” for bacterial vaginosis. A strong fish-like odor is released after one to two drops are added on
the slide.
Question 33
During a well-child exam a 4-year-old girl has a pure-tone audiometry revealing 24 decibels (dB) in the left ear and 48 dB in
the right ear. The nurse practitioner interpret this as:
within normal limits for age
normal hearing in the left ear and moderate hearing loss in the right ear
inconclusive findings because pure tone audiometry is not accurate in children age 5 and younger
mild hearing loss in the left ear and normal hearing in the right ear
The Correct answer is:
Normal hearing in the left ear and moderate hearing loss in the right ear
Pure tone audiometry is appropriate after age 3 years. Further, 0-25 dB = normal, 26-40 dB = mild hearing loss, and 41-55 dB
= moderate hearing loss.
Question 34
A 16 year old patient comes in complaining of weight loss, frequent urination, and excessive thirst. There is a history of Type
1 diabetes in the family. What test should be run?
Fasting blood glucose.
Serum cholesterol.
Serum albumin.
Vitamin B12.
Correct answer: Fasting blood glucose Fasting blood glucose will help determine if the patient has diabetes. The family history
increases the patient's risk of diabetes. The complaints could indicate diabetes.
Question 35
A 16-year-old mildly obese adolescent presents requesting a letter stating she should not participate in gym class because of
her asthma. The most appropriate response is to:
246
Excuse her from outdoor activities only as to avoid pollen exposure.
Write the note to excuse her from indoor activities only to avoid dust mite exposure.
Write the note because gym class participation could trigger asthma symptoms.
Remind her that with appropriate asthma care, she can participate in gym class and that exercise is part of a healthy lifestyle.
The Correct answer is:
Remind her that with appropriate asthma care, she can participate in gym class and that exercise is part of a healthy lifestyle
Asthma is a common chronic disorder that is complex but treatable. Exercise is a necessary part of a healthy lifestyle,
especially for a developing teen that is overweight. With the use of a short-acting beta2 agonist inhaler, this condition can be
controlled and managed during activity.
Question 36
When treating a patient with oral Vitamin B12, which drug interaction will result in deceased absorption of vitamin B12?
aminoglycosides
colchicine
potassium supplements
all of the above
The Correct answer is:
All of the above
In addition to colchicine, aminoglycosides and potassium supplements, ascorbic acid may destroy the vitamin B12 supplement
within one hour of ingestion. Therefore, these drugs should not be taken concomitantly with oral vitamin B12.
Question 37
Which of the following would be considered a reversible cause of dementia?
high fever
infections
metabolic derangements
none of the above
Correct answer:
none of the above
None of the other choices are reversible causes of dementia. Dementia is not reversible! Delirium is reversible and the first
three choices are reversible causes for delirium.
Question 38
The FDA has approved three NSAIDS for primary dysmenorrhea. Which of the following is one of these NSAIDS?
ibuprofen
indomethacin
naproxen sodium
all of the above
Correct answer:
all of the above
All of the first three choices are NSAIDS that have been approved by the FDA for primary dysmenorrhea. NSAIDS are
effective in 75% to 90% of cases.
Question 39
A patient comes to the clinic with a burn on her arm caused by spilling boiling water on it. The FNP knows that it is important
to estimate the body surface area (BSA) affected by the burn. She also knows that a person’s palmar surface represents a BSA
of which of the following?
1%
2%
3%
4%
Correct answer:
1%
With burns of any sort, it is necessary and important to estimate the BSA affected by the burn. A helpful guide in estimating
the BSA is that a person’s palmar surface represents a BSA of 1% throughout the life span.
Question 40
247
The descriptors for the levels of evaluation and management (E/M) services recognize seven components that are used in
defining the levels of E/M services. Which of the following is NOT one of these components?
history
counseling
time
expense
Correct answer:
expense
Expense is not one of the seven components used in defining the levels of E/M services. The seven components are: history,
examination, medical decision making, counseling, coordination of care, nature of presenting problem and time.
Question 41
You are counseling the mother of four children who have been diagnosed with pediculosis humanus capitis infestation. With
the management of this condition, the nurse practitioner knows to advise the mother of all of the following EXCEPT:
All carpets and furniture must be vacuumed and cleaned thoroughly.
Children may return to school or day care a week after treatment.
Seal up all unwashable items, such as stuffed animals and large comforters for 10 days.
The hair must be combed with a fine-toothed comb after treating to remove adherent nits.
The Correct answer is:
Children may return to school or day care a week after treatment
The children may return to school or day care the day after treatment.
Question 42
What is a CAM modality or medicine?
an untested drug
a consistent and accepted modality or medicine
a generic drug
a complementary or alternative modality or medicine
Correct answer:
a complementary or alternative modality or medicine
Research suggests that 40 – 50% of patients are currently using a form of complementary or alternative therapy. NPs as
providers need to learn about common CAM treatments and particularly how some herbal products interact with prescription
drugs.
Question 43
You are treating a patient with asthma in the clinic. They tell you they received a new medication in an emergency room when
they were out of town visiting family. Which of the following should be avoided in a patient with asthma?
naproxen
topical hydrocortisone
amlodipine
timolol ophthalmic drops
The Correct answer is:
Timolol ophthalmic drops
Timolol ophthalmic solution drop is a beta blocker which is known to precipitate asthma exacerbation in patients. Even though
it is administered in the eye, there is considerable absorption through the mucous membranes, resulting in systemic effects.
The remaining answer choices have no specific contraindications for patients with asthma.
Question 44
The theoretical concept that says that a person who feels susceptible to a disease and believes that he will benefit from
changing his behavior is more likely to perform the healthier behavior is which of the following concepts?
the self-efficacy theory
the health belief model
the systems theory
the family systems theory
248
Correct answer:
the health belief model
The theoretical concept that says that a person who feels susceptible to a disease and believes that he will benefit from
changing his behavior is more likely to perform the healthier behavior is the health belief model. This model attempts to
explain and predict health behaviors. This is done by focusing on the attitudes and beliefs of individuals.
Question 45
Marketing refers to determining the needs and desires of the prospective consumer and designing NP services to meet those
needs. The key elements of marketing include the four Ps which are:
product, price, place, and prescription
profession, place, price and production
product, price, place and promotion
profession, price, place and prescription
Correct answer:
product, price, place and promotion
The key elements of marketing include the four Ps. These are: product (the unique role of the NP); price (cost advantage in the
NP-delivered care); place (competitive advantage of a practice site or after-hours services); and promotion (processes devoted
to negotiating a position in practice).
Question 46
Which of the following would NOT be a barrier to therapeutic communication?
probing
interpreting
changing the subject
seeking clarification
Correct answer:
seeking clarification
Seeking clarification involves either stating that what a patient said was misunderstood or asking the patient to repeat the
conversation or basic idea of the message. It is a good therapeutic communication technique. The other choices are not.
Question 47
At what age would you expect a child to jump up and down?
12-14 months
16 months
18 months
24-28 months
Correct answer:
24-28 months
You would expect a child to be able to jump up and down by the age of 24-28 months. At this age the child should also be able
to run about and kick a ball, as well as build a block tower of six cubes.
Question 48
All of the following are considered components of family functioning EXCEPT:
stable and safe physical environment
adequate financial resources
appropriate supervision
being a member of a “traditional” family
Correct answer:
being a member of a “traditional” family
This is not considered a component of family functioning. Modern day has seen a variation in family structure that does not
necessarily affect family functioning. Family functioning is more directly related to healthy growth and development than is
family structure. Components of family functioning include provision of a stable and safe physical environment as well as
financial and emotional resources necessary to provide supportive and nurturing care with appropriate supervision and
249
guidance.
Question 49
All of the following are characteristic of irritable bowel syndrome EXCEPT:
no detectable structural abnormalities
absence of rectal bleeding
fever with weight loss
no elevation of CRO or ESR
Correct answer:
fever with weight loss
With IBS there is usually no fever or weight loss. This is more common in inflammatory bowel diseases such as Crohn’s
disease and ulcerative colitis.
Question 50
A middle aged postmenopausal woman is in the office with questions regarding hormone therapy. You explain to her that
hormone therapy users may experience:
an increase in breast cancer rates with long-term use
a 10% increase in bone mass
no change in the occurrence of osteoporosis
reduction in high-density lipoprotein cholesterol
The Correct answer is:
An increase in breast cancer rates with long-term use
Postmenopausal hormone therapy can help reduce the risk of postmenopausal fracture by 50% by minimizing further bone
loss. The benefits must be balanced against the noted increased risk of breast cancer and other problems.
Question 51
The concept that defines what a nurse practitioner can do in that role is known as which of the following?
standard of care
scope of practice
ethical principle
moral principle
Correct answer:
scope of practice
The NP’s scope of practice, outlined in the nurse practice act, identifies who the NP is, what the NP can do in that role, and
where the NP can legally provide care. Actions that exceed the legal boundaries of practice in a particular state are considered
to be violations of the nurse practice act.
Question 52
The FNP has a patient with seasonal allergic rhinitis. The FNP would most likely tell this patient to avoid the allergen by
staying indoors except for:
in the morning
after a rain shower
during the night
The patient should stay indoors all day during allergy season.
Correct answer:
after a rain shower
After a rain shower, the air is relatively cleansed of offending allergens. Pollen counts are generally the highest early in the
morning because these substances are released during the night.
Question 53
You are examining the testicles of a 25-year-old patient. When touching the right inner thigh of this patient the testicle is
elevated toward the body on that side. This is known as which of the following?
priapism
cremasteric reflex
250
blue dot sign
phimosis
Correct answer:
cremasteric reflex
The cremasteric reflex happens when the testicle is elevated toward the body in response to stroking the ipsilateral inner thigh.
This is the thigh on the same side as the testicle.
Question 54
The nurse practitioner will work at several different job sites over the course of her career. One such site is a federally
qualified health center (FQHC). Which of the following is NOT true regarding this type of facility?
Clinics and facilities can be called a federally qualified health center if they accept Medicare and Medicaid and work toward
public health service.
They originated from community health centers and migrant health centers established under the Public Health Service Act.
Services covered in these centers are health screenings, diagnostic tests immunizations, annual physical examinations and
preventive health education.
They were established in 1992 to provide health promotion and preventive services and access to primary care services for
Medicare beneficiaries.
The Correct answer is:
Clinics and facilities can be called a federally qualified health center if they accept Medicare and Medicaid and work toward
public health service
Clinics and facilities must meet certain requirements and receive a federal grant in order to be called federally qualified health
centers.
Question 55
The majority of breast cancers occur in which area of the breast?
beneath the nipple and areola
upper outer quadrant
lower outer quadrant
upper inner quadrant
The Correct answer is:
Upper outer quadrant
The upper outer quadrant is the most common site for breast cancer. The second most common site for breast cancer is
beneath the nipple and areola.
Question 56
Certain foods are known to be potential dietary triggers influencing the onset or severity of migraine symptoms. When
counseling a patient on these, which is NOT considered a trigger?
onions
ripened cheeses
vanilla ice cream
caffeinated beverages
The Correct answer is:
Vanilla ice cream
Vanilla ice cream is not on the list as triggers for a migraine headache. However, "chocolate" is. Also included as trigger foods
are onions, caffeinated beverages, ripened cheeses, sour cream, sausage, bologna, salami, pepperoni, hot dogs, pizza, chicken
liver, herring, pickled foods, monosodium glutamate, yeast products, nuts or nut butters, various beans, figs, raisins, papayas,
avocados, red plums, citrus foods, bananas, alcoholic beverages and aspartame
Question 57
Your patient is taking a hiking trip in a wooded area with friends. She asks you how she can protect herself from Lyme disease
during this trip. You would tell her which of the following will help?
Wear light colored clothing so ticks are easily detected.
Wear long pants tucked into socks.
Wear long-sleeved shirt tucked into waist.
251
all of the above
Correct answer:
all of the above
All of the first three choices will help to prevent Lyme disease. This is an infection caused by Borrelia burgdorferi, a
spirochete. It is the most common arthropod-borne disease in the United States.
Question 58
Which of the following drugs is a drug that is injected directly into the penis for correction of erectile dysfunction?
alprostadil
Muse
taldalafil
vardenafil
Correct answer:
alprostadil
Alprostadil (Caverject) causes vasodilation and is highly effective as a treatment for erectile dysfunction. Muse is a pellet
inserted into the urethra and can achieve the same effect. Taldalafil (Cialis) and vardenafil (Levitra) are oral treatments.
Question 59
A 24-year-old sexually active female comes to the clinic complaining of lower abdominal pain and cramping. She tells the
FNP that she has not had a period for about 7 weeks. The pain worsens when the woman is lying down on her back. The FNP
might suspect which of the following conditions/diseases?
ovarian cancer
Candida vaginitis
ectopic pregnancy
atrophic vaginitis
Correct answer:
ectopic pregnancy
The most likely of the choices is an ectopic pregnancy. Risk factors include: a history of PID, tubal ligation and older age.
This is the leading cause of death for women in the United States in the first trimester of pregnancy.
Question 60
The fact that an adult nurse practitioner is not legally authorized to care for children is an example of defining which of the
following?
scope of practice
standards of practice
ethical standards
civil responsibilities
Correct answer:
scope of practice
The scope of practice is determined by state statutes, boards of nursing, educational preparation, and common practice within a
community. Broad variation exists from state to state.
Question 61
The Centers for Disease Control and Prevention (CDC) recommends screening all women for sexually transmitted diseases
between the ages of:
12 and 16
16 and 20
20 and 24
24 and 30
Correct answer:
20 and 24
The CDC recommends screening all women for sexually transmitted diseases between the ages of 20 and 24. This would be
252
considered a secondary prevention measure to detect disease early to minimize bodily damage.
Question 62
Which of the following would be the most important source of information to determine whether a patient is suffering from
Alzheimer’s disease (AD)?
mental status examination
CBC
MRI
clinical history
Correct answer:
clinical history
With AD, you would be interested in the symptoms and the progression of symptoms characteristic of AD. Therefore, of all
the choices, the clinical history would be most informative.
Question 63
When counseling the mother of a toddler on lead toxicity and poisoning, what should the nurse practitioner understand that is
most likely to lead to this condition?
If the toddler lives near an electric generating plant, he or she is at risk.
If the toddler is developmentally disabled, he or she is at risk.
If the toddler lives in a 15-year old home with copper plumbing, he or she is at risk.
If the toddler lives in a 100-year old home that is being remodeled, he or she is at risk.
The Correct answer is:
If the toddler lives in a 100-year old home that is being remodeled, he or she is at risk
The toddler is at risk if he or she is exposed to lead-based paint. This paint has not been available in the United States in 30
years. Also, copper pipes are not a risk to the toddler as lead-lined pipes are. A toddler living near a lead factory would be at
risk, not one living near a electric generating plant. A developmental disability does not increase lead toxicity risk.
Question 64
The FNP has an adult patient with diabetes whose blood pressure is 145/95. Which stage of hypertension does this indicate?
normal
pre-hypertension
Stage II
Stage I
Correct answer:
Stage I
Stage I hypertension is characterized by a systolic pressure of 140 – 159 mm Hg and a diastolic pressure of 90 – 99 mm Hg.
The treatment recommendation for this stage is usually a thiazide diuretic and for those with diabetes, either ACE inhibitors or
ARBs, because of their renal protective properties.
Question 65
There are different psychosocial periods in adolescence. In which period would achievement of identity and intimacy in
relationships take place?
pre-adolescence
early adolescence
middle adolescence
late adolescence
Correct answer:
late adolescence
Late adolescence takes place between 18 and 21 years. During this period vocation and career choices become important.
Achievement of identity and intimacy in relationships are hallmarks of this period.
Question 66
An elderly lady with rheumatoid arthritis (RA) is in the office and placed on prednisone 5 mg PO qd. In educating her about
this medication, which of the following is important for the nurse practitioner to include?
253
The medication should be taken about 30 minutes before eating.
When the symptoms of arthritis subside, she will be able to stop this medication.
Increased fluid intake is important for the prevention of renal damage by the use of steroids.
It is important to continue the medication as prescribed, even if the redness and swelling decreases or goes away.
The Correct answer is:
It is important to continue the medication as prescribed, even if the redness and swelling decreases or goes away
The client should be informed that the symptoms will decrease when the medication is effective, but the client should maintain
her dose as prescribed. The medication is not influenced by fluids and should be taken with food to avoid gastrointestinal
upset.
Question 67
Regarding the scientific method for conducting research which uses the null hypothesis, the nurse practitioner knows this is
statistically based and the correct format for the null hypothesis is:
Group "A" is less than group "B".
Group "A" is greater than group "B".
There is a 95% probability that group "A" is different from group "B".
There is no significant difference between the two groups.
The Correct answer is:
There is no significant difference between the two groups
The research hypothesis may take the other forms.
Question 68
Which of the following types of pneumonia has a gradual onset, with symptoms that include a low grade fever, sore throat,
cough, wheeze and chest pain with cough?
bacterial pneumonia
viral pneumonia
community acquired pneumonia
atypical pneumonia
Correct answer:
atypical pneumonia
Atypical pneumonia is an infection of the lungs by atypical bacteria. It is more common in children and young adults. It is also
known as “walking pneumonia.”
Question 69
Which of the following would not be a first-line test for a female with cholecystitis?
CBC
pregnancy test
urinalysis
serum amylase and lipase levels
Correct answer:
serum amylase and lipase levels
Serum amylase and lipase levels would not be first line diagnostic tests. They might be indicated for further testing in a patient
with a certain history or physical examination findings.
Question 70
A 9-year-old African-American child is brought to the clinic by his mother. She tells the FNP that the child has patchy white
spots on his scalp with “black dots” and his scalp appears scaly and is itchy. The FNP knows that these are signs of which of
the following diseases/conditions?
tinea pedis
tinea corporis
tinea cruris
tinea capitis
Correct answer:
tinea capitis
254
Tinea infections are infections of superficial keratinized tissue (skin, hair and nails) by tinea yeast organisms. Tinea capitis is
more common in dark-skinned children and is characterized by patchy alopecia with “black dots” (broken hair shafts) and
fragile hair shafts. Children also have fine scales on the scalp and the scalp is frequently itchy.
Question 71
The nurse practitioner is aware that hepatitis B is a public health concern. He has the understanding that which of the
following groups needs routine hepatitis B (HBsAg) screening?
college students
hospital laboratory workers
pregnant women
recipients of hepatitis B vaccine series
The Correct answer is:
Pregnant women
Infants who become infected with HBV have a 25% lifetime chance of developing hepatocellular carcinoma or cirrhosis. This
is why pregnant women should be screened for HBV at the first prenatal visit, regardless of their vaccine history.
Question 72
Which of the following medications would you prescribe for a patient who suffers from recurrent gout as a maintenance
treatment?
Indocin
Anaprox DS
colchicine
Zyloprim
Correct answer:
Zyloprim
Of the choices given, Zyloprim is the only maintenance medication. The other choices are associated with the acute phase of
gout
Question 73
You are prescribing Zithromax for a child who weighs 66 lb. The initial dose is 10 mg/kg per day. This dose amounts to which
of the following?
300 mg/day
660 mg/day
330 mg/day
600 mg/day
Correct answer:
300 mg/day
Since the initial recommended dose is 10 mg/kg per day, you first need to convert the child’s weight to kg. This is done by
dividing 66 lb by 2.2 kg. (There are 2.2 kg to each lb). The result is 30, which is then multiplied by 10 mg to give you a dose
of 300 mg/day.
Question 74
A patient comes to the clinic with a burn on his leg that is about three times the palmar surface of his hand. The body surface
area (BSA) of the burn can be estimated at:
3%
6%
9%
12%
Correct answer:
3%
A person’s palmar surface represents a BSA of 1% throughout his life span. Therefore, a burn that is 3 times the palmar
surface represents a BSA of 3%.
Question 75
255
When should an infant be able to lift his head and hold his head erect?
1 – 2 months
3 - 4 months
5 - 6 months
6 - 8 months
Correct answer:
1 – 2 months
At 1 – 2 months an infant should be able to lift his head and hold it erect. He should also be regarding faces, following objects
through a visual field, smiling spontaneously and recognizing his parents.
Question 76
You are conducting an assessment with a 40 year old male patient. He notes that he has lost 5 pounds in the last month, though
he has not been dieting. He also says that he is urinating more frequently, though this may be because he is drinking so much
water due to being thirsty all the time. He also reports being frequently tired. This patient should be further assessed for which
of the following?
Hypoglycemia.
Hyperglycemia.
Myocardial infarction.
Anemia.
Correct Answer: Hyperglycemia This patient's symptoms are consistent with hyperglycemia. Further tests, including a blood
glucose test, should be ordered in order to arrive at a correct diagnosis. The patient may also report that cuts and sores heal
more slowly.
Question 77
A 32-year-old Asian man presents with sneezing, postnasal drip, sore throat, and watery eyes. He states he has been doing
yard work for the last couple of days. What do these symptoms suggest?
acute bronchitis
atypical pneumonia
influenza
allergic rhinitis
The Correct answer is:
Allergic rhinitis
The symptoms presented are classic for allergic rhinitis. The signs and symptoms of acute bronchitis include a cough and
colored sputum. An individual with atypical pneumonia would present with a fever and cough. Then, a patient with influenza
would present with an acute onset of fever, chills, and malaise.
Question 78
Which of the following is a system of controlled oversight and authorization of services and benefits provided to patients?
clinical ladder
case management
triage
clinical guidelines
The Correct answer is:
Case management
Case management is a special type of communication that is a necessity for healthcare providers. It is the process of
interviewing and teaching patients and for sharing or clarifying information with others involved in the patient's care.
Question 79
Your patient has a yellow, triangular thickening of the bulbar conjunctiva which you have diagnosed as a pinguecula. The
bulbar conjunctiva is which of the following?
mucosal lining inside the eyelids
mucosal lining inside the mouth
roof of the mouth
mucosal lining covering the eyes
Correct answer:
mucosal lining covering the eyes
The bulbar conjunctiva is the mucosal lining covering the eyes. The mucosal lining inside the eyelids is the palpebral
256
conjunctiva.
Question 80
You have a 20-year-old patient complaining of headache, fatigue, and sore throat. Upon examination you observe a macular
red rash on his trunk and upper arms. He tells you that this rash occurred a few days after he began to have a sore throat and
headache. Your most likely diagnosis of the following choices would be:
Fifth disease
mononucleosis
measles
roseola
Correct answer:
mononucleosis
A patient with mononucleosis is likely to have a macular/popular red, morbilliform rash on the trunk and upper arms. It may
also involve the face. The patient complains of headache, fatigue, and sore throat before the rash occurs.
Question 81
All of the following statements about Lyme disease are accurate EXCEPT:
It is the least common arthropod-borne disease in the United States.
It is transmitted by prolonged attachment of 2 days of more by and infected tick.
Prevalence is associated with increased deer population in endemic areas.
Lyme disease is a multisystem disorder.
Correct answer:
It is the least common arthropod-borne disease in the United States.
This is not correct. Lyme disease is the most common arthropod-borne disease in the United States. it is caused by Borrelia
burgdorferi, a spirochete.
Question 82
Which test is the most important diagnostic lab value to diagnose iron deficiency anemia?
serum folate level
serum ferritin level
red blood cell (RBC) count
direct Coombs
The Correct answer is:
Serum ferritin level
Serum ferritin levels correlate with the total body iron stores because it is the major iron storage protein. Its value is reduced in
iron deficiency anemia. Serum folate measures the folic acid level. Direct Coombs measures in vivo RBC coating by
immunoglobulins.
Question 83
Which family theory explains current family situations in terms of past relationships and family histories?
cognitive development theory
Bowen family systems theory
family systems theory
communications theory
Correct answer:
Bowen family systems theory
The Bowen family systems theory connects one’s past family experiences with current behaviors. It also suggests that multiple
factors interacting across time influence family functioning and identifies the interactions among biological, genetic,
psychological and sociological factors that influence human behaviors.
Question 84
Which of the following would NOT be an indication for hospitalization of an adult with community-acquired pneumonia?
respiratory rate less than 20 breaths per minute
age older than 60 years
altered mental status
257
failure to respond to outpatient treatment within 72 hours
Correct answer:
respiratory rate less than 20 breaths per minute
This is not an indication for hospitalization for an adult with community-acquired pneumonia. The other choices are
indications along with: not having resources for self-care at home; severe underlying disease such as heart failure; and severe
electrolyte abnormality.
Question 85
When counseling a patient with allergic rhinitis, you prescribe Cromolyn. The nurse practitioner understands that its
mechanism of action is a (an):
mast cell stabilizer
vasoconstrictor
leukotriene modifier
anti-immunoglobulin E antibody
The Correct answer is:
Mast cell stabilizer
Cromolyn is a mast cell stabilizer in a nasal spray that is effective at preventing symptoms of allergic rhinitis. This drug does
not acutely relieve symptoms. Instead, it takes around a week of use to achieve symptom relief.
Question 86
Your patient has experienced a TIA. You understand that there are certain risk factors for TIAs that include all of the
following EXCEPT:
atherosclerosis
peptic ulcer disease
cardiac valve problems
oral contraceptive use
Correct answer:
peptic ulcer disease
Risk factors for TIA include: carotid artery and other forms of atherosclerosis; structural cardiac problems, such as valvular
problems that lead to increased risk of embolization; and hypercoagulable conditions, such as oral contraceptive use.
Question 87
Which of the following is an appropriate plan of care for a patient who has been diagnoses with mononucleosis?
Hospitalization with IV fluids.
A short course of high-dose antibiotics.
Oral antibiotics.
Rest and home care.
Correct Answer: Rest and home care The standard care protocol for mono is rest and self-care at home. OTC painkillers such
as Tylenol may be used for symptom management. Fluids and rest are key to recovery.
Question 88
During a health assessment with a male patient, he tells you that he has not had a bowel movement in three days, although he
usually has one each day. He says that the new medication he is taking makes it more difficult to pass feces. Which of the
following is an appropriately worded nursing diagnosis for this patient?
Patient is constipated.
Elimination, altered.
Elimination, fecal, altered.
Elimination, fecal, altered, r/t change in medication.
Correct Answer: Elimination, fecal, altered, r/t change of medication It is key to indicated not only that the patient's
elimination pattern is altered, but that the patient notes that this change has occurred along with the change in his medication.
You must also specify that it is fecal elimination that is altered so that appropriate remedies, and evaluation of the patient's
medication, can be undertaken.
Question 89
Which of the following types of communication would NOT be considered therapeutic interaction?
opinion
value-free
258
advice-free
reassurance-free
Correct answer:
opinion
The purpose of therapeutic interaction with patients is to allow them the autonomy to make choices when appropriate.
Therapeutic communication should be just facts without opinion. It should be value-free, advice-free and reassurance-free.
Question 90
A patient presents in the office with some abnormal findings. The nurse practitioner (NP) suspects a rheumatological disorder,
possibly systemic lupus erythematous (SLE). The NP knows that the frequent occurring symptoms of SLE are:
fever, arthralgia, arthritis and skin rashes
generalized hair growth
green areas on the fingers
Swelling across the back
The Correct answer is:
Fever, arthralgia, arthritis and skin rashes
The clinical symptoms, such as fever, arthralgia or joint pain and arthritis, may all be present in patients diagnosed with
systemic lupus erythematosus (SLE). A butterfly rash and lymphadenopathy occur in less than half the cases. Pulmonary
effusion, hepatomegaly, splenomegaly, and Raynaud's syndrome occur in less than a third of the patients with SLE. In
addition, patients with SLE may experience alopecia, and not hair growth, with flare-ups with this condition. Then, red areas,
and not green areas, may emerge on the fingers and palms of the hands.
Question 91
A 56-year-old Caucasian female is in the office inquiring about menopause. What is the best way to define menopause and
counsel her?
Menopause is the last menstrual period.
Menopause is the completion of 12 months of amenorrhea after the last menstrual period.
Menopause is indicated by a FSH level of 30 and estradiol level of 30.
Menopause does not cease the ability for natural reproduction.
The Correct answer is:
Menopause is the completion of 12 months of amenorrhea after the last menstrual period
Standardized developed definitions for menopause events were created in 2001 by the Stages of Reproductive Aging
Workshop. These standards state that menopause is defined as the completion of 12 months of amenorrhea after the last
menstrual period. Due to the fact that FSH levels are variant, it is not the best indicator of menopause.
Question 92
In terms of adult/geriatric screening recommendations, which of the following is NOT a generally accepted recommendation?
periodic TSH in women
periodic skin exam in older adults with sun exposure
routine screening for testicular cancer
digital rectal exam annually for men older than 40
Correct answer:
routine screening for testicular cancer
No evidence supports routine screening of asymptomatic men for testicular cancer. The old recommendation was for clinical
exam every 3 years for men ages 20 – 39 and annually for those over age 40.
Question 93
The expected date of delivery for a woman who has had her last menstrual period on July 5, 2011 is which of the following?
April 12, 2012
March 12, 2012
March 5, 2012
April 5, 2012
Correct answer:
April 12, 2012
259
The FNP would use Naegele’s rule to calculate the expected date of delivery. She would subtract 3 months from July and add
7 days to the date (5th) of the last period. This would put the date of delivery at April 12, 2012.
Question 94
A patient who has been prescribed Adderall should be educated to notify his healthcare provider if he experiences which of the
following side effects?
Nausea.
Headache.
Teeth grinding.
Difficulty sleeping.
Correct Answer: Teeth grinding The patient should notify his healthcare practitioner if he experiences teeth grinding.
Difficulty sleeping, nausea, and headache may occur with Adderall, but are not typically a cause for concern. If these increase
or worsen, the patient should notify his healthcare provider.
Question 95
All of the following are signs of delirium in your patient EXCEPT:
altered level of consciousness from baseline
memory impairment
perceptual disturbance
loss of cognitive function
Correct answer:
loss of cognitive function
Loss of cognitive function is not associated with delirium which has an acute onset. It is associated with dementia, which is
defined as chronic loss of intellectual or cognitive function of sufficient severity to interfere with social or occupational
function.
Question 96
The FNP has a 39-year-old female patient who wants to get the flu vaccine. She asks the FNP about getting the nasal-spray
vaccine instead of a shot, since she has an aversion to needles. The FNP knows that all of the following are true about the
nasal-spray flu vaccine EXCEPT:
It contains weakened live influenza viruses instead of killed viruses.
The viruses in the nasal-spray flu vaccine can grow in the nose and throat as well as in the lower respirator tract.
This type of vaccine is approved for use in healthy people 2 to 49 years old.
It contains three different influenza viruses that are sufficiently weakened as to be incapable of causing disease.
Correct answer:
The viruses in the nasal-spray flu vaccine can grow in the nose and throat as well as in the lower respirator tract.
The viruses in the live attenuated influenza vaccine (LAIV) are cold adapted and temperature sensitive. As a result the viruses
in the nasal-spray flu vaccine can grow in the nose and throat but not in the lower respiratory tract where the temperature is
higher.
Question 97
Of the following, which type of lesion is associated with psoriasis?
macule
vesicle
plaque
wheal
Correct answer:
plaque
Psoriasis has plaque-type skin lesions. They are raised lesions, > 1 cm, and may be the same or different color from the
surrounding skin.
Question 98
When you examine a young child you find that one phase of his development is out of synchrony with others. This is known
as which of the following?
delay
260
dissociation
deviance
disorder
Correct answer:
dissociation
Diagnosis of developmental delays includes three levels: delay, dissociation, and deviance. Dissociation occurs when one
phase of development is out of synchrony with others.
Question 99
According to the CDC an extremely obese person would have a Body Mass Index (BMI) of:
greater than or equal to 18.5
greater than or equal to 25
greater than or equal to 30
greater than or equal to 40
Correct answer:
greater than or equal to 40
An extremely obese person would have a Body Mass Index (BMI) of greater than or equal to 40. A person of healthy weight
would have a BMI of 18.5 to 24.9.
Question 100
The nurse practitioner will often have to treat systemic anaphylaxis. What is the most common clinical manifestation of this?
persistent vertigo
nausea and vomiting
headache
urticaria
The Correct answer is:
Urticaria
Anaphylaxis is typically manifested as systemic IGE-mediated reaction in response to exposure to an allergen. It is
characterized by urticaria, angioedema, widespread vasodilation and bronchodilation.
test 13 aanp
Question 1
You have an 84-year-old female patient. Which of the following findings are you least likely to see in a patient of this age?
high-frequency hearing loss
increase in peak expiratory flow
decrease in the GFR
slower metabolism of drugs
Correct answer:
increase in peak expiratory flow
As far as respiration and lung function goes in the elderly, there is less elasticity and fewer cilia in the lungs. Peak expiratory
flow decreases, and residual volume increases. Total lung capacity, however, remains unchanged.
Question 2
Which of the following statements about sleep is least accurate?
The average newborn sleeps about 18 hours a day.
The elderly tend to sleep less, usually and average of 6.5 hours a day.
The sleep-wake cycle is controlled by the circadian rhythm located in the parathyroid gland.
A sleep cycle consists of REM and NREM sleep.
Correct answer:
The sleep-wake cycle is controlled by the circadian rhythm located in the parathyroid gland.
261
This is the least accurate statement. The sleep-wake cycle is controlled by the circadian rhythm located in the hypothalamus.
Two of the neurotransmitter associated with sleep are serotonin and gamma-aminobutyric acid.
Question 3
The nurse practitioner knows that all of the following are nonpharmacologic treatments for a patient with acute
glomerulonephritis EXCEPT:
Avoid high-potassium foods.
Treat them as inpatient until edema and hypertension are under control.
Restrict protein in presence of azotemia and metabolic acidosis.
Encourage increased fluid intake.
The correct answer is:
Encourage increased fluid intake
The patient’s fluid intake should be restricted to only the amount the patient requires to replace lost fluids. The remaining
answer choices are included in the treatment of acute glomerulonephritis.
Question 4
Your patient has been diagnosed with Bell’s palsy. The cranial nerve involved in this disease is which of the following?
CN V
CN VI
CN VII
CN VIII
Correct answer:
CN VII
CN VII is the facial nerve. Dysfunction of this nerve gives the characteristic findings of Bell’s palsy (facial asymmetry, droop
of mouth, absent nasolabial fold, impaired eyelid movement).
Question 5
The nurse practitioner (NP) is examining a child for a sore throat. Which of the following statements is true regarding tonsils?
Most cases of tonsillitis are caused by beta-hemolytic streptococcal infection.
Hypertrophied tonsils in children usually represent a normal finding.
Tonsils enlarge as the child grows older.
Children with large tonsils are more prone to tonsillitis than those with small tonsils.
The Correct answer is:
Hypertrophied tonsils in children usually represent a normal finding
Enlarged tonsils are common in young children and as the child grows older, the tonsils will recede in size. Only about 25% of
tonsillitis is caused by streptococcal infection. Further, children with large tonsils are more prone to tonsillitis than those with
small tonsils is a false statement.
Question 6
A patient presents with a severe itching, hives, and sore throat. Her lips begin to swell and she is wheezing. What is the
appropriate action?
Give the patient aspirin.
Give the patient Benadryl.
Tell the patient to rest.
Call 911.
Correct answer: Call 911 The patient appears to be going into anaphylactic shock. Unless in an emergency room already, call
911. The patient may also require a shot of epinephrine.
Question 7
In terms of child development, which of the following statements is most accurate?
Attainment of developmental landmarks in one area always runs parallel with another area of development.
Development is independent of the maturation of the nervous system.
Development occurs in a foot to head direction.
The sequence of development is basically the same in all children but the rate varies.
The sequence of development is basically the same in all children but the rate varies.
This is the most accurate statement. The other three choices are inaccurate. Attainment of developmental landmarks in one
area does not always run parallel with another area of development. Development is dependent on the maturation of the
262
nervous system. Development occurs in a head to foot direction
Question 8
An infant is brought in with a temperature of 99.7 F. The parents insist that this is higher than average. What is the appropriate
response?
Run a CBC.
Call a consult.
Continue the assessment.
Tell the parents they are overreacting.
Correct answer: Continue the assessment An infant's body temperature typically runs a little high. 104 degrees F is considered
a fever. Temperature can fluctuate with individuals, so it is important to complete the assessment.
Question 9
Which of the following theorists developed a model to explain why healthy people do or do not take advantage of screening
programs?
Bandura
Becker
Maslow
Prochashka & DiClemente
Correct answer:
Becker
Becker developed the health belief model which is used to explain why healthy people do or do not take advantage of
screening programs. It involves variables such as perceptions of susceptibility and seriousness of a disease, benefits of
treatment, perceived barriers to change, and expectations of efficacy.
Question 10
Which of the following is NOT one of the basic requirements/allowances of the HIPAA privacy rule?
Providers releasing patient information for marketing purposes must explain to the patient how the information will be used,
to whom it will be disclosed, and the time frame.
Providers and their staff are restricted to conveying the “minimum necessary information” about patients.
Providers may disclose health information to oversight agencies.
In general, providers may disclose psychotherapy notes without patient consent.
Correct answer:
In general, providers may disclose psychotherapy notes without patient consent.
There are special rules for psychotherapy notes. In general, patient authorization is required in order to disclose psychotherapy
notes to carry out treatment, payment or healthcare operations
Question 11
The most common joint disease in North America is which of the following?
osteoarthritis
bursitis
rheumatoid arthritis
meniscal tear
Correct answer:
osteoarthritis
Osteoarthritis is the most common joint disease in North America. It is a degenerative condition that manifests without
systemic manifestations or acute inflammation. The most problematic joint involvement is in the hip and knee.
Question 12
Participants in a research study who do not have the disease or condition that is being studied, but who are included for
comparison are:
case series
case subjects
cross sectionals
controls
263
The Correct answer is:
Controls
Controls are commonly used in many different types of research studies. Additionally, case series refer to an observational
study where patients with interesting characteristics are studied. Case subject is another term for participants. Cross sectionals
are types of observational studies where a particular characteristic is studied at one time rather than over a period of time.
Question 13
An 81 year old patient with gastropathy presents to the NP with pain. The patient states at one time, earlier in life, she was
given an NSAID for pain and asked if she could have it again. The NP said, "Unfortunately, due to your gastropathy condition,
I am unable to offer NSAIDs as an option." At what risk would the NP be putting the patient if she prescribed an NSAID?
Decreased effectiveness of diuretics.
Fluid retention.
Hypoxia.
Ulcer and bleeding risk.
Correct answer: Ulcer and bleeding risk Prescribing medications to patients with various diseases could put the patient at risk
for additional conditions. For example, a patient with HTN cannot be prescribed NSAIDs because they may cause the
decreased effectiveness of diuretics.
Question 14
Which of the following is one of the factors in Bowen’s Family System Theory?
adequacy of growth
changes in social roles, such as children caring for parents
connecting one’s past family experiences with current behaviors
confrontation with death
Correct answer:
connecting one’s past family experiences with current behaviors
Part of Bowen’s Family System Theory is connecting one’s past family experiences with current behaviors. Also part of the
theory are: identifying multiple factors interacting across time that influence family functioning; and identifying the
interactions among biological, genetic, psychological and sociological factors that influence human behaviors.
Question 15
When you say something to a patient such as, “Let me be sure that I understand what you are saying,” this is which of the
following forms of therapeutic communication?
validation
caring
active listening
honesty
Correct answer:
validation
Validation is listening to the patient and responding congruently in order to be sure that you have the same understanding as
the patient. Active listening is hearing and interpreting language, noticing nonverbal and paraverbal enhancements, and
identifying underlying feelings.
Question 16
You are treating a 64-year-old man who has the following PSA levels. What can be concluded about the following annual
readings?
Year 1: 3.8 ng/mL Year 2: 4.5 ng/mL Year 3: 5.5 ng/mL
There is a steady increase that is worrisome.
There is a steady increase but it is not worrisome.
They are all within normal range.
None are within normal range.
There is a steady increase that is worrisome
The prostate gland produces an antigen called prostate-specific antigen (PSA). The PSA test detects the amount of PSA in the
patient’s blood. Typically, the normal range for the PSA level is below 4 ng/mL in an elderly male and below 2.5 ng/mL in a
264
younger male. In the case of the 64-year-old patient, his PSA level during year 1 was 3.8 ng/mL, which is in the normal range.
However, starting at year 2, the patient’s PSA level increased too 4.5 ng/mL, which is outside of the normal range. Then, it
elevated to 5.5 ng/mL in year 3, demonstrating his PSA level is steadily increasing outside of the normal range of 4 ng/mL.
This elevation in PSA levels is a sign that some condition is causing the patient’s PSA level to rise, such as prostatitis, benign
prostatic hyperplasia (BPH) or even prostate cancer, for instance. Therefore, the patient should be referred for further testing,
including a referral to an urologist for a prostate biopsy to rule out prostate cancer. Statistics reveal that about two thirds of
elderly male individual’s develop prostate cancer, when their PSA levels are elevated more than 10 ng/ mL. Further, statistics
suggest that about 25% of males who have elevated PSA levels between 4 to 10 ng/mL have a condition contributing to the
increase in the PSA.
Question 17
A murmur is suspected in an elderly patient. As soon as the stethoscope is placed on the patient's chest, a quiet sound is heard.
Which of the following grades is this type of murmur?
III/VI.
IV/VI.
V/VI.
II/VI.
Correct answer: II/VI Each of the preceding murmur grades is defined below: III/VI - Moderately loud IV/VI - Loud with
palpable thrill V/VI - Very loud and heard with the stethoscope partially off the chest; palpable thrill II/VI - Quiet but heard as
soon as the stethoscope is placed on the chest
Question 18
Which of the following is likely to cause an attack of gout?
alcohol binges
fasting
low-calorie diets
all of the above
Correct answer:
all of the above
All of the first three choices are likely to cause an attack of gout. Generous fluid intake is important in gout patients as is a low
purine diet.
Question 19
The nurse practitioner will need to be aware of infectious diseases and the concerns regarding the recognition and management
of them. Which of the following is important regarding the professional role related to infectious disease recognition and
management?
Be alert to endemic diseases.
Be alert to diseases that may be the result of bioterrorism.
Know the local health department guidelines for reporting communicable diseases.
All of the above
Correct answer:
All of the above.
All of the first three choices are important regarding the professional role related to infectious disease recognition and
management. They are all items for a nurse practitioner to be aware of.
Question 20
Which of the following statements about acute lymphocytic leukemia is least accurate?
Peak incidence is at 8 – 12 years of age.
Males are affected more than females by a 1.3 to 1 ratio.
ALL accounts for 75% to 80% of all childhood leukemias.
There are approximately 3500 new cases each year in the United States.
Correct answer:
Peak incidence is at 8 – 12 years of age.
This is the least accurate statement. Peak incidence is at 3 – 5 years of age. ALL is an abnormal proliferation of immature
lymphocytes in the bone marrow. There are approximately 4 cases per 100,000 children every year.
Question 21
265
Most of the drugs used in pregnancy are Category B drugs. The FNP has a patient in the third trimester of pregnancy. Which
of the following drugs is a category C drug that should not be prescribed for this patient?
insulin
Colace
Sulfa drugs
thyroid hormone
Correct answer:
Sulfa drugs
Sulfa drugs are considered Category C drugs in the third trimester because of increased risk of hyperbilirubinemia. NSAIDs
are also contraindicated in the third trimester because they block prostaglandins.
Question 22
When prescribing oral antibiotics to a female patient, which of the following should NOT be part of patient education?
The patient should be advised that antibiotics can interfere with oral contraceptives.
The patient should be advised that oral antibiotics can increase the risk of yeast infection.
The patient should be advised to stop taking oral contraceptives.
The patient should be advised to use a backup form of birth control.
Correct Answer: The patient should be advised to stop taking oral contraceptives The patient should be advised to continue
taking her oral contraceptives, but that the antibiotics may interfere with them. She should use a back-up form of birth control
while on the antibiotics and for a short time after finishing her dose.
Question 23
Which of the following is a voluntary process that indicates that the clinician has met some predetermined standard for
specialization?
licensure
certification
education
none of the above
Correct answer:
certification
Certification is a voluntary process. It does not grant the individual any legal authority to practice. Its primary purpose is to
denote that the person holding the certification has met a high standard for competency in a specialty area of nursing practice.
Question 24
You are treating a 24-year-old man with infectious mononucleosis with tonsillar hypertrophy, exudative pharyngitis, difficulty
swallowing, and a patent airway. You know that you should prescribe:
prednisone
acyclovir
amoxicillin
ibuprofen
The Correct answer is:
Prednisone
Infectious mononucleosis is an acute systemic viral illness usually caused by Epstein-Barr virus, a DNA herpes virus that
typically enters the body via oropharyngeal secretions and infects B lymphocytes. Treatment includes a corticosteroid, such as
prednisone 40 to 60 mg/day for 3 days.
Question 25
A 22-year-old female is in the clinic. She has a newborn and is breastfeeding. When she inquires about birth control, the nurse
practitioner understands that the best choice for this patient is:
triphasic preparation
1/35 preparation
1/50 preparation
progestin-only preparation
266
The Correct answer is:
Progestin-only preparation
Estrogen inhibits milk production and progestin-only preparations are ideal for the breastfeeding mother because they do not
contain estrogen. Without estrogen, they pose no threat to milk production.
Question 26
A herpes zoster infection of which cranial nerve can result in blindness?
CN 4
CN 5
CN 6
CN 7
Correct answer:
CN 5
CN 5 is the cranial nerve associated with ophthalmic function. If this nerve is infected with the herpes zoster organism it can
result in blindness
Question 27
Asking the patient to slide the tongue from side to side in an exam tests which nerve?
Vagus.
Trigeminal.
Hypoglossal.
Trochlear.
Correct answer: Hypoglossal The hypoglossal nerve controls the tongue. Listening to annunciation, looking at the tongue, and
having the patient move it from side to side is how it is assessed. It is important to examine to tongue for any lesions that may
cause the patient to hold the tongue in a certain way.
Question 28
The mucosal lining covering the eyes is known as which of the following?
bulbar conjunctiva
buccal mucosa
palpebral conjunctiva
cataract
Correct answer:
bulbar conjunctiva
The bulbar conjunctiva is the mucosal lining covering the eyes. The palpebral conjunctiva is the mucosal lining inside eyelids.
Question 29
You have a patient who would benefit greatly from cessation of smoking. You understand that which of the following doubles
the probability that this patient will be successful?
nicotine replacement therapy
behavioral strategies
nicotine replacement therapy along with bupropion
nicotine replacement therapy along with behavioral strategies
Correct answer:
nicotine replacement therapy along with bupropion
When a smoker is ready to quit, combination therapies seem to be more effective than any method alone. Nicotine replacement
therapy and bupropion double the probability of success.
Question 30
The factor that served as the reason for initiating the role of nurse practitioner in the mid 1960s was which of the following?
There was a shortage of physicians to provide for the health care needs of underserved children.
The feminist movement pushed for the further involvement of nurses in patient care.
There was an overabundance of nurses who suggested that they are capable of doing many of the things that physicians do.
all of the above
267
Correct answer:
There was a shortage of physicians to provide for the health care needs of underserved children.
The nurse practitioner role was initiated in the mid-1960s because of a physician shortage. Dr. Henry Silva, a pediatrician, and
Dr. Loretta Ford, a nursing educator, envisioned the NP role as a means to ensure primary health care for children.
Question 31
Lead screening in children is considered which of the following?
primary prevention
secondary prevention
tertiary prevention
derivative prevention
Correct answer:
secondary prevention
Secondary prevention includes interventions at the subclinical stage, directed at early detection of the illness or problem to
reduce the severity of the disease. Lead screening, genetic testing in newborns, vision and hearing screening, and
mammography are some examples of secondary prevention.
Question 32
When checking the dentition of a child, you would expect the first permanent tooth to appear when?
5 – 6 months
12 months
2 ½ years
5 – 7 years
Correct answer:
5 – 7 years
The first permanent teeth appear at about 5 – 7 years of age (first molar). The first tooth should appear at about 5 – 6 months.
Question 33
The pathogenicity of an organism is which of the following?
its ability to invade and multiply in a susceptible host
its ability to produce disease
the severity of disease that an organism can produce
its ability to produce a lasting and effective immunity
Correct answer:
its ability to produce disease
The pathogenicity of an organism is its ability to produce disease. Infectivity is its ability to invade and multiply in a
susceptible host. Virulence is the severity of disease that an organism can produce. Immunogenicity is its ability to produce a
lasting and effective immunity.
Question 34
Bell’s palsy is an acute paralysis of which cranial nerve?
CN III
CN V
CN VII
CN XII
Correct answer:
CN VII
Bell’s palsy is an acute paralysis of CN VII. This nerve is the facial nerve. Because Bell’s palsy can be a complication of
Lyme disease, appropriate antibody testing should be obtained in a patient presenting with Bell’s palsy.
268
Question 35
The subclinical stage in the course of a disease is which of the following?
stage of susceptibility
stage of presymptomatic disease
stage of clinical disease
stage of disability
Correct answer:
stage of presymptomatic disease
The course of disease development, expression, and progression in a person over time involves several stages. The subclinical
stage is the stage of presymptomatic disease.
Question 36
Which of the following concepts about family functioning and structure is least accurate?
The incidence of single parent families, childhood poverty and homelessness has decreased in recent years.
Family functioning is more directly related to healthy growth and development than is family structure.
One of the components of family functioning is the provision of financial and emotional resources.
There has been an increase in the rate of divorce and remarriage in recent years.
Correct answer:
The incidence of single parent families, childhood poverty and homelessness has decreased in recent years.
This statement is not accurate. The incidence of single parent families, childhood poverty and homelessness has increased in
recent years.
Question 37
You have done a lipid profile for an 18-year-old female patient. You would expect that her normal triglyceride level will be in
which of the following ranges?
30 – 102
31 – 104
33 – 108
40 – 136
Correct answer:
40 – 136
This is the normal triglyceride level for a female between 15 and 19 years of age. For this same patient, cholesterol should be
between 124 and 209; LDL between 61 and 141; and HDL between 36 and 76.
Question 38
The NP does not believe it is best to prescribe Halcion to her 70 year old patient. Taking Halcion would MOST likely have
what adverse effect in a patient of this age?
Impairment of psychomotor performance.
Confusion.
Depression.
Fatigue.
Correct answer: Impairment of psychomotor performance Various medications are likely to cause greater adverse reactions in
the elderly population. Impairment of psychomotor performance and sedation are common in the elderly with Halcion use
Question 39
With a 55-year-old African-American male who has sustained an injury at the fifth thoracic vertebra (T5), the nurse
practitioner understands that he will most likely not be able to:
achieve lower body strength and coordination for walking
have adequate upper body strength to drive a car
maintain the upper body coordination required for eating
perform coordinated movements with his hands, such as writing
The Correct answer is:
Achieve lower body strength and coordination for walking
T5 injuries do not affect the coordination or capacity of the upper body, arms, and hands, as the lower body is paralyzed. The
client should be able to do all the activities listed except walk.
269
Question 40
Which of the following statements about otitis media with effusion is least accurate?
Patients typically have a sense of fullness in the ear.
The patient will have a popping or crackling sound in the ear with yawning.
Antibiotic therapy is indicated.
Symptoms are variable ranging from none to severe.
Correct answer:
Antibiotic therapy is indicated.
This is not accurate. Antibiotic therapy is not routinely indicated for serous otitis media. Antibiotics are necessary for acute
otitis media.
Question 41
Which of the following types of murmurs in children is NOT an innocent murmur?
pulmonary flow murmur of the newborn
carotid bruit
systolic regurgitant murmur
venous hum
Correct answer:
systolic regurgitant murmur
A systolic regurgitant murmur begins with S1 and usually lasts throughout systole. It is caused by blood flow from a chamber
that is at a higher pressure throughout systole than the receiving chamber.
Question 42
In the management of Parkinson Disease (PD), surgical intervention such as deep brain stimulation surgery can be helpful to
manage disease-related symptoms:
that are related to memory loss
to ease the tremors and bradykinesia
when medication therapy is not tolerated or helpful
as a last resort when all other options have been exhausted
The Correct answer is:
When medication therapy is not tolerated or helpful
Deep brain stimulation surgery for Parkinson Disease is helpful in making the "off" state more like movement in the "on"
state, and is helpful in the reduction of levodopa-induced dyskinesias. As with other therapies, expert consolation should be
sought, and all options should be thoroughly discussed with the patient before pursuing surgical intervention.
Question 43
A 20-year-old college student is in the clinic with complaints of weakness. She is 15% below ideal body weight. She reports
doing well in her classes, drinking alcohol every night and coffee throughout the day. She is bradycardic and gets dizzy when
she stands up. What other findings may be seen in this patient?
sleep apnea
amenorrhea
hypertension
mitral regurgitation
The Correct answer is:
Amenorrhea
This patient has anorexia nervosa and is far below ideal body weight. She exhibits evidence of poor nutrition and health. More
than 90% of patients with anorexia are amenorrheic. Hypotension is more common than hypertension and many patients with
anorexia also exhibit mitral valve prolapse, not regurgitation.
Question 44
A patient at high risk refuses a DRE from a female nurse. What is the best question to ask?
Do you understand the seriousness of prostate cancer?
Why don't you want to take care of yourself?
270
Would you allow a male to examine you?
Are you afraid of the exam?
Correct answer: Would you allow a male to examine you? Digital rectal exams (DRE) can make patient uncomfortable. Some
patients are uncomfortable with a women performing the procedure. Likewise, some men are not comfortable with men
performing the exam. It is important to always consider the patient's comfort.
Question 45
A child with suppurative conjunctivitis is in the office for treatment. The nurse practitioner has knowledge that all of the
following ophthalmic preparations can be used to cure this EXCEPT:
ciprofloxacin
penicillin
bacitracin-polymyxin B
erythromycin
The Correct answer is:
Penicillin
A patient with a presumptive diagnosis of suppurative conjunctivitis requires antimicrobial therapy. Penicillin is not in an
ophthalmic form and is not indicated as an oral or injectable form.
Question 46
Your patient has a burn on his right thigh that has a white and leathery skin texture. This is consistent with which of the
following types of burn?
third degree
second degree
first degree
partial thickness
Correct answer:
third degree
The pain of a third degree burn may be minimal but the burns are usually surrounded by areas of painful first- and seconddegree burns. The skin is usually white and leathery.
Question 47
A female patient is in the office for her annual gynecological exam and she is inquiring about breast self-exam (BSE). Since
she has bilateral breast implants, she "does not know what to feel for." The most appropriate response would be:
Acknowledge the difficulty of breast self-exam with implants.
Explain the need for regular mammograms for patients with implants.
Suggest she involve her sexual partner in assessing her breasts.
Review the steps of breast self-exam until she feels comfortable with this process.
The Correct answer is:
Review the steps of breast self-exam until she feels comfortable with this process
This patient should be informed about the normal feel of implants and become knowledgeable about the normal feel of her
breasts. Mammography is not a substitute for breast self-exam. The remaining answer choices are good suggestions, but not
the most appropriate.
Question 48
Which of the following signs and symptoms has the lowest likelihood of representing acute coronary syndrome secondary to
coronary artery disease?
chest discomfort reproduced by palpation
pulmonary edema
left arm pain or discomfort
transient mitral regurgitation murmur
Correct answer:
chest discomfort reproduced by palpation
Chest discomfort reproduced by palpation without any of the high or intermediate-likelihood features accompanying it has the
lowest likelihood of representing acute coronary syndrome secondary to coronary artery disease. The other choices represent
signs and symptoms with a high likelihood of representing acute coronary syndrome secondary to coronary artery disease.
Question 49
271
The following statement is TRUE regarding reimbursement for nurse practitioners:
As a salaried employee, the nurse practitioner may be credited to the collaborating physicians, giving her a "ghost" provider
status.
Nurse practitioners are reimbursed for their services under all plans except federally funded school-based clinics.
The nurse practitioner has no restrictions by the Health Care Financing Agency.
There is no risk for fraud and abuse charges with state and national policy for nurse practitioner billing procedures.
The Correct answer is:
As a salaried employee, the nurse practitioner may be credited to the collaborating physicians, giving her a "ghost" provider
status
Nurse practitioners are reimbursed for their services provided at federally funded school-based clinics. The nurse practitioner
has considerable restrictions by the Health Care Financing Agency and there is much risk for fraud and abuse when dealing
with incorrect billing for providers.
Question 50
A 65-year-old female patient comes to the clinic complaining of bowel and bladder incontinence and a numbness of the
buttocks area. The FNP might suspect that his patient has which of the following diseases/conditions?
supraspinatus tendonitis
osteoarthritis
rheumatoid arthritis
cauda equina syndrome
Correct answer:
cauda equina syndrome
This condition involves acute pressure on a sacral nerve root that results in inflammatory and ischemic changes to the nerve. It
is considered a surgical emergency since the patient needs sacral spine decompression to avoid permanent nerve damage. This
patient should be referred to the ER.
Question 51
When assessing for an urinary tract infection in a geriatric patient, it is important for the nurse practitioner to know:
There is always a fever present.
It is an uncommon finding in catheterized patients.
Elderly people may or may not demonstrate symptoms other than mental status change with this type of infection.
Dysfunctional voiding pattern or infrequent voiding are not contributing factors to the development of a urinary tract
infection.
The Correct answer is:
Elderly people may or may not demonstrate symptoms other than mental status change with this type of infection
Geriatric patients with a urinary tract infection usually do not have a fever. It is common in patients with catheters and
dysfunctional voiding pattern and infrequent voiding are contributing factors in the development of a urinary tract infection.
Question 52
Margaret Mahler developed the neo-Freudian theory of psychological birth of the infant. Which of the following phases of
Mahler’s “psychological birth” takes place at about 24 – 36 months?
consolidation
rapprochement
differentiation and practicing
symbiosis
Correct answer:
consolidation
According to Mahler consolidation is part of the separation-individuation phase that takes place from 6 to 36 months of age.
Consolidation takes place from 24 – 36 months of age. It is defined by the increased ability to cope with separations through
symbolic play.
Question 53
Abortive treatment for migraine headaches would include all of the following EXCEPT:
Cafergot
NSAIDs
272
Imitrex
Inderal
Correct answer:
Inderal
All of the choices are abortive treatments for migraine headaches except this one. Inderal is a prophylactic treatment taken
daily or BID. Tricyclic antidepressants such as Elavil might also be used as a prophylactic treatment.
Question 54
Which of the following is a precursor lesion of squamous cell carcinoma?
actinic keratoses
Stevens-Johnson syndrome
erythrema multiforme
erythema migrans
Correct answer:
actinic keratoses
Actinic keratoses are precursor lesions of squamous cell carcinoma. They appear as dry, red lesions with a rough texture and
are usually located in sun-exposed areas of skin such as the cheeks, nose, face, neck, arms and back. They are more common
in light-skinned individuals.
Question 55
The FNP is instructing a patient with gout about the high-purine foods to avoid. She would tell him to avoid all but which of
the following foods?
beans
pasta
scallops
asparagus
Correct answer:
pasta
Foods high in purines should be avoided by those who suffer gout attacks. Pasta is low in purines, not high. All of the other
choices are high purine foods along with others that include: organ meats, game meats, spinach and more.
Question 56
Which of the following is NOT an absolute contraindication to postmenopausal estrogen therapy?
seizure disorder
unexplained vaginal bleeding
acute liver disease
neruo-ophthalmologic vascular disease
Correct answer:
seizure disorder
All of the choices are absolute contraindications to postmenopausal estrogen therapy except seizure disorders. In the case of
seizure disorders, estrogen therapy can be used with caution, considering if the benefit outweighs the risk.
Question 57
A 30-year-old female heterosexual patient who has had a mastectomy tells the FNP that she feels mutilated and “less of a
woman” because of the operation. Which of the following responses is most appropriate?
Many women have had mastectomies and are every bit the woman they were before the operation. You are too.
Don’t think about the operation; just be happy that you are on the road to recovery.
You should be happy that you’re alive; not worrying about how you look.
You’re feeling like the mastectomy has affected your body image in a negative way and that maybe men will not find you so
attractive.
Correct answer:
You’re feeling like the mastectomy has affected your body image in a negative way and that maybe men will not find you so
attractive.
This is a therapeutic response. It is restating and clarifying what the patient has said. The other responses are barriers to
effective therapeutic communication.
273
Question 58
If your patient has Class II heart failure as set out in the New York Heart Association (NYHA) functional classes for heart
failure, which of the following symptoms is likely?
no dyspnea with exertion
dyspnea with maximal exertion
dyspnea with minimal exertion
dyspnea at rest
Correct answer:
dyspnea with maximal exertion
The NYHA ranks heart failure into four classes. Dyspnea with maximal exertion is ranked as Class II heart failure.
Question 59
The influenza vaccine should not be given to a person with a history of anaphylactic reaction to which of the following?
baker's yeast
neomycin
streptomycin
egg
Correct answer:
egg
Persons with a history of anaphylactic reaction to eggs should not be given the influenza vaccine, either nasal spray or
injected. A person with a history of anaphylactic reaction to baker’s yeast should not be given the Hepatitis B vaccine. A
person with a history of anaphylactic reaction to neomycin should not be given the IPV, MMR or varicella vaccine. A person
with a history of anaphylactic reaction to streptomycin should not be given the IPV or smallpox vaccine.
Question 60
The theory that holds that nursing care must be congruent with the client’s culture is the theory of which of the following?
Watson
Leininger
Orem
Roy
Correct answer:
Leininger
Madeline Leininger’s theory of cultural care diversity and universality states that nursing care must be congruent with the
client’s culture. To achieve congruence, the nurse functions within the areas of cultural care preservation, cultural care
negotiation, and cultural care repatterning.
Question 61
“Healthcare providers” under HIPAA include all but which of the following?
skilled nursing facilities
retirement homes
home health agencies
hospice programs
Correct answer:
retirement homes
Retirement homes are not considered “healthcare providers” under HIPAA. “Healthcare providers” under HIPAA include:
hospitals, skilled nursing facilities, comprehensive outpatient rehabilitation facilities, home health agencies, hospice programs,
nurse practitioners, certified nurse-midwives, clinical nurse specialists, psychologists, clinical social workers, certified
registered nurse anesthetists and physicians and physician assistants.
Question 62
274
Organized creation of beneficial change to attain unprecedented levels of performance is which of the following?
risk management
quality improvement
quality assurance
none of the above
Correct answer:
quality improvement
Quality improvement is the organized creation of beneficial change to attain unprecedented levels of performance. It differs
from quality assurance in that it is continuous rather than episodic
Question 63
Which of the following statements about structural functional theory of families is the least accurate?
Rank order within families is not a component of structure.
Families are social systems.
In optimally functioning families, members take on predictable roles.
Disease or ill health can interfere with the family’s ability to carry out its internal functions.
Correct answer:
Rank order within families is not a component of structure.
All of the statements are accurate except this one. Internal family subsystems function as a microcosm of society, reflecting
the larger sphere of human needs. Rank order within families is a component of structure, such as the ordering of children by
birth in the family.
Question 64
What is the lifetime risk to the average American man of having latent prostate cancer?
10%
3%
67%
40%
Correct answer:
40%
The average American man has a 40% lifetime risk of latent prostate cancer. He also has an approximate 10% risk of clinically
significant disease and an approximate 3% risk of dying of prostate cancer.
Question 65
An adolescent female with breast budding and sparse straight pigmented pubic hair along the medial border of the labia is at
what Tanner stage of sexual maturity?
Stage I
Stage II
Stage III
Stage IV
The Correct answer is:
Stage II
The five stages of sexual maturity for both males and females is called Tanner Staging. Stage II is where there is straight,
pigmented pubic hair and breast budding. The areolar size increases during the 2nd stage as well
Question 66
In relation to structural functional theory, which of the following is NOT an example of a suprasystem?
relationships with family
relationships with friends
relationships with religious organizations
relationships with the healthcare system
275
The Correct answer is:
Relationships with family
A suprasystem forms OUTSIDE of the family and reflect functional needs that are not met within the family. Examples of a
suprasystem include relationships with all of the following: friends, religious organizations, the healthcare system, schools,
teachers, civic organizations and other groups.
Question 67
A 17-year-old with nodulocystic acne is employed as a fry cook at a restaurant. What would you do to help him manage his
acne?
benzoyl peroxide plus erythromycin
refer to a dermatologist for possible prescription of isotretinoin (Accutane)
a change in occupation
Retin-A plus minocycline
Correct answer:
refer to a dermatologist for possible prescription of isotretinoin (Accutane)
Nodulocystic acne is the most severe form of acne vulgaris characterized by nodules and cysts. These can be palpated under
the skin. Topical agents are not effective as a treatment and occupation is irrelevant. Referral to a dermatologist is indicated.
Question 68
You are doing a well-child examination and the child is 3-years-old. The nurse practitioner has an understanding that head
circumference should be measured up until what age?
12 months
15 months
24 months
36 months
The Correct answer is:
24 months
Head circumference is routinely measured at each periodic well-child visit until he has reached 2 years of age. During the first
24 months of life, the head grows most quickly, and its growth should be measured for adequacy. This should be recorded on a
growth chart so that changes can be evaluated and appropriate growth can be determined. The CDC charts show head growth
up to 36 months, but the standard recommendations for routine measurements generally specify the first 24 months.
Question 69
An 18-year-old female comes to the clinic complaining of a sore throat and fatigue that has persisted for several months. She
also has some upper abdominal pain. The FNP’s examination reveals enlarged cervical nodes, erythema of the pharynx and
red tonsils. The most likely diagnosis is:
rhinitis medicamentosa
acute mononucleosis
acute sinusitis
allergic rhinitis
Correct answer:
acute mononucleosis
Mononucleosis is an infection by the Epstein-Barr virus that is transmitted through saliva by intimate oral contact. The classic
triad to look for is pharyngitis, cervical lymphadenopathy and fatigue.
Question 70
Which of the following generalized allergic reactions is considered the most serious?
urticaria
angioedema
anaphylaxis
extravascular lesions
Correct answer:
anaphylaxis
Anaphylaxis is an acute systemic reaction manifested by sudden onset of pruritis, generalized flush, urticaria, respiratory
distress, and vascular collapse. It results from an antigen exposure in a sensitized person.
276
Question 71
You are conducting an assessment with a patient who reports heavy menstrual bleeding and pelvic pain. A pelvic exam reveals
that her uterus is larger than normal. She is not pregnant. Which of the following procedures would you order to confirm a
diagnosis of uterine fibroids?
Uterine biopsy.
External sonogram.
Hysterosonogram.
Pelvic X-ray.
Correct Answer: Hysterosonogram This procedure, in which the uterus is filled with saline and a transvaginal pelvic
ultrasound is performed, is often used to diagnose uterine fibroids. If the patient reports other complaints, such as bowel or
urinary problems, she may require other tests to rule out other causes.
Question 72
All of the following statements about Kurt Lewin’s model of change management are accurate EXCEPT:
The first stage of the model is to prepare for a change.
The second stage is all about unfreezing and getting motivated for change.
Change is not an event but a process.
The final stage of the model is establishing stability.
Correct answer:
The second stage is all about unfreezing and getting motivated for change.
This is not an accurate statement. The second stage (change/transition) is about making the changes that are needed after the
first stage of unfreezing.
Question 73
Risk factors for oral cancer include all of the following EXCEPT:
recent trauma to the mouth
smoking
alcohol abuse
human papillomavirus type 16
Correct answer:
recent trauma to the mouth
This is not a risk factor for oral cancer. The risk factors include advancing age, tobacco and alcohol abuse and most recently,
they include chronic infection with human papillomavirus type 16.
Question 74
A 34-year-old female with multiple sclerosis (MS) is in the office. She is now under your care after relocating from another
state to live with relatives who can help her. The nurse practitioner understands that which of the following is NOT part of her
management plan:
Physical and occupational therapy.
Complex treatment regimen must be coordinated with a neurologist.
Acute exacerbations should be treated with prednisone 60-80 mg/day for 1 week, taper over 2-3 weeks.
Corticosteroids are often used for maintenance.
The Correct answer is:
Corticosteroids are often used for maintenance
Corticosteroids are only for acute exacerbations, not for maintenance. The remaining answer choices are treatment options for
this condition.
Question 75
Which of the following is NOT true of the plan of care for a pregnant patient who has been diagnosed with trichomoniasis?
The patient should not take Flagyl under any circumstances.
The patient may be prescribed Flagyl if she is past the first trimester of pregnancy.
The patient does not need to be immediately treated if she does not display symptoms.
The patient should be educated that the infection may lead to a lower birth rate.
277
Correct Answer: The patient should not take Flagyl under any circumstances Many healthcare professionals feel that patients
who are in the first trimester of pregnancy should not take Flagyl for trichomoniasis. However, patients past this point may
take Flagyl. Patients who are asymptomatic do not need to be treated, but those with symptoms should be. Trichomoniasis can
lead to low birth weight in babies born to infected mothers.
Question 76
The Rinne test is part of the evaluation of which of the following cranial nerves?
CN IX
CN VIII
CN III
CN I
Correct answer:
CN VIII
CN VIII is the auditory or vestibulocochlear nerve. When this nerve does not function properly, hearing or balance is
impaired. The Rinne test is part of the evaluation of CN VIII.
Question 77
Of the following qualitative designs which one uses interviews and inductive analysis to seek understanding of lived
experience?
historical
grounded theory
phenomenologic
experimental
Correct answer:
phenomenologic
There are six common qualitative designs for research: phenomenologic, ethnographic, grounded theory, historical, random
sample, and non-random sample. Phenomenologic research uses interviews and inductive analysis to seek understanding of
lived experience.
Question 78
A mother brings her 8-month-old child to the FNP because she thinks that the child is behind in developmental milestones.
The FNP tells her that an 8-month-old child should typically be doing all but which of the following?
sitting without support
placing cubes in a cup
making hand-to-hand transfers
imitating “bye-bye”
Correct answer:
placing cubes in a cup
The 8-month-old child should be doing all of the activities in the choices except this one. Placing cubes in a cup is something
that a 12 – 15-month-old child would do. Other milestones for the 6 – 8-month-old child are having stranger and separation
anxiety, pulling feet into the mouth, closing lips in response to dislike of food, rolling back-to-stomach and stomach-to back,
chaining together syllables such as “dada,” recognizing “no,” and coughing or snorting to attract attention.
Question 79
At what age should a healthy older adult be vaccinated with the pneumovax vaccine?
60 years old
65 years old
55 years old
70 years old
Correct answer:
65 years old
You should start vaccinating healthy older adults at the age of 65 years. The patient only needs one dose per lifetime if he or
she is healthy.
278
Question 80
You are managing a 74-year-old patient who has osteoarthritis and chronic pain. Which of the following medications increases
the risk of a gastrointestinal related ulceration?
warfarin
celecoxib
pravastatin
thiazide diuretic
The Correct answer is:
Celecoxib
Celecoxib is a nonsteroidal anti-inflammatory drug (NSAID). Further, NSAIDs increase the risk of upper and lower
gastrointestinal ulcerations. Warfarin does not increase the risk of ulceration, but if one occurs, the risk of bleeding is
increased. Statins and diuretics are not considered ulcerogenic.
Question 81
If there is no reliable history of chicken pox, when can the varicella vaccine be given?
after 12 years of age
after 5 years of age
after 1 year of age
after 3 months of age
Correct answer:
after 1 year of age
If there is no reliable history of chicken pox, the varicella vaccine can be given after 1 year of age. Children less than 12 years
of age need only one dose. Older children need two doses one month apart.
Question 82
When a nurse practitioner seeks to obtain clinical privileges with a healthcare organization the process is known as which of
the following?
certification
licensure
credentialing
collaborative agreement
credentialing
Credentialing refers to documentation that the NP has met the agency’s specified criteria for hospital privileges. Some
institutions require proof of skill before they allow the NP to perform certain procedures.
Question 83
You are analyzing the results of a blood glucose test. The patient is an adult female. Her blood glucose reading is 60 mg/dL.
Which of the following is true of this patient's blood glucose reading?
It is far too high.
It is in the normal range.
It is slightly elevated.
It is lower than normal.
Correct Answer: Lower than normal The mean blood glucose level in adult humans is 72mg/dL. Blood glucose below
70mg/dL can be harmful and is indicative of hypoglycemia. Further diagnostic tests should be run to arrive at a correct
diagnosis.
Question 84
Which of the following types of injuries is most likely to reveal specific radiographic findings?
Jones fracture
nursemaid’s elbow
gamekeeper’s thumb
tennis elbow
Correct answer:
Jones fracture
279
Jones fractures are foot fractures. They are associated with a painful, swollen lateral foot with pain elicited on palpation of the
fifth metatarsal. Radiographic findings show this fracture.
Question 85
You have a 55-year-old African-American female urgently brought to the clinic with angioedema. It is reported the
angioedema started less than an hour ago. You notice that she is breathing without difficulty. What is the most likely cause of
this?
penicillin
metformin
ramipril
aspirin
The Correct answer is:
Ramipril
Angioedema is common in African-American patients who take ACE inhibitors. It is an unpredictable event but once it occurs
the medication must be discontinued and never prescribed again. Swelling of the lips, face, eyelids, tongue, and larynx
characterizes angioedema. Question 86
Which of the following is NOT part of the ethical decision making process for the nurse practitioner?
Moral concepts such as advocacy, accountability, loyalty, caring, compassion, and human dignity are the foundations of
ethical behavior.
Duty to help others, beneficence, is a foundational component of ethical behavior.
Ethical behavior incorporates respect for the individual and his or her autonomy.
The ethical behavior of nurses has been defined for professional nursing in an American Practice Act policy statement.
The Correct answer is:
The ethical behavior of nurses has been defined for professional nursing in an American Practice Act policy statement
The ethical behavior of nurses was defined by the American Nurses Association and not the American Practice Act. The
remaining answer choices are part of ethical decision-making process of a nurse practitioner
Question 87
An infant will not smile responsively until about which of the following ages?
1 – 1 ½ months
2 – 3 ½ months
3 – 4 months
4 ½ – 5 months
Correct answer:
1 – 1 ½ months
In terms of social skills, a child of 1 – 1 ½ months should be able to smile responsively. At about 5 months of age the child
will even smile at his mirror image.
Question 88
A person with a blood pressure reading of 150/95 mm Hg is classified as which of the following?
normal
stage 2 hypertension
prehypertension
stage 1 hypertension
Correct answer:
stage 1 hypertension
Stage 1 hypertension is characterized by a systolic pressure of 140-159 or a diastolic pressure of 90-99 or both. Hypertension
is due to an increase in peripheral arterial resistance.
Question 89
There is a greater recognition of the use by patients of complementary and alternative modalities and medicines (CAM) today
than ever before. Which of the following is an inaccurate statement regarding the practice environment involving CAM?
Research suggests that 40-50% of patients are currently utilizing some form of complementary or alternative therapy.
280
There is a wealth of evidenced based research that supports CAM.
Nurse practitioners, as providers, need to learn about common CAM treatments and about how some herbal products interact
with prescription medications.
The National Center for Complementary and Alternative Medicine is a federal government's lead agency for scientific
research on CAM.
The Correct answer is:
There is a wealth of evidenced based research that supports CAM
There is a dearth of research supporting such treatment modalities
Question 90
There is a difference in minor depression and major depression. The nurse practitioner understands that this is:
the number of symptoms present
presence of suicidal ideations
the length of time symptoms have lasted
the severity of symptoms
The Correct answer is:
The number of symptoms present
Major depression is diagnosed when at least 5 symptoms out of 9 are identified by the practitioner. Minor depression is
characterized by 2 to 4 of the nine symptoms and these symptoms must be present for at least 2 weeks nearly every day.
Question 91
You are examining a patient who has a cardiac murmur. Which of the following is most likely to be found if this patient’s
murmur is associated with aortic stenosis?
Grade 1-3/6 early to mid systolic murmur
Grade 2-3/6 systolic ejection murmur
Grade 1-3/4 high-pitched blowing diastolic murmur
Grade 1-4/6 harsh systolic murmur
Correct answer:
Grade 1-4/6 harsh systolic murmur
Aortic stenosis causes a Grade 1-4/6 harsh systolic murmur. It is usually in a crescendo-decrescendo pattern and it is heard
best at the second right intercostal space.
Question 92
You have a client with GERD. Which of the following statements indicates that she has NOT understood how to manage the
condition?
I should eat smaller meals.
I should elevate the head of my bed.
I should avoid tight clothes.
I should avoid exercise.
Correct answer:
I should avoid exercise.
The person with GERD should exercise daily. She should also stop smoking and lose weight if more than 130% of ideal body
weight, among other things.
Question 93
Your 30-year-old female patient is suffering from primary dysmenorrhea. You will prescribe the treatment of choice for this
condition which is which of the following?
NSAID
acetaminophen
aspirin
magnesium
Correct answer:
NSAID
281
NSAIDs are the treatment of choice for primary dysmenorrhea. You would tell the patient to take NSAIDs 2 days or more
before onset of menses, at regular intervals around the clock until no longer necessary. You would advise her to take them
with food to avoid GI upset.
Question 94
In terms of treating seizure disorders, the FNP understands that all of the following are true in regard to using antiepileptic
drugs EXCEPT:
Utilize the least number of medications.
Begin with a combination of drugs for more rapid resolution.
Maintain a maximum level of alertness with the fewest number of seizures.
Obtain a baseline of physical status.
Correct answer:
Begin with a combination of drugs for more rapid resolution.
A clinician would not begin treatment for seizures with a combination of drugs. He or she would begin with a single drug so as
to be able to assess side effects more easily.
Question 95
You are treating a 48-year-old male who is suffering with alcoholism. The first most helpful approach for him is:
Inform the patient of the long-term health consequences of alcohol abuse.
Tell the patient to stop drinking.
Refer the patient to Alcoholics Anonymous.
Counsel the patient that alcohol abuse is a treatable disease.
The correct answer is:
Counsel the patient that alcohol abuse is a treatable disease
The first approach for the nurse practitioner is to counsel the patient and family that alcoholism is a lifelong but treatable
disease. In addition, asking about current drinking habits and associated consequences to health with each visit is important
and are appropriate next steps. Then, the remaining answer choices are also important interventions after the nurse practitioner
counsels the patient and the family regarding alcoholism as a treatable disease.
Question 96
Which of the following are elements of a broad-based risk management program?
System of contract review that helps to avoid assuming liabilities that should be borne by others.
Hazardous materials compliance program as part of a comprehensive safety and security system.
Early-warning/incident reporting program to identify elements of risk.
All of the above.
The Correct answer is:
All of the above
These and other elements combine to produce a program of systematic risk identification, analysis, treatment, and evaluation,
with the overall goal of loss prevention.
Question 97
You are managing a patient who has a cat bite. The nurse practitioner knows that which of the following medications should
be prescribed?
topical bacitracin
topical hydrocortisone 2.5%
oral amoxicillin clavulanate (Augmentin)
oral acyclovir
The Correct answer is:
Oral amoxicillin clavulanate (Augmentin)
The most common infective agents with a cat bite are Pasteurella multocida and Staphylococcus aureus. The agents of choice
for treatment are Augmentin, cefuroxime or doxycycline. The other answer choices would be ineffective and unnecessary.
Question 98
A seizure with the typical onset of 3 – 15 years that is characterized by an awake state with abnormal motor, sensory,
autonomic, or psychic behavior is which of the following?
282
simple partial or focal
myoclonic
tonic-clonic
absence
Correct answer:
simple partial or focal
Simple partial or focal seizures are characterized by an awake state with abnormal motor, sensory, autonomic, or psychic
behavior. Movement can affect anY part of the body. It can be localized or generalized.
Question 99
During an abdominal exam, what is an abnormal finding?
The liver is tender.
The spleen is not palpable.
There are bowel sounds.
There is tympany.
Correct answer: The liver is tender The liver should not be tender in a normal abdominal assessment. The other answers can be
normal in abdominal exams.
Question 100
You are testing a client suspected of having Parkinson’s disease. You support the client’s elbow with his forearm extended in
your hand and grasp his wrist, flexing the forearm. You find rigid resistance to this motion. This is considered which of the
following?
cogwheeling
Myerson’s sign
Wilson’s sign
pill rolling
Correct answer:
cogwheeling
Cogwheeling is resistance to passive movement. It is best felt in the elbow, wrist, and neck.
quiz 12 aanp
Question 1
The U.S. Food and Drug Administration (FDA) and the Department of Health and Human Services (DHHS) have regulations
governing research activities on human subjects. The principal investigator is responsible for:
securing a signed research consent form from each participant
appearing before the Institutional Review Board to present the study and secure approval to proceed with the subject
recruitment at the facility
report back to the Institutional Review Board if a subject is injured during the course of the study
all of the above
The Correct answer is:
All of the above
Each of the answer choices are all basic requirements for conducting research at healthcare facilities
Question 2
A 72 year old patient explains to the NP that she thinks she has diarrhea. Her stools are bloody and watery with mucus.
Testing shows the stools are positive for blood and leukocytes. What will MOST likely be the diagnosis?
Shigella.
Adenovirus.
Campylobacter jejuni.
Salmonella.
283
Correct answer: Campylobacter jejuni is a bacterial diarrhea that is very common in children ages 1 - 5 but can occur at any
age. Vomiting and fever are noticeable symptoms of the illness.
Question 3
Which of the following systemic corticosteroids would be considered of medium potency?
Betamethasone, 0.6 -0.75 mg
hydrocortisone, 20 mg
dexamethasone, 0.75 mg
prednisone, 5 mg
Correct answer:
prednisone, 5 mg
Prednisone, 5 mg would be considered a medium potency corticosteroid. Other medium potency corticosteroids include:
methylprednisolone, 4 mg; triamcinolone, 4 mg; and prednisolone, 5 mg.
Question 4
The nurse practitioner knows that medical management for a brown recluse spider bite includes:
Prescribe cephalexin (Keflex) 500 mg PO tid x 7 days.
Prescribe amoxicillin-clavulanate (Augmentin) 250 mg PO tid x 14 days if there are signs of infection.
Have the patient return to the clinic for follow-up in 2 weeks.
Approximate the edges of the wound together with suture.
Correct answer:
Prescribe amoxicillin-clavulanate (Augmentin) 250 mg PO tid x 14 days
Amoxicillin-clavulanate (Augmentin) is an excellent choice for empiric treatment of animal bites if there are signs of
infection. Cephalexin is not indicated due to resistant strains of Pasteurella multocida, an organism present in dog and cat
bites. An infected bite should be followed up daily until the infection clears. Open wound management is indicated, but not
suturing.
Question 5
The FNP has diagnosed a patient as having herpes keratitis. He refers the patient to an ophthalmologist. The FNP understands
that the ophthalmologist will treat the patient with which of the following?
Zovirax
steroid ophthalmic drops
support therapy only
none of the above
Zovirax
The ophthalmologist will treat the patient with Zovirax or Valtrex BID until the problem is resolved. The ophthalmologist
would not treat the patient with steroid ophthalmic drops because this would worsen the condition.
Question 6
You have an African-American male patient who is 42 years old. His older brother has been diagnosed with prostate cancer
recently, and he asks when he should begin prostate cancer screening. You would tell him which of the following?
He is too young to begin screening.
He should not worry about screening because being of the African-American race reduces his risk.
He should have begun screening when he was 21 years old.
He should begin immediately because he has a higher risk of prostate cancer.
He should begin immediately because he has a higher risk of prostate cancer.
Normally a man should begin prostate cancer screening at age 50. But since this patient has a brother who has prostate cancer
and he is also of the African-American race he has a higher risk and should begin yearly screening immediately. Age 40 is the
recommended age for high risk patients.
Question 7
In terms of billing for Medicare patients which of the following statements is incorrect?
There are no consequences for selecting an inappropriate code for a patient visit if done unintentionally.
Each CPT code has corresponding levels of required history taking, physical examination and medical decision making.
NPs are responsible for ensuring that the billing for their services matches the level of care given.
284
The consequence of selecting an inappropriate code may include loss or restriction of the NP’s license by the board of
nursing.
There are no consequences for selecting an inappropriate code for a patient visit if done unintentionally.
NP’s are expected to know how to bill correctly. Ignorance is a poor defense. There may be consequences for selecting an
inappropriate code if auditors find that more errors were made in overcoding than undercoding visits.
Question 8
A short-acting anticholinergic medication can be used alone or in combination with a short-acting beta agonist to manage
symptoms of which disease?
benign prostatic hyperplasia
tachyarrhythmias
chronic obstructive pulmonary disease
glaucoma
Chronic obstructive pulmonary disease
First line treatment for patients with chronic obstructive pulmonary disease who have intermittent symptoms of shortness of
breath is an anticholinergic alone or in combination with a beta agonist. These medications both improve lung function.
Question 9
For patients with fibromyalgia, the recommended initial dose of Lyrica is which of the following?
100 mg/day.
300 mg/day.
1000 mg/day.
150 mg/day.
Correct Answer: 150 mg/day The initial dose of Lyrica for neuropathic pain associated with fibromyalgia is 150 mg/day. This
is given in 50 mg doses, three times per day. Patients may build up to larger doses as needed, with the maximum dose not to
exceed 300 mg/day.
Question 10
Depression is a common mental disorder among the general population. Which of the following patients are more likely to
experience depression?
A patient who has had a stroke.
A patient who has lost a finger.
A patient who has a femur fracture.
A patient who has had a lumbar discectomy.
The Correct answer is:
A patient who has had a stroke
Central nervous system diseases are associated with higher rates of depression. These include stroke, dementia, multiple
sclerosis, and Parkinson's disease. Depression worsens the outcome of physical illness.
Question 11
“Tennis elbow” is a painful condition that arises as a result of injury to the extensor tendon at the lateral epicondile. Therapy in
the first 3 to 4 weeks should be conservative and includes all but which of the following?
avoidance of the precipitating activity
application of appropriate splints
arm cast
NSAIDs
Correct answer:
arm cast
An arm cast should be used only if symptoms persist. Use of an arm cast limits arm movement even further. Local
corticosteroid injection may be helpful if symptoms persist beyond 6 to 8 weeks or are particularly severe.
Question 12
The learning theory that views learning as the product of stimulus and response and is not concerned with individual internal
factors in the learner is which of the following?
behaviorist theory
285
cognitive theory
humanistic theory
affective theory
behaviorist theory
The behaviorist theory focuses on modifying the stimulus or changing the reinforcement that occurs after the response. This
theory values detailed measurable instructional objectives and immediate, positive reinforcement.
Question 13
Bowel sounds in a healthy adult patient should sound like all but the following?
Gurgling.
Bubbling.
High pitch.
Loud splashing.
Correct Answer: Loud splashing Normal bowel sounds are high-pitched, bubbling, gurgling sounds. They may vary in
intensity and volume. Loud splashing may indicate an obstruction or other condition.
Question 14
The family theory stating that family members maintain involvement in relationships on the basis of rewards and costs is
which of the following?
interactionist perspective
family ecologic perspective
family stress perspective
social exchange perspective
social exchange perspective
The social exchange perspective holds that family members maintain involvement in relationships on the basis of rewards and
costs. Other common family theories include: structural-functional; family systems; developmental perspective; interactionist
perspective; family ecologic perspective; and family stress perspective.
Question 15
The FNP has an 87-year-old patient who lost his wife three months ago and now lives alone. He tells the FNP that he is happy
that his family comes to visit him often and takes him out to dinner sometimes because he can no longer drive. This patient is
exhibiting:
dependency on others
despair at not having the ability to go anywhere without someone driving him
positive adaptation to his loss by enjoying what his family does for him
an unhealthy grief response
Correct answer:
positive adaptation to his loss by enjoying what his family does for him
This man is adjusting to the changes that have occurred as a result of the loss of his wife. The patient is not totally dependent
on others since he lives alone. He is not exhibiting any despair or unhealthy response to grief.
Question 16
You have a young patient who has a blood lead level of 14 mg/dL. You would do all of the following for this patient
EXCEPT:
Assess risk factors and rescreen the patient every 6 months.
Perform complete medical assessment with KUB and long bone X-ray examination.
Rescreen the patient every month.
none of the above
Correct answer:
Assess risk factors and rescreen the patient every 6 months.
You would assess risk factors and rescreen the patient every 6 months. You should also provide education for the family
including diet, and education about cleaning and ways to protect the child from lead hazards.
Question 17
286
A 30-year-old female comes to the clinic complaining of a small raised area on the left eyelid. The FNP diagnoses a
hordeolum. The treatment for this condition would include all of the following EXCEPT:
warm compresses to the eye for 10 minutes three to four times a day
application of erythromycin ointment to the affected lid
incision and drainage if needed
intralesion corticosteroid injection if needed
intralesion corticosteroid injection if needed
A hordeolum is also known as a stye and is usually caused by a staphylococcal infection of a hair follicle on the eyelid. The
intralesion corticosteroid injection would not be used for a stye. It is used for a chalazion if other treatment is not successful. A
chalazion is an inflammatory eyelid condition that may not involve infection but can follow hordeolum.
Question 18
When assessing a patient with acute chest pain, the most life threatening dysrhythmia experienced by a patient with an acute
MI is:
atrial fibrillation
third-degree heart block
ventricular tachycardia
ventricular fibrillation
The Correct answer is:
Ventricular fibrillation
The most life-threatening dysrhythmia is ventricular fibrillation, although the other dysrhythmias occur after a myocardial
infarction (MI). The majority of out-of-hospital deaths from a MI are caused by ventricular fibrillation.
Question 19
When counseling a menopausal woman with many risk factors for osteoporosis, the nurse practitioner knows that which of the
following are preventative measures she should be instructed on regarding osteoporosis?
smoking cessation and weight bearing exercise
optimal caloric intake and Vitamin E supplements
exercise, weight gain and a high fat diet
organic supplements and adequate Vitamin B12 intake
The Correct answer is:
Smoking cessation and weight bearing exercise
Although the mechanism is not known, research has shown that cigarette smoking accelerates skeletal bone loss. Therefore,
smoking cessation is important in the prevention of osteoporosis as well as other diseases and conditions. Exercise is
necessary at least 30 minutes, three times a week, to maintain bone density and it has been shown to decrease the incidence of
hip fractures. Walking increases hip and spine density. Further, Vitamin D and calcium intake along with smoking cessation
prevents osteoporosis.
Question 20
Principles that are evidence-based and written by expert panels and/or specialty organizations such as the American Cancer
Society are known as which of the following?
standards of practice
scope of practice
clinical guidelines
collaborative principles
Correct answer:
clinical guidelines
Clinical guidelines are written by expert panels and/or specialty organizations. They are evidence-based and act as treatment
guidelines. A few examples of diseases with treatment guidelines include hypertension, hyperlipidemia, and pneumonia.
Question 21
In Erikson’s stages of psychosocial development, failure in the stage of generativity leads to which of the following?
feelings of shame and doubt
mistrust
stagnation
role confusion
287
Correct answer:
stagnation
Erikson’s stages of psychosocial development theory maintains that how well individuals accomplish developmental tasks will
determine their success in accomplishing other tasks as they get older. Failure in the stage of generativity leads to stagnation.
Question 22
In interacting with a patient, which of the following phrases should NOT be used?
“Tell me about . . .”
“What are your thoughts . . .?”
“What are you feeling?”
“If it were me I would . . .”
Correct answer:
“If it were me I would . . .”
Some phrases should not be used in communicating with patients. “If it were me I would . . .” is one of them. Useful phrases
are those that promote therapeutic interaction and keep it open, genuine and patient-centered.
Question 23
Which of the following is NOT a conceptual model for advanced practice nursing?
Benner’s model
Shuler’s model
Calkin’s model
Ford model
Correct answer:
Ford model
There is no Ford model of NP practice; however, Loretta Ford established the first pediatric nurse practitioner program.
Question 24
A 65-year-old female has come to the clinic complaining of a scaly red rash on one of her nipples that will not go away. She
tells the FNP that it has become crusty and sometimes bleeds. Examination shows a discharge from the nipple and a small
lump. Which of the following diseases/conditions is likely to be diagnosed?
breast cancer
fibrocystic breast
Lymphogranuloma Venereum
Paget’s disease of the breast
Correct answer:
Paget’s disease of the breast
Paget’s disease of the breast is often reported by older females. The disease presents as a scaly rash resembling eczema on the
nipple or the nipple and areola that does not heal. It can eventually have crusting, ulceration and/or bleeding and is sometimes
itchy.
Question 25
You have a patient in the office with symptoms of aortic regurgitation. The nurse practitioner understands which of the
following is characteristic of this?
an acute onset of shortness of breath in the fifth or sixth decade
a long asymptomatic period with sudden death usually during exercise
long asymptomatic period followed by exercise intolerance, then dyspnea at rest
dyspnea on exertion for a long period of time before sudden cardiac death
The Correct answer is:
Long asymptomatic period followed by exercise intolerance, then dyspnea at rest
Aortic regurgitation (AR) is characterized by the patient having a long asymptomatic period with slowing activities.
Eventually, the patient develops shortness of breath. Then, left ventricle failure occurs.
Question 26
Your patient has an anxiety disorder. You will prescribe benzodiazepine therapy for this patient. Which of the following drugs
would fall into this class of medication?
Luvox
Lexapro
288
Zoloft
Klonopin
Correct answer:
Klonopin
Klonopin is a benzodiazepine therapy. All of the other drugs are SSRI/SNRI therapy (antidepressants).
Question 27
Which of the following characteristics is least associated with Still’s murmur, a vibratory murmur?
can be described as a “twanging string”
usually detected at 8 – 14 years of age
intensity may increase with fever
not accompanied by a thrill
Correct answer:
usually detected at 8 – 14 years of age
Still’s murmur is usually detected at 3 – 6 years of age. It is heard best at the middle left sternal border or between the left
lower sternal border and apex when the patient is in the supine position.
Question 28
Which of the following skin conditions affects mainly men 30 years of age and older?
nevi
melanoma
seborrheic keratosis
actinic keratosis
Correct answer:
seborrheic keratosis
Seborrheic keratosis affects mainly men 30 years of age and older. It is the proliferation of immature keratinocytes and
melanocytes. They are beige, brown, or black plaques with a velvety or warty surface.
Question 29
Which of the following statements in regard to interpreting developmental landmarks and milestones is least accurate?
Loss of developmental milestones previously achieved is a significant finding in any developmental assessment.
Early attainment of gross motor skills is not a significant indicator of advanced intellectual development.
Early attainment of gross motor skills usually precludes a diagnosis of mental retardation.
Language and fine motor skills are not indicators of intellectual development.
Correct answer:
Language and fine motor skills are not indicators of intellectual development.
This statement is inaccurate. Language and fine motor skills are sensitive indicators of intellectual development.
Question 30
As an NP you must follow certain ethical principles. If you make a decision for the patient based on the rationale that it is in
the patient’s best interest you are violating the ethical principle of:
autonomy
beneficence
paternalism
justice
Correct answer:
autonomy
Autonomy refers to the potential to be self-determining. Paternalism is the term used when the health care provider makes a
decision for a patient based on the rationale that it is in the patient’s best interest. It is a barrier to patient autonomy.
Question 31
An elderly woman is brought to the clinic by her daughter with whom she lives. The daughter tells the FNP that her mother
gets very excited, confused and sometimes even combative every evening. It is most likely that this patient is suffering from
which of the following?
289
delirium
sundowning
TIAs
none of the above
sundowning
“Sundowning” is seen in both delirium and dementia patients. Starting at dusk (or sundown) the patient becomes very agitated,
confused and combative. These symptoms resolve in the morning.
Question 32
Which of the following statements about dysphagia is least accurate?
It can be attributed to normal aging.
It may be a structural or neuromuscular problem.
It may present as difficulty swallowing solids or liquids.
It may present as chest pain.
Correct answer:
It can be attributed to normal aging.
This is the least accurate statement. Dysphagia is not attributed to normal aging. It is a difficulty in swallowing and having
food pass from the mouth down the esophagus to the stomach.
Question 33
You have a 3-year-old patient with scabies. You prescribe permethrin 5% (Elimite) for the patient. You would tell the parent
of this child all of the following in regards to using this medication EXCEPT:
Precede the treatment with a warm bath.
Apply cream to the entire body while the skin is wet from shower or bath.
Leave the cram on 8 to 12 hours and then remove by showering.
One ounce of the cream is sufficient for one application.
Correct answer:
Apply cream to the entire body while the skin is wet from shower or bath.
This is not proper procedure for application of the medicine. The child should be dried thoroughly and the skin temperature
should be cooled down before applying the cream. A single application of the cream is usually effective.
Question 34
The interval between the receipt of an infection and the maximal communicability of the host is which of the following?
generation time
immunity period
transmission rate
carrier time
Correct answer:
generation time
Generation time is the interval between receipt of infection and the maximal communicability of the host. It is applied to both
subclinical and clinical infections, and is used to describe and analyze the spread of infectious disease.
Question 35
A mother of a child with scabies is asking about the condition. The nurse practitioner knows to advise her that:
Casual contact with an infected person is likely to result in infestation.
The scabies mites can live for many weeks away from the host.
The bedding used by an infected person must be destroyed.
Close personal contact with an infected person is usually needed to contract this disease.
The Correct answer is:
Close personal contact with an infected person is usually needed to contract this disease
Scabies is a communicable skin disease caused by a host-specific mite, generally requiring close personal contact to result in
contagion. Contact with unwashed bedding and clothing from an affected person can result in the infection. The bedding can
be washed in hot water or placed in a clothes dryer to eliminate the scabies mites. The mites do not survive longer than 3 or 4
days away from a host.
Question 36
290
Of the following drugs prescribed, which is a cholinesterase inhibitor that comes in the form of a pill or syrup only?
Galantamine.
Hydrocodone.
Rivastigmine.
Donepezil.
Correct answer: Galantamine There are three cholinesterase inhibitors commonly prescribed by doctors. These are Donepezil,
Galantamine and Rivastigmine. Hydrocodone is a type of medication that treats moderate to severe pain as well as cough
Question 37
A patient who has been prescribed nifedipine for kidney stones should be advised not to take the medication with which of the
following?
Milk.
Coffee.
Food.
Grapefruit juice.
Correct Answer: Grapefruit juice Grapefruit juice interferes with the ability of the liver to break down nifedipine. Patients can
take this medication with food or without, and with other beverages.
Question 38
A patient's chief complaint is abdominal pain. What could mistakenly be associated with abdominal pain ?
Digestive upset.
Chest pain.
Headache.
Appendicitis.
Correct answer: Chest pain Chest pain can be confused with abdominal pain. When a patient presents with abdominal pain, It
is important to rule out heart conditions in the assessments.
Question 39
About how many viral illnesses per year are common in infants and toddlers?
up to 10
2
up to 15
3
Correct answer:
up to 10
Frequent (up to 10) viral illnesses per year are common in infants and toddlers. This is especially true for upper respiratory
illnesses.
Question 40
Which of the following pharmacologic agents should the nurse practitioner use in the elderly hypertensive patient who has a
new onset of congestive heart failure (CHF)?
rehabilitation heat therapy
muscle relaxants
ACE inhibitors
intensive radiation blockers
The Correct answer is:
ACE inhibitors
There are several types of medications used to treat congestive heart failure (CHF), such as beta-blockers, diuretics,
angiotensin-converting enzyme (ACE) inhibitors or angiotensin II receptor blockers. The ACE inhibitors have been shown to
prolong life in patients with congestive heart failure (CHF) by improving overall cardiac function. Rehabilitation heat therapy
and muscle relaxants are used for treating conditions other than CHF, such as osteoarthritis.
Question 41
In terms of the psychosocial development of infants, the New York Longitudinal Study (NYLS) defined nine dimensions of
temperament. Which of the following is NOT one of them?
activity
synchrony
291
approachability
mood
Correct answer:
synchrony
According to the NYLS, the nine dimensions of temperament are: activity, rhythmicity, approachability, adaptability,
intensity, threshold of arousal, mood, distractibility and attention. Synchrony is not one of them. This is part of the parentinfant interaction and attachment.
Question 42
When a pregnant woman’s uterine fundus first rises above the symphysis pubis, the woman is about how many weeks
pregnant?
12 weeks
16 weeks
20 weeks
24 weeks
Correct answer:
12 weeks
At 12 weeks the uterine fundus first rises above the symphysis pubis. Fetal heart tones are heard by Doppler by 10 – 12 weeks.
Question 43
The highest risk of Kawasaki disease is in which of the following ethnic groups?
Asian
Mediterranean
African-American
Hispanic
Correct answer:
Asian
The highest risk of Kawasaki disease is among Asian children, especially those of Japanese or Korean ethnicity. It is the
leading cause of acquired heart disease in the United States.
Question 44
You have recommended speech therapy for a stroke patient. This is which of the following types of prevention?
prophylactic
primary
secondary
tertiary
Correct answer:
tertiary
Tertiary prevention is any type of rehabilitation such as cardiac rehab, physical therapy, or speech therapy. Tertiary prevention
in general is rehabilitation and avoidance of further bodily damage
Question 45
An adult patient suffered head trauma. There is a positive reaction to the startle reflex. What type of assessment is necessary?
Dietary.
Neurological.
Cardiac.
Digestive.
Correct answer: Neurological The startle reflex is an infant reflex. In adults, it can be a sign of brain damage. A neurological
assessment is necessary.
Question 46
What reflex should not be present in a five month old at an assessment?
Moro reflex.
292
Plantar grasp.
Blinking reflex.
Stepping reflex.
Correct answer: Stepping reflex The Stepping reflex should only be apparent in the first three months. The other answers are
still normal for an infant of five months. The presence of this reflex requires investigation.
Question 47
The nurse practitioner understands that the most common class of medications used to treat a patient with systolic dysfunction
post-myocardial infarction is:
ACE inhibitors
thiazide diuretics
beta blockers
loop diuretics
The correct answer is:
ACE inhibitors
Angiotensin-converting–enzyme (ACE) inhibitors are commonly used in patients with systolic dysfunction because they
reduce morbidity and mortality. Also, ACE inhibitors improve symptoms of fatigue, shortness of breath, and exercise
intolerance. While the loop diuretics improve symptoms, they do not alter long-term prognosis with systolic dysfunction.
Further, beta blockers can potentially worsen systolic dysfunction and should not be used as solo agents in this treatment.
Question 48
Which of the following is NOT a tertiary prevention measure?
cardiac rehabilitation
Alcoholics Anonymous
pain management program
testing for STDs
Correct answer:
testing for STDs
Testing for STDs is a secondary prevention measure, i.e., one that is used for detection of disease early enough to minimize
bodily damage. Tertiary prevention is rehabilitation and avoidance of further bodily damage. Examples of tertiary prevention
include pain management programs, cardiac rehabilitation, and support groups.
Question 49
At what age would you begin cholesterol screening for someone with hereditary risk factors?
20
30
40
50
Correct answer: 20 Cholesterol screening is important. A twenty year old patient should be screened for cholesterol if there is
a family history of heart disease. Other risk factors may require early or more frequent screening.
Question 50
You have the job of monitoring patients who have certain high-cost or chronic conditions to help the hospital to best manage
the health care costs of these patients. This is known as which of the following?
case management
risk management
utilization review
accreditation
Correct answer:
case management
Case management is the process of monitoring patients who have certain high-cost or chronic conditions to help third party
payers and/or hospitals best manage the health care costs associated with the condition. Case management is very useful for
third part payers and hospitals.
Question 51
Which of the following would be considered a primary hyperlipidemia?
caused by hypothyroidism
related to medication use
293
related to fat intake
genetic form
Correct answer:
genetic form
Hyperlipidemia (also called dyslipidemia) is either primary (genetic forms) or secondary. Secondary forms are related to fat,
calorie, and alcohol intake; medication use; or caused by metabolic diseases such as hypothyroidism, diabetes mellitus or
nephrosis.
Question 52
In terms of translational science, the distribution of the intervention to health care providers or to the public by way of research
reports or clinical guidelines is known as which of the following?
early translation
dissemination
late translation
clinical trial
Correct answer:
dissemination
The first stage of translational research is referred to as early translation. If this stage proves beneficial it can move into late
translation and then into dissemination, which involves the broader distribution of the intervention (e.g., new drug, medical
device, educational materials) to health care providers or to the public, by way of research reports, clinical guidelines,
professional education and training, or by informing changes in policy.
Question 53
Which of the following drugs has been demonstrated to have use in preventing dementia?
Aricept
Exelon
Cognex
none of the above
Correct answer:
none of the above
All of the drugs mentioned in the first three choices can be used either alone or together to have a minor and time-limited
effect in dementia care. The use of these products and others to prevent dementia are currently not supported.
Question 54
The person designated by a patient in a power of attorney for healthcare to make his future health care decisions in the event
that he becomes mentally incompetent is known as which of the following?
next of kin
guardian ad litem
proxy
all of the above
Correct answer:
proxy
A proxy is the person designated by a patient to make his future health care decisions via a power of attorney for health care,
in the event that he becomes mentally incompetent. It may or may not be the patient’s next of kin.
Question 55
Most men with a gonococcal infection will have:
dysuria
no symptoms
blood-tinged semen
proctitis
Correct answer:
no symptoms
Most men with gonococcal infection have no symptoms. In women, presentation typically includes dysuria with a milky to
purulent, occasionally blood-tinged, vaginal discharge.
294
Question 56
Which of the following lesions in the older adult are precancerous skin lesions that are small rough, pink-to-reddish and are
located in sun-exposed areas?
actinic keratoses
seborrheic keratoses
senile purpura
lentigines
Correct answer:
actinic keratoses
Actinic keratoses are small rough pink-to reddish lesions that do not heal. They are located in sun-exposed areas such as the
cheeks, nose, back of neck, arms, chest, etc. They are more common in light-skinned individuals. They are precancerous skin
lesions.
Question 57
Which of the following gastrointestinal diseases/conditions can be described as a gastrinoma located on the pancreas or the
stomach which secretes gastrin, which stimulates high levels of acid production in the stomach resulting in development of
multiple and severe ulcers in the stomach and duodenum?
Crohn’s disease
Zollinger-Ellison syndrome
colon cancer
irritable bowel syndrome
Correct answer:
Zollinger-Ellison syndrome
Zollinger-Ellison syndrome is characterized by a gastrinoma located on the pancreas or the stomach which secretes gastrin,
which stimulates high levels of acid production in the stomach resulting in development of multiple and severe ulcers in the
stomach and duodenum. The patient will have complaints of epigastric to mid-abdominal pain. Stools may be a tarry color.
Screening is by serum fasting gastrin level.
Question 58
A patient who lost his job six weeks ago comes to you complaining that he is sad most of the time and has difficulty
concentrating. He also complains of headaches. This is affecting his job search. This patient is most likely suffering from
which of the following?
dysthymic disorder
adjustment disorder
major depression
bipolar disorder
Correct answer:
adjustment disorder
Adjustment disorder is a less severe form of depression in which symptoms occur within 3 months of an identifiable stressor
such as a job loss. It is characterized by mild sadness, inability to concentrate, excess worry, and somatic complaints.
Question 59
All of the following are necessary for the NP to be granted a DEA license for prescribing controlled substances EXCEPT:
the NP has no felony record
the NP has practiced for at least 5 years
the NP has a practice site
state law must permit NPs to prescribe controlled substances
Correct answer:
the NP has practiced for at least 5 years
There is no minimum time requirement for granting of a DEA license for prescribing controlled substances. All of the other
choices are requirements for granting of the license.
Question 60
Illnesses caused by organisms that attack and invade vulnerable persons are communicable or infectious diseases. Which of
the following statements about these types of diseases is inaccurate?
295
The involve identification of the causative agents.
They rely on microbiology principles in understanding the life cycle of the organism.
They focus on intervention at vulnerable phases in the course of the disease.
It is unnecessary to focus on the life cycle of the organism.
Correct answer:
It is unnecessary to focus on the life cycle of the organism.
This is the inaccurate statement. Communicable or infectious diseases do focus on the life cycle of the organism.
Question 61
A child of 24 – 36 months usually sleeps for how long each night?
12 – 14 hours
10 – 12 hours
8 – 10 hours
7 or 8 hours
Correct answer:
10 – 12 hours
A child of 24 – 36 months of age sleeps on an average of 10 – 12 hours. By 5 – 10 years of age the child sleeps 8 – 10 hours.
Question 62
Which of the following is NOT a type of managed care plan?
Health Maintenance Organization
Preferred Provider Organization
Point of Service Plan
Indemnity Plan
Correct answer:
Indemnity Plan
There are four common types of managed care plans. They are: Health Maintenance Organizations (HMOs); Preferred
Provider Organizations (PPOs); Point of Service Plans (POSs); and integrated delivery systems.
Question 63
The patient right of confidentiality has exceptions when society determines that the need for information outweighs the
principle of confidentiality. Which of the following is NOT one of these exceptions?
when a parent demands to view a record regarding an adolescent's treatment for STDs
in cases of suspected child abuse
to attorneys involved in litigation
release of records to insurance companies
The Correct answer is:
When a parent demands to view a record regarding an adolescent's treatment for STDs
STD treatment for the adolescent is protected confidential material. The remaining answer choices are all examples of
exceptions. Others include subpoenas, summonses, answering court orders, meeting state requirements for mandatory
reporting of diseases or when the patient reveals an intent to harm himself or someone else.
Question 64
A 29-year-old Caucasian male calls your office. He tells you he just came in from the woods and discovered a tick on his
upper right thigh. He reports self-removal of the tick and now the area is slightly red. What should you advise him to do?
He should come to the office for a ceftriaxone (Rocephin) injection.
He should be prescribed doxycycline.
He needs no treatment.
He needs a topical scrub to prevent Lyme Disease.
The Correct answer is:
He needs no treatment
To develop Lyme Disease from a tick bite, many factors must be present. The tick must belong to Ixodes species and must
have been attached for at least 48 hours before the disease can spread. There is no need for prophylactic treatment in this case
because the tick has not been present long enough.
Question 65
296
You are counseling a mother who has her 7-month-old infant in the office with the complaint of "spitting up his formula." The
nurse practitioner (NP) knows this to be a common thing for infants but the mother conveys she has "put him on goat's milk."
The NP is concerned because she knows that goat's milk places the infant at risk of developing:
scurvy
rickets
megaloblastic anemia
botulism
The Correct answer is:
Megaloblastic anemia
This condition can develop secondary to folic acid deficiency. Goat's milk does not contain the appropriate level of folic acid
needed for the infant. Scurvy is caused by a lack of ascorbic acid and rickets is a result of a diet lacking in Vitamin D.
Botulism is food poisoning due to an endotoxin produced by the bacillus Clostridium botulinum.
Question 66
A nurse practitioner is HIV positive and she is employed in a privately owned clinic. She performs wellness exams on
ambulatory adults. What is required of this professional?
She is under obligation to inform the patient of her HIV status if she performs invasive procedures.
She is under no obligation to inform anyone of her HIV status.
She is obligated to inform her employer of her HIV status.
She is obligated to inform her patients of her HIV status.
The Correct answer is:
She is under no obligation to inform anyone of her HIV status
The nurse practitioner's health information is protected health information, just as her patient's health information is protected.
She is not under obligation to inform anyone of her HIV status, including her employer, the patient, or the State Board of
Nursing. As long as the performance of her job does not impose unnecessary health risks to anyone, she is protected by this
law.
Question 67
A patient who has been prescribed methimazole for hyperthyroidism should be advised to notify his doctor immediately if he
experiences which of the following?
Nausea.
Stomach upset.
Easy bruising.
Mild rash.
Correct Answer: Easy bruising Rarely, methimazole can cause blood disorders. Easy bruising is a sign of this. The patient
should notify his doctor immediately. The other effects listed are mild and usually not cause for concern, though if they persist
or increase the patient should notify his doctor.
Question 68
The CDC recommends STD screening for all sexually active adults and for which of the following?
females over the age of 65
males over the age of 65
males age 20 to 40
females age 20 – 24
Correct answer:
females age 20 – 24
The CDC recommends STD screening for all sexually active adolescents and females age 20 to 24 years. Multiple sexual
partners and inconsistent condom use are risk factors of STD.
Question 69
With prepatellar bursitis, which of the following should be considered as a first-line-therapy?
minimizing the offending activity
applying ice to the affected area for 15 minutes at least 4 times a day
NSAIDs
bursal aspiration
Correct answer:
bursal aspiration
297
With prepatellar bursitis, bursal aspiration should be considered as a first-line therapy. This is because this procedure affords
significant pain relief and allows the bursa to reapproximate. The therapies in the other three choices are appropriate for
bursitis in other sites.
Question 70
When counseling patients who smoke cigarettes, which of the following is NOT good management advice?
For asymptomatic adults, instruct the patient on how it is easier to stop now than when symptomatic.
For teens, instruct on bad breath and stained teeth.
For parents, instruct on increased respiratory infections among the children of smokers.
For pregnant women, instruct on the increased rate of spontaneous abortion.
The Correct answer is:
For asymptomatic adults, instruct the patient on how it is easier to stop now than when symptomatic
For symptomatic adults, you should instruct on heart disease risk, risks of lung cancer, and facial wrinkles. For new smokers,
you should instruct that it is easier to stop now than after smoking a while.
Question 71
Which of the following would be a factor that makes a woman more susceptible to premature labor?
age of 33 years
age at first pregnancy 19 years of age
UTI
Caucasian race
Correct answer:
UTI
Of the choices, having a UTI is the factor that would increase a woman’s risk of premature labor. There are a host of other
factors some of which are: smoking more than 11 cigarettes daily; exposure to job-related teratogens and strenuous activity;
less than a high school education; and previous premature labor.
Question 72
The FNP has an elderly patient who has decided not to take any further action to treat her advanced pancreatic cancer. She
wants to enjoy what time she has left. She tells the FNP that she hopes that she has made a good decision. What is the best
thing that this FNP can say to the patient in regard to her decision?
You have given this a lot of thought so your decision is probably right.
Not knowing about how a decision will affect one’s life can be worrisome.
You can always change your mind.
Why don’t you seek further guidance in this decision?
Correct answer:
Not knowing about how a decision will affect one’s life can be worrisome.
Of the choices given, this is the best statement that the FNP could make. It recognizes the patient’s feelings. The other
statements deny the patient’s feelings.
Question 73
When assessing a 6-year-old girl with fever, abdominal tenderness, and an abnormal urinalysis, the mother tells you the child
had a recent "strep" infection about 3 weeks ago and did not finish her antibiotics. The nurse practitioner should suspect:
an antibiotic reaction
sepsis
gastroenteritis
acute glomerulonephritis
The Correct answer is:
Acute glomerulonephritis
The symptoms of glomerulonephritis are fever, abdominal or flank pain, edema of the face, hands and feet, abrupt onset of
hematuria, oliguria, or anuria. It is most commonly seen in children with a history of recent Group A beta hemolytic
streptococcal infection, such as pharyngitis or impetigo. The symptoms do not indicate an antibiotic reaction or sepsis. Further,
gastroenteritis usually causes nausea and/or vomiting.
Question 74
You are testing a 75-year-old male with the Mini-Mental Exam (MME). You will be testing him with all of the following
EXCEPT:
298
orientation
attention and calculation
writing a 100 word paragraph
copying a design
Correct answer:
writing a 100 word paragraph
There are five subject areas tested in the MME. They are orientation; attention and calculation; recall; writing a short sentence;
and copying a design.
Question 75
A 34-year-old African-American male comes to the clinic after taking an undetermined amount of street heroin. Before you
transfer this patient to a psychiatric treatment facility, what is the drug of choice you should use for opioid overdose?
Clonidine (Catapres)
naltrexone HCl (ReVia)
methadone (Dolophine)
Naloxone (Narcan)
The Correct answer is:
Naloxone (Narcan)
Naloxone (Narcan) is a narcotic antagonist that is used for the reversal of narcotic depression, including respiratory
depression. Naltrexone is a narcotic detoxification adjunct. Clonidine (Catapres) is a central alpha agonist for hypertension.
Methadone is used to treat opioid addiction and not an overdose.
Question 76
A 35-year-old male comes to the clinic with a skin lesion which he tells the FNP came from a tick bite he got on a recent
camping trip. The FNP suspects early Lyme disease. The FNP knows that this disease is caused by which of the following
organisms?
Borrelia burgdorferi
Rickettsia rickettsii
Sarcoptes scabiei
Neisseria meningitides
Correct answer:
Borrelia burgdorferi
Early Lyme disease (erythrema migrans) is a skin lesion caused by the bite of an Ixodes tick infected with Borrelia
burgdorferi. If it is left untreated, the infection becomes systemic and affects multiple organ systems.
Question 77
Folic acid requirements are increased during all of the following EXCEPT:
pregnancy
childhood
recovery from illness
hypertension
Correct answer:
hypertension
Folic acid need is increased in times of accelerated tissue growth and repair, such as in childhood, pregnancy, recovery from
serious illness and recovery from hemolytic anemia. The folic acid requirements increase from the baseline of twofold to
fourfold. Although studies indicate that folic acid helps to prevent high blood pressure, the need for folic acid is not increased
in someone with hypertension.
Question 78
You are testing a 5-year-old child for visual acuity. You use the Snellen chart. The child should be standing at what distance
from the chart?
5 feet
10 feet
15 feet
20 feet
299
Correct answer:
20 feet
You should obtain visual acuity (V/A) and binocular vision by age 4 to 5 years. The child using the Snellen chart should be at
20 feet when reading it. A failed test is V/A 20/40 or greater in either eye, or if there is a two-line discrepancy between the
eyes.
Question 79
Which of the following statements about fibromyalgia is incorrect?
It is four to seven times more common in women than in men.
The diagnosis of fibromyalgia is supported by the presence of tender points in specific locations.
Biochemical changes noted in the central nervous system in a person with fibromyalgia include high serotonin levels and
decreased levels of substance P.
Fibromyalgia affects at least 2% of the general population.
Correct answer:
Biochemical changes noted in the central nervous system in a person with fibromyalgia include high serotonin levels and
decreased levels of substance P.
This is an incorrect statement. Biochemical changes noted in the central nervous system in a person with fibromyalgia include
low serotonin levels and elevated levels of substance P. These changes likely contribute to the diffuse hypersensitivity to pain.
Question 80
The anterior fontanels will usually close at which age?
9 months – 2 years
5 months – 1 year
1 – 2 months
3 – 4 months
Correct answer:
9 months – 2 years
The posterior fontanels will close first at about 1 – 2 months. The anterior fontanels close at about 9 months – 2 years.
Question 81
A 45-year-old African-American male reports chest pain that is sudden and severe. Also, the male patient describes the pain as
"tearing." You notice that the pain is accompanied by a decrease in peripheral pulses. You understand that this may indicate a
diagnosis of:
pericarditis
aortic dissection
acute MI
angina
The Correct answer is:
Aortic dissection
Aortic dissection almost invariably begins with sudden onset of severe chest pain that is tearing or ripping in quality and is
accompanied by absent or decreased peripheral pulses and neurologic deficits. The pain of angina and acute myocardial
infarction (MI) is usually described as "pressure" by the patient. Pericarditis produces pain that is more gradual in its onset.
Question 82
What is included when taking the medical history of an infant?
Prenatal history.
Parent occupation.
Daycare.
Home.
Correct answer: Prenatal history Prenatal history is important to note when taking the medical history of an infant. The other
answers are part of the social assessment.
Question 83
The following are all ethical concepts for NPs except for:
beneficence (the duty to help others)
accountability ( responsible for own actions)
300
accomplishment (sense of self worth)
confidentiality ( the right to keep records private)
The Correct answer is:
Accomplishment (sense of self worth)
Confidentiality, accountability and beneficence are all ethical concepts along with non-malfeasance (duty to do no harm),
human dignity (respect of the patient), and compassion (concern for the patient). Self worth, personal accomplishment are
concepts related to the nurse practitioner's ego and are not ethical concepts.
Question 84
Which of the following would NOT be a complication of Lyme disease?
Lyme gastritis
Lyme carditis
Lyme meningitis
Lyme encephalitis
Correct answer:
Lyme gastritis
This is not a complication of Lyme disease. Complications can include: Lyme carditis; Lyme meningitis; Lyme encephalitis;
Lyme arthritis; and facial nerve paralysis.
Question 85
In terms of secondary skin lesions, which of the following types of lesions are permanent skin changes as a result of newly
formed connective tissue?
lichenification
crust
scar
scale
Correct answer:
scar
A scar is a permanent skin changes as a result of newly formed connective tissue. A lichenification is induration and
thickening of skin; a crust is a dry mass of exudate; and a scale is a dry, greasy fragment of dead skin.
Question 86
A patient in the clinic is found to have 2 palpable, tender, left pre-auricular nodes that are about 0.5 cm in diameter. You
expect to find what in this patient?
ulceration on the tongue
ear infection
sore throat
conjunctivitis
The correct answer is conjunctiviitis. The eyes are drained by the pre-auricular lymph nodes and they can be palpated near the
ear. These swell in response to eye infections, allergies, or foreign bodies in the eye.
Question 87
All of the following are true regarding developmental theory EXCEPT:
The pace of development varies among individuals.
Stress does not impact development.
All development is patterned, orderly, and predictable with both a purpose and a direction.
Development is continuous throughout life, although the degree of change in many areas decreases after adolescence.
The Correct answer is:
Stress does not impact development
Physical and mental stress during periods of critical developmental change, such as puberty, may make a person particularly
susceptible to outside stressors.
Question 88
In assessing the vision of a toddler which of the following tests might be done?
Snellen test
determine ability to follow an object
301
Jaeger test
blink reflex
Correct answer:
determine ability to follow an object
The FNP should determine the toddler’s ability to follow an object as part of a vision screening for a toddler or older infant.
The FNP might also perform a corneal light reflex test, perform a cover/uncover test and assess red reflex.
Question 89
Which of the following foods is NOT a dietary trigger that influences the severity of migraine symptoms?
pears
avocados
bananas
citrus foods
Correct answer:
pears
Pears are not one of the dietary triggers that influence the severity of migraine symptoms. The other choices are triggers along
with such things as: ripened cheeses; sausage; pizza; herring; and many more.
Question 90
Which of the following is most consistent with the diagnosis of anxiety?
diminished cognitive ability
nausea
consistent early morning wakening
difficulty initiating sleep
The Correct answer is:
Difficulty initiating sleep
Anxiety is a normal human emotion that is an important part of fear response. It can also be protective and heighten an
individual's senses when an individual encounters a dangerous situation. The remaining answer choices are not consistent with
anxiety but difficulty initiating sleep is.
Question 91
In terms of issues of confidentiality and the right to consent without parental involvement, which of the following persons
would NOT be considered an emancipated minor in the United States?
a 15-year-old who wants to be treated for painful urination
a 16-year-old who has enlisted in the Army
a 14-year-old who is married
a 15-year-old single mother who has one child
Correct answer:
a 15-year-old who wants to be treated for painful urination
An emancipated minor can give full consent, sign contracts and other legal documents as an adult would. The criteria for an
emancipated minor in the United States are: any minor who is married, any minor who is a parent, and any minor who is
enlisted in the U.S. military. The 15-year-old who wants to be treated for painful urination is not an emancipated minor.
Question 92
In diagnosing a toothache (pulpitis), the nurse practitioner has knowledge that all of the following are necessary except:
Systemic manifestations do not occur from dental caries and pulpitis.
You should inquire about fever and chills.
Dental caries are the most frequent type of injury that causes pulpitis.
There is diverse flora involved in the infectious process, including gram-positive anaerobes and bacteroides.
The Correct answer is:
Systemic manifestations do not occur from dental caries and pulpitis
Many patients present in the clinic setting with a toothache or related symptoms. Systemic manifestations often occur and they
include lymphadenopathy, malaise, fever, and pain. The remaining answer choices are true regarding this condition.
Question 93
302
Which of the following statements does NOT apply to the general systems theory of family development?
Within the family system are boundaries that are open, closed or operate at random.
A change affecting one part of a family manifests itself as change in the whole family system.
Families pass through eight chronological stages from beginning family to old age.
Families strive for homeostasis that reflects a balance between change and stability.
Correct answer:
Families pass through eight chronological stages from beginning family to old age.
This is not part of the general systems theory of family development. It is part of the developmental theory established by
Duvall. It holds that success in one task sets the stage for success in subsequent tasks
Question 94
The NP has diagnosed her elderly patient with shigella. Which of the following medications would she MOST likely prescribe
her?
Metronidazole.
Erythromycin.
Ampicillin.
Trimethoprim-sulfamethoxazole.
Correct answer: Trimethoprim-sulfamethoxazole The preceding drugs are typically used in treating the following: Campylobacter jejuni: Erythromycin - Salmonella: Ampicillin - Shigella: Trimethoprim-sulfamethoxazole - Giardia lamblia:
Metronidazole
Question 95
The FNP has a patient with peripheral vascular disease. During examination of this patient the FNP can expect to find all of
the following EXCEPT:
cool skin
decreased pulses
hairless, shiny skin on the affected area
decreased capillary refill time
decreased capillary refill time
In a patient with PVD (peripheral vascular disease) the FNP would not find a decreased capillary refill time; he would find an
increased capillary refill time. In addition to the other choices, the FNP is likely to find: hyperpigmentation on the affected
area; atrophic changes due to chronic poor circulation; and bruits over partially blocked arteries.
Question 96
Which of the following statements about tuberculosis is incorrect?
Tuberculosis occurs only in the lungs.
Transmission of tuberculosis is by respiratory droplets.
The majority of cases in the United States are reactivated infections.
A TB infection is controlled by an intact immune system.
Correct answer:
Tuberculosis occurs only in the lungs.
This statement is incorrect. The most common site of infection is the lungs (85%), but infection can also occur in the kidneys,
brain, lymph nodes, adrenals, bone and more.
Question 97
In a trial, an expert witness will establish the standard of care for a nurse practitioner. The expert's opinion is based on:
facility policies and procedures
national norms for the specialty
professional literature
all of the above
The Correct answer is:
All of the above
303
With respect to nursing specialties, the standard of care is usually a national one. Therefore, an expert witness may reference
facility policies, books and journals to provide an opinion on nursing standard of care. Also, for a trial, physician input may be
sought. In some cases, physicians may even be allowed to testify about the standard of care.
Question 98
A 15-year-old Caucasian male presents in the clinic with a reddened, pruritic rash. The nurse practitioner understands that the
most common sites for adolescent atopic dermatitis are:
forehead, scalp, and cheeks
wrists, ankles, and cubital core
face, neck, back, and antecubital fossae
palmar creases and extensor surface of legs
The Correct answer is:
Face, neck, back and antecubital fossae
In the adolescent and the young adult population, common sites for a pruritic rash are the popliteal and antecubital fossae,
face, neck, upper arms and back, dorsa of the hand, feet, fingers, and toes.
Question 99
Osteoporosis is indicated by a bone density score of which of the following?
less than -2.5
between -1 and -2.5
greater than -1
none of the above
Correct answer:
less than -2.5
Interpretation of bone density testing is based on a T score, which refers to the number of standard deviations above or below
the average value in young adults of the same gender. Osteoporosis is a bone density indicated by a T score less than -2.5. It is
also indicated by the presence of fragility fractures, irrespective of the bone density test results.
Question 100
Which of the following medications would be used as a topical treatment for fibromyalgia?
trazodone
capsaicin
nortriptyline
pregabalin
Correct answer:
capsaicin
Topical treatments such as capsaicin are often helpful treatment adjuncts for a person with fibromyalgia. Medications such as
trazodone can be helpful in improving sleep latency and duration. Tricyclic antidepressants such as nortriptyline have been
shown to minimize symptoms. Pregabalin (Lyrica) are approved by the FDA for pain reduction in fibromyalgia.
quiz 11 aanp
Question 1
Erythema migrans in Lyme disease has a pattern of a ring. This is indicated by which of the following terms?
annular
confluent
coalescent
reticular
Correct answer:
annular
304
Annular is the term used for the ring pattern of skin lesions. This is typical of the skin condition in Lyme disease.
Question 2
Which of the following is NOT one of the three major requirements under the Occupational Safety and Health Act (OSHA)?
Wear protective wear when at risk for blood and other body fluid handling.
Collect hazardous waste in separate, clearly marked trash cans.
Properly dispose of hazardous waste.
Do not pay remuneration for referral or business reimbursable by a federal healthcare program.
Correct answer:
Do not pay remuneration for referral or business reimbursable by a federal healthcare program.
This is not part of OSHA. It refers to anti-kickback statutes. OSHA is concerned with building safety and safe clinical
practices.
Question 3
Which of the following does not play a significant role in the development of varicose veins?
diabetes mellitus
wearing constricting garments
weakness of the walls of the vein
pregnancy
Correct answer:
diabetes mellitus
Diabetes does not play a significant role in the development of varicose veins. An inherited venous defect of either a valvular
incompetence or a weakness in the walls of the vessel likely plays a significant role. In addition, situations that cause high
venous pressure, such as wearing constricting garments and pregnancy, contribute to their development.
Question 4
Which of the following are you most likely to consider for a 3-month-old infant with developmental dysplasia of the hip?
open reduction
Pavlik harness
spica cast
preliminary traction
Correct answer:
Pavlik harness
A Pavlik harness permits a relaxed motion of the hip while maintaining a flexed and abducted position for natural
development of joint space, until the capsule tightens in about 6 weeks. After age 6 months the other choices may be
considered.
Question 5
In which of the following situations might there be an exception to guaranteed confidentiality of a patient?
in cases of suspected child abuse
when next of kin requests a patient’s records
when discussing the patient with another medical professional not involved in the care of the patient
none of the above
Correct answer:
in cases of suspected child abuse
There are exceptions to guaranteed confidentiality when society determines that the need for information outweighs the
principle of confidentiality. These situations include: when records are released to insurance companies; to attorneys involved
in litigation; in answering court orders, subpoenas or summonses; in meeting state requirements for mandatory reporting of
diseases or conditions; in cases of suspected child abuse; or if a patient reveals an intent to harm someone.
Question 6
Which of the following procedures would you recommend for a patient with a kidney stone that has not passed on its own?
Lorthotomy.
Lithotrispy.
305
Laparoscopy.
Lithotomy.
Correct Answer: Lithotrispy This procedure uses shock waves to break up kidney stones. They can then pass more easily on
their own. This procedure is used when kidney stones have not passed after what the healthcare practitioner deems a sufficient
amount of time.
Question 7
The FNP is communicating with a patient about controlling her blood sugar. The patient tells the FNP that she is afraid that
she will have to take insulin shots soon. Which of the following statements would demonstrate using the therapeutic
communication technique of reflection?
What would it be like for you to have to take insulin shots every day?
If that is necessary a specialist will show you just what to do.
Maybe you should be more vigilant about your diet.
Don’t worry. That’s not likely to happen.
Correct answer:
What would it be like for you to have to take insulin shots every day?
In using reflection, the FNP takes one or two words said by the patient to reflect back to the patient for consideration. By
asking her, “What would it be like for you to have to take insulin shots every day?” the FNP is using reflection.
Question 8
Knowledge concerning health literacy is part of the nurse practitioner practice environment. This is where the patient and
family understand and act on health information. Which of the following is TRUE regarding health literacy?
Health literacy is now recognized as a small contributor to health outcome.
The Institute of Medicine (IOM) has launched efforts to help nurse practitioners and physicians teach patients.
The Institute of Medicine (IOM) and the Agency for Health Care Research and Quality (AHRQ) have launched efforts to
quantify and offer solutions to the problems that result from inadequate health literacy.
None of the above are correct.
The Correct answer is:
The Institute of Medicine (IOM) and the Agency for Health Care Research and Quality (AHRQ) have launched efforts to
quantify and offer solutions to the problems that result from inadequate health literacy.
Health literacy is now recognized as one of the largest contributors to health outcome. The IOM and AHRQ are organizations
who are making efforts to solve the problems of health literacy
Question 9
Which of the following options for the business structure of a practice involves every person being liable for the debts and
legal liabilities of other persons in the entity?
partnership
limited liability company
sole proprietorship
corporation
Correct answer:
partnership
In a partnership, which is a business relationship involving two or more individuals or business entities, each partner is liable
for the debts and legal liabilities of the other partners. Partners share profits, decision making, administration and workload in
some way agreeable to all partners.
Question 10
Contraindications to cows’ milk-based formulas include all of the following EXCEPT:
milk protein sensitivity
soy protein allergy
lactose intolerance
galactosemia
Correct answer:
soy protein allergy
306
Soy protein allergy would be a contraindication to the use of soy protein isolate formulas. All of the other choices would be
contraindications for use of cows’ milk-based formulas.
Question 11
The leading cause of death in the 45 to 54 year age group is which of the following?
heart disease
suicide
malignant neoplasms
cerebrovascular disease
Correct answer:
malignant neoplasms
Malignant neoplasms are the leading cause of death in the 45-54 year age group. Heart disease is second.
Question 12
When treating a patient with atypical depression, the nurse practitioner is aware that the following class of drugs is most likely
to precipitate a hypertensive crisis:
monoamine oxidase (MAO) inhibitor
phenothiazines
narcotic analgesics
barbiturates
The correct answer is:
Monoamine oxidase (MAO) inhibitor
Patients who eat tyramine-rich foods while taking an monoamine oxidase (MAO) inhibitor drug are at risk for hypertensive
crisis. Examples of these tyramine-rich foods that have undergone an aging process are cheese, wine, beer, salami, and yogurt.
Further, the catecholamines from these foods are released from the nerve endings, thus causing a hypertensive crisis. The
remaining answer choices identify drugs that cause low blood pressure.
Question 13
The care giver of an elderly patient with Alzheimer's informs the NP that the patient requires community resources to help
with her care. Who would the NP MOST likely refer the care giver to?
Physical therapist.
Geropsychiatrist.
Clinical psychologist.
Social worker.
Correct answer: Social worker Of the options listed, the social worked would typically be the most skilled in securing
community resources for the patient. The other specialists can help in areas such as patient behavioral issues and patient
physical activity.
Question 14
An 81 year old male patient complains of abdominal pain and slimy stools. Testing shows leukocytes and blood in the stools.
What diagnosis will the NP MOST likely make?
Shigella.
Adenovirus.
Campylobacter jejuni.
Salmonella.
Correct answer: Salmonella Salmonella is a type of diarrhea that can occur at any age. Stools are characterized with a "rotten"
or "egg" odor.
Question 15
Which of the following types of headaches typically lasts several weeks to months?
tension headaches
cluster headaches
migraine headaches without aura
migraine headaches with aura
Correct answer:
307
cluster headaches
Cluster headaches have a tendency to occur daily in groups or clusters. The clusters usually last several weeks to months, then
disappear form months to years.
Question 16
Enuresis is involuntary urination after a child has reached the age when bladder control is usually attained. Management of
primary nocturnal enuresis includes all of the following EXCEPT:
limiting fluid intake before bedtime
double voiding before bedtime
avoiding punishment
withholding verbal praise so as not to call attention to the problem
Correct answer:
withholding verbal praise so as not to call attention to the problem
Verbal praise is an excellent means of motivational therapy for a child with primary nocturnal enuresis. Other means of
motivational therapy include a reward system and keeping a dryness calendar.
Question 17
Your patient complains of eye pain that has come on suddenly and tells you that light bothers him and his vision seems
blurred. Your examination reveals a rash on the side of the temple and on the tip of his nose. This is which of the following
diseases?
hordeolum
chalazion
herpes keratitis
primary open-angle glaucoma
Correct answer:
herpes keratitis
Herpes keratitis is damage to the corneal epithelium caused by the herpes virus, commonly shingles. The patient usually has
acute onset of eye pain, photophobia, and blurred vision in the affected eye.
Question 18
A 30 year old female patient presents abdominal pain as her chief complain. She also mentions alternating between
constipation and diarrhea. There is no fever. What do you need to pay close attention to in the assessment?
Diet and exercise.
Diet.
Exercise.
Depression.
Correct answer: Diet and exercise The patient's lifestyle can affect digestion. A diet that is low in fiber can contribute to IBS
and other problems. A sedentary lifestyle will also negatively affect digestion.
Question 19
A preschooler is in the office with her mother. The mother is concerned because the child has begun to stutter. The nurse
practitioner (NP) knows that she should do what?
Give the child verbal exercises to perform at home.
Reassure the mother that stuttering is normal in a preschooler.
Refer the child to a speech pathologist.
Encourage the mother to correct the child when she stutters.
Correct answer:
Reassure the mother that stuttering is normal in a preschooler
It is normal for preschoolers to repeat whole words or phrases as if stuttering. It is inappropriate to refer to a speech
pathologist. Further, patients should not correct or criticize a child who does this. Verbal exercises would not help and could
contribute to low self-esteem.
Question 20
308
Precocious puberty in girls is defined as the onset of secondary sexual characteristics before the child is how many years old?
10 years old
9 years old
8 years old
6 years old
Correct answer:
8 years old
Precocious puberty in girls has long been defined as the onset of secondary sexual characteristics before the child’s 8th
birthday. In boys, it is before the 9th birthday.
Question 21
Of the following anti-diabetic agents, which one is a biguanide?
Diabinese
Glucophage
Acarbose
Prandin
Correct answer:
Glucophage
Glucophage is a biguanide. It decreases gluconeogenesis and decreases peripheral insulin resistance.
Question 22
Of the models and theories related to health care, which one is used to explain why healthy people do or do not take advantage
of screening programs?
Maslow’s Hierarchy of Needs
Trans-theoretical Model of Change
Self-efficacy Theory Model
Health Belief Model
Correct answer:
Health Belief Model
The Health Belief Model involves variables such as perceptions of susceptibility and seriousness of a disease, benefits of
treatment, perceived barriers to change, and expectations of efficacy. It is used to explain why healthy people do or do not take
advantage of screening programs.
Question 23
You are seeing a 22-year-old female in the office with elbow pain. A diagnosis of lateral epicondylitis is made based on this
knowledge:
pain that is worse with elbow flexion
reduced joint range of motion (ROM)
decreased hand grip strength
electric-like pain elicited by tapping over the median nerve
The Correct answer is:
Decreased hand grip strength
With lateral epicondylitis (tennis elbow), the patient will complain of pain over the lateral epicondyle that increases with
resisted wrist extension, especially with the elbow. Hand grip is often weak on the affected side. The remaining answer
choices are not descriptive of this condition.
Question 24
You have done a cholesterol screening for a young female patient, aged 12. Which of the following results would NOT be
considered normal for this child?
cholesterol = 188
LDL = 155
HDL = 40
triglycerides = 100
Correct answer:
LDL = 155
309
A result of LDL = 155 is not considered normal for a 12-year-old female. LDL should range between 70 – 140.
Question 25
Which of the following medications or treatments is least likely to be prescribed for head lice?
permethrin 1% cream rinse
nit removal with a fine tooth comb
Lindane 1% shampoo
soaking brushes and combs in Nix shampoo
Correct answer:
Lindane 1% shampoo
This is least likely to be prescribed because of neurotoxic effects. It is definitely contraindicated in infants, toddlers, or any
patient with neuro or seizure disorder.
Question 26
Your patient has moderate COPD. Which of the following medications are you least likely to prescribe for him at this stage of
the disease?
Atrovent
extended-release theophylline
Ventolin
triamicinolone
Correct answer:
extended-release theophylline
For moderate COPD regular treatment with one or more of the following is indicated: anticholinergic bronchodilator such as
Atrovent; inhaled beta2-selective agonist such as Ventolin; and inhaled corticosteroid such as triamicinolone. Extendedrelease theophylline is used in severe COPD to improve respiratory muscle performance.
Question 27
You have called a patient back in after a pap smear for re-evaluation. Her results showed reactive cellular changes. The nurse
practitioner has knowledge that this is associated with:
drying artifact
use of oral contraceptives
inflammation
use of estrogen vaginal cream
The Correct answer is:
Inflammation
Reactive cellular changes are most often associated with inflammation. Other associated causes are radiation, IUD use,
atrophic vaginitis, and diethylstilbestrol exposure in utero. Birth control pills and estrogen vaginal creams do not cause
reactive changes.
Question 28
Which of the following pharmacological treatments for allergic rhinitis is not an intranasal corticosteroid?
beclomethasone
cromolyn sodium
budesonide
fluticasone
Correct answer:
cromolyn sodium
Intranasal cromolyn sodium is not an intranasal corticosteroid. It is a topical antiinflammatory agent.
Question 29
The nurse practitioner understands that after starting vitamin B12 therapy for a patient, what would be expected at a 2 week
follow-up visit?
310
reduced RBCs, WBCs, and platelets
increased macrocytosis and anisocytosis
increased hemoglobin/hematocrit and reticulocyte count
ferritin level of 40 ng/ml
The Correct answer is:
Increased hemoglobin/hematocrit and reticulocyte count
With vitamin B12 therapy, the nurse practitioner will see an increase in the hemoglobin/hematocrit and reticulocyte count.
Other improvements will include improved appetite and decrease in neurological symptoms, such as gait disturbances,
weakness, and paresthesias
Question 30
Which of the following organizations can exclude NPs from being designated as primary care providers?
MCOs
Medicare
TRICARE
Federal Employees Health Benefits Program
Correct answer:
MCOs
Managed care organizations (MCOs) frequently have excluded NPs from being designated as primary care providers carrying
their own case loads. Thus, in many MCOs, the only option for NPs is that of being a salaried employee.
Question 31
The mean weight gain of a female in puberty (10 – 14 years) is which of the following?
52 lbs
38 lbs
30 lbs
55 lbs
Correct answer:
38 lbs
The weight gain for a female during puberty (10 to 14 years) is between 15 and 55 lbs. The mean weight gain is 38 lbs. Males
gain 15 – 65 lbs during puberty (11 – 16 years) and the mean weight gain is 52 lbs.
Question 32
The diagnosis of hypertension (HTN) should be established on the basis of:
at least five readings one month apart
one reading taken in three different positions
one reading of 140 mm Hg systolic and 90 mm Hg diastolic or above
at least three hypertensive readings one week apart
The correct answer is:
At least three hypertensive readings one week apart
The diagnosis of hypertension (HTN) from a single measurement of blood pressure (BP) elevation should not be done. A
minimum of three readings with an average systolic BP of 140 mm Hg and diastolic of 90 mm Hg establishes the diagnosis.
An average of two or more readings should be taken at each of the two or more visits following an initial screening.
Question 33
An increase in creatinine from 1 to 2 mg/dL is typically seen with what percentage loss in renal function?
10%
25%
50%
75%
The Correct answer is:
50%
Creatinine is the end product of creatinine metabolism, which arises from skeletal muscle. Because creatinine excretion by a
healthy kidney is very efficient, measurement of creatinine is used as a surrogate marker of kidney function. That much of an
311
elevation is consistent with a 50% loss of renal function.
Question 34
Which of the following is NOT true regarding healthcare issues in the United States?
Most of Americans have access to healthcare and are being adequately treated.
Of all children, 13% are uninsured.
It is estimated that 45+ million Americans are underserved because of limited access to healthcare.
Many of the working poor cannot obtain healthcare because of the high costs of insurance, fees and medications.
The Correct answer is:
Most of Americans have access to healthcare and are being adequately treated
There is a large portion of Americans who do not have access to healthcare.
Question 35
You observe that a patient's muscle strength is perfectly normal when tested. What grade would you give it?
0 of 5.
2 of 5.
3 of 5.
5 of 5.
Correct answer: 5 of 5 Muscle strength is observed and graded on a five-point scale. Five is normal. Anything below five is
weaker than normal.
Question 36
A patient who comes to you displaying rapid speech and frequent change of topic, restlessness and selective inattention is
suffering from which of the following?
mild anxiety
panic anxiety
moderate anxiety
severe anxiety
Correct answer:
moderate anxiety
Moderate anxiety involves a narrowed focus of perception. The patient would show selective inattention and hesitation;
diminished problem solving; rapid speech with frequent change of topic; and muscle tension and restlessness.
Question 37
The fee paid by a managed care organization (MCO) to a healthcare provider, per patient, per month, for care of an MCO
member is which of the following?
“incident to” services
capitation
fee for service
none of the above
Correct answer:
capitation
Capitation is the fee paid by a managed care organization (MCO) to a healthcare provider, per patient, per month, for care of
an MCO member. Capitated fees for primary care vary, based on a patient’s age and sex. An NP wishing to provide care for a
Medicare patient enrolled in an MCO applies to the MCO for admission to the organization’s provider panel.
Question 38
Charles Darwin wrote On the Origin of Species about human evolution. He also is known for which of the following?
developing the first standardized measurement of intelligence
developing the “baby journal” as a systematic method to document observed behavioral development
developing the maturational-organismic theory
none of the above
Correct answer:
developing the “baby journal” as a systematic method to document observed behavioral development
312
In addition to writing On the Origin of Species, Charles Darwin is known for the first use of a “baby journal” as a systematic
method to document observed behavioral development. Alfred Binet developed the first standardized measurement of
intelligence. Arnold Gesell developed the maturational-organismic theory.
Question 39
A patient on levothyroxine for hypothyroidism presents in the clinic with complaints of chest pain after only taking this
medication 3 weeks. What would be the appropriate thing to do?
Decrease dose of levothyroxine, order an ECG, and consult with your collaborating physician.
Prescribe an anxiolytic agent.
Schedule the patient for a sedimentation rate test.
Discontinue levothyroxine because chest pain is a contradiction to its continuance.
The Correct answer is:
Decrease dose of levothyroxine, order an ECG, and consult with your collaborating physician
Decreasing the dose of levothyroxine and evaluating the client's cardiac status are appropriate interventions in this situation as
well as involving the collaborating physician. It would be inappropriate to stop thyroid replacement. An anxiolytic may be
helpful but is not the sole necessary intervention.
Question 40
The FNP has a patient who will be undergoing chemotherapy. She confides to the FNP that this is causing her quite a bit of
worry. Which of the following statements would NOT be considered appropriate therapeutic communication for this patient?
Tell me what you would like to know about your treatment.
Tell me what the doctor told you and maybe I can be more helpful.
What will it be like for you when you go for your chemotherapy?
Don’t worry, I will have the doctor come in and explain what is going to happen.
Correct answer:
Don’t worry. I will have the doctor come in and explain what is going to happen.
This is not therapeutic communication. The FNP is making light of the patient’s fears and dismissing them by telling her not to
worry.
Question 41
Treatment of a patient with heatstroke may include all of the following EXCEPT:
use of tepid sprays
fanning
rapid cooling by ice packing
appliance of cold packs to selected areas
Correct answer:
rapid cooling by ice packing
Rapid cooling by ice packing is discouraged because this can stimulate cutaneous vasoconstriction, inhibiting heat loss. Also,
rehydration should be aggressive but with careful monitoring.
Question 42
Upon evaluation of a 56-year-old Caucasian female who presents with stress incontinence, she asks questions regarding
etiology and management. The nurse practitioner has knowledge that factors that contribute to this condition include:
pelvic floor weakness
urinary tract infection
detrusor overactivity
urethral stricture
The Correct answer is:
Pelvic floor weakness
Stress incontinence is a result from weakness of the pelvic floor and urethral muscles and is more common form of
incontinence in women. If a urinary tract infection (UTI) is present, it should be treated but is not the cause of this condition.
Urge incontinence is a result of detrusor overactivity. Additionally, a urethral obstruction is caused by a stricture.
Question 43
313
Which of the following would not be considered a body change related to menopause?
atrophied ovaries
labia and vagina become atrophic and thinner
urinary incontinence
palpable ovaries
Correct answer:
palpable ovaries
Palpable ovaries are considered an abnormal finding in menopausal women. After several years of menopause, the ovaries are
atrophied and a smaller size. Ovarian cancer must be ruled out when a palpable ovary is found.
Question 44
Research on human subjects requires adherence to ethical principles and protection of human rights. These ethical principals
include which of the following?
freedom from harm
freedom from exploitation
right to self-determination
all of the above
Correct answer:
all of the above
All of the choices are correct. Ethical principles for research on human subjects include: freedom from harm, freedom from
exploitation, the right to self-determination, the right to full disclosure, the right to fair treatment and the right to privacy
Question 45
You are examining a 5-month old infant. You would expect to see all of the following in this infant EXCEPT:
loss of tonic neck reflex
able to release an object on request
raises head and chest with arms extended
able to show displeasure by facial expressions
Correct answer:
able to release an object on request
You would not expect the child to be able to release an object on request. This happens around 9 months of age
Question 46
Which of the following statements is incorrect in regard to treatment of rheumatoid arthritis?
The goal of treatment of patients with RA is to reduce inflammation and pain, while preserving function and preventing
deformity.
Water exercise is helpful.
Behavioral management is important.
Traditional DMARDS to minimize the risk of joint damage include both methotrexate and NSAIDs.
Correct answer:
Traditional DMARDS to minimize the risk of joint damage include both methotrexate and NSAIDs.
As helpful as NSAIDs are in symptom control, these products do not alter the underlying disease process. They are not one of
the traditional disease-modifying anti-rheumatic drugs (DMARDs).
Question 47
The NP explains to her patient that the medication she is prescribing for his hyperlipidemia can result in a 10 to 25% decrease
in his LDL, 15 to 35% increase in his HDL and 20 to 50% decrease in his Triglycerides. Which of the following is he
prescribing?
Lipitor.
Gemfibrozil.
Nicotinic acid.
Colestipol.
Correct answer: Nicotinic acid The expected outcomes of the preceding medications are as follows: Lipitor: LDL: 20-60%
314
decrease HDL: 5-15% increase Triglyceride: 10-40% decrease Gemfibrozil: LDL: 5-15% decrease HDL: 14-20% increase
Triglyceride: 20-50% decrease Nicotinic acid: LDL: 10-25% decrease HDL: 15-35% increase Triglyceride: 20-50% decrease
Colestipol: LDL: 10-20% decrease HDL: 3-5% decrease Triglyceride: May increase
Question 48
The type of contract that is required in some states to specify the ways in which the NP and physician agree to work together is
which of the following?
independent contractor agreement
collaborative practice agreement
scope of practice agreement
none of the above
Correct answer:
collaborative practice agreement
Collaborative practice agreements are required in some states. This agreement specifies the ways in which the NP and
physician agree to work together, capitalizing on the professional roles of both for the mutual benefit of the patient.
Question 49
In the course of inspecting the oral cavity of a 50-year-old male who is a heavy smoker you find a lesion that you suspect may
be malignant. It is most likely that this lesion is:
painful and red
flat, white, and painful
tender, mobile, and raised
painless and firm
Correct answer:
painless and firm
An oral cancer is usually characterized by a relatively painless, firm ulceration or raised lesion. The lymphadenopathy
associated with oral cancer consists of immobile nodes that are non-tender when palpated.
Question 50
A 13-year-old male presents with tragal tenderness. He has a history of frequent swimming. Swimmer's ear is diagnosed. What
other symptom might he have?
hearing loss
fever
otic itching
kyphosis
The Correct answer is:
Otic itching
Otitis externa is another name for swimmer's ear. It represents an infection of the external canal. This is characterized by tragal
tenderness with light touch of the tragus on the affected side. It is treated with a topical agent along with keeping the ear canals
out of water until the condition is resolved.
Question 51
You have an adult female patient who complains of fatigue, palpitations and lightheadedness. She is tall in stature and thin.
Upon examination you detect a normal sinus rhythm with an S2 “click” followed b a systolic murmur. The diagnosis will most
likely be which of the following?
atrial fibrillation
paroxysmal atrial tachycardia
congestive heart failure
mitral valve prolapse
Correct answer:
mitral valve prolapse
Mitral valve prolapse is more common in tall and thin adult females. It is characterized by a normal sinus rhythm with an S2
“click” followed b a systolic murmur. It is diagnosed by a cardiac ultrasound. These patients have a higher risk of
315
thromboemboli and infective endocarditis.
Question 52
Which of the following types of headache has a higher risk of suicide in males?
cluster headaches
migraine headaches
muscle tension headaches
temporal arteritis
Correct answer:
cluster headache
A cluster headache is a sudden onset of severe “ice-pick” headaches behind one eye that occurs several times a day. The
attacks happen at the same times daily and the cause is unknown. There is a higher risk of suicide in males who have this type
of headache as compared with the other types of chronic headaches.
Question 53
You are conducting an assessment with a patient who presents with a severe sore throat. Upon examination, you see that her
tonsils touch her uvula. How would you rate this patient's tonsils?
2+
3+
5+
2
Correct Answer: 3+ Tonsils which touch the uvula are rated 3+. Further tests will be necessary to diagnose the cause of the
patient's tonsilar swelling
Question 54
In terms of accreditation, which of the following statements is NOT true?
The Joint Commission on Accreditation of Healthcare Organizations has accredited and monitored hospital practice through
an accreditation program.
The accreditation program is mandatory for health plans and medical groups.
Through accreditation, committees set standards and conduct site visits.
Clinics and medical practices are not routinely accredited.
Correct answer:
The accreditation program is mandatory for health plans and medical groups.
This is not true. The accreditation program is voluntary for health plans and medical groups as well as for clinics. However,
health plan accreditation is becoming a business necessity and clinics and medical practice accreditation may become a
standard in the future.
Question 55
In examining a pregnant woman, if the FNP finds that the uterus is about grapefruit size, she knows that it is most likely that
the woman is in what week of pregnancy?
8th
16th
12th
20th
Correct answer:
12th
At 12 weeks the uterus is about softball or grapefruit size. It is rising above symphysis pubis and the uterine fundus is palpable
through the abdominal wall.
Question 56
You are treating an elderly patient with stasis ulcerations. In counseling on the etiology and management of this, the nurse
practitioner knows that the most frequent cause is:
venous insufficiency
diabetes mellitus
fungal dermatitis
arterial insufficiency
316
The Correct answer is:
Venous insufficiency
Stasis ulcers are most commonly caused by venous insufficiency and less commonly caused by arterial insufficiency, diabetes
mellitus, and fungal infections. Poor venous return causes lower extremity edema, which in turn leads to decreased tissue
perfusion and resulting risk of ulcer.
Question 57
Mammography routine screening should begin for women who have no increased risk of breast cancer at what age?
30-years-old
35-years-old
40-years-old
45-years-old
The Correct answer is:
40-years-old
A mammography is a screening tool that identifies breast cancer. The National Institutes of Health (NIH) and the American
Cancer Society recommends the starting age for an annual mammography screening at 40.
Question 58
Which of the following tests is considered the gold standard for diagnosing endometriosis?
MRI
pelvic ultrasound
laparoscopy
none of the above
Correct answer:
laparoscopy
Laparoscopy is considered the gold standard for diagnosing endometriosis. It enables direct visualization of endometrial
implants.
Question 59
The person who brings a malpractice suit against the nurse practitioner must prove all of the following EXCEPT:
duty
acceptance of risk
proximate cause
damages
Correct answer:
acceptance of risk
This is not part of what a plaintiff must prove in a malpractice suit. The plaintiff has the burden of proving the following four
elements: duty, breach of duty, proximate cause, and damages.
Question 60
A patient with sensitive skin had a TB skin test but has no symptoms. He returns with redness and swelling that appears
positive. What is the next step?
Prescribe medication.
Schedule a chest x-ray.
Repeat the test.
Quarantine the patient.
Correct answer: Schedule a chest x-ray A chest x-ray shows signs of TB and is more accurate than the skin test. The skin test
may appear positive if there is exposure to the illness or allergic reaction. Because the patient has no symptoms, the chest xray is recommended.
Question 61
The optimum daily intake of calcium for a woman who is 30 years old is which of the following?
1200 mg
800 mg
317
1500 mg
1000 mg
Correct answer:
1000 mg
For women between the ages of 25 to 50 years the optimal daily intake of calcium is 1000 mg. After 50 years, women who are
postmenopausal need 1500 mg per day.
Question 62
The nurse practitioner is treating a middle-aged female who has a history of duodenal ulceration. The patient is also currently
positive for a Helicobacter pylori infection. The Family Nurse Practitioner would prescribe which of the following to prevent
recurrence?
a proton pump inhibitor
instant relief antacids around the clock
antimicrobial therapy
a histamine 2 receptor antagonist
The Correct answer is:
antimicrobial therapy
There are several treatment options for a peptic ulcer, including antacids, proton pump inhibitor and Histamine-2 (H2)
blockers. Since the patient has a history of duodenal ulceration and has tested positive for a Helicobacter pylori infection, the
patient may be prescribed an antibiotic to treat the bacterial infection and a proton pump inhibitor to reduce the acid
production. A peptic ulcer is a sore that forms in the stomach or duodenum lining and penetrates the lining with stomach acid,
along with the juices produced during digestion. A duodenal ulcer is the more common form of a peptic ulcer with up to 90%
of the patients diagnosed with it having the Helicobacter pylori infection as the bacterial cause
Question 63
You have a patient who has recurrent, repetitive, and intrusive thoughts that are extremely difficult for her to control. This
patient has which of the following disorders?
panic disorder
obsessive-compulsive disorder
social phobia
generalized anxiety disorder
Correct answer:
obsessive-compulsive disorder
Patients with obsessive-compulsive disorder (OCD) have recurrent, repetitive, and intrusive thoughts and/or behaviors that are
extremely difficult or impossible to control. They are unreasonable, excessive, and interfere significantly with a person’s
ability to function.
Question 64
Any condition or substance such as hyperthyroidism or caffeine that irritates or overstimulates the heart can cause which of the
following conditions?
hypertension
atrial fibrillation
PVD
DVT
Correct answer:
atrial fibrillation
Atrial fibrillation is caused by any condition or substance that irritates or overstimulates the heart. Examples are
hyperthyroidism, cocaine, caffeine, or alcohol.
Question 65
In using warfarin as a long-term therapy in patients with DVT which of the following statements is incorrect?
Warfarin acts against coagulation factors II, VII, IX and X.
Prothrombin time is used as a measure of the efficacy of warfarin.
INR prolongation is seen in about 5 to 6 days after the first warfarin dose.
Approximately 2 – 10% of patients taking warfarin develop hemorrhage.
318
Correct answer:
INR prolongation is seen in about 5 to 6 days after the first warfarin dose.
This is incorrect. INR prolongation is seen in about 48 - 72 hours after the first warfarin dose. INR is the international
normalized ration. The INR range during warfarin therapy should be 2.0 to 3.0.
Question 66
The part of the joint anatomy that acts as padding and is filled with synovial fluid is which of the following?
articular cartilage
bursae
ligaments
synovial space
Correct answer:
bursae
The bursae are present on the anterior and posterior areas of a joint. They act as padding and are filled with synovial fluid.
Question 67
An elderly patient appears distraught and explains in an interview that there are no friends or family nearby. What is this
patient at risk for?
Heart disease.
Chest pain.
Headache.
Depression.
Correct answer: Depression Lack of socialization is evident in the interview. Loss of socialization contributes to depression.
This patient is at risk for depression.
Question 68
The FNP has a patient who has been diagnosed with a serious illness. The patient demands to know from the the FNP why
she is the one to have this disease. She vehemently states that she is too young to have to contend with a serious illness. Which
of the following stages of grief is this patient demonstrating?
depression
denial
anger
bargaining
Correct answer:
anger
This patient is displaying the anger stage of grief. The five stages of grief include: denial, anger, bargaining, depression and
acceptance.
Question 69
For a healthcare professional to obtain blood at the request of law enforcement officials without a patient’s consent which of
the following must be present?
The suspect must be under arrest.
A delay in drawing blood would lead to destruction of evidence.
The test is performed in a reasonable manner.
all of the above
Correct answer:
all of the above
All of the first three choices are factors that must be present. Two other conditions must also be present: the likelihood that the
blood drawn will produce evidence for criminal prosecution; and the test is reasonable and not medically contraindicated.
Question 70
A 60-year-old male comes to the clinic complaining of a sudden onset of severe frontal headache with eye pain. He tells the
FNP that his vision is blurred and his eyes are tearing. He also sees halos around lights. Which of the following conditions
would the FNP most likely suspect?
primary closed-angle glaucoma
primary open-angle glaucoma
319
subconjunctival hemorrhage
macular degeneration
Correct answer:
primary closed-angle glaucoma
Primary closed-angle glaucoma is a sudden blockage of aqueous humor causing markedly increased IOP causing ischemia and
permanent loss of vision. The symptoms reported by the patient point to primary closed-angle glaucoma more than any of the
other choices.
Question 71
When treating Peyronie's disease, which of the following should be considered for the initial treatment?
trial of vitamin E
oxybutynin (Ditropan)
circumcision
surgery
The Correct answer is:
Trial of Vitamin E
In Peyronie's disease, inflammation of the penis causes fibrous tissue to contract and distort the shape of the penis. With severe
curvature of the penis, sexual functioning is impaired, as an erection is painful and in some cases hindered. The initial
treatment for Peyronie’s disease can include vitamin E, which helps reduce the thickened fibrous tissue and promote healing.
Additional treatment options are Para-aminobenzoate, corticosteroids and radiation therapy. Surgery is not recommended
unless the distortion of the penis is severe. Surgically removing the scar tissue can lead to erectile dysfunction.
Question 72
A nurse practitioner (NP) has worked at a large hospital as an RN and now, as a new NP. She has developed an NP-managed
clinic for hospital employees and is employed by the hospital. The NP is described as a(n):
risk taker
nurse specialist
intrapreneur
entrepreneur
The Correct answer is:
Intrapreneur
An intrapreneur is someone who is able to carve out a specialty role within an existing organization, healthcare setting, or
business setting. An entrepreneur is someone who assumes the financial and personal risks of owning their own business,
which they also operate.
Question 73
The nurse practitioner is treating a patient in a long-term facility whose roommate has been diagnosed with active tuberculosis.
The patient should be started on chemoprophylaxis:
within 72 hours after PPD skin test results are obtained
only if PPD skin test results are positive
within 3 months if the repeated skin test is positive
as soon as possible and initiated at the time of the screening skin testing
The Correct answer is:
As soon as possible and initiated at the time of the screening skin testing
Chemoprophylaxis is initiated at the time of the screening skin testing. This testing should be repeated in 3 months if initial
test results are negative. If the second skin testing is negative, chemoprophylaxis can be stopped.
Question 74
The nurse practitioner (NP) is examining a 2-month-old infant in the office and suspects infantile colic. A complete physical
examination reveals no abnormalities. What further study might the NP order?
blood culture
CBC and differential
chest radiograph
none of the above
The Correct answer is:
None of the above
320
Once careful evaluation has been done and no evidence of other problems are noted, it is important to explain to the parents
that no medication is available to treat the condition. The parents need to be informed that the colic most often resolves with
no problems by age 3 months and no testing is necessary.
Question 75
The following are all reasons to keep a child out of school or daycare EXCEPT for which of the following?
diarrhea with blood or mucus
purulent conjunctivitis
streptococcal pharyngitis, until 3 days after initiation of treatment
impetigo, until 24 hours after initiation of treatment
The Correct answer is:
Streptococcal pharyngitis, until 3 days after initiation of treatment
The child may go back to school 24 hours after initiation of treatment for streptococcal pharyngitis. The remaining answer
choices are all correct. Other childcare exclusions are: (1) pediculosis, until after the first treatment is given, (2) scabies, until
after treatment is given, (3) varicella, until all lesions are crusted over, (4) pertussis, until after 5 days of antibiotics therapy,
(5) hepatitis A, until 1 week after illness began and (6) tuberculosis, until no longer infectious.
Question 76
A 5-year-old girl (weight 18 kg) is diagnosed with bilateral otitis media. Her last ear infection was 6 months ago, and she had
no known drug allergies. An appropriate medication to prescribe would be:
corticosteroid otic solution 3 gtt both ears x 10 days
amoxicillin 75-90 mg/kg/day bid x 10 days
doxycycline 250 mg 1 tsp PO tid x 10 days
ampicillin 40-50 mg/kg/day tid x 7 days
The Correct answer is:
Amoxicillin 75-90 mg/kg/day bid x 10 days
The recommended treatment for otitis media in a child is amoxicillin 75-90 mg/kg/day or for this child at 18 kg, 250 mg/5ml,
2.75 tsp bid x 10 days. The other medications are not appropriate for the treatment of acute otitis media.
Question 77
You are providing care to a patient who has experienced an MI. The results of his treadmill stress test are poor, and treatment
with aspirin and heparin have been ineffective. What is likely to be the next intervention for this patient?
Coronary bypass surgery.
Angioplasty.
Cardiac catherization.
Heart transplant.
Correct Answer: Angioplasty Angioplasty, the insertion of a catheter with a balloon on the end into an artery or vein to
remove an obstruction, is the preferred nonsurgical intervention. For patients with poor stress test results and who have not
responded to anticoagulant therapy, angioplasty is likely to be a next step. Coronary bypass may follow if angioplasty proves
unsuccessful.
Question 78
The telescoping of part of the bowel into another part of the bowel is known as which of the following?
Meckel’s diverticulum
incarcerated inguinal hernia
Hirschsprung’s disease
intussusception
Correct answer:
intussusception
Intussusception is the telescoping of part of the bowel into another. It results in decreased blood flow. Ischemia and infarction
of the intestine may occur if it is not treated. Ileocolic intussusception is most common.
Question 79
You are using the CAGE questionnaire to evaluate a client and manage their disease process. The nurse practitioner knows the
major advantage of this is:
321
sensitivity
specificity
brevity
interpretation
The Correct answer is:
Brevity
CAGE stands for Cut, Annoyed, Guilty, and Eye opener. This is used in screening for alcohol abuse. It consists of 4 questions
related to the patient's history. The "C" is for asking if the patient felt the need to cut down. The "A" is to ask if the patient was
annoyed by criticism about his drinking. The "G" is for being guilty about the drinking. The "E" is for eye opener and if the
patient has a need for this. The majority of alcoholics will respond to yes to at least 2 of these questions.
Question 80
Which of the following statements would be incorrect in terms of a patient’s right to informed consent?
A patient has a right to consent to care being given.
A patient does not have the right to refuse care.
Informed consent involves disclosure of material risks of care.
Informed consent requires that there be no coercion in getting a patient to consent to care.
Correct answer:
A patient does not have the right to refuse care.
A patient has a right to consent to the care being given and a right to refuse care that is offered. The FNP has a legal
responsibility to give a patient enough information about risks and benefits of care being offered so that the patient can make
an informed decision to accept or refuse care.
Question 81
In Urie Bronfenbrenner’s person-place-process model of human growth and development which system would include home
and school?
exosystem
macrosystem
microsystems
mesosystem
Correct answer:
microsystems
Microsystems include the immediate settings within which a child spends time during development. Home and school fall into
this system.
Question 82
Which of the following would be considered a high daily dose of the inhaled corticosteroid Flunisolide 250 mcg/puff?
>1250 mcg
>750 mcg
>500 mcg
>2000 mcg
Correct answer:
>2000 mcg
A high daily dose of the inhaled corticosteroid Flunisolide 250 mcg/puff would be >2000 mcg. A low daily dose of this
medication would be 500 – 1000 mcg; a medium daily dose would be 1000 – 2000 mcg.
Question 83
Which of the following skin conditions would be most indicative of Kawasaki disease?
smooth round papules 5 mm in size
Koplik’s spot
honey-colored crusted lesions
bright red and swollen skin palms and soles
Correct answer:
bright red and swollen skin palms and soles
This is the condition most indicative of Kawasaki disease. You may also see strawberry tongue and bright red conjunctivitis
322
with no exudate.
Question 84
All of the following are significant risks for a woman for developing a urinary tract infection EXCEPT:
drinking excessive caffeinated beverages
frequent sexual activity
frequent constipation
taking showers instead of baths
The Correct answer is:
Taking showers instead of baths
No risks for the development of urinary tract infection occurs for adult women while taking a shower. However, bubble baths
are risk factors for urinary tract infections in children. Then, the remaining answer choices, such as frequent constipation,
frequent sexual activity and drinking excessive caffeinated beverages, represent risk factors for developing urinary tract
infections.
Question 85
A female patient who has been treated for a recurrent vaginal yeast infection should be educated to do all but which of the
following?
Wear cotton underwear.
Douche daily.
Avoid douches and douche sprays.
Wear loose-fitting pants.
Correct Answer: Douche daily Patients with recurrent yeast infections should avoid douching. It can force bacteria into the
vagina and irritate vaginal tissues. This patient should be educated to wear loose fitting pants and cotton underwear and to
avoid douching and douche sprays.
Question 86
Which of the following is an absolute contraindication for fibrinolysis in ST segment elevation myocardial infarction?
history of chronic hypertension
prior intracranial hemorrhage
pregnancy
active peptic ulcer
Correct answer:
prior intracranial hemorrhage
A history of prior intracranial hemorrhage is an absolute contraindication for fibrinolysis in ST segment elevation myocardial
infarction. The other choices are relative contraindications.
Question 87
A mother brings in her 2-year-old male child who is hoarse and has a “strange-sounding cough.” She tells you that the child
has recently had a bad cold and that the cough came on suddenly the previous night. He does not have a fever. Which of the
following is the most likely diagnosis?
epiglottis
bacterial tracheitis
spasmodic croup
laryngotracheitis
Correct answer:
spasmodic croup
Spasmodic croup occurs in children aged 3 months to 3 years. It may be postviral and occurs suddenly, usually at night. The
child has a barking cough but no fever.
Question 88
A patient who has recently been prescribed Savella for fibromyalgia should be made aware of all but which of the following
potential side effects?
Risk of abnormal bleeding.
Constipation.
323
Diarrhea.
Insomnia.
Correct Answer: Diarrhea Diarrhea is not among the common reported side effects of Savella. Constipation and insomnia are
commonly reported. Savella does elevate the risk of abnormal bleeding, which is a potentially more serious side effect.
Question 89
Which of the following would be considered a clinical manifestation of anaphylaxis?
urticaria
hypotension
headache
all of the above
Correct answer:
all of the above
All of the choices are clinical manifestations of anaphylaxis. Other indicators are: upper airway edema; dyspnea and
wheezing; itch without rash; angioedema; flush; dizziness and syncope; GI symptoms; substernal pain; and seizure
Question 90
The stage of group dynamics in which group members try to establish their own personal identities, and conflict over goals is
dominant is which of the following?
forming
performing
norming
storming
Correct answer:
storming
In the storming stage there is normally some anxiety as members try to create an impression and to test each other, thereby
establishing their own identities. During this stage, competition over the leadership role and conflict over goals are dominant.
Question 91
You are educating a man on the use of sildenafil (Viagra) and understand that patient education includes:
This medication helps almost all men regain erectile function.
With the use of the medication, sexual stimulation is also needed to achieve an erection.
It will take approximately 1 hour before a spontaneous erection will occur.
This medication works great for problems with libido.
The Correct answer is:
With the use of the medication, sexual stimulation is also needed to achieve an erection.
Viagra does not help all men who experience erectile dysfunction (ED). The half-life of Viagra is approximately 4 to 5 hours.
While the effectiveness of Viagra occurs around 1 hour post ingestion, it does not lead to a spontaneous erection. The
medication does not increase sex drive or libido.
Question 92
Your patient is complaining of facial pain that she says begins on the side of her mouth and then shoots up toward her ear. You
suspect which of the following?
cluster headaches
facial migraine
trigeminal neuralgia
herpes zoster
Correct answer:
trigeminal neuralgia
Trigeminal neuralgia is a recurrent unilateral facial pain syndrome. It most often affects the maxillary division of the
trigeminal nerve. Patients complain of pain on one side of the face and may have a small degree of numbness.
Question 93
Which of the following is NOT one of the principles of structural functional family theory?
Families are social systems.
In well functioning families members take on predictable roles.
324
Individuals in the family adopt norms, values and cultural traditions.
Families are composed of large numbers with characteristics of large-group behavior.
Correct answer:
Families are composed of large numbers with characteristics of large-group behavior.
This is not one of the principles of structural functional family theory. It is actually opposite. Families are composed of small
numbers with characteristics of small-group behavior.
Question 94
For treating a simple skin infection, the nurse practitioner knows that which of the following is an appropriate first line
therapy?
metoprolol
cephalexin
inderal
verapamil
The Correct answer is:
Cephalexin
Two first generation antibiotics used to treat skin and skin structure infections are cephalexin and cefadroxil. These antibiotics
do not cover organisms that produce beta-lactamase.
Question 95
You are prescribing an acid suppression drug for your patient who suffers from GERD. Which of the following is NOT an
acid suppression drug?
Prilosec
Tagamet
Axid
Pepcid
Correct answer:
Prilosec
Prilosec is a proton pump inhibitor. All of the other choices are acid suppressants.
Question 96
Which of the following drugs is NOT a selective serotonin reuptake inhibitor (SSRI)?
Prozac
Paxil
Elavil
zoloft
Correct answer:
Elavil
Elavil is not an SSRI. It is a tricyclic antidepressant and unlike SSRIs it is not used as a first line of treatment for depression.
Question 97
Which of the following is NOT correct concerning electronic prescriptions, commonly known as e-prescriptions?
Electronic prescriptions are transmittable through one technical device at present.
Electronic prescriptions can enhance patient safety and compliance, improve prescribing accuracy and cut costs.
The DEA currently inhibits the use of electronic prescriptions for controlled substances.
Electronic prescribing systems are securely linked to pharmacies and major health plans.
The Correct answer is:
Electronic prescriptions are transmittable through one technical device at present
There are many systems available and the e-prescriptions can be delivered through a variety of devices. Hand held devices,
tablet formatted computers, desktop computers, laptop computers and personal digital assistants all can deliver electronic
prescriptions.
Question 98
325
Which of the following drugs would more likely be prescribed for stress incontinence rather than urge incontinence?
Ditropan
Santura
Enablex
Premarin
Correct answer:
Premarin
Hormone therapy is sometimes prescribed for stress incontinence. Conjugated estrogen (Premarin) or Progestin
(Medroxyprogesterone) (if the uterus is present) may be prescribed.
Question 99
Which of the following is most likely to cause chronic pelvic pain rather than acute pelvic pain?
PID
leiomyomas
ovarian cyst
renal calculi
Correct answer:
leiomyomas
Of the choices given leiomyomas are the most likely to cause chronic rather than acute pelvic pain. Other causes of chronic
pelvic pain include: endometriosis, malignancy of the uterus, ovary, or colon; adhesions; and interstitial cystitis.
Question 100
The nurse practitioner knows that it is developmentally important to do what during the physical examination of an adolescent
female?
Discuss only the major areas of abnormality.
Verbally affirm normalcy.
Verbally address problems of sexually transmitted diseases.
Maintain a comfortable silence.
The Correct answer is:
Verbally affirm normalcy
During the adolescent years there is a need to discuss the physical changes that the patient is undergoing. Further, verbal
affirmation of normalcy is necessary to decrease anxiety.
quiz 14 aanp
Question 1
The most common complication, and the cause of most pertussis-related deaths is which of the following?
heart failure
secondary bacterial pneumonia
viral pneumonia
none of the above
Correct answer:
secondary bacterial pneumonia
Infants are at the highest risk for having pertussis-associated complications. The most common complication, and the cause of
most pertussis-related deaths, is secondary bacterial pneumonia.
Question 2
The FNP has a 6-year-old male patient with impetigo. She prescribes antibiotic therapy for the child and tells his mother that
he should not return to school until:
one week after initiation of antibiotic therapy
3 days after initiation of antibiotic therapy
24 hours after initiation of antibiotic therapy
the impetigo has resolved
326
Correct answer:
24 hours after initiation of antibiotic therapy
The treatment of impetigo is important to minimize risk of infectious transmission. Children with impetigo should be kept out
of school or day care for 24 hours after initiation of antibiotic therapy. Family members should also be checked for lesions.
Question 3
You are discussing sleep with an aging adult. It is important the nurse practitioner understands:
Increased delta or stage IV sleep occurs with age.
Decreased sleep latency occurs with age.
An aging adult has decreased rapid-eye-movement (REM) sleep.
There are decreased nocturnal awakenings with the older adult.
The Correct answer is:
An aging adult has decreased rapid-eye-movement (REM) sleep
REM sleep is associated with skeletal muscle atonia and dreaming. It occurs in three of four regular spaced 10 to 15 minute
cycles and starts 120 minutes after the onset of sleep. With age, REM decreases. Delta sleep, or stage IV, is a deep sleep and
also decreases with age. Nocturnal awakening and sleep latency increases with age.
Question 4
While assessing a patient, a NP noticed symptoms that appeared similar to those of a patient with Alzheimer's. Rather than
immediately treating the patient, the NP ordered more testing. Which symptom did the NP LEAST likely observe?
Decreased response time.
Increased risk for falls.
Increased cognitive performance.
Decreased memory.
Correct answer: Increased cognitive performance The opposite is true. Patients experiencing symptoms of Alzheimer's of lack
of sleep, generally battle with decreased cognitive performance.
Question 5
At which stage of group dynamics can the group be considered as having matured?
performing
norming
adjournment
storming
Correct answer:
performing
At this stage the roles of individual members are accepted and understood. It can be said that the group has matured and it is at
this point that it becomes effective.
Question 6
All of the following are nonpharmacologic treatment options for people with rheumatoid arthritis to complement their
medication routine except:
dietary modification and increase in fluid intake to >3 liters/day
gain weight to prevent further bone loss
complete bed rest during an acute attack
do not exercise a hot inflamed joint
The Correct answer is:
Gain weight to prevent further bone loss
Obese patients with rheumatoid arthritis (RA) would benefit from weight loss or maintaining their current weight in order to
prevent joint overload. Further, the answer choices, dietary modification and increase in fluid intake to >3 liters/day, engage in
complete bed rest during an acute attack and avoiding exercise with a hot inflamed joint, are all good measures to help with
rheumatoid arthritis (RA) symptoms.
Question 7
When treating a patient with suspected cancer, the nurse practitioner understands that the most common cancer deaths in males
and females is directly related to:
colon cancer
lung cancer
327
brain cancer
leukemia
The Correct answer is:
Lung cancer
Lung cancer is the leading cause of mortality in males and females diagnosed with the condition than any other cancer type.
One of the major causes of lung cancer is cigarette smoking. Patients with lung cancer may show symptoms of coughing that
is persistent. In some patients, they cough up blood or have streaks of blood in the sputum. Other symptoms of lung cancer are
fatigue, pain in the chest, weight loss and weakness.
Question 8
A 52-year-old male patient presents in the office and you notice a yellowish plaque on his upper eyelid that is painless. What
should the nurse practitioner look for?
liver function studies
lipid levels
sedimentation rate
vision in the affected eye
The Correct answer is:
Lipid levels
The yellowish painless plaque on the patient's upper eyelid is descriptive of xanthelasma. Additionally with xanthelasma, the
plaque is slightly raised and well-circumscribed on the upper eyelid. Further, one or both eyelids may be affected.
Xanthelasma is often associated with lipid disorders but may occur independent of any systemic or local disease. However,
xanthelasma does not affect vision.
Question 9
An adult female comes to your clinic complaining of a rash on her hands. You ask her if there have been any changes in
detergent, lotion, etc. that come in contact with her hands. She tells you that she changed her brand of dish detergent recently
and that she washes all her dishes by hand. Which of the following would you diagnose for this patient?
contact dermatitis
atopic dermatitis
scabies
psoriasis
Correct answer:
contact dermatitis
Contact dermatitis is a cutaneous reaction to an external substance such as an irritant or allergen. It appears as an asymmetric
distribution of red, raised, and/or inflamed rash, or rash only on exposed areas. 80% of cases are due to universal irritants such
as soap and detergent.
Question 10
Patient medical records play a critical role in utilization review and quality assurance. The nurse practitioner understands that
all of the following are correct statements regarding quality assurance EXCEPT:
Patients' records are a primary source of data for such utilization review and other quality assurance activities.
State laws, statutes, and regulations do not vary on utilization review and quality assurance principles.
Accreditation organizations have standards related to quality assurance and utilization review.
Many states have enacted laws protecting the confidentiality of medical records used in utilization management and quality
assurance functions.
The Correct answer is:
State laws, statues, and regulations do not vary on utilization review and quality assurance principles
State laws vary widely on utilization review. Additionally, statues and regulations mandating quality assurance activities vary
in terms of level of detail.
Question 11
Excluding NPs from being designated as primary care providers, as has been done in some managed care organizations, results
in all of the following EXCEPT:
The NP must work independently of the primary care physician.
The NP becomes a “ghost” provider.
Job security can be threatened since the NP cannot document services provided and revenue generated.
All of the above
Correct answer:
The NP must work independently of the primary care physician.
328
As a result of excluding NPs from being designated as primary care providers, the only option for NPs is to become a salaried
employee. As a salaried employee, the NP contributions are often not visible and may be credited to the collaborating
physician.
Question 12
A pregnant patient is concerned about genetic abnormalities. When is it too late to perform a CVS test?
11 weeks.
6 weeks.
5 weeks.
2 weeks.
Correct answer: 11 weeks Chorionic villus sampling (CVS) can identify genetic abnormalities. It uses the placenta and should
not be done after 10 weeks.
Question 13
Which of the following are you least likely to recommend as a nonpharmacologic treatment for hyperlipidemia?
smoking cessation
decreased use of soy products
increased physical activity
moderation of alcohol intake
Correct answer:
decreased use of soy products
Rather than decreasing soy products, a person with hyperlipidemia should increase his intake of soy products. Diet changes
would also be made.
Question 14
Bony nodules on the proximal interphalangeal joints are none as which of the following?
Heberden’s nodes
Bouchard’s nodes
degenerative joint nodes
none of the above
Correct answer:
Bouchard’s nodes
Bouchard’s nodes are bony nodules on the proximal interphalangeal joints. Heberden’s nodes are bony nodules on the distal
interphalangeal joints.
Question 15
Of the following, which is an early stage of Lyme disease?
erythema migrans
tinea versicolor
fragile bulla
“sandpaper” rash
Correct answer:
erythema migrans
Erythema migrans involves red target lesions that expand and grow in size. It is an early stage of Lyme disease.
Question 16
Which of the following drugs used for peripheral vascular disease is a neuropathic analgesic?
Keflex
Dynapen
Cipro
Tegretol
Correct answer:
Tegretol
Tegretol is a neuropathic analgesic. The other drugs are not. Other neuropathic alagesics are Neurontin and Elavil.
Question 17
329
Which of the following screening and assessment tests used in child health supervision is NOT used exclusively for
developmental screening and assessment for infants and/or toddlers?
Denver II
HOME scale
Ages and Stages Questionnairre
First Step
Correct answer:
HOME scale
The HOME scale is not used exclusively for infants/toddlers. It is the Home Observation for Measurement of the Environment
scale. It has infant, preschool and elementary school versions.
Question 18
You are considering prescribing birth control to a 21-year old who has previously been using condoms. Which of the
following is a contraindication to combined oral contraception (COC)?
cigarette smoking
a personal history of untreated hepatitis C
presence of factor V Leiden mutation
first-degree relative with breast cancer
The Correct answer is:
Presence of factor V Leiden mutation
The known thrombotic mutation, factor V Leiden, can lead to venous thromboembolism and combined oral contraceptives are
not to be used. The World Health Organization (WHO) has published four categories of precautions for use of combined oral
contraception (COCs) and category 4 "Refrain From Use" lists the factor V Leiden. Cigarette smoking in those over 35 is a
contraindication but a personal history of hepatitis C and a relative with breast cancer are not contraindications.
Question 19
Quality assurance is a system designed to evaluate and monitor quality of care. Which statement is NOT an accurate
concerning quality assurance?
These programs identify components of structure, process and outcomes of care.
These programs look at organization effectiveness, efficiency and client and provider interactions.
These programs audit financial records and payroll of the organization to evaluate appropriateness of monetary distribution.
These programs promote responsibility and accountability to deliver high-quality care and assist in the evaluation and
improvement of patient care.
The Correct answer is:
These programs audit financial records and payroll of the organization to evaluate appropriateness of monetary distribution
Payroll and financial records of an organization are not evaluated and viewed in a quality assurance program. These programs
are set up to evaluate and monitor the care of patients by providers, ancillary staff, personnel and the facility in general.
Question 20
It is common for the nurse practitioner (NP) to deal with anxiety and depression in the clinic setting. To treat and understand
these conditions, the NP knows that presentation includes:
dry mouth
feeling elated
energetic
psychomotor agitation
The Correct answer is:
Psychomotor agitation
There are many signs and symptoms of anxiety and depression, which are are often seen together in a patient presenting for
primary care. Psychomotor agitation with fidgeting and irritability is often found in patients with these disorders. Being
energetic and feeling elated are more common diagnostic criteria of bipolar disorder. Other symptoms are changes in appetite
and sleep, fatigue, decrease of self-worth, problems with concentration, loss of interest in usual things and thoughts of death.
Question 21
A 67-year-old man is in the clinic with frequent dizziness while standing. Orthostatic hypotension can be diagnosed in an
older adult if the systolic blood pressure decreases:
more than 20 points any time after rising
more than 20 points within one minute after rising
330
any degree drop if the patient becomes weak or dizzy
more than 20 points within three minutes after rising
The Correct answer is:
More than 20 points within three minutes after rising
Orthostatic hypotension, also called postural hypotension, is diagnosed in older adults when the systolic blood pressure drops
20 mm Hg or more within 3 minutes of moving to an upright position. It can be evaluated from lying to sitting or from sitting
to standing. Also, if the systolic reading does not drop but the diastolic drops by 10 mm Hg or more, postural
Question 22
Which of the following pathogens is the most common cause of fatal community-acquired pneumonia?
M. pneumoniae
H. influenzae
Legionella spp.
S. pneumoniae
Correct answer:
S. pneumoniae
S. pneumoniae is the most common cause of fatal community-acquired pneumonia. It is a Gram-positive diplococci.
Question 23
Which of the following conditions is most likely to be found in a person with acute cholecystitis?
fever
jaundice
vomiting
palpable internal joint
The Correct answer is:
Vomiting
Acute cholecystitis symptoms include vomiting, right upper abdomen pain, and nausea. Fever, jaundice and a palpable internal
joint are not seen in patients with cholelithiasis. Cholelithiasis is defined as a condition in which there is the formation of
calculi or gallstones.
Question 24
Which of the following statements about communication in the nursing profession is least accurate?
Communication should have objective and purpose.
Credibility is very important.
Messages should convey something of value to the receiver.
The organizational climate is not important in most communication.
Correct answer:
The organizational climate is not important in most communication.
This is the least accurate statement. The organizational climate is important in most communication. This includes appropriate
timing and physical setting to convey the desired meaning of the communications.
Question 25
Which of the following is an example of objective data used for diagnosis?
Patient complains of dizziness.
Patient's blood pressure is 180/90.
Patient's blood pressure is elevated.
Patient is hypertensive.
Correct Answer: Patient's blood pressure is 180/90 Objective data must be measurable and verifiable. It is key to collect both
objective and subjective data to form correct differential diagnoses and then to determine the correct diagnosis and course of
further testing or treatment for a patient. The ability to synthesize both objective and subjective data is a key skill for NPs.
Question 26
When counseling a male patient on impotence, the nurse practitioner understands the following to be true:
It is caused by infections only.
It is not the result of vascular problems.
331
It can be caused by antihypertensives.
It is not the result of multicausal factors.
The Correct answer is:
It can be caused by antihypertensives
Impotence is caused by many different things and is not specifically caused only by an infection. It can be caused by
antihypertensives, like nonselective beta-blockers, that decrease vascular resistance and central effect. Other known causes are
infections, vascular disorders, trauma, and psychological disorders.
Question 27
You are treating a 60-year-old depressed female and started an SSRI. When should another antidepressant be tried if there is
no response?
5-10 days
2-4 weeks
4-6 weeks
8-12 weeks
The Correct answer is:
8-12 weeks
Most authorities agree that if there is no response by 8-12 weeks at a maximum therapeutic dose, a different antidepressant
should be tried. The time frame is adequate to achieve effectiveness.
Question 28
When treating a 26-year-old Caucasian female for secondary amenorrhea, it is important for the nurse practitioner to have
knowledge that:
Approximately 30% of patients with secondary amenorrhea have associated genetic abnormality.
Pregnancy is the most common cause of secondary amenorrhea.
It is commonly seen in Turner syndrome.
It is often cause by a sexually transmitted disease.
The Correct answer is:
Pregnancy is the most common cause of secondary amenorrhea
Secondary amenorrhea is a minimum of 3 missing cycles or 6 months of cessation of menses after menarche is established.
The most common cause of secondary amenorrhea is pregnancy with other causes being polycystic ovary disease,
hypothalamic amenorrhea, thyroid dysfunction and hyperprolactinemia being others. Approximately 30% of the patients with
primary amenorrhea, not secondary amenorrhea, have associated genetic abnormalities. Further, primary amenorrhea, and not
secondary amenorrhea, is commonly seen in Turner syndrome. Sexually transmitted disease does not cause this.
Question 29
Your patient has a squamous cell carcinoma. You understand that this presents as which of the following?
scar-like lesion, whitish/yellow, smooth and shiny
firm to hard erythematous, scaly nodule
blue, brown, or black waxy papule
none of the above
Correct answer:
firm to hard erythematous, scaly nodule
Squamous cell carcinoma usually presents as firm to hard, erythematous, scaly or ulcerated nodules. Squamous cell
carcinomas are isolated, keratotic, eroded, ulcerated lesions that grow rapidly.
Question 30
You are providing care to a female patient who reports having 1 to 2 migraines per month. She asks if there are any foods she
should avoid. Which of the following foods are NOT known to trigger migraines in some people?
Dairy.
Bleu cheese.
Lean red meat.
Red wine.
Correct Answer: Lean red meat Lean meat and poultry are not known to trigger migraines. Dairy, especially full-fat dairy, can
trigger migraines for some people. Cheeses containing tyramine, including bleu cheese, can also be triggering. Alcohol,
especially red wine and beer, may also be trigger foods for some people. The patient should note any specific triggers she has
as well and avoid or limit those foods.
Question 31
332
As an NP you understand that the anatomy of the skin involves three layers and that the fat and sweat glands are located in
which of the following layers?
epidermis
dermis
bottom epidermis
subcutaneous
Correct answer:
subcutaneous
The skin has three layers: epidermis (top and bottom), dermis, and subcutaneous. The subcutaneous layer includes fat, sweat
glands, and hair follicles.
Question 32
When recommending cardiopulmonary conditioning for a patient, you would calculate the target heart rate for 30 minutes of
exercise. If a patient is 50 years old what would his target heart rate be?
170
153
136
119
Correct answer:
136
The target heart rate that should be sustained is found by subtracting the patient’s age from 220 and multiplying the result by
0.8. In this case 220 - 50 = 170; and 170 x 0.8 = 136.
Question 33
Which of the following is NOT a normal physiologic change during pregnancy?
Cardiac output increases by 1/3 by the last two trimesters.
Plasma volume decreases by 50% by the end of the third trimester.
Decreased systolic and diastolic pressure occurs in the first trimester.
There is decreased peristalsis from progesterone effects.
Correct answer:
Plasma volume decreases by 50% by the end of the third trimester.
This would not be a normal physiologic change during pregnancy. Plasma volume increases by 50% by the end of the third
trimester. Hemoglobin and hematocrit are decreased because of the hemodilution from increased plasma volume. And the
heart is displaced upward and to the left in the late second to third trimester.
Question 34
Which of the following is a pansystolic murmur that is heard best at the apex or the apical area, radiates to the axilla and is a
loud-blowing and high-pitched murmur?
tricuspid regurgitation
aortic stenosis
mitral regurgitation
none of the above
:
Correct answer:
mitral regurgitation
Mitral regurgitation is a pansystolic murmur that is heard best at the apex or the apical area, radiates to the axilla and is a loudblowing and high-pitched murmur. The diaphragm of the stethoscope is used to detect mitral regurgitation.
Question 35
Before deciding to provide a detailed reference on a former employee, it would be important for the Family Nurse Practitioner
to consider which of the following?
If the comments are perceived as negative and the former employee becomes aware of them, the nurse could be subjected to
a lawsuit for defamation.
If the employee exhibits unsafe patient care practices, it may be wiser to risk legal action from the employee than to subject
future patients to this unsafe practitioner.
The requirements of the human resources department should be determined.
all of the above
333
The Correct answer is:
All of the above
If the human resources department has established guidelines for the handling of references, it would be wise to follow them.
It is also an effective mechanism to deflect potentially problematic queries by passing requests to the department most
equipped to circumspect about particular released information. Without protective state legislation, employers should be
prudent about sharing comments on former employees' work histories.
Question 36
Your patient has an intense and brief sharp stabbing pain on one cheek that is aggravated by cold food, cold air and chewing.
Of the following which is most likely the cause?
migraine
trigeminal neuralgia
temporal arteritis
cluster headache
Correct answer:
trigeminal neuralgia
Trigeminal neuralgia (Tic Douloureux) is a unilateral headache caused by impingement or inflammation of the trigeminal
nerve (CN5). It is rare before age 35 and peaks at age 60.
Question 37
Sigmund Freud developed a stage theory of psychosocial development. At which age does a person reach the genital stage?
preschool
school age
adolescence
post-adolescence
Correct answer:
adolescence
Adolescence is the genital stage according to Freud. The stages are: infancy – oral; toddler – anal; preschool – phallic; school
age – latency; and adolescence – genital.
Question 38
Gross painless hematuria is the most common presenting sign of bladder cancer. Persistent microscopic hematuria is the only
finding in about what percentage of individuals presenting with bladder cancer?
20%
25%
35%
50%
Correct answer:
20%
Persistent microscopic hematuria is the only finding in about 20% of individuals presenting with bladder cancer. Irritative
voiding symptoms and urinary frequency without fever are reported occasionally. Abdominal mass is palpable only with
advanced disease.
Question 39
Which of the following diseases/conditions has symptoms of fever, fatigue and weight loss; bleeding gums, nosebleeds, pallor,
easy bruising, and petechiae in addition to bone pain?
acute hemorrhage
acute leukemia
Hodgkin’s lymphoma
neutropenia
Correct answer:
acute leukemia
Acute leukemia is a cancer of the hematopoietic progenitor cells. The patient can have fever, fatigue and weight loss; bleeding
gums, nosebleeds, pallor, easy bruising, and petechiae in addition to bone pain.
334
Question 40
You are seeing an uncircumcised client that presents with a complaint of not being able to retract the foreskin over the glans
penis. What is the likely diagnosis?
phimosis
paraphimosis
lateral phimosis
Peyronie's disease
The Correct answer is:
Phimosis
Phimosis results in the patient's inability to retract the foreskin from behind the glans penis. This condition occurs at any age
and is the result of poor hygiene and chronic infection.
Question 41
Madeline Leininger envisioned blending the disciplines of nursing and anthropology to enable a cultural perspective for
nursing. This is known as which of the following?
ethnocentric nursing
transcultural nursing
enculturation
cultural transference
Correct answer:
transcultural nursing
Transcultural nursing is a term coined by Leininger. It involves blending nursing and anthropology in both theory and practice
to enable a cultural perspective for the professional practice of nursing. It helps nurses to provide care that is culturally
competent.
Question 42
You have initiated treatment for your patient with fibromyalgia syndrome (FMS). You should do a follow-up visit:
within 48 hours
within 1 week
in 2 weeks
in a month
Correct answer:
in a month
A follow-up visit in one month is appropriate for this patient. This should continue monthly until improvement in symptoms is
noticed.
Question 43
An example of a secondary prevention measure for a 55-year-old healthy and physically active female is which of the
following?
yearly flu shot
recommending the use of bicycle helmets
educating her about skin cancer from sun exposure
doing a mammogram and Pap smear
Correct answer:
doing a mammogram and Pap smear
Secondary prevention measures include activities provided to identify and treat asymptomatic persons who might be at risk for
a particular disease or condition. Doing a mammogram and Pap smear is a secondary prevention measure. All of the other
choices are primary prevention measures.
Question 44
When counseling a mother on her newborn that presents with a concerning skin lesion, the nurse practitioner has knowledge
that:
Erythema toxicum or infant acne is extremely rare and should be referred to a specialist.
Milia or prickly heat rash only occurs in warm regions of the world.
A Mongolian spot is a blue to black-colored patch or stain that is commonly located on the face of Caucasian children.
Strawberry hemangiomas are raised vascular lesions with a bright red color and up to 95% involute spontaneously.
335
The Correct answer is:
Strawberry hemangiomas are raised vascular lesions with a bright red color and up to 95% involute spontaneously
Erythema toxicum is common and there is no need for referral as it resolves spontaneously. Milia rash occurs with overheating
of the infant in all regions of the world. Mongolian spots are common in African American, Asian, Hispanic, and Native
American infants and are usually in the lumbosacral area.
Question 45
There are still many health care disparities in the United States. A setting with a shortage of personal health services is known
as which of the following?
underrepresented minorities
disadvantaged
vulnerable
medically underserved community
Correct answer:
medically underserved community
A medically underserved community is considered to be a setting with a shortage of personal health services.
Underrepresented minorities are racial and ethnic populations whose representation among the health professions is lower than
their proportion of the general population. Disadvantaged means inhibited from knowledge, skills and abilities to participate in
the healthcare system as a provider or recipient. Vulnerable mean that a person is open to physical, emotional and/or
socioeconomic harm.
Question 46
A person with which type of skin would need longer periods of sun exposure to produce vitamin D?
light skin
freckled skin
medium complexion
darker skin
Correct answer:
darker skin
In terms of vitamin D synthesis, a person with darker sin would need longer periods of sun exposure to produce vitamin D. A
deficiency of vitamin D in pregnancy results in infantile rickets.
Question 47
Which of the following statements about tendonitis and its treatment is false?
With rotator cuff involvement, the likelihood of concurrent bursitis is low.
When the hand or wrist is affected, splinting and NSAIDs are reasonable first-line therapies.
Achilles tendonitis may necessitate treatment with a posterior splint.
There is a 10% risk of tendon rupture with recurrent Achilles tendonitis.
Correct answer:
With rotator cuff involvement, the likelihood of concurrent bursitis is low.
This statement is false. With rotator cuff involvement, the likelihood of concurrent bursitis is high. Treatment includes limiting
overhead movement and intrabursal corticosteroid injection.
Question 48
During an assessment of a patient, which of the following does NOT indicate depression?
Weight change.
New interests.
Loss of energy.
Loss of appetite.
Correct answer: New interests Depressed people tend to withdraw and loss interest in their passions. The other answers are
indicators that a patient is suffering from depression.
Question 49
Your adult female patient comes to you complaining of varicose veins. After confirming the diagnosis you would recommend
which of the following?
336
refrain from elevating the leg
high sodium diet
Vasotec
elastic compression stockings up to the knee
Correct answer:
elastic compression stockings up to the knee
For varicose veins you might recommend elastic compression stockings up to the knee. You would also recommend weight
loss if necessary and prescribe hydrochlorothiazide when there is fluid retention.
Question 50
You are crafting a plan of care for a 32 year old female patient who has large uterine fibroids. These are causing her intense
discomfort and anemia from heavy periods. She says she hopes to have children within the next several years. Which of the
following procedures is the best choice for this patient?
Partial hysterectomy.
Myomectomy.
Fibroid embolization.
Uterine ablation.
Correct Answer: Myomectomy A myomectomy removes fibroids but leaves the uterus intact. For patients who have child
bearing plans, this is often the best choice. Healthy pregnancies are common after myomectomy. However, fibroids may
return.
Question 51
Which of the following managed care systems is most associated with capitation?
PPO
HMO
POS
fee-for-service
Correct answer:
HMO
Health maintenance organizations have access to services that is based on a fixed monthly fee per enrollee. Control for
utilization is maintained by the HMO.
Question 52
Which of the following characteristics would be most associated with “atypical” pneumonia?
There is usually a sudden onset.
Fever and chills are common.
Lung exam shows dullness, with signs of consolidation.
Pleuritic pain is rare.
Correct answer:
Pleuritic pain is rare.
This is the characteristic of the choices that is most associated with “atypical” pneumonia. Atypical pneumonia usually has a
gradual onset. Fever and chills are rare; and there are often minimal abnormal findings in a lung exam.
Question 53
In terms of spontaneous abortions, where there is vaginal bleeding and cramping but the cervix remains closed, it would be
considered a:
threatened abortion
inevitable abortion
complete abortion
incomplete abortion
Correct answer:
threatened abortion
When there is vaginal bleeding and cramping but the cervix remains closed it is a threatened abortion. It is possible in this case
that the pregnancy can be salvaged. In an inevitable abortion the cervix is dilated. In a complete abortion the placenta and fetus
are expelled completely. In an incomplete abortion placental products remain in the uterus and the cervix remains dilated.
337
Question 54
A 12-year-old female patient with Strep pharyngitis has received a prescription for amoxicillin. She has not improved in 72
hours. What course of action is acceptable?
A macrolide antibiotic should be prescribed.
The antibiotic should be changed to a first generation cephalosporin.
The patient should wait another 24 hours for improvement.
A penicillin or cephalosporin with beta lactamase coverage should be considered.
Correct answer:
A penicillin or cephalosporin with beta lactamase coverage should be considered
According some clinical studies, after the administration of antibiotics, such as amoxicillin, the patient diagnosed with
streptococcal pharyngitis should have improvement in symptoms within 48-72 hours. If not, the provider may need to
prescribe the patient a different antibiotic. With the 12-year-old female patient, since a streptococcal organism was diagnosed
and amoxicillin was ineffective, the prescriber should consider that the patient has a resistant streptococcal organism and beta
lactamase coverage is necessary.
Question 55
A father brings in his teenage son. He is sure that the boy is sick because he is lethargic even though he is in bed 8 hours a
night. The boy says that he is tired. When asked if he is sleeping the full 8 hours, he says that he does not fall asleep
immediately. What is a probable explanation?
He has a cold.
He is sleep deprived.
He is taking drugs.
He is faking for attention.
Correct answer: He is sleep deprived Many teenagers need close to nine hours of sleep a night. Additionally, their natural
sleep patterns are later, and he probably is awake later than his father believes. Sleep deprivation is common among teenagers.
Question 56
Persons who are allergic to latex products are often allergic to all of the following EXCEPT:
apples
kiwi
bananas
avocado
The Correct answer is:
Apples
It has been noted that those persons with latex allergy also often have a cross-allergy to kiwi, bananas and avocado. Apples
pose no cross-allergy threat, making this the right answer in this case.
Question 57
Racial and ethnic populations whose representation among the health professions is lower than their proportion of the general
population indicates which of the following?
health disparity
vulnerability
underserved minorities
underrepresented minorities
Correct answer:
underrepresented minorities
Underrepresented minorities consist of racial and ethnic populations whose representation among the health professions is
lower than their proportion of the general population. Underserved minorities are those whose access to healthcare is limited.
Question 58
It has been determined that an elderly patient is suffering from long term insomnia. Which of the following is LESS likely to
be considered a long term cause of the condition?
Nocturia.
Environmental changes.
Congestive heart failure.
Alcohol and substance abuse.
338
Correct answer: Environmental changes There are various causes of insomnia in the elderly. While some may be the cause of
short term insomnia and others long term insomnia, long term insomnia is more common in the elderly than short term.
Question 59
According to the CMS guidelines, medical decision making has three components. Which of the following is NOT one of
these components?
making a diagnosis
counseling patient
choosing treatment options
reviewing data
Correct answer:
counseling patient
Medical decision making has three components: making a diagnosis, choosing treatment options and reviewing data. While
clinicians are expected to document patient counseling and coordination of services, there are no specific guidelines for this
documentation.
Question 60
You are treating a patient with a persistent cough. Which of the following is NOT an accurate statement regarding when to
consult or refer this patient?
Refer if patient is at high risk for malignancy.
Refer if patient refuses to stop smoking.
Refer to a pulmonologist if bronchoscopy is indicated.
Refer if cough is accompanied by blood for at least 2-3 days after there has been treatment initiated for a respiratory
infection.
The Correct answer is:
Refer if patient refuses to stop smoking
The patient's refusal to stop tobacco use is not a reason for referral or consultation, but a reason to counsel.
Question 61
Excessive thick scaling on the scalp of younger infants is which of the following?
erythema toxicum
milaria
seborrheic dermatitis
nevus flammeus
Correct answer:
seborrheic dermatitis
Seborrheic dermatitis (cradle cap) is excessive thick scaling on the scalp of younger infants. It is treated by softening and
removal of the thick scales on the scalp with mineral oil left on the scalp for 10 to 15 minutes before shampooing and
scrubbing off the scales
Question 62
At about what age does the Moro reflex disappear in an infant?
1 – 2 months
4 – 6 months
6 – 12 months
12 – 24 months
Correct answer:
4 – 6 months
The Moro Reflex is the startle reflex. Sudden loud noise causes symmetric abduction and extension of the arms followed by
adduction and flexion of the arms over the body. It disappears by 4 to 6 months.
Question 63
The Global Initiative for Chronic Obstructive Lung Disease (COPD) guidelines specify which medications are indicated for
certain stages of COPD. Which is indicated for all stages of the disease?
an inhaled corticosteroid
a short-acting inhaled bronchodilator
theophylline
guaifenesin
339
The Correct answer is:
A short-acting inhaled bronchodilator
Albuterol and pirbuterol are beta 2 agonists that are bronchodilator. They are indicated as rescue drugs for treatment of acute
bronchospasm for any of the stages of chronic obstructive lung disease.
Question 64
Which type of seizure is characterized by unusual sense of smell or taste, visual or auditory hallucinations, and stomach upset
followed by a vague stare and facial movements, muscle contraction and relaxation?
tonic-clonic (grand mal)
myoclonic
complex partial
absence (petit mal)
The Correct answer is:
Complex partial
The onset age of complex partial seizures occurs at any age. Patients with complex partial seizures have aura that is
characterized by unusual sense of smell or taste, visual or auditory hallucinations, stomach upset followed by muscle
contraction and relaxation and autonomic signs. This can progress to loss of consciousness.
Question 65
In advising a pregnant woman, which of the following would be incorrect?
Always wear a seat belt (below the uterine fundus).
Avoid exercise in hot and humid weather.
Relax in a hot tub or sauna when stressed.
Eat no raw shellfish or raw meat.
Correct answer:
Relax in a hot tub or sauna when stressed.
The FNP would not advise a pregnant woman to use a hot tub or sauna. She would be advised not to use them.
Question 66
Part of the job of an FNP is educating parents on safety measures for children. Which type of car seat or restraint is
recommended for toddlers and preschoolers?
seat belt
booster
rear-facing convertible
forward-facing convertible, combination
Correct answer:
forward-facing convertible, combination
Toddlers and preschoolers (children 1 year old and weighing at least 20 lb) can ride in a convertible, combination, forwardfacing seat. It is best to ride rear-facing as long as possible, however.
Question 67
As a nurse practitioner, you understand that rabies has a nearly 100% fatality rate. Which of the following characteristics is
evidence of this?
communicability
immunogenicity
pathogenicity
virulence
Correct answer:
virulence
Virulence is defined as the severity of disease that an organism can produce, measured by criteria such as number of days in
bed or the frequency of serious sequelae including death. Rabies virus is highly virulent (nearly 100% fatality rate); poliovirus
is moderately virulent; varicella and rhinovirus are of low virulence (almost zero fatality rate).
Question 68
In the course of examining a 6 month old infant for a well-baby checkup you note that he has persistent fisting. You
understand that this indicates which of the following?
The child is right on schedule for development.
340
This is a warning sign of a possible developmental problem.
This is not average for a 6 month old infant but is no cause for concern.
Fisting is expected for a child of this age.
Correct answer:
This is a warning sign of a possible developmental problem.
Persistent fisting is a warning sign of a possible developmental problem. Other warning signs for a child of this age are:
persistence of hand regard; failure to follow 180° (for both near and far objects); and preference of one hand.
Question 69
The usefulness of abdominal ultrasound to identify pancreatic cancer is limited by:
patient’s age
the presence of intestinal gas
technician’s skill
size of the tumor
Correct answer:
the presence of intestinal gas
Abdominal CT scan is helpful in identifying pancreatic cancer. The usefulness of abdominal ultrasound is limited by the
presence of intestinal gas.
Question 70
A professional designation such as "family practice nurse" is protected by law. This is called:
scope of practice
title protection
certification
licensure
Correct answer:
title protection
Professional designation such as "family practice nurse" is protected by law. It cannot be used by anyone who does not meet
the educational criteria for a family practice nurse. Laws such as this help protect the public from unlicensed persons. This is
called title protection.
Question 71
Treatment options for adult seizure patients include al of the following medications EXCEPT:
gabapentin
phenytoin
carbamazepine
aminophylline
The Correct answer is:
Aminophylline
Numerous standard seizure therapies exist including gabapentin, phenytoin, carbamazepine, ethosuximide, lamotrigine,
topiramate, valproic acid and clonazepam. However, aminophylline is not used to treat seizures.
Question 72
Title XIX of the Social Security Act is also known as which of the following?
Americans with Disabilities Act
Medicare
OSHA
Medicaid
Correct answer:
Medicaid
Title XIX of the Social Security Act is also known as Medicaid which is a federal and state matching program that offers
medical assistance to low income persons, the disabled, blind or members of families with dependent children. It pays for
health care, nursing home, and prescription drugs.
Question 73
The FNP has a pregnant patient whose uterine is grapefruit size. At what stage of pregnancy is this patient most likely to be?
12 weeks
16 weeks
341
10 weeks
8 weeks
Correct answer:
12 weeks
At 12 weeks the uterus is softball or grapefruit size. It rises above the symphysis pubis and the uterine fundus is palpable
through the abdominal wall.
Question 74
Which of the following persons developed the first standardized measurement of intelligence?
G. Stanley Hall
Alfred Binet
Arnold Gesell
Sigmund Freud
Correct answer:
Alfred Binet
Alfred Binet would be considered a proponent of the nature side of the nature vs. nurture debate. He developed the first
standardized measurement of intelligence. The intention was to identify “mentally defective” children needing specialized
education.
Question 75
Of the following choices which condition is most likely to have a positive Gram stain, positive U/A and hyperperfusion on
ultrasound?
testicular torsion
testicular tumor
UTI
epididymitis
Correct answer:
epididymitis
Epididymitis is an infectious process of the epididymis. Of the choices it is the most likely to have a positive Gram stain,
positive U/A, and hyperperfusion on ultrasound. The symptoms have a gradual onset worsening over time.
Question 76
All of the following are factors that would protect a person from the possibility of gastroenteritis EXCEPT:
normal bacterial flora of the intestine
acidity of the stomach
normal motility of the GI tract
taking antacids
Correct answer:
taking antacids
Loss of normal gastric acidity, such as in those taking antacids or H2-receptor antagonists, is a factor that increases
susceptibility to gastroenteritis. Gastric surgery, increased age, and poor physical condition are also factors contributing to
susceptibility.
Question 77
When managing the patient on Lithium therapy, which of the following would be least important for the nurse practitioner to
evaluate?
urinalysis, electrolytes
alanine aminotransferase, aspartate aminotransferase, lactate dehydrogenase
thyroid-stimulating hormone, triiodothyronine, thyroxine
blood urea nitrogen, creatinine
The Correct answer is:
Alanine aminotransferase, aspartate aminotransferase, lactate dehydrogenase
It is not important to evaluate liver function before initiating treatment with lithium. However, lithium has adverse side effects
on renal, cardiac, and thyroid function. Baseline electrolytes are also important to obtain.
Question 78
342
You need to assess cerebellar function in an exam. Which of the following is appropriate?
Patient quickly alternates moving fingers to and from the nose.
Patient sits and resist force on the arms.
Patient slowly walks a straight line in a darkened room.
Patient sits and resist force on the legs.
Correct answer: Patient quickly alternates moving fingers to and from the nose A stroke can affect cerebellar function. A
patient quickly alternates moving fingers to and from the nose to assess cerebellar function. This movement should be
smooth.
Question 79
Which type of hernia is responsible for 99% of all groin hernias in children younger than 3 year of age?
epigastric
umbilical
inguinal
femoral
Correct answer:
inguinal
Inguinal hernias are responsible for 99% of all groin hernias in children younger than 3 years of age. Inguinal hernias are more
common in boys and one-third of inguinal hernias are diagnosed before 6 months of age.
Question 80
You have prescribed Cymbalta for a patient with depression. The patient should be advised to contact a healthcare professional
immediately if she experiences which of the following?
Dry mouth.
Drowsiness.
Fast heart rate.
Vivid dreams.
Correct Answer: Fast heart rate Cymbalta has some uncommon but potentially serious side effects. If the patient experiences a
fast heart rate, she should notify a healthcare practitioner immediately. Other side effects, such as dry mouth, drowsiness, and
vivid dreams, are common but not typically a cause for concern unless they interfere with daily living or do not abate.
Question 81
Patients taking all but which of the following should be advised of the potential for addiction or dependency?
Xanax.
Vicodin.
Tramadol.
Naproxen.
Correct Answer: Naproxen Naproxen is not known to carry a risk of dependency or addiction. Pain killers such as Tramadol
and Vicodin, as well as anti-anxiety drugs such as Xanax, can carry these risks. Patients should be made aware of this risk and
should be educated about how to step off medications when they are no longer needed to avoid withdrawal symptoms.
Question 82
Learning is evaluated by determining whether the objectives for client education are met. Evaluation that occurs while the
educational activity is ongoing is which of the following?
summative
abstract
review
formative
Correct answer:
formative
Formative evaluation occurs while the educational activity is ongoing and enables adjustments to be made to optimize the
likelihood that learning will occur. Summative evaluation occurs at the end of the educational activity.
Question 83
You are examining a 2 year old child at a well baby checkup. You will assess development in which of the following areas?
gross motor
343
fine motor
language
all of the above
Correct answer:
all of the above
Development is typically assessed in four areas. These areas are: gross motor, fine motor, language, and social skills. The most
widely used developmental assessment tool is the Denver II.
Question 84
Which of the following statements best describes the obturator sign in terms of abdominal diseases?
Rotate right hip through full range of motion. Positive if pain with movement or flexion of the hip.
Instruct patient to raise heels then drop them suddenly. Positive if pain is elicited.
With abdominal palpation, the abdominal muscles reflexively become tense and firm.
Deep palpation of the left lower quadrant of the abdomen results in referred pain to the right lower quadrant.
Correct answer:
Rotate right hip through full range of motion. Positive if pain with movement or flexion of the hip.
The obturator sign in the supine position is used for suspected acute appendicitis or any suspected retroperitoneal area acute
process. It is done by rotating the right hip through full range of motion. It is positive if there is pain with movement or flexion
of the hip.
Question 85
Before starting any research project, the FNP must conduct a literature review. The purposes of a literature review include all
of the following EXCEPT:
what is known and not known about a subject
the expected impact of the research
gaps and inconsistencies in the literature about a subject
unanswered questions about the subject
Correct answer:
the expected impact of the research
Although the expected impact of the research may be a consideration for the FNP it is not one of the purposes of a literature
review. The literature review is critical to all evidence-based practice projects.
Question 86
Which of the following does NOT increase a person’s risk for cataracts?
cigarette smoking
eye trauma
allergies
exposure to ultraviolet B rays
Correct answer:
allergies
Factors increasing susceptibility to cataracts include: eye trauma; eye disease; diabetes; aging; cigarette smoking; and
recreational or occupational exposure to ultraviolet B rays. Allergies are not a factor.
Question 87
Which of the following tests is federally mandated for infants?
TSH
Lead screening
PKU
all of the above
Correct answer:
PKU
PKU (phenylketonuria) is a federally mandated test. Severe mental retardation may develop if this is not treated early.
Question 88
Piaget’s cognitive development theory includes all of the following EXCEPT:
sensorimotor
postoperational thinking
344
concrete operations
formal operations
Correct answer:
postoperational thinking
Piaget’s cognitive development theory focuses on intellectual changes which occur in a sequential manner as a result of
continuous interaction with the environment. The stages are: sensorimotor (infancy); preoperational thinking; concrete
operations; and formal operations (involves logic, determines possibilities, problem-solves and makes decisions).
Question 89
Nurse practitioner acts identify who the NP is, what the NP can do in that role, and where the NP can legally provide care.
This is known as which of the following?
standards of care
ethical principles
scope of practice
all of the above
Correct answer:
scope of practice
The NP’s scope of practice, outlined in the nurse practice act, identifies who the NP is, what the NP can do in that role, and
where the NP can legally provide care. Actions that exceed the legal boundaries of practice in a particular state are considered
to be violations of the nurse practice act.
Question 90
At about what age would you expect an infant’s rooting reflex to disappear?
1 month
1 – 2 months
3 – 4 months
12 months
Correct answer:
3 – 4 months
The rooting reflex of an infant appears when the corner of the infant’s mouth is stroked. The infant turns toward the stimulus
and sucks. This reflex disappears by 3 to 4 months.
Question 91
A 21 year old female presents with tachycardia and dilated pupils. You notice that her mood appears joyful, not scared. What
do you need to address in the assessment?
Heart disease.
Diet.
Exercise.
Drug use.
Correct answer: Drug use The feelings of euphoria may be a sign of drug use. Drugs, such as cocaine, in the system can
interact with different treatments. It is essential that you ask the patient about any drugs she has taken.
Question 92
Your patient is terminally ill and suffering greatly. A decision is made by the patient and agreed to by healthcare providers to
stop chemotherapy treatment for this patient. This is considered which of the following?
withholding treatment
withdrawing treatment
letting die
none of the above
Correct answer:
withdrawing treatment
Withdrawing treatment involves removing or stopping a treatment already started. Withholding treatment means never starting
345
a given treatment.
Question 93
A young female patient presents in the office with painful genital lesions. She has been sexually active and has not used
condoms. When counseling her, you include all of the following EXCEPT:
Stop having sexual intercourse until the lesions are no longer infectious.
Sitz baths and cool compresses may offer some relief.
Safe sex with condom use does not reduce the risk of transmission as viral shedding occurs along a wide area of the groin
and perineal region.
There are no pharmacological treatment options.
The Correct answer is:
There are no pharmacological treatment options
There are pharmacological treatment options for genital lesions. Oral antivirals, such as acyclovir (Zovirax) and famciclovir
(Famvir), can be used to prevent and/or lessen severity of breakouts.
Question 94
The document that focuses on reducing the health disparities in the general population of the United States with 10-year
national objectives is which of the following?
HIPPA
Healthy People 2020
OBRA
COBRA
Correct answer:
Healthy People 2020
Healthy People 2020 provides science-based, 10-year national objectives for improving the health of all Americans. For three
decades, Healthy People has established benchmarks and monitored progress over time in order to: encourage collaborations
across sectors; guide individuals toward making informed health decisions; and measure the impact of prevention activities.
Question 95
Which of the following statements about acute pyelonephritis is least accurate?
75% of cases are due to E. coli organism.
Recent diagnosis of PUD is a risk factor.
The most common route of infection is ascension from the bladder.
Urinary catheters are a risk factor.
Correct answer:
Recent diagnosis of PUD is a risk factor.
This is the least accurate statement. Risk factors include urinary tract abnormalities or instrumentation, stones, catheters,
diabetes or other immunocompromised states, recent pyelonephritis, BPH, pregnancy, and fecal incontinence. Also recent
lower UTIs present a risk factor.
Question 96
In terms of determining the reliability of a research instrument, which type uses the measure that items of an instrument
correlate well with other items on the instrument?
test-retest
inter-rater
internal consistency
content correlation
Correct answer:
internal consistency
The measure that items of an instrument correlate well with other items on the instrument is internal consistency. It is one of
the criteria for determining reliability of a research instrument.
Question 97
A 35-year-old Caucasian female is in the office with rosacea. She is asking about treatment options. What is the recommended
treatment for her?
topical 5-fluorouracil
346
low-dose tetracycline
oral ketoconazole
Dilantin
The Correct answer is:
Low-dose tetracycline
Treatment with systemic low-dose tetracycline is a very effective measure for rosacea. Further, topical treatment with
metronidazole or a low-dose steroid cream may also be helpful. In some cases, ketoconazole, the antifungal cream and not an
oral form, is used to treat rosacea. Topical 5-fluorouracil is used to treat actinic keratosis.
Question 98
For male,s approximately how long does it take to go from Tanner stage 2 to stage 5 for genital development?
4 years
6 years
8 years
3 years
Correct answer:
4 years
It takes males approximately 4 years to move from stage two to stage five of the Tanner stages of genital development.
quiz 26 aanp
Question 1
There are many types of managed care systems. The type of system in which discounted fees for service are offered instead of
capitation is which of the following?
preferred provider organization
health maintenance organization
fee-for-service plan
health insurance purchasing cooperative
preferred provider organization
Preferred provider organizations (PPOs) involve discounted fees for service instead of capitation. If a patient selects a provider
outside the network, the difference between the discounted fee and the normal fee is assumed by the client through co-pays or
deductible payments.
Question 2
The FNP has a patient who she suspects has macular degeneration. The FNP may use which of the following tests to check the
central vision of the patient?
Snellen chart
Amsler grid
amplitude of accommodation
slit lamp examination
Correct answer:
Amsler grid
Macular degeneration is a gradual change in the pigment in the macula (area of central vision) resulting in blindness. The FNP
will check central vision with the Amsler Grid test. If macular degeneration is present, the lines in the center of
Question 3
You are providing care to a female patient who is 6 months pregnant. She is suffering from hemorrhoids. Which of the
following may help alleviate her discomfort?
Sleeping on her stomach.
Sleeping on her side.
Sleeping on her back.
Sleeping with the head slightly elevated.
Correct Answer: Sleeping on her side For patients who are not pregnant, sleeping on the stomach can alleviate pressure on
inflamed blood vessels that lead to hemorrhoids. Pregnant patients should instead sleep on their side, as sleeping on the
stomach can be uncomfortable especially in later stages of pregnancy. Warm sitz baths may also help alleviate discomfort
caused by hemorrhoids.
Question 4
347
At which age does a child develop a pincer grasp?
3 months
6 months
9 months
12 months
Correct answer:
9 months
A child develops a pincer grasp which is a fine motor skill around the age of 9 months. The child should also be able to pull
himself or herself up to stand, as well as crawl and bear weight well.
Question 5
Which of the following statements about pulmonary tuberculosis (TB) is false?
About 75% of individuals exposed to the causative organism of TB become infected.
TB is a chronic bacterial infection.
TB is transmitted through aerosolized droplets.
The stage of TB known as primary TB is usually symptom-free.
Correct answer:
About 75% of individuals exposed to the causative organism of TB become infected.
This statement is false. About 30% of individuals exposed to the causative organism of TB become infected. In an
immunocompetent host, when the organism is acquired, an immune reaction ensues to help contain the infection within
granulomas.
Question 6
An elderly patient presents with a flat, nonpalpable colored spot, 4 mm in size. What is the medical term for this spot?
Bulla.
Tumor.
Pustule.
Macule.
Correct answer: Macule Bulla: fluid-filled, elevated, circumscribed lesion larger than 5 mm Tumor: solid, elevated mass larger
than 1 cm Pustule: pus-filled, elevated, circumscribed lesion up to 5 mm in size Macule: flat, nonpalpable colored spot up to 5
mm in size
Question 7
Which of the following measures would be directed particularly against the reservoirs of infection?
insect spraying
water purification
immunization
improved nutrition
Correct answer:
insect spraying
Measures directed against the reservoirs of infection include: insect spraying, isolation and quarantine. Water purification is a
measure to interrupt the transmission of organisms. Immunization and improved nutrition are measures that reduce host
susceptibility.
Question 8
A brief cultural assessment in the CONFHER model would include all of the following EXCEPT:
communication
occupation
nutrition
education
Correct answer:
occupation
The CONFHER model includes: communication, orientation, nutrition, family relationships, health beliefs, education, and
religion. It does not include occupation.
Question 9
348
A 24-year-old female presents with a headache. She describes throbbing pain behind one eye, light sensitivity, nausea and
vomiting, and recurrence right before menstruation. The nurse practitioner suspects which of the following?
temporal arteritis
cluster headache
trigeminal neuralgia
migraine without aura
The Correct answer is:
Migraine without aura
The symptoms and aggravating factor indicate migraine without aura. A cluster headache, more common in middle-aged men,
involves piercing pain with tearing, rhinorrhea, ptosis, and often, miosis on one side. Temporal arteritis involves warmness
over an artery and scalp tenderness at the temporal area. Trigeminal neuralgia involves a very brief intense sharp stabbing
pain.
Question 10
You are providing care to a patient who has been diagnosed with diverticulosis. Which of the following is NOT an appropriate
strategy for this patient to take to manage the constipation that sometimes accompanies this condition?
The patient should use OTC chemical laxatives.
The patient should increase the fiber in her diet.
The patient should incorporate wheat germ into her diet.
The patient should avoid chemical laxatives.
Correct Answer: The patient should avoid OTC chemical laxatives These medications can irritate the colon. The patient
should instead incorporate wheat germ and more fiber into her diet.
Question 11
The nurse practitioner understands that when treating a child on long-term antihistamine therapy there will be a need for
supplements due to decreased absorption of:
iron
magnesium
niacin
calcium
The correct answer is:
Iron
Antihistamines block the absorption of iron, vitamin B12, and folate. Iron, a mineral, helps the body make myoglobin, located
in the muscles and hemoglobin, located in the red blood cells. Additionally, the iron helps the body to store and move oxygen.
Question 12
The classic triad of symptoms for appendicitis includes:
abdominal pain, vomiting, and fever
abdominal pain, constipation, and lethargy
abdominal pain, diarrhea, and confusion
abdominal pain, lethargy, and confusion
Correct answer:
abdominal pain, vomiting, and fever
The classic triad of symptoms for appendicitis includes abdominal pain, vomiting, and fever. Right-sided abdominal pain is
present. Nausea is common, with vomiting almost always following the onset of pain. And fever is present.
Question 13
The FNP has a patient who is a new mother and complains of sore nipples. What advice should the FNP give this mother?
stop breastfeeding
the problem will resolve on its own
supplement breastfeeding with formula
use formula at nighttime feedings
Correct answer:
the problem will resolve on its own
The best advice is to tell this mother that sore nipples are a common problem and that it will resolve itself. The FNP should
not advise any of the other choices.
349
Question 14
During a physical exam, what will help indicate pain?
Face.
Voice.
Hands.
Words.
Correct answer: Face Patients do not always cry out when they are in pain. Monitoring facial expressions will give an accurate
physical exam.
Question 15
Which of the following statements about the MMR vaccine is false?
The MMR vaccine is safe to use during lactation.
The MMR vaccine is a live, attenuated vaccine.
The recommended schedule for early childhood immunization is two immunizations one month apart.
Systemic reaction to the MMR vaccine is rare.
Correct answer:
The recommended schedule for early childhood immunization is two immunizations one month apart.
This statement is false. The recommended schedule for early childhood immunization is two doses of MMR vaccine given
between 12 and 15 months and between 4 and 6 years. Two immunizations one month apart are recommended for older
children who were not immunized earlier in life.
Question 16
Of the following choices, which is most consistent with findings in patients with Parkinson Disease (PD)?
masklike facies and continued cognitive function
tremor at rest and bradykinesia
excessive muscle spasms and exaggerated gait
relaxed posture and poor muscle tone
The Correct answer is:
Tremor at rest and bradykinesia
The diagnosis of Parkinson Disease (PD) is made by clinical evaluation. Further, this condition consists of a combination of
six cardinal features: bradykinesia (slowness in the execution of movement), tremor at rest, rigidity, flexed posture, loss of
postural reflexes, and masklike facies.
Question 17
You might prescribe all of the following for a patient with sickle cell anemia EXCEPT:
folic acid
iron
oxygen for hypoxia
pain reliever
Correct answer:
iron
You would not prescribe iron for a patient with sickle cell anemia. It increases Hgb S production. Hgb S deforms the RBCs
into a sickle shape, the sickle cells hemolyze, and clusters of sickle cells occlude small blood vessels.
Question 18
Which of the following osteoporosis treatments stops bone loss but does not rebuild bone?
Fosamax
Evista
Teriparatide
Miacalcin
Correct answer:
Miacalcin
Miacalcin (calcitonin salmon) stops bone loss but does not rebuild bone. It is a good choice for relief of bone pain from
vertebral fractures.
350
Question 19
Medical records must be kept safe. It is essential that each healthcare organization establish an effective procedure for
safeguarding them. The following are all true statements regarding medical records EXCEPT:
Medical records should be stored in a secure, restricted-access location.
Competent medical records personnel should review every record before it is examined by the patient or patient
representative.
An original medical record should be sent with the patient to specialist appointments and for their attorney to review as
necessary.
Anyone who is not an authorized employee or staff member should not be allowed to examine a medical record.
The Correct answer is:
An original medical record should be sent with the patient to specialist appointments and for their attorney to review as
necessary
An original medical record should never be sent with the patient to a specialist’s appointment. The original medical record
remains on the facility’s premises unless the organization has received a Subpoena Duces Tecum, court order or a subpoena
and a court order to bring the medical record to a court of law. Alternatively, if the medical record has reached the state and
federal law’s identified retention period, the original medical record can be stored offsite or destroyed.
If the patient needs information from the medical record to take to the specialist, the nurse practitioner’s office can make a
copy of what is needed for the referral and send the photocopies of the medical. This process holds true for an attorney. The
attorney will only receive copies of the original medical record. Or, if the patient signs appropriate consent forms, the attorney
can come to the medical office and review the original medical record.
Also, the healthcare provider, nurses or any other staff cannot take original medical records home or to another facility to
complete charting or other documentation in the medical record. All charting has to be completed in the facility where the
original medical exists. By removing the original medical record from the facility, it can be lost, accidentally destroyed or
even stolen, for instance. This can lead to breaches of confidentiality and privacy laws, not to mention it is hard to reproduce
an original medical record if it is lost.
In the case of office space restrictions, some physician offices and hospitals may not have sufficient amounts of space to store
all of their medical records. Therefore, storage or retention of the original medical records becomes an issue. There are many
options to storing medical records including storing them offsite, microfilming, scanning the medical records and storing them
on CD ROM’s, disks and more. In this case, the nurse practitioner will need to review state and federal laws, as the laws
govern how medical records can be retained and the compliance guidelines required if the physical, original medical record
has to be stored offsite. Certain medical record documents are permanently kept, such as records of surgical procedures and
master patient indexes. Then, depending on state laws, diagnostic images, for instance x-ray films, can be kept for 5 years
before the images are archived or destroyed. Destruction of medical records are also dictated by federal and state laws. Some
destruction methods include shredding the medical record, for example.
Question 20
You have a patient who is reluctant to talk to anyone. You wish to draw her out and so ask her to tell you about her family.
This is using which of the following techniques for therapeutic communication?
reflection
open-ended comment
restating
focusing
Correct answer:
open-ended comment
Using an open-ended comment or question is a therapeutic communication technique. Questions cannot be answered with a
one-word answer. It allows the patient to decide what content is relevant.
Question 21
Your 70-year-old female patient complains of pain when she walks. The pain, she says, is relieved when she sits or lies down.
She tells you that this pain has become gradually worse and now she finds it difficult to walk very far. You diagnosis is likely
to be which of the following?
peripheral vascular disease
superficial thrombophlebitis
351
deep vein thrombosis
Raynaud’s phenomenon
Correct answer:
peripheral vascular disease
Peripheral vascular disease (PVD) is a gradual narrowing or occlusion of the medium to large arteries resulting in permanent
ischemic damage to an extremity. Skin may be cool, and there may be decreased-to-absent pulses (popliteal, dorsal pedis, and
posterior tibialis), among other signs.
Question 22
A patient who has been prescribed Flomax should be advised to seek immediate medical attention if he experiences which of
the following?
Fast heartbeat.
Runny nose.
Weakness.
Ejaculatory problems.
Correct Answer: Fast heartbeat This is a potentially serious side effect of Flomax. Patients experiencing this should seek
immediate medical attention. Less serious side effects include runny nose, drowsiness, weakness, problems ejaculating, and
blurred vision.
Question 23
All of the following are sources of legal risks for the nurse practitioner EXCEPT:
patients
peer review
procedures
quality of medical records
The Correct answer is:
Peer review
According to the American Nurses Credentialing Center, patients, procedures and quality of medical records are all sources of
legal risk and liability for nurse practitioners.
Question 24
The nurse practitioner is seeing a 28-year-old female who took L-thyroxin prior to becoming pregnant. Now that the diagnosis
of pregnancy is made, what should be done?
She should be switched to a supplement with a category B rating.
She should continue it and have monthly TSH levels evaluated.
She can continue it during pregnancy without concern.
It should be discontinued during pregnancy.
Correct answer:
She should continue it and have monthly TSH levels evaluated
L-thyroxin is used to treat hypothyroidism. It is safe during pregnancy, so it can be continued. However, during pregnancy, the
patient's TSH levels needs to be monitored. If the patient's TSH levels rise to a hypothyroid state, the fetal growth can be
affected.
Question 25
Electronic patient records raise specific legal concerns, many of which are concepts of the evolving status of traditional rights
and responsibilities associated with ownership and control of patient information. All of the following are accurate statements
regarding legal issues related to electronic patient records EXCEPT:
A computerized medical record system must be designed, installed, and maintained to preserve both the confidentiality and
the integrity of patient health information.
The Joint Commission on Accreditation of Healthcare Organizations (JCAHO) has standards related to patient record
confidentiality and integrity set up for organizations to adhere to.
When medical record data is transmitted across state lines, it is always the state where the patient was treated in which the
law applies.
Computerization of patient data increases the risk of unauthorized disclosure of medical information.
The Correct answer is:
When medical record data is transmitted across state lines, it is always the state where the patient was treated in which the law
applies
352
During the transfer process, it is not always clear which state's law applies or which courts will have jurisdiction if a dispute
arises over disclosure of an individual's health information.
Question 26
Which of the following best describes anisocoria?
a fine tremor of the hand with unknown cause
a unilateral headache caused by impingement
a unilateral pain in the temporal area
a condition in which the two pupils are not of equal size
Correct answer:
a condition in which the two pupils are not of equal size
Anisocoria can best be described as a condition in which the two pupils are not of equal size. A fine tremor of the hand with
unknown cause is benign familial tremor. A unilateral headache caused by impingement is trigeminal neuralgia. A unilateral
pain in the temporal area is temporal arteritis.
Question 27
Your 20-year-old female patient has bulimia. You might manage her condition by all of the following EXCEPT:
MAO inhibitors
SSRIs
TCAs
cognitive behavioral therapy
Correct answer:
MAO inhibitors
MAO inhibitors should be avoided for those with bulimia. They have a potential for severe food interactions and hypertensive
crisis.
Question 28
There is a 19-year-old female patient in the office and it is determined she has iron deficiency anemia. If this anemia has
occurred in the past 3-4 months, what might be expected?
a multititude of white and red blood cells
the red blood cells are small
a decreased total iron binding capacity
an elevated serum ferritin
The Correct answer is:
The red blood cells are small
Iron deficiency anemia happens when the reserve of iron stored in the body are decreased and in some cases depleted. The iron
helps the body make red blood cells. With iron deficiency anemia, the individual may experience weakness, have shortness of
breath and the skin is pale. When the iron stores in the body are completely gone, the red blood cells may appear small as well
as the quantity of the red blood cells are very few. Further, serum ferritin is a measure of the iron in storage. Then, the total
iron binding capacity is always increased in a patient with iron deficiency anemia.
Question 29
The nurse practitioner's standards of practice are established for what purpose?
They are meant to regulate and control nurse practitioner practice.
They limit liability of nurse practitioners.
They protect nurse practitioners from frivolous lawsuits.
They are set up to promote autonomous practice.
:
The Correct answer is:
They are meant to regulate and control nurse practitioner practice
For all professionals (nurses, physicians, dentists, etc.) the standards of practice are set up to regulate and control practice.
Their purpose is to provide accountability for the professionals and to help protect the public from unethical and unsafe
practice.
Question 30
Which of the following statements are NOT true concerning entries in the medical record?
Maintaining a complete medical record is important to comply with licensing and accreditation requirements.
353
Medical record entries should be made in clear and concise language that can be understood by healthcare professionals who
treat the patient.
Allegations of improper care can always be defended without documentation in a medical record if staff witnessed the care.
Maintaining a complete medical record enables the healthcare provider to establish that a patient has received adequate care.
The Correct answer is:
Allegations of improper care can always be defended without documentation in a medical record if staff witnessed the care
Some courts have dismissed claims involving incomplete records. However, some cases have been decided without
documentation of the allegation in the medical record.
Question 31
A 6-year-old patient has been discharged from the hospital to home on DDAVP (desmopressin acetate) for diabetes insipidus
after removal of a pituitary tumor. The nurse practitioner (NP) notices that the child is lethargic and has 4+ deep tendon
reflexes on examination. The NP suspects:
interaction with OTC cough medicine
water intoxication
increased vasopressor effect
noncompliance with therapy
The Correct answer is:
Water intoxication
Desmopressin acetate (DDAVP) promotes reabsorption of water in the renal tubules, which can lead to water intoxication.
Other signs of this are behavioral changes and disorientation.
Question 32
The test done to assess for meningeal irritation where the FNP bends the patient’s neck toward the chest to see if the patient
reflexively flexes the hips and knee is which of the following tests?
Kernig sign
Romberg test
Brudzinski sign
Snellen test
Correct answer:
Brudzinski sign
The test done to assess for meningeal irritation where the FNP bends the patient’s neck toward the chest to see if the patient
reflexively flexes the hips and knee is the Brudzinski sign. It is positive if the patient reflexively flexes the hips and knee to
relieve pressure and pain (due to inflammation of the lumbar nerve roots).
Question 33
Which of the following statements about acute myelogenous leukemia (AML) is least accurate?
It accounts for 75% of all childhood leukemias.
There is a relatively constant incidence of AML from birth through adolescence.
The ratio of AML to ALL (acute lymphocytic leukemia) is 1:4.
It is an abnormal proliferation of immature myeloid stem cells in the bone marrow.
Correct answer:
It accounts for 75% of all childhood leukemias.
This is the least accurate statement. AML accounts for 15% to 20% of childhood leukemia. ALL accounts for 75% to 80% of
all childhood leukemias.
Question 34
Which of the following is least likely to exacerbate edema in your patient?
laxative abuse
corticosteroids
calcium agonists
diuretics
Correct answer:
diuretics
Diuretics are indicated for marked peripheral edema, pulmonary edema, CHG, and inadequate dietary salt restriction. Abuse of
diuretics, however, can lead to edema.
354
Question 35
In terms of liver function tests, the normal serum AST (also known as SGOT) is which of the following?
5 – 50 u/L
50 – 100 u/L
2.0 or higher
5.0 or higher
Correct answer:
5 – 50 u/L
Serum AST for adults to elderly has a normal range of 5 – 50 u/L. It is present in the liver, heart muscle, skeletal muscle,
kidney, and lung. It is not specific for liver injury because it is also elevated in other conditions.
Question 36
You have received the results of a platelet count test for a patient. It indicates that her platelet count is 13,000/mm3. Which of
the following is true of this patient, based on these results?
Her platelet count is far too high.
Her platelet count is low but within the acceptable range.
The patient is at risk for serious bleeding.
The patient's platelet count is normal.
Correct Answer: The patient is at risk for serious bleeding This patient's platelet count is lower than 20,000/mm3, the
threshold for concern for serious bleeding. It will be a key next step to discover why the patient's platelet count is so low and
take immediate steps both to reduce her risk of injury and to address the underlying cause of the low count.
Question 37
Sometimes when evaluating a patient with acute appendicitis, appendiceal rupture occurs. The clinical findings most
consistent with this occurrence include all of the following EXCEPT:
fever that is greater than 102°F
abdominal pain for less than a 24 hour duration
marked leukocytosis on CBC with WBC greater than 20,000mm³
palpable abdominal mass
The Correct answer is:
Abdominal pain for less than a 24 hour duration
Appendiceal perforation is usually associated with fever greater than 102, a marked leukocytosis with total WBC count often
exceeding 20,000 mm³, peritoneal inflammation, symptoms lasting longer than 24 hours and an ill-defined right lower
quadrant abdominal mass.
Question 38
The nurse practitioner must be sensitive to cultural influences on health and health promotion. Which of the following is NOT
true regarding cultural healthcare considerations?
Ethnicity is based on race, tribe, or nation with which a person identifies and this influences beliefs and behaviors.
Family structure and values have an impact on the health encounter.
Traditional health beliefs should be allowed even if they compromise the patient's well-being or the well-being of their
family.
More comprehensive care can be provided if the nurse practitioner is aware of the cultural traditions surrounding their
patient.
The Correct answer is:
Traditional health beliefs should be allowed even if they compromise the patient's well-being or the well-being of their family
Traditional health beliefs that compromise a patient's well-being should be instructed upon and corrected for the safety of the
family. An example would be giving small amounts of home-made "moonshine" as a cough suppressant to small children.
This should be avoided due to lead toxicity.
Question 39
A mother is asking questions regarding giving her 4-month-old infant fruit juice. The nurse practitioner understands that the
following statements are accurate concerning fruit juice EXCEPT:
Fruit juice is an excellent replacement for infant formula if an infant is allergic to cow's milk.
Juice should not be given before 6 months of age and intake should be limited to 1 to 2 ounces.
Pureed fruit should be given instead of juice if possible.
355
Fruit juice should not replace breast milk or infant formula in the diet because it is high in carbohydrate and low in the other
nutrients.
The Correct answer is:
Fruit juice is an excellent replacement for infant formula if an infant is allergic to cow's milk
Fruit juice is not an excellent alternative to infant formula. If an infant is allergic to cow’s milk, the nurse practitioner can
suggest to the mother other acceptable alternatives, such as feeding the infant soy-milk formula. Further, fruit juice should
only be given to infants who are 6 months old and older. Therefore, the nurse practitioner should inform the mother that the 4month-old is not at an appropriate age to drink fruit juice.
Question 40
The resistance of a group to invasion and spread of an infection because a large portion of the group is immune is known as
which of the following?
generation immunity
reservoir immunity
herd immunity
direct immunity
Correct answer:
herd immunity
Herd immunity is the resistance of a group to invasion and spread of an infectious agent because a large portion of the group is
immune. It decreases the likelihood of an epidemic in the area.
Question 41
First generation antihistamines readily cross the blood-brain barrier. Which of the following drugs is a first generation
antihistamine?
loratadine
diphenhydramine
desloratadine
levocetirizine
Correct answer:
diphenhydramine
Diphenhydramine (Benadryl) is a first generation antihistamine. Another first generation antihistamine is chlorpheniramine
(Chlor-Trimeton). The other choices are all second generation antihistamines.
Question 42
You are taking the medical history of a patient who is suspected of having Addison’s disease. Which of the following data in
his history is most likely to support this diagnosis?
weight gain
salt cravings
hyperglycemia
increased libido
Correct answer:
salt cravings
Of the choices, this is the one most likely to support Addison’s disease. Weight loss, hypoglycemia and decreased libido
would also support the diagnosis.
Question 43
A toddler is brought in with a low grade fever, infection of the upper airway, and hoarseness. What test may be ordered if the
clinical exam is not conclusive?
X-ray.
CBC.
Spinal tap.
Throat culture.
Correct answer: X-ray The toddler's symptoms may indicate croup. The clinical exam typically confirms this. An X-ray of the
lateral neck may be ordered.
Question 44
356
Your patient with benign prostatic hypertrophy (BPH) is taking zinc as a supplement to improve the condition. He has been
taking this for 6 weeks so far. Which of the following should also be added in this case?
vitamin B12
potassium
caffeine
copper
Correct answer:
copper
If zinc is taken for more than one month, copper at 1 to 2 mg/day should be added. Zinc depletes the body’s stores of copper.
Question 45
The levels of E/M services recognize four types of medical decision making. Which of the following is NOT one of these four
types?
low complexity
high complexity
minimal
straightforward
Correct answer:
minimal
The four types of medical decision making recognized by the levels of E/M services are: straightforward, low complexity,
moderate complexity and high complexity. Medical decision making refers to the complexity of establishing a diagnosis
and/or selecting a management option.
Question 46
In terms of fee-for-service Medicare payments, the fee schedule is determined using a system called a resource-based relative
value scale (RBRVS). There are three components to a relative value. Which of the following is NOT one of these
components?
Geographic practice cost index component
Practice expense component
Work component
Malpractice component
Correct answer:
Geographic practice cost index component
The three components to a relative value are: a practice expense component; a work component; and a malpractice component.
Each component is adjusted geographically, using three separate geographic practice cost indexes.
Question 47
Which of the following statements about the MMR vaccine is incorrect?
The MMR is not safe to use during lactation.
Its use in pregnant women is discouraged.
Systemic reaction to the MMR vaccine is rare.
The MMR vaccine is typically well tolerated with rare reports of transient adverse reactions.
Correct answer:
The MMR is not safe to use during lactation.
This is incorrect. The MMR is safe to use during lactation. Its use in pregnant women is discouraged because of the possible
risk of passing the virus on to the unborn child. This risk exists in theory but has not been noted in ongoing observation
Question 48
Part of becoming a nurse practitioner is the licensure process. The following statements are related to the history and
development of the licensure process and all are correct EXCEPT:
In the early 1900s, the first nursing statutes were actually nurse registration acts.
The healthcare reform movement of the 1990s forced the nursing profession to increase professional accountability.
Nurse Practice Acts have evolved over time.
Applicants for licensure as an advanced practice nurse do not need a license as a registered nurse.
The Correct answer is:
Applicants for licensure as an advanced practice nurse do not need a license as a registered nurse
357
Applicants do need a license as a registered nurse to obtain one as an advance practice nurse. Additionally, they must hold
current national certification in the appropriate advance practice nurse specialty.
Question 49
The FNP has a 64-year-old female patient who inquires about the Zoster vaccine for shingles. The FNP might tell her all of the
following EXCEPT:
The vaccine is prepared from a live, attenuated strain of varicella-zoster virus.
Reactivation of the varicella virus is not related to a decline in varicella-zoster virus-specific immunity.
The use of zoster vaccine significantly reduces the risk of shingles.
The vaccine should be used even with a history of shingles.
Correct answer:
Reactivation of the varicella virus is not related to a decline in varicella-zoster virus-specific immunity.
This statement is incorrect. Reactivation of the varicella virus seems to be related to a decline in varicella-zoster virus-specific
immunity. Therefore, the use of zoster vaccine significantly reduces shingles risk.
Question 50
In Maslow’s hierarchy of needs which one ranks highest?
safety and security
love and belonging
self-actualization
self-esteem
Correct answer:
self-actualization
Self-actualization is the highest level of Maslow’s hierarchy of needs. Self-actualizing people are self-aware, concerned with
personal growth, less concerned with the opinions of others and interested fulfilling their potential.
Question 51
A 16-year-old male is in the office. He has a insect bite on his left forearm and you suspect a brown recluse spider bite. What
medical management would you provide?
ice pack and elevation of the area
active and passive range of motion (ROM) to the area
avoidance of antihistamines
warm moist soaks to the affected area
The Correct answer is:
Ice pack and elevation of the area
Ice packs are preferred to heat to decrease the edema. The area should be immobilized and tetanus toxoid given.
Antihistamines may reduce the swelling and relieve site itching.
Question 52
Fractures of the basilar area of the skull can present with bruising over the mastoids. This bruising is known as which of the
following?
Adam’s sign
Christmas tree bruise
Koplik’s sign
Battle’s sign
Correct answer:
Battle’s sign
Battle’s sign is a bruising over the mastoid area that is a sign of fractures of the basilar area of the skull. Look for clear golden
fluid draining from the nose and/or ear.
Question 53
A mother brings her 2-year-old girl to you. After careful assessment, you diagnose bronchiolitis. Which choice would NOT be
part of patient management?
nebulized bronchodilators
antibiotics
358
oral steroids
antipyretics
The Correct answer is:
Antibiotics
Bronchiolitis is a common viral infection and antibiotics are not effective. Since it is characterized by wheezing,
bronchodilators, especially nebulized, are a good choice for treatment. Oral steroids are controversial in bronchiolitis but are
commonly used when significant edema in the small airways is suspected. Antipyretics to treat symptoms of fever, and not the
bronchiolitis, are necessary in most cases.
Question 54
A person who has been prescribed three daily medications for asthma – an antiinflammatory agent, a long-acting
bronchodilator, and an oral glucocorticoid is most likely to have which category of asthma?
severe persistent
moderate persistent
mild persistent
mild intermittent
Correct answer:
severe persistent
Severe persistent asthma involves continual symptoms which limit activity. For this stage of asthma three daily medications
are usually prescribed, along with a short-acting beta-agonist.
Question 55
Which of the following is the preferred treatment fro pregnant or lactating women who have Lyme disease?
doxycycline 100 mg bid x 10 to 21 days OR
tetracycline 250 to 500 mg qid x 10 to 21 days
erythromycin 250 mg qid x 10 to 21 days
amoxicillin 250 to 500 mg tid x 10 to 21 days
Correct answer:
amoxicillin 250 to 500 mg tid x 10 to 21 days.
The therapeutic plan for pregnant, lactating females and children younger than 8 years is different from others. The
pharmacologic treatment for them is amoxicillin 250 to 500 mg tid x 10 to 21 days.
Question 56
Your patient has chills, fever, and vaginal discharge along with a tender uterus upon examination. She tells you that she has
just had an abortion. Your diagnosis is most likely to be which of the following?
Mittelschmerz
endometritis
ovarian cyst
leiomyoma
Correct answer:
endometritis
This patient most likely has endometritis. This is an inflammation or irritation of the lining of the uterus (the endometrium). It
is not the same as endometriosis.
Question 57
An adult patient presents with the acute onset of diffused pink-to-red skin on the site of a skin trauma. He tells the FNP that
the area has enlarged over the last few days. The skin is indurated, swollen and tender. Which of the following is the most
likely diagnosis?
candidiasis
contact dermatitis
actinic keratoses
acute cellulitis
Correct answer:
acute cellulitis
359
Acute cellulitis is a skin infection caused by Gram-positive bacteria. Points of entry are skin breaks such as abrasions or
surgical wounds. The classic case has an acute onset.
Question 58
In managing a patient with a substance abuse problem, the patient tells you, "I wish I had never used cocaine, it has ruined my
life." What is the most appropriate response by the nurse practitioner?
"Things will work out, don't you worry."
"You shouldn't be so hard on yourself. You can change."
"You should think before you do something."
"It sounds like you've thought a lot about your cocaine use."
The Correct answer is:
"It sounds like you've thought a lot about your cocaine use"
The nurse practitioner's statement is open ended. It allows for the patient to discuss and clarify his feelings. Further, the
remaining statements are inappropriate responses.
Question 59
In which of the following cases would the FNP refer a patient to a specialist for low back pain? A patient with:
acute neck pain
bladder dysfunction
reduced range of movement
stiffness
Correct answer:
bladder dysfunction
If the patient has a bladder dysfunction, or a limb or bowel dysfunction, prompt referral to a specialist is needed. Surgery is
usually considered only if severe symptoms persist beyond 3 months.
Question 60
A 8-year-old male child is called back into the office because his stool specimen for ova and parasites (O & P) came back
positive. The nurse practitioner knows what regarding enterobiasis?
The parasite causes pruritus around the anus because the females exit at night to lay eggs on the skin.
This parasite is a protozoan and the source is usually contaminated water and it is spread by fecal-oral contamination.
The parasite is in the soil and enters the body through the feet.
Eggs of this parasite enter the body by ingestion of dirt or from unwashed vegetables that contain the eggs in the dirt.
The correct answer is:
The parasite causes pruritus around the anus because the females exit at night to lay eggs on the skin
This infestation is also known as "pinworms" which reside in the intestine. Hookworm larvae reside in the soil and enter the
body through the feet. When dirt is ingested, there is a risk of contacting roundworm. Giardiasis from Giardia lamblia results
from ingesting this organism through contaminated water or fecal-oral transmission.
Question 61
There are a number of theories of aging. Which of the following theories holds that the aging process decreases T cells so
there is a rise in the incidence of infection?
Activity theory
Immune theory
Free-radical theory
Wear-and-tear theory
Correct answer:
Immune theory
The immune theory holds that the aging process affects the immune system and decreases T cells so there is a rise in the
incidence of infection. It is one of the biologic theories of aging.
Question 62
A female patient just saw her sister battle, and survive, ovarian cancer. She is concerned that she may have a genetic
predisposition. What test can you order to confirm or disprove her suspicions?
BRCA2.
360
CA-125.
PSA.
CT scan.
Correct answer: BRCA2 The breast cancer gene 2 (BRCA2) is also related to ovarian cancer. The breast cancer gene 1 is
another genetic factor, as is HNPCC. A genetic profile of the patient and her sister will identify this risk factor.
Question 63
The theory that outlines eight consecutive stages in the family life cycle that offers a predictive overview of the activities that
occur in families over time is called the:
developmental theory
family systems theory
self-efficacy theory
none of the above
Correct answer:
developmental theory
The developmental theory outlines eight consecutive stages in the family life cycle that offers a predictive overview of the
activities that occur in families over time. It serves as a basis for anticipatory guidance when assessing and teaching families.
Question 64
Long-term care insurance is a relatively new concept designed to meet the needs of the growing elderly population. Which of
the following statements about long-term care is least accurate?
The likelihood of older adults requiring long-term care at some point in time in their lives is approximately 50%.
The average stay for long-term care is about 19 months.
Long-term care insurance generally provides coverage for approved care in nursing facilities and assisted living.
Long-term care insurance was developed by the federal government to meet the long-term and chronic health care needs of
older adults.
Correct answer:
Long-term care insurance was developed by the federal government to meet the long-term and chronic health care needs of
older adults.
This statement is not accurate. Long-term care insurance was developed by private insurance companies to meet the long-term
and chronic health care needs of older adults. It is designed to pay for long-term health services when multiple chronic health
problems occur that require custodial care not covered by Medicare or other insurance.
Question 65
It is considered precocious puberty in a male if puberty starts before what age?
9 years
10 years
12 years
13 years
Correct answer:
9 years
For boys, puberty starts at 9 years. It is considered precocious puberty if it begins before 9 years and delayed puberty if no
testicular growth is seen by 14 years.
Question 66
Which of the following is NOT a normal stooling pattern for infants/children?
Breastfed babies stool less often.
Neonates stool more often than four times per day.
Four-month-olds produce two stools per day.
Four-year-olds produce one stool per day.
Correct answer:
Breastfed babies stool less often.
This is not correct. Breastfed babies may stool after every feeding. Bottle-fed babies stool less often.
Question 67
361
Patients with which of the following conditions should not be prescribed Cipro?
Mysathenia gravis.
Multiple sclerosis.
Fibromyalgia.
Interstitial cystitis.
Correct Answer:
Myasthenia gravis
Patients with this condition should not take Cipro. It may cause the condition to become worse.
Question 68
You are explaining to a 67-year-old female patient with osteoporosis which foods are the best dietary sources for calcium.
Which of the following foods would have the lowest content of calcium?
collard greens, cooked
cottage cheese (low-fat) 4 oz
oysters, raw (13 – 19)
Swiss cheese, 1 oz.
Correct answer:
cottage cheese (low-fat) 4 oz
Cottage cheese (low-fat) 4 oz has about 78 mg of calcium. All of the other choices have much more. The optimum calcium
intake for women older than 65 years is 1500 mg.
Question 69
A patient that has been transferred into your care from another provider presents with a history of migraine headaches. She is
on Tylenol with codeine for treatment of these headaches but nothing for abortive therapy. The nurse practitioner understands
that a good agent to prescribe the patient for abortive therapy is:
ketorolac (Toradol) 100 mg IM
amitriptyline (Elavil) 100 mg PO
sumatriptan (Imitrex) 6 mg IM
ergotamine (Ergostat) 2 mg SL
The Correct answer is:
Ergotamine (Ergostat) 2 mg SL
Ergotamine sublingual at 2 mg is the correct dose for abortive therapy for migraine headaches. With Ketorolac, dosing
guidelines indicates administration of 30-60 mg for pain. However, Ketorolac does not help with abortive therapy.
Sumatriptan is given subcutaneously (SC) or by mouth (PO), not IM. Also, sumatriptan is a good medication for abortive
migraine therapy. Amitriptyline is not used in abortive therapy.
Question 70
A pregnant patient presents with abdominal pain. What test should you use to determine the cause?
Ultrasound.
CBC.
MRI
CT scan.
Correct answer: Ultrasound An ultrasound will not damage a fetus. It is part of prenatal care. The other imaging answers may
harm the fetus and are not recommended for pregnant women.
Question 71
The most common cancer death in females is which of the following?
lung cancer
leukemia
breast cancer
colon cancer
Correct answer:
lung cancer
Lung cancer is the most common type of cancer death in females as well as males.
Question 72
362
Which of the following would NOT be recommended by the FNP as a measure to prevent colonic diverticulosis and
diverticulitis?
regular aerobic exercise
foods low in uric acid
adequate hydration
a high-fiber diet
Correct answer:
foods low in uric acid
Foods low in uric acid are what is recommended for those suffering from gout. The other three choices are recommendations
for preventing colonic diverticulosis and diverticulitis. They help to increase bowel motility and tone.
Question 73
Your patient is a newly arrived immigrant from China. He speaks English, but has a limited vocabulary. He does not
understand certain medical terms. This is a concern for which of the CONFHER model aspects?
orientation
education
communication
health beliefs
Correct answer:
communication
Communication includes language, dialect, nonverbal, and social customs. You should be concerned with the patient’s ability
to understand common health terms.
Question 74
The blisters that you will find with second-degree burns are known by which of the following terms?
wheals
bullae
pustules
purpura
Correct answer:
bullae
Bullae are fluid-filled lesions, ≥ 1 cm. They are blisters. These are found with second-degree burns.
Question 75
What is the leading cause of death for all ages and genders in the United States?
cancer
heart disease
accidents
congenital abnormalities
Correct answer:
heart disease
Heart disease is the leading cause of death in all ages and genders. The leading cause of death for adolescents is motor vehicle
crashes. The leading cause of death in infants younger than 12 months is congenital anomalies and the leading cause of death
in children 12 months to 9 years is unintentional injuries.
Question 76
You prescribe a short-acting anticholinergic medication that can be used alone or in combination with a short-acting beta
agonist that the patient has already. What disease are you trying to manage?
benign prostatic hyperplasia
glaucoma
chronic obstructive pulmonary disease
tachyarrhythmias
The Correct answer is:
Chronic obstructive pulmonary disease
363
Patients with chronic obstructive pulmonary disease who have intermittent symptoms need a first line treatment for shortness
of breath which is an anticholinergic alone or in combination with a beta agonist. Both these medications will improve lung
function.
Question 77
The patient has the right to confidentiality. The following are all accurate statements regarding patient confidentiality
EXCEPT:
The individual's right to privacy is respected when requesting or responding to a request for a medical record.
HIPAA was set up by the Department of Health and Human Services and stands for Health and Insurance Portability
Advisory Association.
The statute requires that the provider discuss confidentiality issues with the patient and establish consent.
The provider shall not discuss any information with anyone unless the patient has given consent to do so.
The Correct answer is:
HIPAA was set up by the Department of Health and Human Services and stands for Health and Insurance Portability Advisory
Association
HIPAA stands for Health and Insurance Portability and Accountability Act, and not Advisory Association. HIPAA emerged in
1996 and the Department of Health and Human Services plays a role in governing this act. Further, HIPAA establishes
national standards for electronic healthcare transactions and national identifiers for providers, health plans, and employers.
Question 78
In a child who is developing normally, which of the following gross motor skills are you likely to see first?
lifts shoulders while prone
rolls over
has head control
moves head side to side
Correct answer:
moves head side to side
An infant who is developing normally will move his head from side to side at 2 weeks of age. The other three choices are
skills learned from 2 months to 4 months in the average infant.
Question 79
Which of the following patients has the highest risk factor for peptic ulcer disease?
a 65-year-old female who drinks 4 cups of coffee per day
a 45-year-old male who has one or two alcoholic drinks per day
a 55-year-old male who has cirrhosis as a result of alcohol abuse
none of the above
Correct answer:
a 55-year-old male who has cirrhosis as a result of alcohol abuse
This is the patient who has the greatest risk factor for peptic ulcer disease. Coffee drinking and occasional alcohol use are not
risk factors for peptic ulcer disease. H. pylori is also found in individuals with asymptomatic gastritis and dyspepsia without
ulceration.
Question 80
You are treating a female patient with an acute urinary tract infection (UTI). The nurse practitioner understands that she must
discuss risk factors. The risk factor for UTI in women include:
obesity
hot tub use
wearing pantyhose
low lactobacilli colonization
The Correct answer is:
Low lactobacilli colonization
Lactobacilli produce hydrogen peroxide and lactic acid, providing the periurethral area and vagina with a pH that inhibits
bacterial growth, blocks potential sites of attachment and is toxic to uropathogens. A normal component of periurethral flora
protects against urinary tract infection (UTI). Douching, obesity, use of hot tubs, and wearing pantyhose have not been shown
364
to increase UTI risk.
Question 81
All of the following are part of the structural functional theory of families EXCEPT:
Families are social systems that form interdependent and independent relationships.
Suprasystems form outside of the family and reflect needs not met within the family.
Internal family subsystems function as a microcosm of society.
Families pass through eight stages of development.
Correct answer:
Families pass through eight stages of development.
This is not one of the tenets of structural functional family theory. It is a part of a developmental theory proposed by Duvall
Question 82
Which of the following would NOT be considered one of the four major factors influencing healthcare delivery services?
payers
insurers
patients
suppliers
Correct answer:
patients
The four major factors influencing healthcare delivery services are: payers (including individual healthcare consumers);
insurers; providers; and suppliers. Patients who receive the healthcare services are not considered a factor.
Question 83
Which of the following statements about chronic myelogenous leukemia is least accurate?
It is rare in children.
The median age at onset is 21 years.
It progresses from chronic to accelerated and then to the blast phase.
It accounts for 1 – 5% of all childhood leukemias.
Correct answer:
The median age at onset is 21 years.
The median age at onset for chronic myelogenous leukemia is 45 years. It is rare in children but if it is seen in children it is
almost always before 2 years of age.
Question 84
For alcohol withdrawal symptoms, the preferred benzodiazepine is lorazepam. This is used when there is concomitant history
of:
folate-deficiency anemia
hepatic disease
seizure disorder
multiple substance abuse
The Correct answer is:
Hepatic disease
Librium or Valium are good therapeutic agents for patients with adequate hepatic function. However, lorazepam is the
preferred agent for those with impaired hepatic function because of its short half life.
Question 85
Licensure for nurse practitioners is mandatory for which of the following reasons?
to protect the nurse practitioner
to establish a scope of practice
to protect against malpractice
to protect the public
Correct answer:
to protect the public
The purpose of licensure is to protect the public by ensuring a minimum level of professional competence. It is not to protect
365
the nurse or nurse practitioner.
Question 86
If a 74 year old patient with CHF is prescribed Metformin, what could be the possible outcome?
Decreased effectiveness of diuretics.
Fluid retention.
Hypoxia.
Ulcer and bleeding risk.
Correct answer:
Hypoxia
Prescribing medications to patients with various diseases could put the patient at risk for additional conditions. For example, a
patient with gastropathy cannot be prescribed NSAIDs because they may cause increased ulcer and bleeding risk.
Question 87
Hypothyroidism is least likely to be a cause of abnormal uterine bleeding in which of the following age groups?
over 45 years of age
35 – 45 years of age
25 – 35 years of age
14 – 25 years of age
Correct answer:
14 – 25 years of age
Hypothyroidism is the least likely to be a cause of abnormal uterine bleeding in the 14 – 25 year old age group. It is also not
likely in those under 14 years of age.
Question 88
When treating a male patient with orchitis, the nurse practitioner has knowledge that the most common causative agent for this
is:
mumps
rubeola
arbovirus
echovirus
The Correct answer is:
Mumps
The mumps virus is responsible for causing orchitis. Arbovirus and echovirus are implicated in meningitis and encephalitis.
Rubeola is associated with complications of otitis media, croup, encephalitis, and pneumonia.
Question 89
Which of the following is the most frequent type of hearing loss in the elderly?
presbycusis
cerumen impaction
ototoxicity
noise trauma
Correct answer:
presbycusis
Presbycusis is the most frequent type of hearing loss among the elderly. Annual hearing tests should be conducted after age 65
years.
Question 90
You have prescribed a monoamine oxidase inhibitor (MAOI) for your patient with depression. You would tell this patient to
avoid which of the following foods?
those high in calorie
those high in glucose
those high in tyramine
those high in potassium
Correct answer:
366
those high in tyramine
Persons taking MAOIs should avoid foods with high tyramine. These include: aged cheese, red wine, chocolate, and fermented
foods such as beer.
Question 91
With the geriatric client, which assessment tool should the nurse practitioner use to evaluate balance and gait problems?
Index of Independence of Activities of Daily Living Scale
Tinetti Balance and Gait Evaluation
Lawton & Brody Balance and Coordination Scale
Instrumental Activities of Daily Living Scale
The Correct answer is:
Tinetti Balance and Gait Evaluation
The Tinetti scale and gait evaluation is an activity-based test that asks the patient to perform tasks, such as sitting and rising
from a chair, turning, and bending. The Index of Independence o Activities of Daily Living helps to identify daily activities
with which the patient needs assistance. The Instrumental Activities of Daily Living Scale assesses complex tasks.
Question 92
The second-most common cause of hearing loss is which of the following?
TM perforation
noise trauma
ototoxicity
cerumen impaction
Correct answer:
noise trauma
Noise trauma is the second most common cause of hearing loss. Sounds louder than 85 dB are potentially damaging to the
cochlea, especially with prolonged exposure.
Question 93
According to the National Asthma Education and Prevention Program Expert Panel Report 3 (NAEPP EPR-3) all of the
following are goals of asthma care EXCEPT:
minimal or no chronic symptoms such as cough and wheeze
no limitations on activities
no or minimal side effects while optimal medications are given
fully reverse impaired lung function
Correct answer:
fully reverse impaired lung function
This is incorrect. The formerly held concept in asthma was that no matter how severe the airway obstruction, the process was
fully reversible. The newer disease model acknowledges that continued airway inflammation contributes to significant and
potentially permanent airway remodeling and fixed obstruction. The NAEPP EPR-3 goal therefore is to prevent progressive
loss of lung function, not fully reverse impaired lung function.
Question 94
When counseling a patient who currently has a testicular torsion, the Family Nurse Practitioner educates him on the best
treatment approach. Which of the following is recommended as the first treatment option?
orchiopexy
avoidance of further testicular trauma
use of a scrotal support device
avoiding frequent sexual activity
The Correct answer is:
Orchiopexy
Testicular torsion is a urological emergency caused by a twisting of the testis and spermatic cord around a vertical axis. The
first approach before using a supportive device is surgery. Surgery (orchiopexy) is performed to untwist the testis allowing
both testes to be brought down and tacked lower in the scrotum to avoid subsequent torsion. Avoiding further trauma after the
surgical procedure, a scrotal support device may be helpful to relieve the discomfort but will not prevent recurrence.
Avoidance of sexual activity will not prevent testicular torsion.
Question 95
You are seeing a mother and her 12-month-old child in the office for a well-child examination. An example of a primary
prevention measure for a 12-month-old healthy infant would be:
367
doing a routine urinalysis at the well-child exam
educating the mother on home care and treatment of the child's active diaper dermatitis rash
administering the first dose of the measles, mumps, and rubella vaccine (MMR)
administering the first dose of the meningococcal vaccine (MCV)
The Correct answer is:
Administering the first dose of the measles, mumps, and rubella vaccine (MMR)
The measles, mumps, and rubella vaccine is an immunization, which is a primary prevention measure. The vaccine is first
given at 12 months of age. A urinalysis is not part of the 12 month well-child exam unless clinically indicated. Educating and
treating active diaper dermatitis is a tertiary measure. The meningococcal vaccine is not administered until 2 years of age
Question 96
In terms of marketing the nurse practitioner role, the elements of marketing include the four Ps. All of the following are one
of the four Ps EXCEPT:
product
price
potential
place
Correct answer:
potential
This is not one of the four Ps of marketing the NP role. The four Ps include: product, price, place, and promotion.
Question 97
Treatment options for lymphogranuloma venereum include all but which of the following?
tetracycline
doxycycline
erythromycin
penicillin
Correct answer:
penicillin
Penicillin is not one of the treatment options for lymphogranuloma venereum, which is an STI caused by Chlamydia
trachomatis types L1 to L3. As with all STIs a critical part of care is discussion of prevention strategies, such as condom use.
Question 98
A parent brings a 3 year old in for a follow-up appointment. The child was treated for bronchitis earlier. What sign in the
parent interview could indicate the child is showing an emergent condition?
The child is drinking fluids.
The child has gained weight.
The child is lethargic.
The child is active.
Correct answer: The child is lethargic Lethargy could indicate that the child is still sick or developing an emergent condition.
The other answers are positive indicators that the child's health is improving.
Question 99
The normal stomach pH is which of the following?
7
9
5
2
Correct answer:
2
The normal stomach pH is about 2, which kills many swallowed bacteria and viruses. Gastric acid production is about 1 to 2
mEq/hr in a resting empty stomach. This increases to 30 to 50 mEq/hr after a meal
Question 100
Which of the following is indicative of elder abuse?
burns or abrasions
threatening behavior by caregiver
financial discrepancies
all of the above
368
Correct answer:
all of the above
The most common form of elder abuse is neglect. It can include physical abuse, psychological abuse, financial abuse, and
sexual abuse.
quiz 27 aanp
Question 1
The cause of the highest mortality among teenagers in the U.S. is:
smoking
motor vehicle or unintentional injuries
suicide
congenital heart disease
The Correct answer is:
Motor vehicle or unintentional injuries
The number one cause of mortality in this age group is motor vehicle accidents or unintentional injuries. The second is death
from suicide.
Question 2
Which organization accredits various entities and has standards concerning medical record confidentiality?
National Committee for Quality Assurance (NCQA)
Joint Commission on Accreditation of Healthcare Organizations (JCAHO)
American Medical Association (AMA)
Department of Health, Education and Welfare (HEW)
The Correct answer is:
The Joint Commission on Accreditation of Healthcare Organizations (JCAHO)
JCAHO requires entities to develop a process to protect confidentiality of medical record information among other things.
NCQA accredits managed care entities and has their own standards of confidentiality. The American Medical Association and
the Department of Health and Education and Welfare are not involved with medical record confidentiality.
Question 3
You are educating the parent of a teenage girl who has been diagnosed with anorexia nervosa. You would NOT tell the parent
which of the following?
Approximately 90% of patients are female.
The maximum frequency of occurrence is 13 – 14 years of age.
It is associated with depression in 65% of cases.
It is associated with OCD in 26% of cases.
Correct answer:
The maximum frequency of occurrence is 13 – 14 years of age.
This is not an accurate statement. The maximum frequency of occurrence is 17 – 18 years of age. About 85% of cases have
onset usually at 14 – 18 years of age.
Question 4
A patient with fatigue, migratory arthralgia and cranial nerve palsies. These symptoms are consistent with what stage of Lyme
Disease?
1
2
3
None of these.
Correct answer: 2 There are three stages of Lyme Disease. Stage 1 is known for: fever, chills, myalgia and erythema migrans
Stage 2 is known for: fatigue, migratory arthralgia and cranial nerve palsies Stage 3 is known for: memory, sleep and mood
problems
Question 5
Which of the following tests is used to diagnose macular degeneration?
369
Weber test
Rinne test
Snellen chart
Amsler grid
Correct answer:
Amsler grid
Central vision is tested using the Amsler grid for persons suspected of having macular degeneration. If the lines in the center
of the grid are distorted this is a positive sign of macular degeneration.
Question 6
Which of the following conditions in the patient’s past medical history is least likely to be associated with congestive heart
failure (CHF)?
renal disease
cancer
hypertension
severe anemia
Correct answer:
cancer
Cancer is not normally associated with CHF. A past medical history of any of the following may be associated with CHF:
CAD, COPD, renal disease, diabetes, hypertension, previous MI, valvular disease, severe anemia, and more.
Question 7
Which of the following types of hepatitis is NOT transmitted via blood and body fluids?
Hepatitis C
Hepatitis A
Hepatitis D
Hepatitis B
Correct answer:
Hepatitis A
Hepatitis A is not transmitted by blood and body fluids. It is transmitted through a fecal-oral route.
Question 8
The New York Longitudinal Study defines nine dimensions of temperament. Which of the following traits is included in these
nine dimensions?
sociability
synchrony
morality
sensitivity
Correct answer:
sensitivity
Sensitivity is one of the nine dimensions of temperament. The others are: activity, rhythmicity, approach (or withdrawal),
adaptability, intensity, mood, distractibility and attention span.
Question 9
You have a patient with back pain but you suspect malingering. In order to rule this out, you use Waddell’s signs. All of the
following are part of this standardized group of nonorganic physical signs EXCEPT:
axial loading
regionalization
reproducing pain
simulation
Correct answer:
reproducing pain
370
Reproducing pain is done in Magnuson’s test for malingering. Waddell’s signs include all of the other three choices plus:
overreaction, distraction and tenderness.
Question 10
The theory that a person who believes that he can succeed in performing an action that will result in a positive outcome is
more likely to perform the healthier behavior is which of the following?
Systems Theory
Self-Efficacy Theory
Health Belief Model
Family Systems Theory
Correct answer:
Self-Efficacy Theory
The Self-Efficacy Theory holds that a person who believes that he can succeed in performing an action that will result in a
positive outcome is more likely to perform the healthier behavior. This concept was put forth by Albert Bandura in the 1970s.
Question 11
If an adult presents with the classic triad of fever, headache and stiff neck, which of the following diseases/conditions would
these symptoms most likely indicate?
migraine headache
bacterial meningitis
mononucleosis
chronic epidural hematoma
Correct answer:
bacterial meningitis
Of the choices given, the most likely diagnosis would be bacterial meningitis. As with most forms of infectious disease,
however, atypical presentation in older adults is common. In particular, stiff neck and fever are often absent.
Question 12
You are caring for an elderly patient who has had a cerebrovascular accident (CVA) and has urinary incontinence. The family
should be taught to:
insert a Foley catheter
establish a scheduled voiding pattern
restrict fluid intake
reposition the patient often to reduce the discomfort of urgency
The Correct answer is:
Establish a scheduled voiding pattern
The patient needs a reestablished pattern for regularity to assist in maintaining bladder control. A Foley catheter would expose
the patient to infection and fluids should not be restricted.
Question 13
Which of the following responses in a physical exam could indicate damage or inflammation of the spinal cord?
Weak reflexes.
Peripheral neuropathy.
Hyper flexion.
Gag reflex.
Correct answer: Hyper flexion Hyper flexion indicates that the nerves are hyperactive. This is typically indicative of problems
in the spinal cord. Weak reflexes can indicate peripheral neuropathy.
Question 14
After talking to you about his smoking and drinking, your client tells you that he will think about changing his habits but he is
not ready yet. You help him to examine the benefits and obstacles that would be associated with his change. This stage in the
transtheoretical model of change is which of the following?
maintenance
preparation
contemplation
precontemplation
Correct answer:
contemplation
Contemplation involves considering a behavioral change in the foreseeable future, possibly within 6 months. The role of the
371
NP includes continuing consciousness raising, providing information, praising for considering, and exploring ways to reduce
barriers and obstacles to change.
Question 15
Madeline Leininger envisioned blending the disciplines of nursing and anthropology in both theory and practice to enable a
cultural perspective for the professional practice of nursing. Her theory is known as which of the following?
enculturation
cultural blindness
transcultural nursing
melting pot theory
Correct answer:
transcultural nursing
Madeline Leininger promoted the theory of transcultural nursing. Although not all advanced practice nurses practice the full
scope of transcultural nursing as a specialization, all are encouraged to provide care that is culturally competent.
Question 16
You have an adult patient of Scottish descent who reports that she has noticed a red, acne-like area around her nose and
mouth. She tells you that it comes and goes. Your most likely diagnosis of the following choices would be:
cheilosis
rosacea
cystic acne
tinea barbae
Correct answer:
rosacea
Rosacea is seen more in adults of Celtic background with pale skin, blue eyes, and red-to-blond hair. The patient usually
complains of chronic, small acne-like papules and pustules that erupt around the nose, mouth, and chin, which become
exacerbated episodically.
Question 17
You are seeing a 3-year-old female child in the office. Her mother is concerned about her not being able to elevate her left arm
without crying out. A clavicular fracture is suspected. Which of the following is NOT true regarding a fracture of the clavicle?
This is one of the most common fractures that occur in childhood.
The patient may avoid moving the arm on the injured side or may angle the head toward the injured side to relax the trapezius
muscle.
There is usually no visible or palpable deformity upon examination and it is rarely seen on an x-ray.
The mechanisms of injury is usually a fall on the outstretched hand or direct blow to the shoulder area.
The Correct answer is:
There is usually no visible or palpable deformity upon examination and it is rarely seen on an x-ray
There is most always a visible and a palpable deformity seen and the x-ray findings will show a visible separation of the
clavicular bone. Further, the remaining answer choices are all true of this type of fracture.
Question 18
According to Duvall, families pass through eight chronological stages, and success in one task sets the stage for success in
subsequent tasks. Which of the following is NOT one of Duvall’s stages of development?
family with old age and retirement
launching center family
childbearing family
prenatal family
Correct answer:
prenatal family
This is not a stage of Duvall’s theory of families. His stages are: beginning family, childbearing family, family with preschool
children, family with schoolchildren, family with teenagers, launching center family, family with middle-aged parents and
family with old age and retirement.
Question 19
Which of the following herbal over-the-counter medicines is NOT used for anxiety and/or depression?
Kava kava
St. John’s Wort
372
Valerian root
none of the above
Correct answer:
none of the above
All of the choices except this one are herbal remedies for anxiety and/or depression. St. John’s Wort acts like MAOI, SSRI, or
TCA. Kava kava has action at GABA receptors similar to benzodiazepines. Valerian root also has action similar to
benzodiazepines.
Question 20
Which of the following is a modifiable environmental factor affecting the health and health care of patients?
climate
altitude
air pollution
none of the above
Correct answer:
air pollution
Air pollution is one of the factors affecting health that can be modified. Other modifiable factors include: water fluoridation,
water contamination, crime, poverty and transportation.
Question 21
A milder form of Stevens-Johnson syndrome that produces pink-to-red targetlike lesions, wheals, and blisters, with no
mucosal involvement is which of the following?
erythema multiforme
actinic keratosis
erythema migrans
shingles
Correct answer:
erythema multiforme
Erythema multiforme is a milder form of Stevens-Johnson syndrome that produces pink-to-red targetlike lesions, wheals, and
blisters, with no mucosal involvement. The clinician should look for a history of antibiotic and other drug treatment such as
sulfa drugs, penicillins, and other drugs.
Question 22
A good quality assurance program should do which of the following?
identify educational needs
improve the documentation of care
reduce the clinician’s overall exposure to liability
all of the above
Correct answer:
all of the above
A good quality assurance program would include all of the first three choices. These programs also look at organizational
effectiveness, efficiency, and client and provider interactions.
Question 23
An elderly male has a diagnosis of left-sided congestive heart failure (CHF). The nurse practitioner would identify which of
the following as a common condition associated with CHF?
ventricular dysrhythmias
peripheral vascular disease
untreated hypertension
chronic obstructive pulmonary disease
The Correct answer is:
Untreated hypertension
Untreated hypertension causes significant increased work of the left ventricle. Further, the years of untreated high blood
pressure eventually causes congestive heart failure (CHF).
Question 24
373
When treating the patient with heartburn symptoms, the nurse practitioner understand that the drug that is most likely to
produce rapid relief for a patient is:
H2 blockers
proton pump inhibitors
sucralfate
antacids
The Correct answer is:
Antacids
Antacids produce the most rapid change in gastric pH and the most rapid relief of symptoms. H2 blockers and proton pump
inhibitors may take many hours before relief is realized. Therefore, H2 blockers are not adequate for immediate relief. Further,
sucralfate does not affect gastric pH.
Question 25
You want to test a child for color-blindness. You would use which of the following?
Snellen chart
Ishihara chart
Kawasaki chart
Snellen picture chart
Correct answer:
Ishihara chart
The Ishihara chart is used for screening color blindness. The Snellen Chart is used for screening central distance vision if the
child knows the alphabet; if not the Snellen picture chart is used.
Question 26
Which of the following would NOT be considered a good strategy for improving access to health care?
Train more specialists and fewer primary care providers.
Establish community health centers.
Increase the number of minority providers.
Establish migrant health centers.
Correct answer:
Train more specialists and fewer primary care providers.
This would not be a good strategy for improving access to health care. More primary care providers are needed in order to
improve access for those underserved populations.
Question 27
In terms of human growth and development in infancy, which of the following would be a normal finding?
Head circumference is approximately 2 cm larger than chest during the first year of life.
During childhood the chest is usually 10 – 12 cm larger than head circumference.
The posterior fontanel is always palpable at birth.
The size of the anterior fontanel should be no larger than 7 -8 cm in diameter.
Correct answer:
Head circumference is approximately 2 cm larger than chest during the first year of life.
This would be a normal finding. The other choices are not. During childhood the chest is usually 5 - 7 cm larger than head
circumference. The posterior fontanel is rarely palpable at birth. The size of the anterior fontanel should be no larger than 4 -5
cm in diameter.
Question 28
Which of the following statements about pityriasis rosea is accurate?
Pityriasis is mainly a disorder of those with Mediterranean heritage.
It occurs in men more than in women.
There is no racial predilection.
It is most common in those over 65 years of age.
Correct answer:
There is no racial predilection.
374
This is the accurate statement. Pityriasis is mainly a disorder of fair-skinned persons. It occurs in women more than in men. It
is common in ages 10 to 35
Question 29
You are treating a patient with acute sinusitis. You understand that the “gold standard” first line medication for this condition
is which of the following?
cephalosporins
trimethoprim sulfamethoxazole
Ceftin
amoxicillin
Correct answer:
amoxicillin
Acute sinusitis is an infection of the sinuses by Gram-positive and Gram-negative bacteria. For treating acute sinusitis,
amoxicillin is the “gold standard” for any age group.
Question 30
Which of the following statements about the approaches to healthcare issues is incorrect?
Reducing health disparities is a major concern.
There is decreasing emphasis on culturally competent care and evidence-based practice.
More than 45 million Americans are underserved because of limited access to care.
Causes of infant mortality have changed over the years.
Correct answer:
There is decreasing emphasis on culturally competent care and evidence-based practice.
This is incorrect. The opposite is true. There is increasing emphasis on culturally competent care and evidence-based practice.
Question 31
Which of the following is the least likely risk factor for suicide?
black race
male
increasing age
physical illness
Correct answer:
black race
Being of a white race is more likely to be a factor in suicide. Besides the other three choices risk factors include: substance
abuse, living alone, and less education.
Question 32
A 78-year-old woman who presents with incontinence, is prescribed Detrol. She has the most common type of incontinence
among the elderly population. What type of incontinence is it?
Urge.
Stress.
Functional.
Transient.
Correct answer: Urge There are pharmalogical and non-pharmacological aids that assist with alleviating this type of
incontinence. Behavioral therapy such as gentle bladder stretching and pharmacological care such as being prescribed
VESIcare and Detrol have proven effective.
Question 33
Which of the following is the leading cause of cancer deaths in women?
breast cancer
lung cancer
colon cancer
pancreatic cancer
375
Correct answer:
lung cancer
Lung cancer is the leading cause of cancer deaths in women. Breast cancer is the second leading cause.
Question 34
Which of the following danger signals of pregnancy and childbirth is characterized by a patient who is older than 25 years,
multipara and in the third trimester of pregnancy complaining of severe headaches, edema of the face and hands, abdominal
pain, nausea and/or vomiting and fatigue and having elevated liver enzymes and low platelets?
eclampsia
placenta previa
abruptio placentae
HELLP syndrome
Correct answer:
HELLP syndrome
HELLP syndrome is a rare but serious complication of pregnancy. It is characterized by all of the conditions in the question as
well as hemolysis, elevated liver enzymes and low platelets (HELLP).
Question 35
You are prescribing a high-fiber diet for a patient with diverticulosis. You would recommend all of the following foods as high
in fiber EXCEPT:
bran flakes
brown rice
eggs
kidney beans
Correct answer:
eggs
Eggs are not high in fiber. In fact they are recommended for low-residue diets.
Question 36
You are seeing an 18-year-old woman in the office with a "pimple" on her left eyelid. Upon examination, you find a 2.5 mm
pustule on the lateral boarder of the eyelid. This is most consistent with:
cellulitis
chalazion
blepharitis
hordeolum
The Correct answer is:
Hordeolum
A hordeolum, or stye, is caused by staphylococcal infection of a hair follicle. Cellulitis is a serious complication of a
hordeolum and severe edema and redness would be present. A chalazion is characterized by a hard, nontender swelling of the
upper or lower lid.
Question 37
The FNP is seeing a college-age female who participated in strenuous sports activities in high school. The FNP understands
that all of the following may be a risk for this patient EXCEPT:
menorrhagia
eating disorders
delayed menses
osteoporosis
Correct answer:
menorrhagia
Menorrhagia (heavy menstrual bleeding) is not a common concern for women who take part in strenuous physical activity.
Females who participate in strenuous physical activities are at risk for eating disorders, delayed menses and osteoporosis.
Question 38
A young woman comes in with a new rash on her hands and face. What question will help determine what type of rash this is?
Has your weight changed?
Have you changed cosmetics?
How much water do you drink daily?
376
When where you born?
Correct answer: Have you changed cosmetics? Cosmetics do affect the skin. Changing or adding any new cream, even natural
creams, can cause allergic reactions. Always ask about cosmetics with rashes.
Question 39
You have a 55-year-old male patient who has a history of sickle cell anemia. He has come to you because he has had an
erection that lasted over 4 hours. You understand that this condition is known as which of the following?
erectile dysfunction
epididymis
transillumination
priapism
Correct answer:
priapism
Priapism is a continuous penile erection of at least 4 hours or longer unrelated to sexual desire or stimulation. Persons with a
history of sickle cell anemia or other blood clotting disorders may suffer from this. It may also be the result of taking drugs for
erectile dysfunction.
Question 40
A 32 year old female patient reports feeling achy and nauseous for three weeks. She also says her urine is very dark. Her
temperature is 100.7F. She has a long-term history of multiple male sexual partners. This patient should be further assessed for
which of the following?
Syphilis.
Hepatitis A.
Hepatitis B.
Hepatitis C.
Correct Answer: Hepatitis B This patient's symptoms are consistent with Hepatitis B. Her sexual history also puts her at
elevated risk for the disease, which is sexually transmitted. Further testing should be undertaken to confirm the diagnosis.
Question 41
In managing the client with fibromyalgia, the nurse practitioner includes what teaching instruction?
Take ibuprofen (Motrin) for pain and amitriptyline (Elavil) 1-2 hours before bedtime
Avoid stretching exercises and daily low impact aerobics.
Apply heat or massage "trigger points" to reduce pain.
Diagnostic studies, such as ESR and CBC, are important tools to assess treatment and disease progression.
Correct answer:
Take ibuprofen (Motrin) for pain and amitriptyline (Elavil) 1-2 hours before bedtime
For patients with fibromyalgia, the nurse practitioner should include in the patient teaching that daily, slow, low-impact
aerobics as well as heat and massage is encouraged for pain reduction and better physical functioning. Further, medication
treatment options include tricyclic antidepressants at a low dose, such as amitriptyline (Elavil), which helps reduce symptoms
associated with fibromyalgia. Other medications such as muscle relaxants, for example cyclobenzaprine (Flexeril), pain
relievers and anticonvulsant drugs, such as Lyrica may also be helpful in reducing symptoms associated with fibromyalgia.
Further, diagnostic testing is not a treatment option for fibromyalgia. The diagnosis of fibromyalgia is based on the patient
having pain in 11 or more of the areas identified as tender points, along with other symptoms of fibromyalgia.
Question 42
What is the purpose of the National Practitioner Databank?
It was established to scrutinize members of the healthcare profession and list any malpractice claims that exist against them.
It was established to allow insurance companies ease in credentialing members for their provider networks.
It was established to assure reimbursement for nurse practitioners who are providers of Medicare, Medicaid, and other
federal insurance programs.
It was established to regulate the prescribing practices of nurse practitioners and physicians.
The Correct answer is:
It was established to scrutinize members of the healthcare profession and list any malpractice claims that exist against them.
The National Practitioner Databank was established by the Health Care Quality Improvement Act of 1986. Currently, very few
nurse practitioners are listed in this database.
Question 43
377
Which of the following medications would you prescribe for a 36 year old female with hypothyroidism?
Tapazole.
Synthroid.
PTU.
Propylthiouracil.
Correct Answer: Synthroid Also known as levothyroxine, Synthroid is commonly prescribed for hypothyroidism. The other
choices listed are commonly prescribed for hyperthyroidism.
Question 44
A 5-year-old school age child is in the clinic with varicella. The nurse practitioner has knowledge that the following is the best
approach to relieve discomfort in a child with varicella:
aspirin
acetaminophen
alcohol rub
ibuprofen
The Correct answer is:
Acetaminophen
Aspirin should be avoided in children, especially when they have a viral illness due to Reye syndrome. Ibuprofen is not the
best answer choice as it increases the risk of necrotizing fascitis when used in the varicella patient. Then, an alcohol rub will
irritate the patient's lesions and exacerbate pain.
Question 45
Which of the following is least likely to be associated with a varicocele in your patient?
It is found in the left scrotum.
It is present only in the supine position.
Surgery is curative.
Scrotal support can be helpful for relief of discomfort.
Correct answer:
It is present only in the supine position.
This is the least likely presentation to be associated with a varicocele. A varicocele is present while the man is standing. It
disappears in the supine position
Question 46
You believe that if one family member is dysfunctional, the rest of the family is affected negatively. This theory is known as
which of the following?
self-efficacy theory
health belief model
comparative model
family systems theory
Correct answer:
family systems theory
The family systems theory holds that families develop at a different rate. If one family member is dysfunctional, the rest of the
family is affected negatively. It is derived from the systems theory.
Question 47
It has become more necessary for advanced practice nurses to assume emergency roles during a time of mass casualties from
natural disasters and terrorist attacks. All of the following are true EXCEPT:
The International Nursing Coalition for Mass Casualty Education is an organization established to fund supplies for victims
of a disaster.
The International Nursing Coalition for Mass Casualty Education educates nurses to respond safely and effectively to mass
casualty incidents.
All nurse practitioners are expected to prepare themselves to play a larger role in delivery of care during a time of disaster.
Other countries have had more experience dealing with terrorism than the U.S.
378
Correct answer is:
The International Nursing Coalition for Mass Casualty Education is an organization established to fund supplies for victims of
a disaster.
The purpose of this organization is to improve the ability of all nurses to respond safely and effectively to mass casualty
incidents through identifying existing and emerging roles of nurses and ensuring appropriateness of education. They do not
offer supplies or funding for victims. The other answers are accurate.
Question 48
Which of the following parts of Medicare pays 80% of durable medical equipment such as wheelchairs and walkers?
Part D
Part A
Part B
Medicare does not pay for durable medical equipment.
Correct answer:
Part B
Medicare Part B pays for medically necessary services or supplies (outpatient). This includes durable medical equipment such
as wheelchairs and walkers
Question 49
A patient recently diagnosed with fibromyalgia is about to start taking Lyrica for the condition. She also takes LEV for
epilepsy. This patient should be carefully watched for which of the following?
Suicidal thinking or behavior.
Intestinal bleeding.
Symptoms of stroke.
Increased risk of bleeding.
Correct Answer: Suicidal thinking or behavior Lyrica is know to have a possible drug interaction with antiepileptic drugs
(AEDs), including LEV. The patient should be carefully watched for suicidal thoughts or behaviors, which may result from
this interaction. Changes in behavior or worsening of suicidal thoughts should also be noted in these patients.
Question 50
The FNP is working in a college clinic and sees a student complaining of high fever, headache, stiff neck and vomiting. She
has purple color lesions on her body. Which of the following diseases/conditions would the FNP most likely suspect?
Stevens-Johnson syndrome
erythema multiforme
meningococcemia
Lyme disease
Correct answer:
meningococcemia
The highest incidence of meningococcemia is in college students residing in dormitories. Vaccine is recommended for this
population. Meningococcemia is a serious life-threatening infection spread by respiratory droplets. It has a 20% mortality, but
if treated early, the mortality is less than 5%.
Question 51
Most kidney stones are composed of which of the following?
uric acid
calcium
cystine
struvite
Correct answer:
calcium
Most urinary and kidney stones are composed of calcium (80%). Uric acid accounts for about 5%, and cystine, 2%, with
struvite accounting for even less.
Question 52
379
A vegan complains of GI problems. The patient does not take vitamin supplements. What test should be run?
Blood glucose.
Vitamin C.
Serum albumin.
Vitamin B12.
Correct answer: Vitamin B12 Some vegans have trouble maintaining Vitamin B12 levels. A blood test for B12 is necessary,
particularly if there is GI trouble
Question 53
Which of the following medications is NOT appropriate to prescribe for the management of genital herpes?
Zovirax.
Diflucan.
Valtrex.
Famvir.
Correct Answer: Diflucan Diflucan, or flucanazole, is an oral medication prescribed for the treatment of vaginal yeast
infections. The other choices are suppressive medications prescribed for the management of genital herpes.
Question 54
For which of the following conditions/diseases would the FNP test by using the Phalen’s sign?
Bell’s Palsy
carpal tunnel syndrome
osteoporosis
glaucoma
Correct answer:
carpal tunnel syndrome
Carpal tunnel syndrome is a median nerve compression that is due to swelling of the carpal tunnel. The Phalen’s sign is full
flexion of the wrist for 60 seconds. If the patient’s symptoms are reproduced, it is a positive finding.
Question 55
Lack of which of the following skills in a school-age child would be most likely to cause concern?
formal operational thought
inability to assert his or her independence
inability to reverse a process or action
lack of awareness of his or her body image
Correct answer:
inability to reverse a process or action
The only skill of the three that may cause concern is the inability to reverse a process or action, for instance, the understanding
that if 1 + 1 = 2, then 2 – 1 =1. All of the other skills are attributable to later development.
Question 56
Your pregnant patient has been complaining of heartburn. You would tell her to do all of the following EXCEPT:
eat small, frequent meals
eat papaya or raw almonds after meals
avoid fried or spicy foods
use antacids containing baking soda, aluminum, or high sodium content
Correct answer:
use antacids containing baking soda, aluminum, or high sodium content
Antacids with magnesium hydroxide or magnesium trisilicate may be used. Antacids with baking soda, aluminum, or high
sodium content should be avoided.
Question 57
Which of the following statements about calcium channel blockers (CCBs) is incorrect?
Vasotec is the most commonly used CCB.
CCBs depress heart muscle and the AV node.
Bradycardia is a contraindication to the use of CCBs.
380
Norvasc is a CCB drug.
Correct answer:
Vasotec is the most commonly used CCB.
Vasotec (enalapril) is an ACE inhibitor drug, not a CCB. CCBs block calcium channels in the arterioles resulting in systemic
vasodilation, which results in decreasing PVR. They depress heart muscle and the AV node.
Question 58
Which of the following is the best definition for positive predictive value (PPV)?
a more systematic evaluation using a standardized or generally accepted method leading to recommendations for
intervention
the proportion of those with an abnormality who are correctly identified through screening (true positives)
the proportion of those individuals correctly screened as positive of all those who actually have an abnormality
the proportion of those without an abnormality who are correctly identified as negative through screening (true negatives)
Correct answer:
the proportion of those individuals correctly screened as positive of all those who actually have an abnormality
The positive predictive value (PPV) is defined as the proportion of those individuals correctly screened as positive of all those
who actually have an abnormality. A more systematic evaluation using a standardized or generally accepted method leading to
recommendations for intervention is assessment. The proportion of those with an abnormality who are correctly identified
through screening (true positives) is sensitivity. The proportion of those without an abnormality who are correctly identified as
negative through screening (true negatives) is specificity.
Question 59
A 16-year-old male patient presents in the clinic with abdominal pain, worse with ambulation, nausea, and fever. Next, the
nurse practitioner would test the obturator and iliopsoas muscle to evaluate for which of the following?
cholecystitis
acute appendicitis
inguinal hernia
gastric ulcer
The Correct answer is:
Acute appendicitis
Acute appendicitis is common in this age group and the symptoms are indicative of the condition. The obturator and iliopsoas
muscles are sensitive when
Question 60
Which of the following is NOT true about a durable power of attorney for health care?
It is not limited to the circumstances of terminal illness.
It is not binding except in certain circumstances.
It is often accompanied by a durable power of attorney over financial issues as well.
It is flexible enough to carry out the patient’s wishes throughout the course of an illness.
Correct answer:
It is not binding except in certain circumstances.
This is incorrect. A durable power of attorney for health care is always binding.
Question 61
The incubation period for Hepatitis A is which of the following?
45 - 160 days
7 – 10 days
30 – 40 days
15 – 50 days
Correct answer:
15 – 50 days
Hepatitis A is transmitted through the fecal-oral route in contaminated food and water. Its incubation period is 15 – 50 days.
Hepatitis B and D have incubation periods of 45 – 160 days.
Question 62
381
A 67-year-old diabetic has been taking oral antihyperglycemic medication. The patient is still having poor glycemic control.
You make the decision to start insulin therapy. He weighs 60 kg. What should you order as an initial starting dose?
6 units short-acting insulin at breakfast, continue oral medication
6 units intermediate insulin at bedtime, stop oral medication
6 units long-acting before breakfast, stop oral medication
12 units long-acting insulin at bedtime, continue oral medication
The Correct answer is:
12 units long-acting insulin at bedtime, continue oral medication
The American Diabetic Association (ADA) algorithm for initiation and adjustment of therapy (2006) suggests an intermediate
or long-acting insulin to be started at bedtime or morning as a once daily dose. According to the ADA, the starting dose is
either 10 units or 0.2 units per kilogram. For this 67-year-old diabetic patient, the nurse practitioner starts him on 12 units,
based on the calculation of 0.2 units per kilogram (0.2 X 60 kg). Oral medication should be continued except for discontinuing
sulfonylureas or meglitinides.
Question 63
Which of the following would NOT be considered a risk factor for gastroesophageal reflux disease (GERD)?
hiatal hernia
esophageal clearance
gastric outlet obstruction
underweight
Correct answer:
underweight
Being underweight is not a risk factor for GERD. Obesity is a risk factor.
Question 64
Type 2 diabetes mellitus testing should be considered in all adults who are overweight and have additional risk factors. All of
the following are additional risk factors to consider EXCEPT:
history of cardiovascular disease
women with polycystic ovarian syndrome
member of a high risk population
history of hypotension
Correct answer:
history of hypotension
This is not one of the additional risk factors for type II diabetes. All of the other choices are additional risk factors along with:
physical inactivity, first-degree relative with diabetes, women who delivered an infant weighing over 9 pounds, women who
have had gestational diabetes, hypertension, HDL cholesterol level of < 35 mg/dL, triglyceride level > 250 mg/dL, impaired
glucose tolerance and other clinical conditions associated with insulin resistance.
Question 65
The FNP has a pregnant patient who is diagnosed with chlamydia trachomatis. Which of the following treatments is preferred
for this woman?
azithromycin 1 gm PO single dose
doxycycline 100 mg BID x 7 days
Cipro 500 mg x one dose
erythromycin base QID x 7 days
Correct answer:
azithromycin 1 gm PO single dose
For pregnant women erythromycin base QID x 7 days was the preferred treatment. However the CDC recommendation at the
present time for a pregnant patient who is diagnosed with chlamydia trachomatis is azithromycin 1 gm PO single dose
Question 66
382
If the FNP is testing the visual acuity of a 3-year-old child, which of the following results would be considered in the normal
range?
40/40
20/20
20/40
all of the above
Correct answer:
20/40
Visual acuity for toddlers is normally 20/40. A visual acuity of 20/20 would be exceptional for a toddler. Visual acuity of
40/40 would indicate less than normal visual acuity.
Question 67
You are counseling a 23-year-old with an abnormal pap smear. It is important you discuss the risks of cervical cancer. Which
of the following is NOT a risk factor for cervical cancer?
virginal status
previous high-grade squamous intraepithelial lesion
human papillomavirus
multiple sexual partners
The correct answer is:
Virginal status
A female who has not had sexual intercourse is not at risk for developing cervical cancer. However, the remaining answer
choices are risk factors for cervical cancer.
Question 68
Orthopedic referral would most likely be made in which of the following grades of sprain?
Grade IV
Grade III
Grade II
Grade I
Correct answer:
Grade III
Orthopedic referral would be made in the most serious grade of sprain which is Grade III. There is no Grade IV sprain. Grade
III sprains have a complete ligamentous tear, complete ankle instability, significant swelling and moderate to severe
ecchymosis.
Question 69
The FNP has an adult patient who presents with a rash on the hands and neck that consists of bright red weepy lesions. The
FNP diagnoses eczema. Which of the following should be avoided in the treatment of this disease?
topical steroids
skin lubricants
hot water baths
systemic oral antihistamines
Correct answer:
hot water baths
In treating patients with eczema, the FNP should advise that patient to avoid drying the skin. Hot water baths, harsh
detergents, chemicals and wool clothing should be avoided. Medications for this disease include topical steroids, systemic oral
antihistamines and skin lubricants such as Eucerin, Keri lotion and baby oil.
Question 70
Your 65-year-old female patient has developed a scaly red-colored rash on the nipple. She tells you that she thought it would
just “go away” but that instead it has enlarged and now there is some nipple discharge. You understand that this is which of
the following diseases?
Paget’s disease
breast cancer
fibrocystic breast
sign of STD
383
Correct answer:
Paget’s disease
Paget’s disease of the breast (ductal carcinoma in situ) occurs when the woman develops a chronic scaly red-colored rash
resembling eczema on the nipple that does not heal and slowly enlarges over time. The skin lesion slowly evolves to include
crusting, ulceration, and/or bleeding. Some women complain of itching. Nipple discharge and/or a lump may be present.
Question 71
A 39-year-old Caucasian male is started on Aldactone 50 mg PO qd. He has a longstanding history of hypertension. The nurse
practitioner knows that she should instruct him to call the clinic if which symptoms are experienced?
Decreased reflex response, nausea, and vomiting.
Muscle twitching, numbness of the limbs, and depression.
Increased irritability, abdominal cramping, and lower extremity weakness.
Weight gain, excessive thirst, and fever.
The Correct answer is:
Increased irritability, abdominal cramping, and lower extremity weakness
Aldactone is a potassium-sparing diuretic and the patient should be instructed on the side effects of this medication. Signs or
symptoms, such as lower extremity weakness, may indicate hyperkalemia. Aldactone can place the patient at increased risk for
hyperkalemia. The remaining answer choices have no relevance to Aldactone side effects or mechanism of action.
Question 72
At what age would you schedule a female patient for a mammogram as part of a routine screening?
25
30
40
55
Correct answer: 40 Mammograms become part of routine screenings as after patients reach 40 or older. This number may vary
slightly by institution. A mammogram is typically given every year or two, depending on the risk factors
Question 73
You have a patient diagnosed with acute bronchitis. Which of the following types of drugs are you NOT likely to prescribe?
antitussives
mucolytics
antibiotics
guaifenesin
Correct answer:
antibiotics
Acute bronchitis requires symptomatic treatment only. No antibiotics should be prescribed. Acute bronchitis is characterized
by paroxysms of dry coughing and wheezing, and can last up to 4 to 6 weeks.
Question 74
You are conducting an assessment with a male patient who is complaining of chills and chest pain. He has had a cough for two
weeks, which produces yellow-green sputum. His temperature is 101F. All but which of the following are possible diagnoses
for this patient?
Pertussis.
Viral pneumonia.
Bacterial pneumonia.
Upper respiratory infection.
Correct Answer: Pertussis This patient's symptoms are not in keeping with pertussis, which has a hacking cough and lowgrade fever. This patient's temperature is too high, and the chest pain and productive cough are more indicative of pneumonia
or upper respiratory infection. Sputum cultures may be needed to correctly diagnose this patient.
Question 75
It is important that medical personnel be familiar with the source and use of pharmaceutical drugs. What are the two more
common ways drugs are prescribed to be taken?
Dissolved or crushed.
Orally or intravenously.
In food or drink.
Give the option to take or not to take.
384
Correct answer:
Orally or intravenously
Drugs given orally is the most common method. However, intravenously given drugs are absorbed quicker.
Question 76
Conforming to a standard of what is considered right and good is which of the following?
ethics
behavior
morality
autonomy
Correct answer:
morality
Ethics is the study of standards of conduct and moral judgment. It is the study of morality. Morality is conforming to a
standard of what is considered right and good.
Question 77
The FNP has a pregnant patient who is Rh negative and whose baby is Rh positive. The treatment for the infant antepartum
includes which of the following?
transfusion with Rh negative blood
phototherapy
transfusion of packed red blood cells
administration of gamma globulin
Correct answer:
transfusion with Rh negative blood
Once a diagnosis has been established, antenatal treatment includes transfusion of the fetus with Rh negative blood. All of the
other choices are parts of postpartum interventions.
Question 78
Certification for nurse practitioners is granted by all of the following EXCEPT:
AANP
Nurse Practice Act
ANCC
NCC
Correct answer:
Nurse Practice Act
The Nurse Practice Act of each state defines nursing, including advanced practice nursing, but it is not a certification
organization as are the other three choices. Another organization is the National Association of Pediatric Nurse Associates and
Practitioners (NAPNAP).
Question 79
The theorist who developed a theory of basic needs and human potential is which of the following?
Carl Rogers
Mary Ainsworth
Abraham Maslow
Charles Darwin
Correct answer:
Abraham Maslow
Abraham Maslow developed the theory of basic needs and human potential derived from the study of healthy, creative
individuals. The hierarchy of needs includes: physiologic; safety, security and stability; affiliation, acceptance and love; ego,
self-worth, confidence, competence and success; and self-actualization.
Question 80
In educating a client who has recurrent episodes of gout, you would tell him all of the following EXCEPT:
avoid alcohol binges
avoid fasting
385
avoid low-calorie diets
limit fluid intake
Correct answer:
limit fluid intake
You would encourage generous fluid intake for a person with gout. This helps to avoid kidney stones.
Question 81
Which of the following is NOT part of the triad involved with preeclampsia?
hypertension
proteinuria
anemia
edema
Correct answer:
anemia
Anemia is not part of the triad of symptoms involved with preeclampsia. This condition can cause multi-organ failure and
death. It may start at or after 20 weeks of gestation up to the postpartum period.
Question 82
The nurse practitioner understands that Title XIX of the Social Security Act (Medicaid) is:
Medical assistance for those persons 65 years of age and older only.
Medical assistance for low income persons.
Medical assistance for disabled persons.
Both B and C
The Correct answer is:
Both B and C
Medicaid is an insurance program designed for the low income person, the disabled, the blind, or members of families with
dependent children. It is not set up for those over 65 years of age only.
Question 83
When treating a patient with reactive arthritis (also known as Reiter syndrome), the nurse practitioner knows that in a sexually
active man this will include:
oral steroid therapy
immunosuppressive medications
antimicrobial therapy
antirheumatic medications
The Correct answer is:
Antimicrobial therapy
Treatment includes